Sunteți pe pagina 1din 107

1

Republic of the Philippines


SUPREME COURT
Manila
EN BANC
G.R. No. L-4963

January 29, 1953

MARIA USON, plaintiff-appellee,


vs.
MARIA DEL ROSARIO, CONCEPCION NEBREDA,
CONRADO NEBREDA, DOMINADOR NEBREDA, AND
FAUSTINO NEBREDA, Jr., defendants-appellants.
Priscilo Evangelista for appellee.
Brigido G. Estrada for appellant.
BAUTISTA ANGELO, J.:
This is an action for recovery of the ownership and possession
of five (5) parcels of land situated in the Municipality of
Labrador, Province of Pangasinan, filed by Maria Uson against
Maria del Rosario and her four children named Concepcion,
Conrado, Dominador, and Faustino, surnamed Nebreda, who
are all of minor age, before the Court of First Instance of
Pangasinan.
Maria Uson was the lawful wife of Faustino Nebreda who upon
his death in 1945 left the lands involved in this litigation.
Faustino Nebreda left no other heir except his widow Maria
Uson. However, plaintiff claims that when Faustino Nebreda
died in 1945, his common-law wife Maria del Rosario took
possession illegally of said lands thus depriving her of their
possession and enjoyment.
Defendants in their answer set up as special defense that on
February 21, 1931, Maria Uson and her husband, the late
Faustino Nebreda, executed a public document whereby they
agreed to separate as husband and wife and, in consideration
of their separation, Maria Uson was given a parcel of land by
way of alimony and in return she renounced her right to inherit
any other property that may be left by her husband upon his
death (Exhibit 1).
After trial, at which both parties presented their respective
evidence, the court rendered decision ordering the defendants
to restore to the plaintiff the ownership and possession of the
lands in dispute without special pronouncement as to costs.
Defendants interposed the present appeal.
There is no dispute that Maria Uson, plaintiff-appellee, is the
lawful wife of Faustino Nebreda, former owner of the five
parcels of lands litigated in the present case. There is likewise
no dispute that Maria del Rosario, one of the defendantsappellants, was merely a common-law wife of the late Faustino
Nebreda with whom she had four illegitimate children, her now

co-defendants. It likewise appears that Faustino Nebreda died


in 1945 much prior to the effectivity of the new Civil Code. With
this background, it is evident that when Faustino Nebreda died
in 1945 the five parcels of land he was seized of at the time
passed from the moment of his death to his only heir, his
widow Maria Uson (Article 657, old Civil Code).As this Court
aptly said, "The property belongs to the heirs at the moment of
the death of the ancestor as completely as if the ancestor had
executed and delivered to them a deed for the same before his
death" (Ilustre vs. Alaras Frondosa, 17 Phil., 321). From that
moment, therefore, the rights of inheritance of Maria Uson over
the lands in question became vested.
The claim of the defendants that Maria Uson had relinquished
her right over the lands in question because she expressly
renounced to inherit any future property that her husband may
acquire and leave upon his death in the deed of separation
they had entered into on February 21, 1931, cannot be
entertained for the simple reason that future inheritance cannot
be the subject of a contract nor can it be renounced (1
Manresa, 123, sixth edition; Tolentino on Civil Code, p. 12;
Osorio vs. Osorio and Ynchausti Steamship Co., 41 Phil., 531).
But defendants contend that, while it is true that the four minor
defendants are illegitimate children of the late Faustino
Nebreda and under the old Civil Code are not entitled to any
successional rights, however, under the new Civil Code which
became in force in June, 1950, they are given the status and
rights of natural children and are entitled to the successional
rights which the law accords to the latter (article 2264 and
article 287, new Civil Code), and because these successional
rights were declared for the first time in the new code, they
shall be given retroactive effect even though the event which
gave rise to them may have occurred under the prior legislation
(Article 2253, new Civil Code).
There is no merit in this claim. Article 2253 above referred to
provides indeed that rights which are declared for the first time
shall have retroactive effect even though the event which gave
rise to them may have occurred under the former legislation,
but this is so only when the new rights do not prejudice any
vested or acquired right of the same origin. Thus, said article
provides that "if a right should be declared for the first time in
this Code, it shall be effective at once, even though the act or
event which gives rise thereto may have been done or may
have occurred under the prior legislation, provided said new
right does not prejudice or impair any vested or acquired right,
of the same origin." As already stated in the early part of this
decision, the right of ownership of Maria Uson over the lands in
question became vested in 1945 upon the death of her late
husband and this is so because of the imperative provision of
the law which commands that the rights to succession are
transmitted from the moment of death (Article 657, old Civil
Code). The new right recognized by the new Civil Code in
favor of the illegitimate children of the deceased cannot,
therefore, be asserted to the impairment of the vested right of
Maria Uson over the lands in dispute.

2
As regards the claim that Maria Uson, while her deceased
husband was lying in state, in a gesture of pity or compassion,
agreed to assign the lands in question to the minor children for
the reason that they were acquired while the deceased was
living with their mother and Maria Uson wanted to assuage
somewhat the wrong she has done to them, this much can be
said; apart from the fact that this claim is disputed, we are of
the opinion that said assignment, if any, partakes of the nature
of a donation of real property, inasmuch as it involves no
material consideration, and in order that it may be valid it shall
be made in a public document and must be accepted either in
the same document or in a separate one (Article 633, old Civil
Code). Inasmuch as this essential formality has not been
followed, it results that the alleged assignment or donation has
no valid effect.
WHEREFORE, the decision appealed from is affirmed, without
costs.
Paras, C.J., Pablo, Bengzon, Padilla, Tuason, Montemayor,
Reyes, Jugo and Labrador, JJ., concur.

Republic of the Philippines


SUPREME COURT
Manila
EN BANC
G.R. No. L-44837

November 23, 1938

SOCORRO LEDESMA and ANA QUITCO LEDESMA,


plaintiffs-appellees,
vs.
CONCHITA MCLACHLIN, ET AL., defendants-appellants.
Adriano T. de la Cruz for appellants.
Simeon Bitanga for appellees.
VILLA-REAL, J.:
This case is before us by virtue of an appeal taken by
the defendants Conchita McLachlin, Lorenzo Quitco, Jr.,
Sabina Quitco, Rafael Quitco and Marcela Quitco, from the
decision of the Court of First Instance of Occidental Negros,
the dispositive part of which reads:
For the foregoing considerations, the court renders
judgment in this case declaring Ana Quitco Ledesma an
acknowledged natural daughter of the deceased Lorenzo M.
Quitco, for legal purposes, but absolving the defendants as to
the prayer in the first cause of action that the said Ana Quitco
Ledesma be declared entitled to share in the properties left by
the deceased Eusebio Quitco.
As to the second cause of action, the said defendants
are ordered to pay to the plaintiff Socorro Ledesma, jointly and
severally, only the sum of one thousand five hundred

pesos(P1,500), with legal interest thereon from the filing of this


complaint until fully paid. No pronouncement is made as to the
costs. So ordered.
In support of their appeal, the appellants assign the
following errors allegedly committed by the trial court in its
aforesaid decision:
1. That the trial court erred in holding, that the action for the
recovery of the sum of P1,500, representing the last
installment of the note Exhibit C has not yet prescribed.
2. That the trial court erred in holding that the property
inherited by the defendants from their deceased grandfather by
the right of representation is subject to the debts and
obligations of their deceased father who died without any
property whatsoever.lawphi1.net
3. That the trial court erred in condemning the defendants to
pay jointly and severally the plaintiff Socorro Ledesma the sum
of P1,500.
The only facts to be considered in the determination of
the legal questions raised in this appeal are those set out in the
appealed decision, which have been established at the trial,
namely:
In the year 1916, the plaintiff Socorro Ledesma lived
maritally with Lorenzo M. Quitco, while the latter was still
single, of which relation, lasting until the year 1921, was born a
daughter who is the other plaintiff Ana Quitco Ledesma. In
1921, it seems hat the relation between Socorro Ledesma and
Lorenzo M. Quitco came to an end, but the latter executed a
deed (Exhibit A), acknowledging the plaintiff Ana Quitco
Ledesma as his natural daughter and on January 21, 1922, he
issued in favor of the plaintiff Socorro Ledesma a promissory
note (Exhibit C), of the following tenor:
P2,000. For value received I promise to pay Miss
Socorro Ledesma the sum of two thousand pesos (P2,000).
Philippine currency under the following terms: Two hundred
and fifty pesos (P250) to be paid on the first day of March
1922; another two hundred and fifty pesos (P250)to be paid on
the first day of
November 1922; the remaining one
thousand and five hundred (P1,500) to be paid two years from
the date of the execution of this note. San Enrique, Occ.
Negros, P. I., Jan. 21, 1922.
Subsequently, Lorenzo M. Quitco married the defendant
Conchita McLachlin, with whom he had four children, who are
the other defendants. On March 9, 1930, Lorenzo M. Quitco
died (Exhibit 5), and, still later, that is, on December 15, 1932,
his father Eusebio Quitco also died, and as the latter left real
and personal properties upon his death, administration
proceedings of said properties were instituted in this court, the
said case being known as the "Intestate of the deceased
Eusebio Quitco," civil case No. 6153 of this court.

3
Upon the institution of the intestate of the deceased
Eusebio Quitco and the appointment of the committee on
claims and appraisal, the plaintiff Socorro Ledesma, on August
26, 1935, filed before said committee the aforequoted
promissory note for payment, and the commissioners, upon
receipt of said promissory note, instead of passing upon it,
elevated the same to this court en consulta (Exhibit F), and as
the Honorable Jose Lopez Vito, presiding over the First
Branch, returned said consulta and refrained from giving his
opinion thereon (Exhibit C), the aforesaid commissioners on
claims and appraisal, alleging lack of jurisdiction to pass upon
the claim, denied he same (Exhibit H).
On
November 14, 1933 (Exhibit I), the court
issued an order of declaration of heirs in the intestate of the
deceased Eusebio Quitco, and as Ana Quitco Ledesma was
not included among the declared heirs, Socorro Ledesma, as
mother of Ana Quitco Ledesma, asked for the reconsideration
of said order, a petition which the court denied. From the order
denying the said petition no appeal was taken, and in lieu
thereof there was filed the complaint which gives rise to this
case.
The first question to be decided in this appeal, raised in
the first assignment of alleged error, is whether or not the
action to recover the sum of P1,500, representing the last
installment for the payment of the promissory note Exhibit C,
has prescribed.
According to the promissory note Exhibit C, executed by
the deceased Lorenzo M. Quitco, on January 21, 1922, the last
installment of P1,500 should be paid two years from the date of
the execution of said promissory note, that is, on January 21,
1924. The complaint in the present case was filed on June 26,
1934, that is, more than ten years after he expiration of the
said period. The fact that the plaintiff Socorro Ledesma filed
her claim, on August 26, 1933, with the committee on claims
and appraisal appointed in the intestate of Eusebio Quitco,
does not suspend the running of the prescriptive period of the
judicial action for the recovery of said debt, because the claim
for the unpaid balance of the amount of the promissory note
should no have been presented in the intestate of Eusebio
Quitco, the said deceased not being the one who executed the
same, but in the intestate of Lorenzo M. Quitco, which should
have been instituted by the said Socorro Ledesma as provided
in section 642 of the Code of Civil Procedure, authorizing a
creditor to institute said case through the appointment of an
administrator for the purpose of collecting his credit. More than
ten years having thus elapsed from the expiration of the period
for the payment of said debt of P1,500, the action for its
recovery has prescribed under section 43, No. 1, of the Code
of Civil Procedure.
The first assignment of alleged error is, therefore, wellfounded.
As to the second assignment of alleged error, consisting
in that the trial court erred in holding that the properties

inherited by the defendants from their deceased grandfather by


representation are subject to the payment of debts and
obligations of their deceased father, who died without leaving
any property, while it is true that under the provisions of articles
924 to 927 of the Civil Code, a children presents his father or
mother who died before him in the properties of his grandfather
or grandmother, this right of representation does not make the
said child answerable for the obligations contracted by his
deceased father or mother, because, as may be seen from the
provisions of the Code of Civil Procedure referring to partition
of inheritances, the inheritance is received with the benefit of
inventory, that is to say, the heirs only answer with the
properties received from their predecessor. The herein
defendants, as heirs of Eusebio Quitco, in representation of
their father Lorenzo M. Quitco, are not bound to pay the
indebtedness of their said father from whom they did not inherit
anything.
The second assignment of alleged error is also wellfounded.
Being a mere sequel of the first two assignments of
alleged errors, the third assignment of error is also wellfounded.
For the foregoing considerations, we are of the opinion
and so hold: (1) That the filing of a claim before the committee
on claims and appraisal, appointed in the intestate of the
father, for a monetary obligation contracted by a son who died
before him, does not suspend the prescriptive period of the
judicial action for the recovery of said indebtedness; (2) that
the claim for the payment of an indebtedness contracted by a
deceased person cannot be filed for its collection before the
committee on claims and appraisal, appointed in the intestate
of his father, and the propertiesinherited from the latter by the
children of said deceased do not answer for the payment of the
indebtedness contracted during the lifetime of said person.
Wherefore, the appealed judgment is reversed, and the
defendants are absolved from the complaint, with the costs to
the appellees. So ordered.
Avancea, C.J., Imperial, Diaz, Laurel and Concepcion, JJ.,
concur.

THIRD DIVISION
[G.R. No. 113725. June 29, 2000]
JOHNNY S. RABADILLA, petitioner, vs. COURT OF APPEALS
AND MARIA MARLENA COSCOLUELLA Y BELLEZA
VILLACARLOS, respondents.
DECISION
PURISIMA, J.:

4
This is a petition for review of the decision of the Court of
Appeals, dated December 23, 1993, in CA-G.R. No. CV-35555,
which set aside the decision of Branch 52 of the Regional Trial
Court in Bacolod City, and ordered the defendants-appellees
(including herein petitioner), as heirs of Dr. Jorge Rabadilla, to
reconvey title over Lot No. 1392, together with its fruits and
interests, to the estate of Aleja Belleza.
The antecedent facts are as follows:
In a Codicil appended to the Last Will and Testament of
testatrix Aleja Belleza, Dr. Jorge Rabadilla, predecessor-ininterest of the herein petitioner, Johnny S. Rabadilla, was
instituted as a devisee of 511, 855 square meters of that parcel
of land surveyed as Lot No. 1392 of the Bacolod Cadastre. The
said Codicil, which was duly probated and admitted in Special
Proceedings No. 4046 before the then Court of First Instance
of Negros Occidental, contained the following provisions:
"FIRST
I give, leave and bequeath the following property owned by me
to Dr. Jorge Rabadilla resident of 141 P. Villanueva, Pasay
City:
(a) Lot No. 1392 of the Bacolod Cadastre, covered by Transfer
Certificate of Title No. RT-4002 (10942), which is registered in
my name according to the records of the Register of Deeds of
Negros Occidental.
(b) That should Jorge Rabadilla die ahead of me, the
aforementioned property and the rights which I shall set forth
hereinbelow, shall be inherited and acknowledged by the
children and spouse of Jorge Rabadilla.
xxx
FOURTH
(a)....It is also my command, in this my addition (Codicil), that
should I die and Jorge Rabadilla shall have already received
the ownership of the said Lot No. 1392 of the Bacolod
Cadastre, covered by Transfer Certificate of Title No. RT-4002
(10942), and also at the time that the lease of Balbinito G.
Guanzon of the said lot shall expire, Jorge Rabadilla shall have
the obligation until he dies, every year to give to Maria Marlina
Coscolluela y Belleza, Seventy (75) (sic) piculs of Export sugar
and Twenty Five (25) piculs of Domestic sugar, until the said
Maria Marlina Coscolluela y Belleza dies.
FIFTH
(a) Should Jorge Rabadilla die, his heir to whom he shall give
Lot No. 1392 of the Bacolod Cadastre, covered by Transfer
Certificate of Title No. RT-4002 (10492), shall have the
obligation to still give yearly, the sugar as specified in the
Fourth paragraph of his testament, to Maria Marlina
Coscolluela y Belleza on the month of December of each year.

SIXTH
I command, in this my addition (Codicil) that the Lot No. 1392,
in the event that the one to whom I have left and bequeathed,
and his heir shall later sell, lease, mortgage this said Lot, the
buyer, lessee, mortgagee, shall have also the obligation to
respect and deliver yearly ONE HUNDRED (100) piculs of
sugar to Maria Marlina Coscolluela y Belleza, on each month
of December, SEVENTY FIVE (75) piculs of Export and
TWENTY FIVE (25) piculs of Domestic, until Maria Marlina
shall die, lastly should the buyer, lessee or the mortgagee of
this lot, not have respected my command in this my addition
(Codicil), Maria Marlina Coscolluela y Belleza, shall
immediately seize this Lot No. 1392 from my heir and the
latter's heirs, and shall turn it over to my near desendants, (sic)
and the latter shall then have the obligation to give the ONE
HUNDRED (100) piculs of sugar until Maria Marlina shall die. I
further command in this my addition (Codicil) that my heir and
his heirs of this Lot No. 1392, that they will obey and follow that
should they decide to sell, lease, mortgage, they cannot
negotiate with others than my near descendants and my
sister."
Pursuant to the same Codicil, Lot No. 1392 was transferred to
the deceased, Dr. Jorge Rabadilla, and Transfer Certificate of
Title No. 44498 thereto issued in his name.
Dr. Jorge Rabadilla died in 1983 and was survived by his wife
Rufina and children Johnny (petitioner), Aurora, Ofelia and
Zenaida, all surnamed Rabadilla.
On August 21, 1989, Maria Marlena Coscolluela y Belleza
Villacarlos brought a complaint, docketed as Civil Case No.
5588, before Branch 52 of the Regional Trial Court in Bacolod
City, against the above-mentioned heirs of Dr. Jorge Rabadilla,
to enforce the provisions of subject Codicil. The Complaint
alleged that the defendant-heirs violated the conditions of the
Codicil, in that:
1. Lot No. 1392 was mortgaged to the Philippine National Bank
and the Republic Planters Bank in disregard of the testatrix's
specific instruction to sell, lease, or mortgage only to the near
descendants and sister of the testatrix.
2. Defendant-heirs failed to comply with their obligation to
deliver one hundred (100) piculs of sugar (75 piculs export
sugar and 25 piculs domestic sugar) to plaintiff Maria Marlena
Coscolluela y Belleza from sugar crop years 1985 up to the
filing of the complaint as mandated by the Codicil, despite
repeated demands for compliance.
3. The banks failed to comply with the 6th paragraph of the
Codicil which provided that in case of the sale, lease, or
mortgage of the property, the buyer, lessee, or mortgagee shall
likewise have the obligation to deliver 100 piculs of sugar per
crop year to herein private respondent.
The plaintiff then prayed that judgment be rendered ordering
defendant-heirs to reconvey/return-Lot No. 1392 to the

5
surviving heirs of the late Aleja Belleza, the cancellation of TCT
No. 44498 in the name of the deceased, Dr. Jorge Rabadilla,
and the issuance of a new certificate of title in the names of the
surviving heirs of the late Aleja Belleza.

However, there was no compliance with the aforesaid


Memorandum of Agreement except for a partial delivery of
50.80 piculs of sugar corresponding to sugar crop year 1988
-1989.

On February 26, 1990, the defendant-heirs were declared in


default but on March 28, 1990 the Order of Default was lifted,
with respect to defendant Johnny S. Rabadilla, who filed his
Answer, accordingly.

On July 22, 1991, the Regional Trial Court came out with a
decision, dismissing the complaint and disposing as follows:

During the pre-trial, the parties admitted that:


On November 15, 1998, the plaintiff (private respondent) and a
certain Alan Azurin, son-in-law of the herein petitioner who was
lessee of the property and acting as attorney-in-fact of
defendant-heirs, arrived at an amicable settlement and entered
into a Memorandum of Agreement on the obligation to deliver
one hundred piculs of sugar, to the following effect:
"That for crop year 1988-89, the annuity mentioned in Entry
No. 49074 of TCT No. 44489 will be delivered not later than
January of 1989, more specifically, to wit:
75 piculs of 'A' sugar, and 25 piculs of 'B' sugar, or then
existing in any of our names, Mary Rose Rabadilla y Azurin or
Alan Azurin, during December of each sugar crop year, in
Azucar Sugar Central; and, this is considered compliance of
the annuity as mentioned, and in the same manner will
compliance of the annuity be in the next succeeding crop
years.
That the annuity above stated for crop year 1985-86, 1986-87,
and 1987-88, will be complied in cash equivalent of the number
of piculs as mentioned therein and which is as herein agreed
upon, taking into consideration the composite price of sugar
during each sugar crop year, which is in the total amount of
ONE HUNDRED FIVE THOUSAND PESOS (P105,000.00).
That the above-mentioned amount will be paid or delivered on
a staggered cash installment, payable on or before the end of
December of every sugar crop year, to wit:
For 1985-86, TWENTY SIX THOUSAND TWO HUNDRED
FIFTY (P26,250.00) Pesos, payable on or before December of
crop year 1988-89;
For 1986-87, TWENTY SIX THOUSAND TWO HUNDRED
FIFTY (P26,250.00) Pesos, payable on or before December of
crop year 1989-90;
For 1987-88, TWENTY SIX THOUSAND TWO HUNDRED
FIFTY (P26,250.00) Pesos, payable on or before December of
crop year 1990-91; and
For 1988-89, TWENTY SIX THOUSAND TWO HUNDRED
FIFTY (P26,250.00) Pesos, payable on or before December of
crop year 1991-92."

"WHEREFORE, in the light of the aforegoing findings, the


Court finds that the action is prematurely filed as no cause of
action against the defendants has as yet arose in favor of
plaintiff. While there maybe the non-performance of the
command as mandated exaction from them simply because
they are the children of Jorge Rabadilla, the title holder/owner
of the lot in question, does not warrant the filing of the present
complaint. The remedy at bar must fall. Incidentally, being in
the category as creditor of the left estate, it is opined that
plaintiff may initiate the intestate proceedings, if only to
establish the heirs of Jorge Rabadilla and in order to give full
meaning and semblance to her claim under the Codicil.
In the light of the aforegoing findings, the Complaint being
prematurely filed is DISMISSED without prejudice.
SO ORDERED."
On appeal by plaintiff, the First Division of the Court of Appeals
reversed the decision of the trial court; ratiocinating and
ordering thus:
"Therefore, the evidence on record having established plaintiffappellant's right to receive 100 piculs of sugar annually out of
the produce of Lot No. 1392; defendants-appellee's obligation
under Aleja Belleza's codicil, as heirs of the modal heir, Jorge
Rabadilla, to deliver such amount of sugar to plaintiff-appellant;
defendants-appellee's admitted non-compliance with said
obligation since 1985; and, the punitive consequences
enjoined by both the codicil and the Civil Code, of seizure of
Lot No. 1392 and its reversion to the estate of Aleja Belleza in
case of such non-compliance, this Court deems it proper to
order the reconveyance of title over Lot No. 1392 from the
estates of Jorge Rabadilla to the estate of Aleja Belleza.
However, plaintiff-appellant must institute separate
proceedings to re-open Aleja Belleza's estate, secure the
appointment of an administrator, and distribute Lot No. 1392 to
Aleja Belleza's legal heirs in order to enforce her right,
reserved to her by the codicil, to receive her legacy of 100
piculs of sugar per year out of the produce of Lot No. 1392 until
she dies.
Accordingly, the decision appealed from is SET ASIDE and
another one entered ordering defendants-appellees, as heirs of
Jorge Rabadilla, to reconvey title over Lot No. 1392, together
with its fruits and interests, to the estate of Aleja Belleza.
SO ORDERED."
Dissatisfied with the aforesaid disposition by the Court of
Appeals, petitioner found his way to this Court via the present

6
petition, contending that the Court of Appeals erred in ordering
the reversion of Lot 1392 to the estate of the testatrix Aleja
Belleza on the basis of paragraph 6 of the Codicil, and in ruling
that the testamentary institution of Dr. Jorge Rabadilla is a
modal institution within the purview of Article 882 of the New
Civil Code.
The petition is not impressed with merit.
Petitioner contends that the Court of Appeals erred in resolving
the appeal in accordance with Article 882 of the New Civil
Code on modal institutions and in deviating from the sole issue
raised which is the absence or prematurity of the cause of
action. Petitioner maintains that Article 882 does not find
application as there was no modal institution and the testatrix
intended a mere simple substitution - i.e. the instituted heir, Dr.
Jorge Rabadilla, was to be substituted by the testatrix's "near
descendants" should the obligation to deliver the fruits to
herein private respondent be not complied with. And since the
testatrix died single and without issue, there can be no valid
substitution and such testamentary provision cannot be given
any effect.
The petitioner theorizes further that there can be no valid
substitution for the reason that the substituted heirs are not
definite, as the substituted heirs are merely referred to as "near
descendants" without a definite identity or reference as to who
are the "near descendants" and therefore, under Articles 843
and 845 of the New Civil Code, the substitution should be
deemed as not written.
The contentions of petitioner are untenable. Contrary to his
supposition that the Court of Appeals deviated from the issue
posed before it, which was the propriety of the dismissal of the
complaint on the ground of prematurity of cause of action,
there was no such deviation. The Court of Appeals found that
the private respondent had a cause of action against the
petitioner. The disquisition made on modal institution was,
precisely, to stress that the private respondent had a legally
demandable right against the petitioner pursuant to subject
Codicil; on which issue the Court of Appeals ruled in
accordance with law.
It is a general rule under the law on succession that
successional rights are transmitted from the moment of death
of the decedent and compulsory heirs are called to succeed by
operation of law. The legitimate children and descendants, in
relation to their legitimate parents, and the widow or widower,
are compulsory heirs. Thus, the petitioner, his mother and
sisters, as compulsory heirs of the instituted heir, Dr. Jorge
Rabadilla, succeeded the latter by operation of law, without
need of further proceedings, and the successional rights were
transmitted to them from the moment of death of the decedent,
Dr. Jorge Rabadilla.
Under Article 776 of the New Civil Code, inheritance includes
all the property, rights and obligations of a person, not
extinguished by his death. Conformably, whatever rights Dr.
Jorge Rabadilla had by virtue of subject Codicil were

transmitted to his forced heirs, at the time of his death. And


since obligations not extinguished by death also form part of
the estate of the decedent; corollarily, the obligations imposed
by the Codicil on the deceased Dr. Jorge Rabadilla, were
likewise transmitted to his compulsory heirs upon his death.
In the said Codicil, testatrix Aleja Belleza devised Lot No. 1392
to Dr. Jorge Rabadilla, subject to the condition that the usufruct
thereof would be delivered to the herein private respondent
every year. Upon the death of Dr. Jorge Rabadilla, his
compulsory heirs succeeded to his rights and title over the said
property, and they also assumed his (decedent's) obligation to
deliver the fruits of the lot involved to herein private
respondent. Such obligation of the instituted heir reciprocally
corresponds to the right of private respondent over the
usufruct, the fulfillment or performance of which is now being
demanded by the latter through the institution of the case at
bar. Therefore, private respondent has a cause of action
against petitioner and the trial court erred in dismissing the
complaint below.
Petitioner also theorizes that Article 882 of the New Civil Code
on modal institutions is not applicable because what the
testatrix intended was a substitution - Dr. Jorge Rabadilla was
to be substituted by the testatrix's near descendants should
there be noncompliance with the obligation to deliver the piculs
of sugar to private respondent.
Again, the contention is without merit.
Substitution is the designation by the testator of a person or
persons to take the place of the heir or heirs first instituted.
Under substitutions in general, the testator may either (1)
provide for the designation of another heir to whom the
property shall pass in case the original heir should die before
him/her, renounce the inheritance or be incapacitated to inherit,
as in a simple substitution, or (2) leave his/her property to one
person with the express charge that it be transmitted
subsequently to another or others, as in a fideicommissary
substitution. The Codicil sued upon contemplates neither of the
two.
In simple substitutions, the second heir takes the inheritance in
default of the first heir by reason of incapacity, predecease or
renunciation. In the case under consideration, the provisions of
subject Codicil do not provide that should Dr. Jorge Rabadilla
default due to predecease, incapacity or renunciation, the
testatrix's near descendants would substitute him. What the
Codicil provides is that, should Dr. Jorge Rabadilla or his heirs
not fulfill the conditions imposed in the Codicil, the property
referred to shall be seized and turned over to the testatrix's
near descendants.
Neither is there a fideicommissary substitution here and on this
point, petitioner is correct. In a fideicommissary substitution,
the first heir is strictly mandated to preserve the property and
to transmit the same later to the second heir. In the case under
consideration, the instituted heir is in fact allowed under the
Codicil to alienate the property provided the negotiation is with

7
the near descendants or the sister of the testatrix. Thus, a very
important element of a fideicommissary substitution is lacking;
the obligation clearly imposing upon the first heir the
preservation of the property and its transmission to the second
heir. "Without this obligation to preserve clearly imposed by the
testator in his will, there is no fideicommissary substitution."
Also, the near descendants' right to inherit from the testatrix is
not definite. The property will only pass to them should Dr.
Jorge Rabadilla or his heirs not fulfill the obligation to deliver
part of the usufruct to private respondent.
Another important element of a fideicommissary substitution is
also missing here. Under Article 863, the second heir or the
fideicommissary to whom the property is transmitted must not
be beyond one degree from the first heir or the fiduciary. A
fideicommissary substitution is therefore, void if the first heir is
not related by first degree to the second heir. In the case under
scrutiny, the near descendants are not at all related to the
instituted heir, Dr. Jorge Rabadilla.
The Court of Appeals erred not in ruling that the institution of
Dr. Jorge Rabadilla under subject Codicil is in the nature of a
modal institution and therefore, Article 882 of the New Civil
Code is the provision of law in point. Articles 882 and 883 of
the New Civil Code provide:
Art. 882. The statement of the object of the institution or the
application of the property left by the testator, or the charge
imposed on him, shall not be considered as a condition unless
it appears that such was his intention.
That which has been left in this manner may be claimed at
once provided that the instituted heir or his heirs give security
for compliance with the wishes of the testator and for the return
of anything he or they may receive, together with its fruits and
interests, if he or they should disregard this obligation.
Art. 883. When without the fault of the heir, an institution
referred to in the preceding article cannot take effect in the
exact manner stated by the testator, it shall be complied with in
a manner most analogous to and in conformity with his wishes.
The institution of an heir in the manner prescribed in Article
882 is what is known in the law of succession as an institucion
sub modo or a modal institution. In a modal institution, the
testator states (1) the object of the institution, (2) the purpose
or application of the property left by the testator, or (3) the
charge imposed by the testator upon the heir. A "mode"
imposes an obligation upon the heir or legatee but it does not
affect the efficacy of his rights to the succession. On the other
hand, in a conditional testamentary disposition, the condition
must happen or be fulfilled in order for the heir to be entitled to
succeed the testator. The condition suspends but does not
obligate; and the mode obligates but does not suspend. To
some extent, it is similar to a resolutory condition.
From the provisions of the Codicil litigated upon, it can be
gleaned unerringly that the testatrix intended that subject
property be inherited by Dr. Jorge Rabadilla. It is likewise

clearly worded that the testatrix imposed an obligation on the


said instituted heir and his successors-in-interest to deliver one
hundred piculs of sugar to the herein private respondent,
Marlena Coscolluela Belleza, during the lifetime of the latter.
However, the testatrix did not make Dr. Jorge Rabadilla's
inheritance and the effectivity of his institution as a devisee,
dependent on the performance of the said obligation. It is clear,
though, that should the obligation be not complied with, the
property shall be turned over to the testatrix's near
descendants. The manner of institution of Dr. Jorge Rabadilla
under subject Codicil is evidently modal in nature because it
imposes a charge upon the instituted heir without, however,
affecting the efficacy of such institution.
Then too, since testamentary dispositions are generally acts of
liberality, an obligation imposed upon the heir should not be
considered a condition unless it clearly appears from the Will
itself that such was the intention of the testator. In case of
doubt, the institution should be considered as modal and not
conditional.
Neither is there tenability in the other contention of petitioner
that the private respondent has only a right of usufruct but not
the right to seize the property itself from the instituted heir
because the right to seize was expressly limited to violations
by the buyer, lessee or mortgagee.
In the interpretation of Wills, when an uncertainty arises on the
face of the Will, as to the application of any of its provisions,
the testator's intention is to be ascertained from the words of
the Will, taking into consideration the circumstances under
which it was made. Such construction as will sustain and
uphold the Will in all its parts must be adopted.
Subject Codicil provides that the instituted heir is under
obligation to deliver One Hundred (100) piculs of sugar yearly
to Marlena Belleza Coscuella. Such obligation is imposed on
the instituted heir, Dr. Jorge Rabadilla, his heirs, and their
buyer, lessee, or mortgagee should they sell, lease, mortgage
or otherwise negotiate the property involved. The Codicil
further provides that in the event that the obligation to deliver
the sugar is not respected, Marlena Belleza Coscuella shall
seize the property and turn it over to the testatrix's near
descendants. The non-performance of the said obligation is
thus with the sanction of seizure of the property and reversion
thereof to the testatrix's near descendants. Since the said
obligation is clearly imposed by the testatrix, not only on the
instituted heir but also on his successors-in-interest, the
sanction imposed by the testatrix in case of non-fulfillment of
said obligation should equally apply to the instituted heir and
his successors-in-interest.
Similarly unsustainable is petitioner's submission that by virtue
of the amicable settlement, the said obligation imposed by the
Codicil has been assumed by the lessee, and whatever
obligation petitioner had become the obligation of the lessee;
that petitioner is deemed to have made a substantial and
constructive compliance of his obligation through the

8
consummated settlement between the lessee and the private
respondent, and having consummated a settlement with the
petitioner, the recourse of the private respondent is the
fulfillment of the obligation under the amicable settlement and
not the seizure of subject property.
Suffice it to state that a Will is a personal, solemn, revocable
and free act by which a person disposes of his property, to take
effect after his death.1[25] Since the Will expresses the manner
in which a person intends how his properties be disposed, the
wishes and desires of the testator must be strictly followed.
Thus, a Will cannot be the subject of a compromise agreement
which would thereby defeat the very purpose of making a Will.
WHEREFORE, the petition is hereby DISMISSED and the
decision of the Court of Appeals, dated December 23, 1993, in
CA-G.R. No. CV-35555 AFFIRMED. No pronouncement as to
costs
SO ORDERED.
Melo, J., (Chairman), concur in the separate opinion of Justice
Vitug.

Del Castillo, J.:


The basic questions to be resolved in this case are: Is a
waiver of hereditary rights in favor of another executed by a
future heir while the parents are still living valid? Is an adverse
claim annotated on the title of a property on the basis of such
waiver likewise valid and effective as to bind the subsequent
owners and hold them liable to the claimant?
This Petition for Review on Certiorari under Rule 45 of the
Rules of Court assails the December 12, 2003 Decision of the
Court of Appeals (CA) in CA-G.R. CV No. 70888. Said
Decision modified the June 14, 2001 Summary Judgment of
the Regional Trial Court (RTC) of Quezon City in Civil Case
No. Q-99-38876 by holding respondents Spouses Bienvenido
and Elizabeth Pangan (the Pangans) not solidarily liable with
the other respondents, Spouses Alfredo and Imelda Diaz (the
Diazes) and Reina Comandante (Comandante), to petitioner
Atty. Pedro M. Ferrer (Atty. Ferrer). Likewise assailed is the
CA Resolution dated September 10, 2004 which denied
petitioners as well as respondents Spouses Diaz and
Comandantes respective motions for reconsideration.

Vitug, J., see separate opinion.

The parties respective versions of the factual antecedents are


as follows:

Panganiban, J., join the separate opinion of Justice Vitug.

Version of the Petitioner

Gonzaga-Reyes, J., no part.

Petitioner Atty. Ferrer claimed in his original Complaint that on


May 7, 1999, the Diazes, as represented by their daughter
Comandante, through a Special Power of Attorney (SPA),
obtained from him a loan of P1,118,228.00. The loan was
secured by a Real Estate Mortgage Contract2[8] by way of
second mortgage over Transfer Certificate of Title (TCT) No.
RT-66043[9] and a Promissory Note payable within six months
or up to November 7, 1999. Comandante also issued to
petitioner postdated checks to secure payment of said loan.

Republic of the Philippines


Supreme Court
Baguio City
SECOND DIVISION
ATTY. PEDRO M.
FERRER,
Petitioner,

G.R. No. 165300

Present:
- versus -

SPOUSES ALFREDO DIAZ


and IMELDA DIAZ,
REINA COMANDANTE and
SPOUSES BIENVENIDO

CARPIO, J.,
Chairperson,
BRION,
ABAD,
DEL CASTILLO, and
PEREZ, JJ.

Petitioner further claimed that prior to this or on May 29, 1998,


Comandante, for a valuable consideration of P600,000.00,
which amount formed part of the abovementioned secured
loan, executed in his favor an instrument entitled Waiver of
Hereditary Rights and Interests Over a Real Property (Still
Undivided), the pertinent portions of which read:

x - - - - - -- - - - - - - - - - - - - - - - - - - - - - - - - - - - - - - - - - - - - x

I, REINA D. COMANDANTE, of legal age, Filipino, married,


with residence and postal address at No. 6, Road 20, Project
8, Quezon City, Metro Manila, Philippines, for a valuable
consideration of SIX HUNDRED THOUSAND PESOS
(P600,000.00) which constitutes my legal obligation/loan to
Pedro M. Ferrer, likewise of legal age, Filipino, married to
Erlinda B. Ferrer, with residence and postal address at No. 9,
Lot 4, Puerto Rico Street, Loyola Grand Villas, Quezon City,
Metro Manila, Philippines, by virtue of these presents, do
hereby WAIVE, and/or REPUDIATE all my hereditary rights

DECISION

PANGAN and ELIZABETH


PANGAN,
Respondents.

Promulgated:
April 23, 2010

9
and interests as a legitimate heir/daughter of Sps. Alfredo T.
Diaz and Imelda G. Diaz in favor of said Pedro M. Ferrer, his
heirs and assigns over a certain parcel of land together with all
the improvements found thereon and which property is more
particularly described as follows:
TRANSFER CERTIFICATE OF TITLE
NO. RT-6604 (82020) PR-18887
xxxx
and which property is titled and registered in the name of my
parents Alfredo T. Diaz and Imelda G. Diaz, as evidenced by
Transfer Certificate of Title No. RT 6604 (82020) PR-18887.

(sgd.
REINA D. COMANDANTE

Affiant
On the basis of said waiver, petitioner executed an Affidavit of
Adverse Claim4[12] which he caused to be annotated at the
back of TCT No. RT-6604 on May 26, 1999.
The Diazes, however, reneged on their obligation as the
checks issued by Comandante were dishonored upon
presentment. Despite repeated demands, said respondents
still failed and refused to settle the loan. Thus, petitioner filed
on September 29, 1999 a Complaint for Collection of Sum of
Money Secured by Real Estate Mortgage Contract against the
Diazes and Comandante docketed as Civil Case No. Q-9938876 and raffled to Branch 224 of RTC, Quezon City.
Petitioner twice amended his complaint. First, by including as
an alternative relief the Judicial Foreclosure of Mortgage and,
second, by impleading as additional defendants the Pangans
as the mortgaged property covered by TCT No. RT-6604 was
already transferred under their names in TCT No. N-209049.
Petitioner prayed in his second amended complaint that all the
respondents be ordered to jointly and solidarily pay him the
sum of P1,118,228.00, exclusive of interests, and/or for the
judicial foreclosure of the property pursuant to the Real Estate
Mortgage Contract.
Version of the Respondents
In her Answer to petitioners original complaint, Comandante
alleged that petitioner and his wife were her fellow members in
the Couples for Christ Movement. Sometime in 1998, she
sought the help of petitioner with regard to the mortgage with a
bank of her parents lot located at No. 6, Rd. 20, Project 8,
Quezon City and covered by TCT No. RT-6604. She also
sought financial accommodations from the couple on several
occasions which totaled P500,000.00. Comandante, however,

claimed that these loans were secured by chattel mortgages


over her taxi units in addition to several postdated checks she
issued in favor of petitioner.
As she could not practically comply with her obligation,
petitioner and his wife, presented to Comandante sometime in
May 1998 a document denominated as Waiver of Hereditary
Rights and Interests Over a Real Property (Still Undivided)
pertaining to a waiver of her hereditary share over her parents
abovementioned property. Purportedly, the execution of said
waiver was to secure Comandantes loan with the couple
which at that time had already ballooned to P600,000.00 due
to interests.
A year later, the couple again required Comandante to sign the
following documents: (1) a Real Estate Mortgage Contract over
her parents property; and, (2) an undated Promissory Note,
both corresponding to the amount of P1,118,228.00, which
petitioner claimed to be the total amount of Comandantes
monetary obligation to him exclusive of charges and interests.
Comandante alleged that she reminded petitioner that she was
not the registered owner of the subject property and that
although her parents granted her SPA, same only pertains to
her authority to mortgage the property to banks and other
financial institutions and not to individuals. Petitioner
nonetheless assured Comandante that the SPA was also
applicable to their transaction. As Comandante was still
hesitant, petitioner and his wife threatened to foreclose the
formers taxi units and present the postdated checks she
issued to the bank for payment. For fear of losing her taxi units
which were the only source of her livelihood, Comandante was
thus constrained to sign the mortgage agreement as well as
the promissory note. Petitioner, however, did not furnish her
with copies of said documents on the pretext that they still
have to be notarized, but, as can be gleaned from the records,
the documents were never notarized. Moreover, Comandante
claimed that the SPA alluded to by petitioner in his complaint
was not the same SPA under which she thought she derived
the authority to execute the mortgage contract.
Comandante likewise alleged that on September 29, 1999 at
10:00 o clock in the morning, she executed an Affidavit of
Repudiation/Revocation of Waiver of Hereditary Rights and
Interests Over A (Still Undivided) Real Property, which she
caused to be annotated on the title of the subject property with
the Registry of Deeds of Quezon City on the same day.
Interestingly, petitioner filed his complaint later that day too.
By way of special and affirmative defenses, Comandante
asserted in her Answer to the amended complaint that said
complaint states no cause of action against her because the
Real Estate Mortgage Contract and the waiver referred to by
petitioner in his complaint were not duly, knowingly and validly
executed by her; that the Waiver of Hereditary Rights and
Interests Over a Real Property (Still Undivided) is a useless
document as its execution is prohibited by Article 1347 of the
Civil Code, hence, it cannot be the source of any right or
obligation in petitioners favor; that the Real Estate Mortgage

10
was of doubtful validity as she executed the same without valid
authority from her parents; and, that the prayer for collection
and/or judicial foreclosure was irregular as petitioner cannot
seek said remedies at the same time.
Apart from executing the affidavit of repudiation, Comandante
also filed on October 4, 1999 a Petition for Cancellation of
Adverse Claim (P.E. 2468) Under The Memorandum of
Encumbrances of TCT No. RT-6604 (82020) PR-188875[19]
docketed as LRC Case No. Q-12009 (99) and raffled to Branch
220 of RTC, Quezon City. Petitioner who was impleaded as
respondent therein moved for the consolidation of said case
with Civil Case No. Q-99-38876. On June 24, 2000, Branch
220 of RTC, Quezon City ordered the consolidation of LRC
Case No. Q-12009 (99) with Civil Case No. Q-99-38876.
Accordingly, the records of the former case was forwarded to
Branch 224.
For their part, the Diazes asserted that petitioner has no cause
of action against them. They claimed that they do not even
know petitioner and that they did not execute any SPA in favor
of Comandante authorizing her to mortgage for the second
time the subject property. They also contested the due
execution of the SPA as it was neither authenticated before the
Philippine Consulate in the United States nor notarized before
a notary public in the State of New York where the Diazes have
been residing for 16 years. They claimed that they do not owe
petitioner anything. The Diazes also pointed out that the
complaint merely refers to Comandantes personal obligation
to petitioner with which they had nothing to do. They thus
prayed that the complaint against them be dismissed.
At the Pangans end, they alleged that they acquired the
subject property by purchase in good faith and for a
consideration of P3,000,000.00 on November 11, 1999 from
the Diazes through the latters daughter Comandante who was
clothed with SPA acknowledged before the Consul of New
York. The Pangans immediately took actual possession of the
property without anyone complaining or protesting. Soon
thereafter, they were issued TCT No. N-209049 in lieu of TCT
No. RT-6604 which was cancelled.
However, on December 21, 1999, they were surprised upon
being informed by petitioner that the subject land had been
mortgaged to him by the Diazes. Upon inquiry from
Comandante, the latter readily admitted that she has a
personal loan with petitioner for which the mortgage of the
property in petitioners favor was executed. She admitted,
though, that her parents were not aware of such mortgage and
that they did not authorize her to enter into such contract.
Comandante also informed the Pangans that the signatures of
her parents appearing on the SPA are fictitious and that it was
petitioner who prepared such document.
As affirmative defense, the Pangans asserted that the
annotation of petitioners adverse claim on TCT No. RT-6604
cannot impair their rights as new owners of the subject

property. They claimed that the Waiver of Hereditary Rights


and Interests Over a Real Property (Still Undivided) upon
which petitioners adverse claim is anchored cannot be the
source of any right or interest over the property considering
that it is null and void under paragraph 2 of Article 1347 of the
Civil Code.
Moreover, the Pangans asserted that the Real Estate
Mortgage Contract cannot bind them nor in any way impair
their ownership of subject property because it was not
registered before the Register of Deeds.
All the respondents interposed their respective counterclaims
and prayed for moral and exemplary damages and attorneys
fees in varying amounts.
After the parties have submitted their respective pre-trial briefs,
the Diazes filed on March 29, 2001 a Motion for Summary
Judgment alleging that: first, since the documents alluded to
by petitioner in his complaint were defective, he was not
entitled to any legal right or relief; and, second, it was clear
from the pleadings that it is Comandante who has an
outstanding obligation with petitioner which the latter never
denied. With these, the Diazes believed that there is no
genuine issue as to any material fact against them and, hence,
they were entitled to summary judgment.
On May 7, 2001, petitioner also filed a Motion for Summary
Judgment, claiming that his suit against the respondents is
meritorious and well-founded and that same is documented
and supported by law and jurisprudence. He averred that his
adverse claim annotated at the back of TCT No. RT-6604,
which was carried over in TCT No. 209049 under the names of
the Pangans, is not merely anchored on the Waiver of
Hereditary Rights and Interests Over a Real Property (Still
Undivided) executed by Comandante, but also on the Real
Estate Mortgage likewise executed by her in representation of
her parents and in favor of petitioner. Petitioner insisted that
said adverse claim is not frivolous and invalid and is registrable
under Section 70 of Presidential Decree (PD) No. 1529. In
fact, the Registrar of Deeds of Quezon City had already
determined the sufficiency and/or validity of such registration
by annotating said claim, and this, respondents failed to
question. Petitioner further averred that even before the sale
and transfer to the Pangans of the subject property, the latter
were already aware of the existence of his adverse claim. In
view of these, petitioner prayed that his Motion for Summary
Judgment be granted.
Ruling of the Regional Trial Court
After the filing of the parties respective Oppositions to the said
motions for summary judgment, the trial court, in an Order
dated May 31, 2001, deemed both motions for summary
judgment submitted for resolution. Quoting substantially
petitioners allegations in his Motion for Summary Judgment, it
thereafter rendered on June 14, 2001 a Summary Judgment in
favor of petitioner, the dispositive portion of which reads:

11
WHEREFORE, premises considered, summary judgment is
hereby rendered in favor of plaintiff and against defendants by:

petitioner, in effect affirming in all other respects the assailed


summary judgment, viz:

a)
ORDERING all defendants jointly and solidarily to
pay plaintiff the sum of ONE MILLION ONE HUNDRED
EIGHTEEN THOUSAND TWO HUNDRED TWENTY EIGHT
PESOS (P1,118,228.00) which is blood money of plaintiff;

WHEREFORE, foregoing premises considered, the


Decision of the Regional Trial Court of Quezon City, Branch
224 in Civil Case No. Q-99-38876 is hereby MODIFIED, as
follows:

b)
ORDERING the Honorable Registrar of Deeds of
Quezon City that the rights and interest of the plaintiff over
subject property be annotated at the back of T.C.T. No. N209049;

1. Ordering defendants-appellants Comandante and


Spouses Diaz to jointly and severally pay plaintiff the sum of
Php 1,118, 228.00; and

c)
SENTENCING all defendants to pay plaintiffs
expenses of TEN THOUSAND PESOS (P10,000.00) and to
pay the costs of suit.
IT IS SO ORDERED.
The Pangans, the Diazes, and Comandante appealed
to the CA.6[29] The Pangans faulted the trial court in holding
them jointly and severally liable with the Diazes and
Comandante for the satisfaction of the latters personal
obligation to petitioner in the total amount of P1,118,228.00.
The Diazes and Comandante, on the other hand, imputed error
upon the trial court in rendering summary judgment in favor of
petitioner. They averred that assuming the summary judgment
was proper, the trial court should not have considered the Real
Estate Mortgage Contract and the Promissory Note as they
were defective, as well as petitioners frivolous and nonregistrable adverse claim.
In its Decision dated December 12, 2003, the CA
declared Comandantes waiver of hereditary rights null and
void. However, it found the Real Estate Mortgage executed by
Comandante on behalf of her parents as binding between the
parties thereto.
As regards the Pangans, the CA ruled that the
mortgage contract was not binding upon them as they were
purchasers in good faith and for value. The property was free
from the mortgage encumbrance of petitioner when they
acquired it as they only came to know of the adverse claim
through petitioners phone call which came right after the
formers acquisition of the property. The CA further ruled that
as Comandantes waiver of hereditary rights and interests
upon which petitioners adverse claim was based is a nullity, it
could not be a source of any right in his favor. Hence, the
Pangans were not bound to take notice of such claim and are
thus not liable to petitioner.
Noticeably, the appellate court did not rule on the
propriety of the issuance of the Summary Judgment as raised
by the Diazes and Comandante. In the ultimate, the CA merely
modified the assailed Summary Judgment of the trial court by
excluding the Pangans among those solidarily liable to

2. Ordering defendants-appellants Comandante and


Spouses Diaz to jointly and severally pay plaintiff the amount
of Php10,000.00 plus cost of suit.
SO ORDERED.
Petitioners Motion for Reconsideration7[32] having been
denied by the CA in its Resolution dated September 10, 2004,
he now comes to us through this petition for review on
certiorari insisting that the Pangans should, together with the
other respondents, be held solidarily liable to him for the
amount of P1,118,228.00.
Our Ruling
The petition lacks merit.
Petitioner merely reiterates his contentions in the Motion for
Summary Judgment he filed before the trial court. He insists
that his Adverse Claim annotated at the back of TCT No. RT6604 is not merely anchored on Comandantes Waiver of
Hereditary Rights and Interests Over A Real Property (Still
Undivided) but also on her being the attorney-in-fact of the
Diazes when she executed the mortgage contract in favor of
petitioner. He avers that his adverse claim is not frivolous or
invalid and is registrable as the Registrar of Deeds of Quezon
City even allowed its annotation. He also claims that even
prior to the sale of subject property to the Pangans, the latter
already knew of his valid and existing adverse claim thereon
and are, therefore, not purchasers in good faith. Thus,
petitioner maintains that the Pangans should be held, together
with the Diazes and Comandante, jointly and severally liable to
him in the total amount of P1,118,228.00.
Petitioners contentions are untenable.
The Affidavit of Adverse Claim executed by petitioner reads in
part:
xxxx
1. That I am the Recipient/Benefactor of compulsory heirs
share over an undivided certain parcel of land together with all
the improvements found therein x x x as evidenced by Waiver
of Hereditary Rights and Interests Over A Real Property,
executed by REINA D. COMANDANTE (a

12
compulsory/legitimate heir of Sps. Alfredo T. Diaz and Imelda
G. Diaz), x x x.
2. That in order to protect my interest over said property as a
Recipient/Benefactor, for the registered owners/parents might
dispose (of) and/or encumber the same in a fraudulent manner
without my knowledge and consent, for the owners duplicate
title was not surrendered to me, it is petitioned that this Affidavit
of Adverse Claim be ANNOTATED at the back of the said title
particularly on the original copy of Transfer Certificate of Title
No. RT-6604 (82020) PR-18887 which is on file with the
Register of Deeds of Quezon City.
3. That I am executing this Affidavit in order to attest (to) the
truth of the foregoing facts and to petition the Honorable
Registrar of Deeds, Quezon City, to annotate this Affidavit of
Adverse Claim at the back of the said title particularly the
original copy of Transfer Certificate of Title No. RT-6604
(82020) PR-18887 which is on file with the said office, so that
my interest as Recipient/Benefactor of the said property will be
protected especially the registered owner/parents, in a
fraudulent manner might dispose (of) and/or encumber the
same without my knowledge and consent. (Emphasis ours)
Clearly, petitioners Affidavit of Adverse Claim was based solely
on the waiver of hereditary interest executed by Comandante.
This fact cannot be any clearer especially so when the
inscription of his adverse claim at the back of TCT No. RT6604 reads as follows:
P.E. 2468/T-(82020)RT-6604 - - AFFIDAVIT OF ADVERSE
CLAIM - - Executed under oath by PEDRO M. FERRER,
married to Erlinda B. Ferrer, claiming among others that they
have a claim, the interest over said property as
Recipient/Benefactor, by virtue of a waiver of Hereditary Rights
and Interest over a real property x x x (Emphasis ours)
Therefore, there is no basis for petitioners assertion
that the adverse claim was also anchored on the mortgage
contract allegedly executed by Comandante on behalf of her
parents.
The questions next to be resolved are: Is Comandantes waiver
of hereditary rights valid? Is petitioners adverse claim based
on such waiver likewise valid and effective?
We note at the outset that the validity of petitioners adverse
claim should have been determined by the trial court after the
petition for cancellation of petitioners adverse claim filed by
Comandante was consolidated with Civil Case No. Q-9938876. This is in consonance with Section 70 of PD 1529
which provides:
Section 70. Adverse Claim. Whoever claims any part or
interest in registered land adverse to the registered owner,
arising subsequent to the date of the original registration, may,
if no other provision is made in this Decree for registering the
same, make a statement in writing setting forth fully his alleged
right or interest, and how or under whom acquired, a reference

to the number of the certificate of title of the registered owner,


the name of the registered owner, and a description of the land
in which the right or interest is claimed.
The statement shall be signed and sworn to, and shall state
the adverse claimants residence, and a place at which all
notices may be served upon him. This statement shall be
entitled to registration as an adverse claim on the certificate of
title. The adverse claim shall be effective for a period of thirty
days from the date of registration. After the lapse of said
period, the annotation of adverse claim may be cancelled upon
filing of a verified petition therefor by the party in interest:
Provided, however, That after cancellation, no second adverse
claim based on the same ground shall be registered by the
same claimant.
Before the lapse of thirty days aforesaid, any party in interest
may file a petition in the Court of First Instance where the land
is situated for the cancellation of the adverse claim, and the
court shall grant a speedy hearing upon the question of validity
of such adverse claim, and shall render judgment as may be
just and equitable. If the adverse claim is adjudged to be
invalid, the registration thereof shall be ordered cancelled. If,
in any case, the court, after notice and hearing, shall find that
the adverse claim thus registered was frivolous, it may fine the
claimant in an amount not less than one thousand pesos nor
more than five thousand pesos, in its discretion. Before the
lapse of thirty days, the claimant may withdraw his adverse
claim by filing with the Register of Deeds a sworn petition to
that effect. (Emphasis ours)
Pursuant to the third paragraph of the afore-quoted provision, it
has been held that the validity or efficaciousness of an adverse
claim may only be determined by the Court upon petition by an
interested party, in which event, the Court shall order the
immediate hearing thereof and make the proper adjudication
as justice and equity may warrant. And, it is only when such
claim is found unmeritorious that the registration of the adverse
claim may be cancelled.8[36]
As correctly pointed out by respondents, the records is bereft
of any showing that the trial court conducted any hearing on
the matter. Instead, what the trial court did was to include this
material issue among those for which it has rendered its
summary judgment as shown by the following portion of the
judgment:
x x x it will be NOTED that subject Adverse Claim annotated at
the back of Transfer Certificate of Title No. RT-6604 (82020)
PR-18887, and carried over to defendants-Sps. Pangans Title
No. N-20909, is not merely anchored on defendant Reina
Comandantes Waiver of Hereditary Rights and Interest Over
a Real Property but also on her being the Attorney-In-Fact of
the previous registered owners/parents/defendants Sps.
Alfredo and Imelda Diaz about the Real Estate Mortgage
Contract for a loan of P1,118,228.00 which is a blood money of
the plaintiff. Moreover, subject Adverse Claim in LRC Case

13
No. Q-12009 (99) is NOT frivolous and invalid and
consequently, REGISTRABLE by virtue of Section 110 of the
Land Registration Act (now Section 70 of Presidential Decree
No. 1529). (Emphasis ours)

matter of Comandantes waiver concededly forms part of the


properties that she expect to inherit from her parents upon their
death and, such expectancy of a right, as shown by the facts,
is undoubtedly purely hereditary in nature.

It does not escape our attention that the trial court


merely echoed the claim of petitioner that his adverse claim
subject of LRC Case No. Q-12009 (99) is not frivolous, invalid
and is consequently registrable. We likewise lament the
apparent lack of effort on the part of said court to make even a
short ratiocination as to how it came up with said conclusion.
In fact, what followed the above-quoted portion of the summary
judgment are mere recitals of the arguments raised by
petitioner in his motion for summary judgment. And in the
dispositive portion, the trial court merely casually ordered that
petitioners adverse claim be inscribed at the back of the title of
the Pangans. What is worse is that despite this glaring defect,
the CA manifestly overlooked the matter even if respondents
vigorously raised the same before it.

From the foregoing, it is clear that Comandante and petitioner


entered into a contract involving the formers future inheritance
as embodied in the Waiver of Hereditary Rights and Interest
Over a Real Property (Still Undivided) executed by her in
petitioners favor.

Be that as it may, respondents efforts of pointing out this flaw,


which we find significant, have not gone to naught as will be
hereinafter discussed.
All the respondents contend that the Waiver of
Hereditary Rights and Interest Over a Real Property (Still
Undivided) executed by Comandante is null and void for being
violative of Article 1347 of the Civil Code, hence, petitioners
adverse claim which was based upon such waiver is likewise
void and cannot confer upon the latter any right or interest over
the property.
We agree with the respondents.
Pursuant to the second paragraph of Article 1347 of
the Civil Code, no contract may be entered into upon a future
inheritance except in cases expressly authorized by law. For
the inheritance to be considered future, the succession must
not have been opened at the time of the contract. A contract
may be classified as a contract upon future inheritance,
prohibited under the second paragraph of Article 1347, where
the following requisites concur:
(1)

That the succession has not yet been opened.

(2)
That the object of the contract forms part of the
inheritance; and,
(3)
That the promissor has, with respect to the object, an
expectancy of a right which is purely hereditary in nature.9[38]
In this case, there is no question that at the time of execution
of Comandantes Waiver of Hereditary Rights and Interest
Over a Real Property (Still Undivided), succession to either of
her parents properties has not yet been opened since both of
them are still living. With respect to the other two requisites,
both are likewise present considering that the property subject

In Taedo v. Court of Appeals, we invalidated the contract of


sale between Lazaro Taedo and therein private respondents
since the subject matter thereof was a one hectare of
whatever share the former shall have over Lot 191 of the
cadastral survey of Gerona, Province of Tarlac and covered by
Title T-13829 of the Register of Deeds of Tarlac. It constitutes
a part of Taedos future inheritance from his parents, which
cannot be the source of any right nor the creator of any
obligation between the parties.
Guided by the above discussions, we similarly declare in this
case that the Waiver of Hereditary Rights and Interest Over a
Real Property (Still Undivided) executed by Comandante in
favor of petitioner as not valid and that same cannot be the
source of any right or create any obligation between them for
being violative of the second paragraph of Article 1347 of the
Civil Code.
Anent the validity and effectivity of petitioners adverse claim, it
is provided in Section 70 of PD 1529, that it is necessary that
the claimant has a right or interest in the registered land
adverse to the registered owner and that it must arise
subsequent to registration. Here, as no right or interest on the
subject property flows from Comandantes invalid waiver of
hereditary rights upon petitioner, the latter is thus not entitled to
the registration of his adverse claim. Therefore, petitioners
adverse claim is without any basis and must consequently be
adjudged invalid and ineffective and perforce be cancelled.
Albeit we have already resolved the issues raised by
petitioner, we shall not stop here as the Diazes and
Comandante in their Comment call our attention to the failure
of the CA to pass upon the issue of the propriety of the
issuance by the trial court of the Summary Judgment in favor
of petitioner despite the fact that they have raised this issue
before the appellate court. They argue that summary judgment
is proper only when there is clearly no genuine issue as to any
material fact in the action. Thus, where the defendant
presented defenses tendering factual issue which call for
presentation of evidence, as when he specifically denies the
material allegations in the complaint, summary judgment
cannot be rendered.
The Diazes and Comandante then enumerate the genuine
issues in the case which they claim should have precluded the
trial court from issuing a summary judgment in petitioners
favor. First, the execution of the SPA in favor of Comandante
referred to by petitioner in his complaint was never admitted by

14
the Diazes. They assert that as such fact is disputed, trial
should have been conducted to determine the truth of the
matter, same being a genuine issue. Despite this, the trial
court merely took the word of the plaintiff and assumed that
said document was indeed executed by them. Second,
although Comandante acknowledges that she has a personal
obligation with petitioner, she nevertheless, did not admit that it
was in the amount of P1,118,228.00. Instead, she claims only
the amount of P500,000.00 or P600,000.00 (if inclusive of
interest) as her obligation. Moreover, the Diazes deny
borrowing any money from petitioner and neither did the
Pangans owe him a single centavo. Thus, the true amount of
the obligation due the petitioner and how each of the
respondents are responsible for such amount are genuine
issues which need formal presentation of evidence. Lastly,
they aver that the trial court ignored factual and material issues
such as the lack of probative value of Comandantes waiver of
hereditary rights as well as of the SPA; the fact that
Comandante signed the mortgage contract and promissory
note in her personal capacity; and, that all such documents
were prepared by petitioner who acted as a lawyer and the
creditor of Comandante at the same time.
Rule 35 of the Rules of Court provides for summary judgment,
the pertinent provisions of which are the following:
Section 1. Summary Judgment for claimant. A party seeking
to recover upon a claim, counterclaim, or cross-claim or to
obtain a declaratory relief may, at any time after the pleading in
answer thereto has been served, move with supporting
affidavits, depositions or admissions for a summary judgment
in his favor upon all or any part thereof.
Section 2. Summary Judgment for the defending party. A
party against whom a claim, counterclaim or cross-claim is
asserted or a declaratory relief is sought may, at any time,
move with supporting affidavits, depositions or admissions for
a summary judgment in his favor as to all or any part thereof.
Section 3. Motion and proceedings thereon. The motion shall
be served at least ten (10) days before the time specified for
the hearing. The adverse party may serve opposing affidavits,
depositions, or admissions at least three (3) days before the
hearing. After the hearing, the judgment sought shall be
rendered forthwith if the pleadings, supporting affidavits,
depositions and admissions on file, show that, except as to the
amount of damages, there is no genuine issue as to any
material fact and that the moving party is entitled to a judgment
as a matter of law.
As can be deduced from the above provisions, summary
judgment is a procedural devise resorted to in order to avoid
long drawn out litigations and useless delays. When the
pleadings on file show that there are no genuine issues of facts
to be tried, the Rules of Court allows a party to obtain
immediate relief by way of summary judgment. That is, when
the facts are not in dispute, the court is allowed to decide the
case summarily by applying the law to the material facts.

Conversely, where the pleadings tender a genuine issue,


summary judgment is not proper. A genuine issue is such fact
which requires the presentation of evidence as distinguished
from a sham, fictitious, contrived or false claim.
Here, we find the existence of genuine issues which removes
the case from the coverage of summary judgment. The
variance in the allegations of the parties in their pleadings is
evident.
Petitioner anchors his complaint for sum of money and/or
judicial foreclosure on the alleged real estate mortgage over
the subject property allegedly entered into by Comandante in
behalf of her parents to secure payment of a loan amounting to
P1,118,228.00. To support this claim, petitioner attached to his
complaint (1) the SPA alleged to have been executed by the
Diazes; (2) the Real Estate Mortgage Contract pertaining to
the amount of P1,118,228.00; and, (3) a Promissory Note.
Comandante, in her Answer to petitioners Amended
Complaint, assailed the validity and due execution of the
abovementioned documents. She asserted that the same
were not duly, knowingly and validly executed by her and that it
was petitioner who prepared all of them. Also, although she
admitted owing petitioner, same was not an absolute
admission as she limited herself to an obligation amounting
only to P600,000.00 inclusive of charges and interests. She
likewise claimed that such obligation is her personal obligation
and not of her parents.
The Diazes, for their part, also denied that they executed the
SPA authorizing their daughter to mortgage their property to
petitioner as well as having any obligation to the latter.
Clearly, there are genuine issues in this case which require the
presentation of evidence. For one, it is necessary to ascertain
in a full blown trial the validity and due execution of the SPA,
the Real Estate Mortgage and the Promissory Notes because
the determination of the following equally significant questions
depends on them, to wit: (1) Are the Diazes obligated to
petitioner or is the obligation a purely personal obligation of
Comandante? and, (2) Is the sum of P1,118,228.00 as shown
in the Real Estate Mortgage and the Promissory Note, the
amount which is really due the petitioner?
To stress, trial courts have limited authority to render
summary judgments and may do so only when there is clearly
no genuine issue as to any material fact. When the facts as
pleaded by the parties are disputed or contested, proceedings
for summary judgment cannot take the place of trial. From the
foregoing, it is apparent that the trial court should have
refrained from issuing the summary judgment but instead
proceeded to conduct a full blown trial of the case. In view of
this, the present case should be remanded to the trial court for
further proceedings and proper disposition according to the
rudiments of a regular trial on the merits and not through an
abbreviated termination of the case by summary judgment.

15
WHEREFORE, the petition is DENIED. The assailed
Decision of the Court of Appeals dated December 12, 2003
insofar as it excluded the respondents Spouses Bienvenido
Pangan and Elizabeth Pangan from among those solidarily
liable to petitioner Atty. Pedro M. Ferrer, is AFFIRMED. The
inscription of the adverse claim of petitioner Atty. Pedro M.
Ferrer on T.C.T. No. N-209049 is hereby ordered
CANCELLED. Insofar as its other aspects are concerned, the
assailed Decision is SET ASIDE and VACATED. The case is
REMANDED to the Regional Trial Court of Quezon City,
Branch 224 for further proceedings in accordance with this
Decision.
SO ORDERED.
IN RE: PETITION FOR PROBATE OF LAST WILL AND
TESTAMENT OF BASILIO SANTIAGO,
MA. PILAR SANTIAGO and CLEMENTE SANTIAGO,
Petitioners,
- versus ZOILO S. SANTIAGO, FELICIDAD SANTIAGO-RIVERA,
HEIRS
OF
RICARDO SANTIAGO,
HEIRS
OF
CIPRIANO SANTIAGO, HEIRS OF TOMAS SANTIAGO,
Respondents.
FILEMON SOCO, LEONILA SOCO, ANANIAS SOCO,
URBANO SOCO, GERTRUDES SOCO AND HEIRS OF
CONSOLACION
SOCO,
Oppositors.
CARPIO MORALES, J.:
Basilio Santiago (Basilio) contracted three marriagesthe first
to Bibiana Lopez, the second to Irene Santiago, and the third
to Cecilia Lomotan. Basilio and his first wife bore two
offsprings, Irene and Marta, the mother of herein oppositors
Felimon, Leonila, Consolacion, Ananias, Urbano, and
Gertrudes, all surnamed Soco.
Basilio and his second wife had six offsprings, Tomas,
Cipriano, Ricardo, respondents Zoilo and Felicidad, and
petitioner Ma. Pilar, all surnamed Santiago.
Basilio and his third wife bore three children, Eugenia herein
petitioner Clemente, and Cleotilde, all surnamed Santiago.
After Basilio died testate on September 16, 1973, his daughter
by the second marriage petitioner Ma. Pilar filed before the
Regional Trial Court (RTC) of Bulacan a petition for the probate
of Basilios will, docketed as SP No. 1549-M. The will was
admitted to probate by Branch 10 of the RTC and Ma. Pilar
was appointed executrix.

The will contained the following provisions, among others:


4. Ang mga ari-arian ko na nasasaysay sa itaas ay INIWAN,
IPINAGKAKALOOB, IBINIBIGAY, at IPINAMAMANA ko sa
aking mga nasabing tagapagmana sa ilalim ng gaya ng
sumusunod:
xxxx
c) ang aking anak na si Ma. Pilar ang magpapalakad at
mamamahala ng balutan na nasa Santiago, Malolos, Bulacan,
na nasasaysay sa itaas na 2(y);
d) Sa pamamahala ng bigasan, pagawaan ng pagkain ng
hayop at lupat bahay sa Maynila, ang lahat ng solar sa danay
ng daang Malolos-Paombong na nasa Malolos, Bulacan, kasali
at kasama ang palaisdaan na nasa likuran niyon, ay ililipat sa
pangalan nila Ma. Pilar at Clemente; ngunit ang kita ng
palaisdaan ay siyang gagamitin nila sa lahat at anomang
kailangang gugol, maging majora o roperacion [sic], sa lupat
bahay sa Lunsod ng Maynila na nasasaysay sa itaas na 2(c);
e) Ang lupat bahay sa Lunsod ng Maynila na nasasaysay sa
itaas na 2(c) ay ililipat at ilalagay sa pangalan nila Ma. Pilar at
Clemente hindi bilang pamana ko sa kanila kundi upang
pamahalaan at pangalagaan lamang nila at nang ang sinoman
sa aking mga anak sampu ng apo at kaapuapuhan ko sa
habang panahon ay may tutuluyan kung magnanais na magaral sa Maynila o kalapit na mga lunsod x x x.
f) Ang bigasan, mga makina at pagawaan ng pagkain ng
hayop ay ipinamamana ko sa aking asawa, Cecilia Lomotan, at
mga anak na Zoilo, Ma. Pilar, Ricardo, Cipriano, Felicidad,
Eugenia, Clemente, at Cleotilde nang pare-pareho. Ngunit, sa
loob ng dalawampong (20) taon mula sa araw ng aking
kamatayan, hindi nila papartihin ito at pamamahalaan ito ni
Clemente at ang maghahawak ng salaping kikitain ay si Ma.
Pilar na siyang magpaparte. Ang papartihin lamang ay ang
kita ng mga iyon matapos na ang gugol na kakailanganin
niyon, bilang reparacion, pagpapalit o pagpapalaki ay maawas
na. Ninais ko ang ganito sa aking pagmamahal sa kanila at
pagaaring ibinubuhay ko sa kanila lahat, bukod sa yaon ay sa
kanila ding kapakinabangan at kabutihan.
g) Ang lahat ng lupa, liban sa lupat bahay sa Lunsod ng
Maynila, ay ipinapamana ko sa aking nasabing asawa, Cecilia
Lomotan, at mga anak na Tomas, Zoilo, Ma. Pilar, Ricardo,
Cipriano, Felicidad, Eugenia, Clemente at Cleotilde nang parepareho. Datapwat, gaya din ng mga bigasan, makina at
gawaan ng pagkain ng hayop, ito ay hindi papartihin sa loob ng
dalawampong (20) taon mula sa aking pagpanaw, at
pamamahalaan din nila Ma. Pilar at Clemente. Ang mapaparte
lamang ay ang kita o ani ng nasabing mga pag-aari matapos
bayaran ang buwis at/o patubig at iba pang mga gugol na
kailangan. Si Ma. Pilar din ang hahawak ng ani o salaping
manggagaling dito. (emphasis and underscoring supplied)
The oppositors-children of Marta, a daughter of Basilio and his
first wife, were, on their motion, allowed to intervene.

16
After the executrix-petitioner Ma. Pilar filed a Final
Accounting, Partition and Distribution in Accordance with the
Will, the probate court approved the will by Order of August
14, 1978 and directed the registers of deeds of Bulacan and
Manila to register the certificates of title indicated therein.
Accordingly, the titles to Lot Nos. 786, 837, 7922, 836 and 838
in Malolos, Bulacan and Lot No. 8-C in Manila were transferred
in the name of petitioners Ma. Pilar and Clemente.
The oppositors thereafter filed a Complaint-in-Intervention with
the probate court, alleging that Basilios second wife was not
Irene but a certain Maria Arellano with whom he had no child;
and that Basilios will violates Articles 979-981 of the Civil
Code.
The probate court dismissed the Complaint-in-Intervention,
citing its previous approval of the Final Accounting, Partition,
and Distribution in Accordance with the Will.
The oppositors-heirs of the first marriage thereupon filed a
complaint for completion of legitime before the Bulacan RTC,
docketed as Civil Case No. 562-M-90, against the heirs of the
second and third marriages.
In their complaint, oppositors-heirs of the first marriage
essentially maintained that they were partially preterited by
Basilios will because their legitime was reduced. They thus
prayed, inter alia, that an inventory and appraisal of all the
properties of Basilio be conducted and that Ma. Pilar and
Clemente be required to submit a fresh accounting of all the
incomes of the properties from the time of Basilios death up to
the time of the filing of Civil Case No. 562-M-90.
RTC-Branch 17 decided Civil Case No. 562-M-90 (for
completion of legitime) in favor of the oppositors-heirs of the
first marriage.
On appeal (docketed as CA G.R. No. 45801), the Court of
Appeals, by Decision of January 25, 2002, annulled the
decision of RTC-Branch 17, holding that the RTC Branch 17
dismissal of the Complaint-in-Intervention in SP No. 1549-M
and its August 14, 1978 Order approving the probate of the will
constitute res judicata with respect to Civil Case No. 562-M-90.
Thus the appellate court disposed:

In the interregnum, or on October 17, 2000, respondent-heirs


of the second marriage filed before the probate court (RTCBranch 10) a Motion for Termination of Administration, for
Accounting, and for Transfer of Titles in the Names of the
Legatees. Citing the earlier quoted portions of Basilios will,
they alleged that:
x x x x the twenty (20) year period within which subject
properties should be under administration of [Ma.] Pilar
Santiago and Clemente Santiago expired on September 16,
1993.
Consequently, [Ma.] Pilar Santiago and Clemente Santiago
should have ceased as such administrator[s] way back on
September 16, 1993 and they should have transferred the
above said titles to the named legatees in the Last Will and
Testament of the testator by then. Said named legatees in the
Last Will and Testament are no[ne] other than the following:
xxxx
Said [Ma.] Pilar Santiago and Clemente Santiago should have
also rendered an accounting of their administration from such
death of the testator up to the present or until transfer of said
properties and its administration to the said legatees.
xxxx
Respondents prayed that petitioners be ordered:
1)
To surrender the above-enumerated titles presently in
their names to [the] Honorable Court and to transfer the same
in the names of the designated legatees in the Last Will and
Testament, to wit:
1) asawa, Cecilia Lomotan, at mga anak na
2) Tomas
3) Zoilo
4) Ma. Pilar
5) Ricardo
6) Cipriano

WHEREFORE, premises considered, the Appeal is hereby


GRANTED. The Decision in Civil Case No. 562-M-90 is
hereby ANNULLED on the ground of res judicata. Let the
Decree of Distribution of the Estate of Basilio Santiago remain
UNDISTURBED.
SO ORDERED. (emphasis in the original; underscoring
supplied)
Oppositors-heirs of the first marriage challenged the appellate
courts decision in CA G.R. No. 45801 by petition for review,
docketed as G.R. No. 155606, which this Court denied. The
denial became final and executory on April 9, 2003.

7) Felicidad
8) Eugenia
9) Clemente at
10) Cleotilde
(all surnamed SANTIAGO)
2)
To peacefully surrender possession and administration
of subject properties, including any and all improvements
thereon, to said legatees.

17
3)
To render an accounting of their administration of said
properties and other properties of the testator under their
administration, from death of testator Basilio Santiago on
September 16, 1973 up to the present and until possession
and administration thereof is transferred to said legatees.

Santiago and Clemente Santiago and to issue new ones in the


lieu thereof in the names of Cecilia Lomotan-Santiago, Tomas
Santiago, Zoilo Santiago, Ma. Pilar Santiago, Ricardo
Santiago, Cipriano Santiago, Felicidad Santiago, Eugenia
Santiago, Clemente Santiago, and Cleotilde Santiago.

Opposing the motion, petitioners argued that with the approval


of the Final Accounting, Partition and Distribution in
Accordance with the Will, and with the subsequent issuance of
certificates of title covering the properties involved, the case
had long since been closed and terminated.

Moreover, the Register of Deeds of Manila is hereby


DIRECTED to cancel and consider as no force and effect
Transfer Certificate of Title No. 131044 [Lot No. 8-C] in the
names of Ma. Pilar Santiago and Clemente Santiago and to
issue new ones in lieu thereof in the names of the Heirs of
Bibiana Lopez, the Heirs of Irene Santiago, and the Heirs of
Cecilia Lomotan.

The probate court, finding that the properties in question would


be transferred to petitioners Ma. Pilar and Clemente for
purposes of administration only, granted the motion, by Order
of September 5, 2003, disposing as follows:
WHEREFORE, premises considered, the Motion for
Termination of Administration, for Accounting, and for Transfer
of Titles in the Names of the Legatees dated October 3, 2000
filed by some heirs of the testator Basilio Santiago xxx is
hereby GRANTED. Accordingly, the administratrix [sic] Ma.
Pilar Santiago and Mr. Clemente Santiago are hereby
DIRECTED, as follows:
a.) To surrender the above-enumerated titles presently in
their names to this Honorable Court and to transfer the same in
the names of the designated legatees in the Last Will and
Testament, to wit: 1.) asawa, Cecilia Lomotan at mga anak na
2.) Tomas 3). Zoilo 4.) Ma. Pilar 5.) Ricardo 6.) Cipriano 7.)
Felicidad 8.) Eugenia 9.) Clemente and 10.) Cleotilde all
named SANTIAGO.
b.) To peacefully surrender possession and administration of
subject properties including any and all improvements thereon,
to said legatees; and
c.) To render an accounting of their administration of subject
properties, including any and all improvements thereon, to said
legatees; and
d.) To submit an accounting of their administration of the
above-mentioned estate of the testator or all the above said
lots including the rice mill, animal feeds factory, and all
improvements thereon from August 14, 1978 up to the present.
e.) To submit a proposed Project of Partition, indicating how
the parties may actually partition or adjudicate all the above
said properties including the properties already in the name of
all the said legatees xxx.
x x x x.
Further, the Register of Deeds of Bulacan are hereby
DIRECTED to cancel and consider as no force and effects
Transfer Certificates of Title Nos. T-249177 (RT-46294) [Lot
No. 786], T-249175 (RT-46295) [Lot No. 837], T-249174 (RT46296) [Lot No. 7922], T-249173 (RT-46297) [Lot No. 836], and
T-249176 (RT-46293) [Lot No. 838] in the names of Ma. Pilar

The Motion to Suspend Proceedings filed by Filemon, Leonila,


Ma. Concepcion, Ananias, Urbano and Gertrudes, all
surnamed Soco, dated December 3, 2002, is hereby DENIED
for lack of merit.
Respecting petitioners argument that the case had long been
closed and terminated, the trial court held:
x x x x [I]t is clear from the Last Will and Testament that
subject properties cannot actually be partitioned until after 20
years from the death of the testator Basilio Santiago x x x x. It
is, therefore, clear that something more has to be done after
the approval of said Final Accounting, Partition, and
Distribution. The testator Basilio Santiago died on September
16, 1973, hence, the present action can only be filed after
September 16, 1993. Movants cause of action accrues only
from the said date and for which no prescription of action has
set in.
The principle of res judicata does not apply in the present
probate proceeding which is continuing in character, and
terminates only after and until the final distribution or
settlement of the whole estate of the deceased in accordance
with the provision of the will of the testator. The Order dated
August 14, 1978 refers only to the accounting, partition, and
distribution of the estate of the deceased for the period
covering from the date of the filing of the petition for probate on
December 27, 1973 up to August 14, 1978. And in the said
August 14, 1978 order it does not terminate the appointment of
petitioner[s] Ma. Pilar Santiago and Clemente Santiago as
executrix and administrator, respectively, of the estate of the
deceased particularly of those properties which were prohibited
by the testator to be partitioned within 20 years from his death.
Since then up to the present, Ma. Pilar Santiago and Clemente
Santiago remain the executor and administrator of the estate of
the deceased and as such, they are required by law to render
an accounting thereof from August 14, 1978 up to the present;
there is also now a need to partition and distribute the
aforesaid properties as the prohibition period to do so has
elapsed. (emphasis and underscoring supplied)
Petitioners, together with the oppositors, filed a motion for
reconsideration, which the probate court denied, drawing them
to appeal to the Court of Appeals which docketed it as CA G.R.
No. 83094.

18
The Court of Appeals affirmed the decision of the probate
court, hence, the petition which raises the following grounds:

claim, demand or cause of action between the present petition


and G.R. No. 155606.

I.
CAN THE HONORABLE COURT OF APPEALS REVERSE
ITSELF

A. THE COURT OF APPEALS ERRED IN NOT BINDING


ITSELF WITH ITS PREVIOUS DECISION INVOLVING THE
SAME PARTIES AND SAME PROPERTIES;
B. THE COURT OF APPEALS ERRED IN AFFIRMING THE
RTC AS IT AGREED WITH THE RTC THAT THIS CASE IS
NOT BARRED BY RES JUDICATA;
C. IN C.A.-G.R. NO. 45801, THE HONORABLE COURT OF
APPEALS HELD THAT THERE WAS RES JUDICATA; IN C.A.G.R. CV NO. 83094, THERE WAS NO RES JUDICATA.
II.
GRANTING THAT THE COURT OF APPEALS HAS ALL THE
COMPETENCE AND JURISDICTION TO REVERSE ITSELF,
STILL THE COURT OF APPEALS ERRED IN AFFIRMING
THE RTCS ORDER TO TRANSFER THE MANILA
PROPERTY COVERED BY TCT NO. 131004 TO THE NAMES
OF CECILIA LOMOTAN, TOMAS, ZOILO, MA. PILAR,
RICARDO, CIPRIANO FELICIDAD, EUGENIA, CLEMENTE
AND CLEOTILDE, ALL SURNAMED SANTIAGO. (emphasis
in the original)
The petition lacks merit.
Petitioners argument that the decision of the appellate court in
the earlier CA-G.R. NO. 45801 (upheld by this Court in G.R.
No. 155606) constitutes res judicata to the subsequent CA
G.R. No. 83094 (the subject of the present petition for review)
fails.
Res judicata has two aspects, which are embodied in Sections
47 (b) and 47 (c) of Rule 39 of the Rules of Civil Procedure.
The first, known as bar by prior judgment, proscribes the
prosecution of a second action upon the same claim, demand
or cause of action already settled in a prior action. The second,
known as conclusiveness of judgment, ordains that issues
actually and directly resolved in a former suit cannot again be
raised in any future case between the same parties involving a
different cause of action.
Both aspects of res judicata, however, do not find application
in the present case. The final judgment regarding oppositors
complaint on the reduction of their legitime in CA-G.R. NO.
45801 does not dent the present petition, which solely tackles
the propriety of the termination of administration, accounting
and transfer of titles in the names of the legatees-heirs of the
second and third marriages. There is clearly no similarity of

While as between the two cases there is identity of parties,


conclusiveness of judgment cannot likewise be invoked.
Again, the judgment in G.R. No. 155606 would only serve as
an estoppel as regards the issue on oppositors supposed
preterition and reduction of legitime, which issue is not even a
subject, or at the very least even invoked, in the present
petition.
What is clear is that petitioners can invoke res judicata insofar
as the judgment in G.R. No. 155606 is concerned against the
oppositors only. The records reveal, however, that the
oppositors did not appeal the decision of the appellate court in
this case and were only impleaded pro forma parties.
Apparently, petitioners emphasize on the directive of the
appellate court in CA G.R. No. 45801 that the decree of
distribution of the estate of Basilio should remain undisturbed.
But this directive goes only so far as to prohibit the interference
of the oppositors in the distribution of Basilios estate and does
not pertain to respondents supervening right to demand the
termination of administration, accounting and transfer of titles
in their names.
Thus, the Order of September 5, 2003 by the probate court
granting respondents Motion for Termination of Administration,
for Accounting, and for Transfer of Titles in the Names of the
Legatees is a proper and necessary continuation of the August
14, 1978 Order that approved the accounting, partition and
distribution of Basilios estate. As did the appellate court, the
Court notes that the August 14, 1978 Order was yet to become
final pending the whole settlement of the estate. And final
settlement of the estate, in this case, would culminate after 20
years or on September 16, 1993, when the prohibition to
partition the properties of the decedent would be lifted.
Finally, petitioners object to the inclusion of the house and lot
in Manila, covered by TCT No. 131044, among those to be
transferred to the legatees-heirs as it would contravene the
testators intent that no one is to own the same.
The Court is not persuaded. It is clear from Basilios will that
he intended the house and lot in Manila to be transferred in
petitioners names for administration purposes only, and that
the property be owned by the heirs in common, thus:
e) Ang lupat bahay sa Lunsod ng Maynila na nasasaysay sa
itaas na 2(c) ay ililipat at ilalagay sa pangalan nila Ma. Pilar at
Clemente hindi bilang pamana ko sa kanila kundi upang
pamahalaan at pangalagaan lamang nila at nang ang sinoman
sa aking mga anak sampu ng apo at kaapuapuhan ko sa
habang panahon ay may tutuluyan kung magnanais na magaral sa Maynila o kalapit na mga lunsod sa medaling salita,
ang bahay at lupang itoy walang magmamay-ari bagkus ay
gagamitin habang panahon ng sinomang magnanais sa aking

19
kaapuapuhan na tumuklas ng karunungan sa paaralan sa
Maynila at katabing mga lunsod x x x x (emphasis and
underscoring supplied)
But the condition set by the decedent on the propertys
indivisibility is subject to a statutory limitation. On this point,
the Court agrees with the ruling of the appellate court, viz:
For this Court to sustain without qualification, [petitioners]s
contention, is to go against the provisions of law, particularly
Articles 494, 870, and 1083 of the Civil Code, which provide
that the prohibition to divide a property in a co-ownership can
only last for twenty (20) years x x x x
x x x x Although the Civil Code is silent as to the effect of the
indivision of a property for more than twenty years, it would be
contrary to public policy to sanction co-ownership beyond the
period expressly mandated by the Civil Code x x x x
WHEREFORE, the petition is DENIED.
Costs against petitioners.
SO ORDERED.

SPOUSES CHARLITO COJA and ANNIE MESA COJA,


Petitioners,
-versusHON. COURT OF APPEALS and HEIRS OF FELICIANO
AQUILLO, SR., namely: QUINCIANO VICTOR, SR., LORNA A.
VICTOR, ANTONIO VICTOR, QUINCIANO A. VICTOR, JR.,
SUSANA A. VICTOR, CLARA AQUILLO, CARMENCITA
AQUILLO, AGAPITO AQUILLO, NOEL AQUILLO, ADONIS
AQUILLO, FELICIANO AQUILLO, JR., RONALD AQUILLO
and ALDRIN AQUILLO,
Respondents.
DECISION
AZCUNA, J.:
Before us is a petition for review on certiorari[1] assailing the
Decision of the Court of Appeals (CA) in CA-G.R. CV No.
37583 dated February 5, 2001 and the
Resolution[2] dated November 5, 2001 denying petitioners
motion for reconsideration.
The facts of the case are as follows:
Luz Aquillo Victor (hereafter Luz) and Feliciano Aquillo,
Jr. (hereafter Feliciano Jr.), both deceased, were the legitimate
children of the late spouses Feliciano Aquillo, Sr. (hereafter
Feliciano Sr.) and Lorenza Mangarin Aquillo (hereafter
Lorenza).[3] During their marriage, Feliciano Sr. and Lorenza

acquired a 120-square meter lot located at Poblacion,


Mandaon, Masbate, upon which they built their conjugal
home. The subject lot was covered by Tax Declaration No.
1151 issued in the name of Feliciano Sr.
After the death of Lorenza, Feliciano Sr. cohabited with Paz
Lachica and lived at the aforesaid house. However, after
Lorenzas death, her heirs failed to partition their hereditary
shares in their inheritance.
On February 27, 1960, while Lorenza was cohabiting with
Feliciano Sr., Paz Lachica purchased a 192-square meter lot
covered by Tax Declaration No. 02115 from the heirs of Juan
Rivas. She later sold 40.10 square meters of the property to
Isabel L. de Real leaving her with only 151.9 square meters.
On July 7, 1965, or two (2) days before he died, Feliciano Sr.
married Paz Lachica. After Feliciano Sr. died, his heirs also
failed to partition among themselves their hereditary shares in
their inheritance.
Sometime in 1969, Paz Lachica was issued Tax Declaration
No. 4424 over the remaining 151.9 square meters of the
property covered by Tax Declaration No. 02115. The aforesaid
Tax Declaration was later cancelled by Tax Declaration No.
3443-Rev. On September 10, 1973, Tax Declaration No.
3514 was issued, effectively canceling Tax Declaration No.
3443-Rev. Also, in said Tax Declaration No. 3514, the area
originally covered by Tax Declaration No. 3443-Rev was
increased from 151.9 square meters to 336 square meters,
and it included the 120-square meter property originally
covered by Tax Declaration No. 1151. It also contained an
annotation at the back stating Revised as per request of the
owner to include the excess area for taxation
purposes. Thereafter, Tax Declaration No. 3514 was cancelled
by Tax Declaration No. 1558, which was later cancelled by Tax
Declaration No. 1946, and later cancelled by Tax Declaration
No. 2038.
On December 18, 1986, Paz Lachica and herein
petitioners, Spouses Charlito Coja and Annie Mesa Coja,
executed a Deed of Absolute Sale wherein the former sold the
336-square meter parcel of land covered by Tax Declaration
No. 2038 to the latter. Consequently, Tax Declaration No.
4946 was issued in the name of petitioners, canceling Tax
Declaration No. 2038.
Sometime in 1987, Charlito Coja filed an application for
the issuance of title with the Regional Trial Court (RTC),
Branch 46,Masbate, Masbate (now Masbate City) docketed as
LRC No. N-365. Luz, being one of the heirs of Feliciano Sr.,
opposed the application for registration. Likewise, the Office of
the Solicitor General (OSG) opposed the application. The
OSG alleged, among other things, that the applicant or his
predecessors-in- interest had not been in open, continuous,
exclusive, and notorious possession of the subject land within
the period required by law; and that the documents attached to
or alleged in the application do not constitute competent and
sufficient evidence of a bona fide acquisition of the land or of

20
an open, continuous, exclusive, and notorious possession and
occupation thereof in the concept of an owner.

Once this decision becomes final and executory, let the


corresponding decree of registration issue.

During the pendency of the case, Luz died. She was


substituted by her spouse, Quinciano Victor, Sr., and her
children, Lorna, Antonio, Quinciano Jr., and Susana, all
surnamed Victor.

SO ORDERED.

On November 3, 1989, respondents filed an action for


recovery of possession and ownership with damages,
docketed as Civil Case No. 3904, against the petitioners and
Paz Lachica. Respondents claimed that they are the true and
lawful heirs of the Spouses Feliciano Sr. and Lorenza; that Paz
Lachica refused to deliver the property to its rightful owners
despite repeated demands; that Paz Lachica appropriated the
subject property to herself and had the tax declaration
transferred to her name; that Paz Lachica sold the property to
the Spouses Coja; and that the Spouses Coja failed to deliver
the subject property to the rightful heirs despite repeated
demands.
Upon motion by the plaintiffs, LRC No. N-365 and Civil
Case No. 3904 were consolidated.
In their Answer, defendants therein alleged that Paz
Lachica acquired the subject property before her marriage to
Feliciano Sr. and that she had been in actual and physical
possession of the same for more than fifteen (15) years before
she sold the property to the Spouses Coja; that they acquired
the property by purchasing it from Paz Lachica; that they are
buyers in good faith and for value; and that the property in
question was the paraphernal property of Paz Lachica and,
therefore, plaintiffs therein have no right and interest over the
same.
The parties failed to settle their respective differences
and a joint trial ensued.
On March 11, 1992, the RTC rendered a
decision against the plaintiffs-oppositors and in favor of the
defendants-applicants, the decretal portion of which reads:
WHEREFORE, premises considered, decision is hereby
rendered in favor of the defendants-applicants, to wit:
1.
Ordering the dismissal of the complaint in Civil Case
No. 3904 with costs against the plaintiffs-oppositors;
2.
Declaring the defendants-applicants spouses Charlito
Coja and Annie Mesa, the absolute owner of the land subject
of their application in L.R.C. No. N-365;
3.
Declaring the title of the applicants, spouses Charlito
Coja and Annie Mesa and Sancho Mesa, over the property
designated as Psu-05-005736 together with all the
improvements thereon, CONFIRMED and REGISTERED
pursuant to the provisions of P.D. 1529.

The RTC opined that since the land in question is


registered in the name of Paz Lachica alone, it is assumed that
it is not part of the conjugal partnership properties of Feliciano
Sr., and Lorenza, for if it was their conjugal property, it should
have been registered in their names. As such, when the
Spouses Coja purchased the property from Paz Lachica, they
were of the honest belief that the latter was the true and lawful
owner. Likewise, on the basis of the evidence adduced, the
RTC held that defendants-applicants possess good title proper
for registration and confirmation.
Aggrieved, plaintiffs-oppositors appealed from the decision to
the CA, assigning the following errors:
[1]
THE TRIAL COURT GRAVELY ERRED IN HOLDING
THAT THE DEFENDANTS-APPLICANTS, SPOUSES
CHARLITO COJA AND ANNIE MESA COJA ARE THE TRUE
AND LAWFUL OWNERS OF THE LAND SUBJECT OF THEIR
APPLICATION.
[2]
THE TRIAL COURT GRAVELY ERRED IN
CONCLUDING THAT THE LAND IN QUESTION IS NOT THE
CONJUGAL PARTNERSHIP PROPERTY OF THE COUPLE,
THE LATE SPOUSES FELICIANO AQUILLO AND LORENZA
MANGARIN
[3]
THE TRIAL COURT GRAVELY ERRED IN HOLDING
THAT DEFENDANTS-APPLICANTS POSSESS GOOD TITLE,
PROPER FOR REGISTRATION.
On February 5, 2001, the CA rendered a
Decision reversing and setting aside the decision of the RTC,
the pertinent portion of which reads:
WHEREFORE, premises considered, the decision
dated March 11, 1992 is hereby REVERSED and SET ASIDE,
and a new one entered, as follows:
1.
The sale of the property by Paz Lachica to
Spouses Charlito Coja and Annie Mesa Coja insofar as the
shares of appellants is (sic) concerned is NULLIFIED;
2.
Appellees-applicants are ordered to deliver
possession of the property originally covered by Tax
Declaration No. 1151 to appellants, to the extent of 93.3333
square meters;
3.
Appellee-applicants are ordered to pay appellants
P300.00 per month as reasonable rent for the use of the
property, from the date of filing of the complaint and until
possession thereof is restored to appellants;
4.
The application for registration of title by Charlito
and Annie Coja in L.R.C. No. N-365 is denied; and

21
5.

Costs against appellees.

SO ORDERED.
The CA concluded that the property formerly covered by
Tax Declaration No. 1151, with an area of 120 square meters,
is the conjugal property of Feliciano Sr. and Lorenza having
been acquired during their marriage. Under the law, upon the
death of Lorenza, one-half of said property, or 60 square
meters, was transmitted to her heirs, namely Feliciano Sr.,
Feliciano Jr., and Luz, at 20 square meters each, while the
remaining one-half pertained to Feliciano Sr. alone as his
share in the conjugal property. Upon the death of Feliciano Sr.,
his rights over the property, consisting of his inheritance from
his wife and his share in the conjugal partnership, or a total of
80 square meters, were transmitted to his heirs, Feliciano Jr.,
Luz, and his widow, Paz Lachica. Thus, Paz Lachica is entitled
to only 26.6666 square meters and the heirs of Feliciano Jr.
and Luz are entitled to the remaining 93.3333 square meters of
the subject property. Therefore, Paz Lachica had no authority
to sell their portions of the property.
Applicants-appellees therein filed a motion for
reconsideration but it was denied in the
Resolution dated November 5, 2001.
Hence, this petition, assigning the following errors:
[1] THE HONORABLE COURT OF APPEALS ERRED IN
REVERSING AND SETTING ASIDE THE DECISION OF THE
REGIONAL TRIAL COURT, BRANCH 46, AT MASBATE NOW
MASBATE CITY (ANNEX H) AS THE SAME IS IN
ACCORDANCE WITH LAW AND JURISPRUDENCE; AND
[2]
THE HONORABLE COURT OF APPEALS ERRED IN
DENYING THE APPLICATION FOR LAND REGISTRATION
OF TITLE OF THE PETITIONERS OVER THEIR
RESIDENTIAL AND COMMERCIAL LAND SITUATED AT
POBLACION, MANDAON, MASBATE WHICH SHOULD BE
CONFIRMED AND REGISTERED PURSUANT TO LAND
REGISTRATION LAW IN RELATION TO PD NO. 1529.
Petitioners argue that respondents failed to establish
their case on the basis of the evidence they presented during
the trial. Respondents only presented Tax Declaration No. 1151
which had never been updated since 1945 up to Feliciano Sr.s
death. In addition, his alleged successors-in-interest have not
caused the revision of the said tax declaration nor paid the
taxes to the government up to the present and hence the same
cannot be considered proof of ownership. Since Feliciano Sr.
is not the owner of the property in question, the same cannot
be inherited by the respondents. Moreover, no survey of the
property had been made in the name of Feliciano Sr.
Petitioners add that the subject property was the
paraphernal property of Paz Lachica since she purchased the
property before she married Feliciano Sr. Finally, petitioners
maintain that they are purchasers in good faith and for value

since the subject property was covered by a tax declaration in


Paz Lachicas name when they bought it from her.
The petition lacks merit.
The property subject matter of the contract of sale
between the Spouses Coja and Paz Lachica, is a 336-square
meter parcel of land covered by Tax Declaration No. 2038. This
includes the property bought by Paz Lachica from the heirs of
Juan Rivas, some other parcels of land, and the 120-square
meter lot purchased by Feliciano Sr. and Lorenza during their
marriage.
Article 160 of the Civil Code provides:
Article 160. All property of the marriage is presumed to
belong to the conjugal partnership, unless it be proved that it
pertains exclusively to the husband or to the wife.
All properties acquired during the marriage are thus disputably
presumed to belong to the conjugal partnership. As a
condition for the operation of above article, in favor of the
conjugal partnership, the party who invokes the presumption
must first prove that the property was acquired during the
marriage.
The CA declared that the 120-square meter lot belonged to the
conjugal partnership of Feliciano Sr. and Lorenza because the
spouses acquired it during the subsistence of their marriage
and the property was in fact declared for taxation purposes
during the said period. Thus, the statutory presumption set
forth in Article 160 of the Civil Code became operative. Having
been acquired during the marital union of Feliciano Sr. and
Lorenza, the subject 120-square meter portion of the property
sold by Paz Lachica to the Spouses Coja is presumed to be
the conjugal property of Feliciano Sr. and Lorenza.
The presumption may be rebutted only with strong, clear,
categorical and convincing evidence. There must be strict
proof of the exclusive ownership of one of the spouses, and
the burden of proof rests upon the party asserting it.
Petitioners insist that the property subject of the sale was
exclusively owned by Paz Lachica having been purchased
prior to her marriage with Feliciano Sr. The argument is not
supported by evidence. While it may be correct to argue that
the 216-square meter portion of the 336-square meter subject
of the sale was exclusively owned by Paz Lachica, the same
cannot be sustained as to the 120-square meter portion
originally covered by Tax Declaration No. 1151. Paz Lachica
claims ownership over the property in question on the basis
only of a tax declaration issued in her name. But that is Tax
Declaration No. 3514 which was belatedly issued in the name
of Paz Lachica to include the 120-square meter lot originally
covered by Tax Declaration No. 1151. Revision was done
upon Paz Lachicas request after the death of Feliciano
Sr. The revision of the tax declaration or the issuance of a new
one in her name, did not operate and transfer title of the
subject property to her. The property remained as one that

22
formed part of the conjugal property of Feliciano Sr. and
Lorenza.

because until division is effected his right over the thing is


represented only by an ideal portion.

Upon the death of Lorenza, the conjugal partnership was


terminated. As a result, one half of the property was
automatically reserved in favor of the surviving spouse,
Feliciano Sr. as his share in the conjugal partnership. The
other half, which is the share of Lorenza, was transmitted to
Lorenzas heirs, Feliciano Jr., Luz, and her husband Feliciano
Sr., who is entitled to the same share as that of a legitimate
child.

As such, the only effect of an action brought by a coowner against a co-owner will be to obtain recognition of the
co-ownership; the defendant cannot be excluded from a
specific portion of the property because as a co-owner he has
a right to possess and the plaintiff cannot recover any material
or determinate part of the property. x x x.

The Court agrees in toto with the CAs conclusion:


x x x. Under Article 996 of the Civil Code, upon the death of
Lorenza Mangarin, one-half of said property, or 60 square
meters, is transmitted to her heirs, namely: Feliciano Aquillo,
Sr., Feliciano Aquillo, Jr., and Luz Aquillo, at 20 square meters
each, while the remaining one-half is transmitted to Feliciano
Aquillo, Sr. Upon the death of Feliciano Aquillo, Sr., his rights
over the property, consisting of the 20 square meterinheritance from his late wife and his 60 square meter-share in
the conjugal partnership, or a total of 80 square meters were
transmitted to his heirs, namely: Feliciano Aquillo, Jr., Luz
Aquillo, and his widow, Paz Lachica. The surviving spouse is
entitled to the same share as that of the legitimate children, to
the portion of one-third each or 26.6666 square meters each x
x x. Thus, as a result of the death of Feliciano Aquillo, Sr., a
regime of co-ownership exists among Feliciano, Jr., Luz
Aquillo, and Paz Lachica, with respect to the undivided 80
square meters of the property covered by Tax Declaration No.
1151.
The 120 square meters less the hereditary share of Paz
Lachica which is 26.6666 square meters, or the 93.3333
square meters of the property covered by Tax Declaration No.
1151, belong to the appellants, being the heirs of the late
Feliciano Aquillo, Jr. and Luz Aquillo. x x x.
Considering that Paz Lachica owns only 26.6666 square
meters of the 120-square meter property and the remaining
93.3333-square meter portion thereof is owned by the
respondents, the former could only validly sell the portion
which rightfully belonged to her. However, considering that Paz
Lachica, the predecessor-in-interest of the Spouses Coja, was
a co-owner of the subject 120-square meter property; and
considering further that partition of the property is wanting, this
Court is precluded from directing the Spouses Coja to return
specific portions of the property to respondents. Noteworthy is
the pronouncement on this issue in De Guia v. Court of
Appeals citing Hermogena G. Engreso with Spouse Jose
Engreso v. Nestoria de la Cruz and Herminio de la Cruz:
It is a basic principle in civil law that before a property owned in
common is actually partitioned, all that the co-owner has is an
ideal or abstract quota or proportionate share in the entire
property. A co-owner has no right to demand a concrete,
specific or determinate part of the thing owned in common

A co-owner may file an action for recovery of possession


against a co-owner who takes exclusive possession of the
entire co-owned property. However, the only effect of such
action is a recognition of the co-ownership. The courts cannot
proceed with the actual partitioning of the co-owned
property. In fine, judicial or extrajudicial partition is necessary
to effect physical division of the subject 120-square meter
property.
WHEREFORE, the petition is PARTIALLY
GRANTED. The Decision of the Court of Appeals dated
February 5, 2001 in CA-G.R. CV No. 37583 is AFFIRMED with
the MODIFICATION that the portion ordering petitioners to
deliver possession to respondents of the property originally
covered by Tax Declaration No. 1151 to the extent of 93.3333
square meters is DELETED. In lieu thereof, the co-ownership
between the parties over the subject 120-square meter
property is recognized, to the extent of undivided shares of
93.3333 square meters for respondents and 26.6666 square
meters for petitioners.
No costs.
SO ORDERED.
SECOND DIVISION
[G.R. No. 124099. October 30, 1997]
MANUEL G. REYES, MILA G. REYES, DANILO G. REYES,
LYN AGAPE, MARITES AGAPE, ESTABANA GALOLO, and
CELSA AGAPE, petitioners, vs. COURT OF APPEALS AND
JULIO VIVARES, respondents.
DECISION
TORRES, JR., J.:
Unless legally flawed, a testators intention in his last will and
testament is its life and soul which deserves reverential
observance.
The controversy before us deals with such a case.
Petitioners Manuel G. Reyes, Mila G. Reyes, Danilo G. Reyes,
Lyn Agape, Marites Agape, Estebana Galolo and Celsa Agape,
the oppositors in Special Proceedings No. 112 for the probate
of the will of Torcuato J. Reyes, assail in this petition for review

23
the decision of the Court of Appeals[1] dated November 29,
1995, the dispositive portion of which reads:
WHEREFORE, premises considered, the judgment appealed
from allowing or admitting the will of Torcuato J. Reyes to
probate and directing the issuance of Letter Testamentary in
favor of petitioner Julio A. Vivares as executor without bond is
AFFIRMED but modified in that the declaration that paragraph
II of the Torcuato Reyes' last will and testament, including
subparagraphs (a) and (b) are null and void for being contrary
to law is hereby SET ASIDE, said paragraphs (a) and (b) are
declared VALID. Except as above modified, the judgment
appealed from is AFFIRMED.
SO ORDERED."[2]
The antecedent facts:
On January 3, 1992, Torcuato J. Reyes executed his last will
and testament declaring therein in part, to wit:
xxx
II. I give and bequeath to my wife Asuncion Oning R. Reyes
the following properties to wit:
a. All my shares of our personal properties consisting among
others of jewelries, coins, antiques, statues, tablewares,
furnitures, fixtures and the building;
b. All my shares consisting of one half (1/2) or 50% of all the
real estates I own in common with my brother Jose, situated in
Municipalities of Mambajao, Mahinog, Guinsiliban, Sagay all in
Camiguin; real estates in Lunao, Ginoong, Caamulan,
Sugbongcogon, Boloc-Boloc, Kinoguinatan, Balingoan, Sta.
Ines, Caesta, Talisayan, all in the province of Misamis
Oriental.[3]
The will consisted of two pages and was signed by Torcuato
Reyes in the presence of three witnesses: Antonio Veloso,
Gloria Borromeo, and Soledad Gaputan. Private respondent
Julio A. Vivares was designated the executor and in his default
or incapacity, his son Roch Alan S. Vivares.
Reyes died on May 12, 1992 and on May 21, 1992, private
respondent filed a petition for probate of the will before the
Regional Trial Court of Mambajao, Camiguin. The petitioner
was set for hearing and the order was published in the
Mindanao Daily Post, a newspaper of general circulation, once
a week for three consecutive weeks. Notices were likewise
sent to all the persons named in the petition.
On July 21, 1992, the recognized natural children of Torcuato
Reyes with Estebana Galolo, namely Manuel, Mila, and Danilo
all surnamed Reyes, and the deceaseds natural children with
Celsa Agape, namely Lyn and Marites Agape, filed an
opposition with the following allegations: a) that the last will
and testament of Reyes was not executed and attested in
accordance with the formalities of law; and b) that Asuncion

Reyes Ebarle exerted undue and improper influence upon the


testator at the time of the execution of the will. The opposition
further averred that Reyes was never married to and could
never marry Asuncion Reyes, the woman he claimed to be his
wife in the will, because the latter was already married to Lupo
Ebarle who was still then alive and their marriage was never
annulled. Thus Asuncion can not be a compulsory heir for her
open cohabitation with Reyes was violative of public morals.
On July 22, 1992, the trial court issued an ordering declaring
that it had acquired jurisdiction over the petition and, therefore,
allowed the presentation of evidence. After the presentation of
evidence and submission of the respective memoranda, the
trial court issued its decision on April 23, 1993.
The trial court declared that the will was executed in
accordance with the formalities prescribed by law. It, however,
ruled that Asuncion Reyes, based on the testimonies of the
witnesses, was never married to the deceased Reyes, and,
therefore, their relationship was an adulterous one. Thus:
The admission in the will by the testator to the illicit
relationship between him and ASUNCION REYES EBARLE
who is somebody elses, wife, is further bolstered,
strengthened, and confirmed by the direct testimonies of the
petitioner himself and his two attesting witnesses during the
trial.
In both cases, the common denominator is the immoral
meretrecious, adulterous and adulterous and illicit relationship
existing between the testator and the devisee prior to the death
of the testator, which constituted the sole and primary
consideration for the devise or legacy, thus making the will
intrinsically invalid.[4]
The will of Reyes was admitted to probate except for
paragraph II (a) and (b) of the will which was declared null and
void for being contrary to law and morals. Hence, Julio Vivares
filed an appeal before the Court of Appeals with the allegation
that the oppositors failed to present any competent evidence
that Asuncion Reyes was legally married to another person
during the period of her cohabitation with Torcuato Reyes.
On November 29, 1995, the Court of Appeals promulgated the
assailed decision which affirmed the trial courts decision
admitting the will for probate but the modification that
paragraph II including subparagraphs (a) and (b) were
declared valid. The appellee court stated:
Considering that the oppositors never showed any competent,
documentary or otherwise during the trial to show that
Asuncion Oning Reyes marriage to the testator was
inexistent or void, either because of a pre-existing marriage or
adulterous relationship, the trial court gravely erred in striking
down paragraph II (a) and (b) of the subject Last Will and
Testament, as void for being contrary to law and morals. Said
declarations are not sufficient to destroy the presumption of
marriage. Nor is it enough to overcome the very declaration of
the testator that Asuncion Reyes is his wife.[5]

24
Dissatisfied with the decision of the Court of Appeals, the
oppositors filed this petition for review.
Petitioners contend that the findings and conclusion of the
Court of Appeals was contrary to law, public policy and
evidence on record. Torcuato Reyes and Asuncion Oning
Reyes were collateral relatives up to the fourth civil
degree. Witness Gloria Borromeo testified that Oning Reyes
was her cousin as her mother and the latters father were sister
and brother. They were also nieces of the late Torcuato
Reyes. Thus, the purported marriage of the deceased Reyes
and Oning Reyes was void ab initio as it was against public
policy pursuant to Article 38 (1) of the Family Code. Petitioners
further alleged that Oning Reyes was already married to Lupo
Ebarle at the time she was cohabiting with the testator hence,
she could never contact any valid marriage with the
latter. Petitioners argued that the testimonies of the witnesses
as well as the personal declaration of the testator, himself,
were sufficient to destroy the presumption of marriage. To
further support their contention, petitioners attached a copy of
the marriage certificate of Asuncion Reyes and Lupo Ebarle.[6]
The petition is devoid of merit.
As a general rule, courts in probate proceedings are limited to
pass only upon the extrinsic validity of the will sought to be
probated.[7] Thus, the court merely inquires on its due
execution, whether or not it complies with the formalities
prescribed by law, and the testamentary capacity of the
testator. It does not determine nor even by implication
prejudge the validity or efficacy of the wills provisions.[8] The
intrinsic validity is not considered since the consideration
thereof usually comes only after the will has been proved and
allowed. There are, however, notable circumstances wherein
the intrinsic validity was first determined as when the defect of
the will is apparent on its face and the probate of the will may
become a useless ceremony if it is intrinsically invalid.[9] The
intrinsic validity of a will may be passed upon because
practical considerations demanded it as when there is
preterition of heirs or the testamentary provisions are doubtful
legality.[10] Where the parties agree that the intrinsic validity
be first determined, the probate court may also do so.
[11] Parenthetically, the rule on probate is not inflexible and
absolute. Under exceptional circumstances, the probate court
is not powerless to do what the situation constrains it to do and
pass upon certain provisions of the will.[12]
The case at bar arose from the institution of the petition for the
probate of the will of the late Torcuato Reyes. Perforce, the
only issues to be settled in the said proceeding were: (1)
whether or not the testator had animus testandi; (2) whether or
not vices of consent attended the execution of the will; and (3)
whether or not the formalities of the will had been complied
with. Thus, the lower court was not asked to rule upon the
intrinsic validity or efficacy of the provisions of the will. As a
result, the declaration of the testator that Asuncion Oning
Reyes was his wife did not have to be scrutinized during the
probate proceedings. The propriety of the institution of Oning

Reyes as one of the devisees/legatees already involved inquiry


on the wills intrinsic validity and which need not be inquired
upon by the probate court.
The lower court erroneously invoked the ruling in Nepomuceno
vs. Court of Appeals (139 SCRA 206) in the instant case. In
the case aforesaid, the testator himself, acknowledged his illicit
relationship with the devisee, to wit:
Art. IV. That since 1952, I have been living, as man and wife,
with one Sofia J. Nepomuceno, whom I declare and avow to be
entitled to my love an [sic] affection, for all the things which she
has done for me, now and in the past; that while Sofia J.
Nepomuceno has with my full knowledge and consent, did
comfort and represent myself as her own husband, in truth and
in fact, as well as in the eyes of the law, I could not bind her to
me in the holy bonds of matrimony because of my
aforementioned previous marriage.
Thus, the very tenor of the will invalidates the legacy because
the testator admitted he was disposing of the properties to a
person with whom he had been living in concubinage.[13] To
remand the case would only be a waste of time and money
since the illegality or defect was already patent. This case is
different from the Nepomuceno case. Testator Torcuato Reyes
merely stated in his will that he was bequeathing some of his
personal and real properties to his wife, Asuncion Oning
Reyes. There was never an open admission of any illicit
relationship. In the case of Nepomuceno, the testator admitted
that he was already previously married and that he had an
adulterous relationship with the devisee.
We agree with the Court of Appeals that the trial court relied on
uncorroborated testimonial evidence that Asuncion Reyes was
still married to another during the time she cohabited with the
testator. The testimonies of the witnesses were merely
hearsay and even uncertain as to the whereabouts or
existence of Lupo Ebarle, the supposed husband of
Asuncion. Thus:
The foregoing testimony cannot go against the declaration of
the testator that Asuncion Oning Reyes is his wife. In
Alvarado v. City Government of Tacloban (supra) the Supreme
Court stated that the declaration of the husband is competent
evidence to show the fact of marriage.
Considering that the oppositors never showed any competent
evidence, documentary or otherwise during the trial to show
that Asuncion Oning Reyes marriage to the testator was
inexistent or void, either because of a pre-existing marriage or
adulterous relationship, the trial court gravely erred in striking
down paragraph II (a) and (b) of the subject Last Will and
Testament, as void for being contrary to law and morals. Said
declarations are not sufficient to destroy the presumption of
marriage. Nor is it enough to overcome the very declaration of
the testator that Asuncion Reyes is his wife.[14]
In the elegant language of Justice Moreland written decades
ago, he said-

25
A will is the testator speaking after death. Its provisions have
substantially the same force and effect in the probate court as
if the testator stood before the court in full life making the
declarations by word of mouth as they appear in the will. That
was the special purpose of the law in the creation of the
instrument known as the last will and testament. Men wished
to speak after they were dead and the law, by the creation of
that instrument, permitted them to do so. xxx All doubts must
be resolved in favor of the testators having meant just what he
said. (Santos vs. Manarang, 27 Phil. 209).
Petitioners tried to refute this conclusion of the Court of
Appeals by presenting belatedly a copy of the marriage
certificate of Asuncion Reyes and Lupo Ebarle. Their failure to
present the said certificate before the probate court to support
their position that Asuncion Reyes had an existing marriage
with Ebarle constituted a waiver and the same evidence can no
longer be entertained on appeal, much less in this petition for
review. This Court would no try the case a new or settle
factual issues since its jurisdiction is confined to resolving
questions of law which have been passed upon by the lower
courts. The settled rule is that the factual findings of the
appellate court will not be disturbed unless shown to be
contrary to the evidence on the record, which petitioners have
not shown in this case.[15]
Considering the foregoing premises, we sustain the findings of
the appellate court it appearing that it did not commit a
reversible error in issuing the challenged decision.
ACCORDINGLY, decision appealed from dated November 29,
1995, is hereby AFFIRMED and the instant petition for review
is DENIED for lack of merit.
SO ORDERED.
Regalado, (Chairman), Romero, Puno, and Mendoza,
JJ., concur.
Republic of the Philippines
SUPREME COURT
Manila
FIRST DIVISION
G.R. No. L-62952 October 9, 1985
SOFIA J. NEPOMUCENO Petitioner, vs. THE HONORABLE
COURT OF APPEALS, RUFINA GOMEZ, OSCAR JUGO ANG,
CARMELITA JUGO, Respondents.chanrobles virtual law library
GUTIERREZ, JR., J.:
This is a petition for certiorari to set aside that portion of the
decision of the respondent Court of Appeals (now intermediate
Appellate Court) dated June 3, 1982, as amended by the
resolution dated August 10, 1982, declaring as null and void
the devise in favor of the petitioner and the resolution dated

December 28, 1982 denying petitioner's motion for


reconsideration.
Martin Jugo died on July 16, 1974 in Malabon, Rizal. He left a
last Will and Testament duly signed by him at the end of the
Will on page three and on the left margin of pages 1, 2 and 4
thereof in the presence of Celestina Alejandro, Myrna C.
Cortez, and Leandro Leano, who in turn, affixed their
signatures below the attestation clause and on the left margin
of pages 1, 2 and 4 of the Will in the presence of the testator
and of each other and the Notary Public. The Will was
acknowledged before the Notary Public Romeo Escareal by
the testator and his three attesting witnesses.
In the said Will, the testator named and appointed herein
petitioner Sofia J. Nepomuceno as his sole and only executor
of his estate. It is clearly stated in the Will that the testator was
legally married to a certain Rufina Gomez by whom he had two
legitimate children, Oscar and Carmelita, but since 1952, he
had been estranged from his lawfully wedded wife and had
been living with petitioner as husband and wife. In fact, on
December 5, 1952, the testator Martin Jugo and the petitioner
herein, Sofia J. Nepomuceno were married in Victoria, Tarlac
before the Justice of the Peace. The testator devised to his
forced heirs, namely, his legal wife Rufina Gomez and his
children Oscar and Carmelita his entire estate and the free
portion thereof to herein petitioner. The Will reads in part:
Art. III. That I have the following legal heirs, namely: my
aforementioned legal wife, Rufina Gomez, and our son, Oscar,
and daughter Carmelita, both surnamed Jugo, whom I declare
and admit to be legally and properly entitled to inherit from me;
that while I have been estranged from my above-named wife
for so many years, I cannot deny that I was legally married to
her or that we have been separated up to the present for
reasons and justifications known fully well by them: chanrobles
virtual law library
Art. IV. That since 1952, 1 have been living, as man and wife
with one Sofia J. Nepomuceno, whom I declare and avow to be
entitled to my love and affection, for all the things which she
has done for me, now and in the past; that while Sofia J.
Nepomuceno has with my full knowledge and consent, did
comport and represent myself as her own husband, in truth
and in fact, as well as in the eyes of the law, I could not bind
her to me in the holy bonds of matrimony because of my
aforementioned previous marriage;
On August 21, 1974, the petitioner filed a petition for the
probate of the last Will and Testament of the deceased Martin
Jugo in the Court of First Instance of Rizal, Branch XXXIV,
Caloocan City and asked for the issuance to her of letters
testamentary.
On May 13, 1975, the legal wife of the testator, Rufina Gomez
and her children filed an opposition alleging inter alia that the
execution of the Will was procured by undue and improper
influence on the part of the petitioner; that at the time of the
execution of the Will, the testator was already very sick and

26
that petitioner having admitted her living in concubinage with
the testator, she is wanting in integrity and thus, letters
testamentary should not be issued to her.
On January 6, 1976, the lower court denied the probate of the
Will on the ground that as the testator admitted in his Will to
cohabiting with the petitioner from December 1952 until his
death on July 16, 1974, the Will's admission to probate will be
an Idle exercise because on the face of the Will, the invalidity
of its intrinsic provisions is evident.
The petitioner appealed to the respondent-appellate court.
On June 2, 1982, the respondent court set aside the decision
of the Court of First Instance of Rizal denying the probate of
the will. The respondent court declared the Will to be valid
except that the devise in favor of the petitioner is null and void
pursuant to Article 739 in relation with Article 1028 of the Civil
Code of the Philippines. The dispositive portion of the decision
reads:
WHEREFORE, the decision a quo is hereby set aside, the will
in question declared valid except the devise in favor of the
appellant which is declared null and void. The properties so
devised are instead passed on in intestacy to the appellant in
equal shares, without pronouncement as to cost.
On June 15, 1982, oppositors Rufina Gomez and her children
filed a "Motion for Correction of Clerical Error" praying that the
word "appellant" in the last sentence of the dispositive portion
of the decision be changed to "appellees" so as to read: "The
properties so devised are instead passed on intestacy to the
appellees in equal shares, without pronouncement as to costs."
The motion was granted by the respondent court on August 10,
1982.
On August 23, 1982, the petitioner filed a motion for
reconsideration. This was denied by the respondent court in a
resolution dated December 28, 1982.
The main issue raised by the petitioner is whether or not the
respondent court acted in excess of its jurisdiction when after
declaring the last Will and Testament of the deceased Martin
Jugo validly drawn, it went on to pass upon the intrinsic validity
of the testamentary provision in favor of herein petitioner.
The petitioner submits that the validity of the testamentary
provision in her favor cannot be passed upon and decided in
the probate proceedings but in some other proceedings
because the only purpose of the probate of a Will is to
establish conclusively as against everyone that a Will was
executed with the formalities required by law and that the
testator has the mental capacity to execute the same. The
petitioner further contends that even if the provisions of
paragraph 1 of Article 739 of the Civil Code of the Philippines
were applicable, the declaration of its nullity could only be
made by the proper court in a separate action brought by the
legal wife for the specific purpose of obtaining a declaration of
the nullity of the testamentary provision in the Will in favor of

the person with whom the testator was allegedly guilty of


adultery or concubinage.
The respondents on the other hand contend that the fact that
the last Will and Testament itself expressly admits indubitably
on its face the meretricious relationship between the testator
and the petitioner and the fact that petitioner herself initiated
the presentation of evidence on her alleged ignorance of the
true civil status of the testator, which led private respondents to
present contrary evidence, merits the application of the
doctrine enunciated in Nuguid v. Felix Nuguid, et al. (17 SCRA
449) and Felix Balanay, Jr. v. Hon. Antonio Martinez, et al.
(G.R. No. L- 39247, June 27, 1975). Respondents also submit
that the admission of the testator of the illicit relationship
between him and the petitioner put in issue the legality of the
devise. We agree with the respondents.
The respondent court acted within its jurisdiction when after
declaring the Will to be validly drawn, it went on to pass upon
the intrinsic validity of the Will and declared the devise in favor
of the petitioner null and void.
The general rule is that in probate proceedings, the court's
area of inquiry is limited to an examination and resolution of
the extrinsic validity of the Will. The rule is expressed thus:
xxx xxx xxx
... It is elementary that a probate decree finally and definitively
settles all questions concerning capacity of the testator and the
proper execution and witnessing of his last Will and testament,
irrespective of whether its provisions are valid and enforceable
or otherwise. (Fernandez v. Dimagiba, 21 SCRA 428)
chanrobles virtual law library
The petition below being for the probate of a Will, the court's
area of inquiry is limited to the extrinsic validity thereof. The
testators testamentary capacity and the compliance with the
formal requisites or solemnities prescribed by law are the only
questions presented for the resolution of the court. Any inquiry
into the intrinsic validity or efficacy of the provisions of the will
or the legality of any devise or legacy is premature.
xxx xxx xxx
True or not, the alleged sale is no ground for the dismissal of
the petition for probate. Probate is one thing; the validity of the
testamentary provisions is another. The first decides the
execution of the document and the testamentary capacity of
the testator; the second relates to descent and distribution
(Sumilang v. Ramagosa, 21 SCRA 1369)
xxx xxx xxx
To establish conclusively as against everyone, and once for all,
the facts that a will was executed with the formalities required
by law and that the testator was in a condition to make a will, is
the only purpose of the proceedings under the new code for
the probate of a will. (Sec. 625). The judgment in such

27
proceedings determines and can determine nothing more. In
them the court has no power to pass upon the validity of any
provisions made in the will. It can not decide, for example, that
a certain legacy is void and another one valid. ... (Castaneda v.
Alemany, 3 Phil. 426)
The rule, however, is not inflexible and absolute. Given
exceptional circumstances, the probate court is not powerless
to do what the situation constrains it to do and pass upon
certain provisions of the Will.
In Nuguid v. Nuguid (17 SCRA 449) cited by the trial court, the
testator instituted the petitioner as universal heir and
completely preterited her surviving forced heirs. A will of this
nature, no matter how valid it may appear extrinsically, would
be null and void. Separate or latter proceedings to determine
the intrinsic validity of the testamentary provisions would be
superfluous.
Even before establishing the formal validity of the will, the
Court in Balanay .Jr. v. Martinez (64 SCRA 452) passed upon
the validity of its intrinsic provisions.
Invoking "practical considerations", we stated:
The basic issue is whether the probate court erred in passing
upon the intrinsic validity of the will, before ruling on its
allowance or formal validity, and in declaring it void.
We are of the opinion that in view of certain unusual
provisions of the will, which are of dubious legality, and
because of the motion to withdraw the petition for probate
(which the lower court assumed to have been filed with the
petitioner's authorization) the trial court acted correctly in
passing upon the will's intrinsic validity even before its formal
validity had been established. The probate of a will might
become an Idle ceremony if on its face it appears to be
intrinsically void. Where practical considerations demand that
the intrinsic validity of the will be passed upon, even before it is
probated, the court should meet the issue (Nuguid v. Nuguid,
64 O.G. 1527, 17 SCRA 449. Compare with Sumilang vs.
Ramagosa L-23135, December 26, 1967, 21 SCRA 1369;
Cacho v. Udan L-19996, April 30, 1965, 13 SCRA 693).
There appears to be no more dispute at this time over the
extrinsic validity of the Will. Both parties are agreed that the
Will of Martin Jugo was executed with all the formalities
required by law and that the testator had the mental capacity to
execute his Will. The petitioner states that she completely
agrees with the respondent court when in resolving the
question of whether or not the probate court correctly denied
the probate of Martin Jugo's last Will and Testament, it ruled:
This being so, the will is declared validly drawn. (Page 4,
Decision, Annex A of Petition.)
On the other hand the respondents pray for the affirmance of
the Court of Appeals' decision in toto.

The only issue, therefore, is the jurisdiction of the respondent


court to declare the testamentary provision in favor of the
petitioner as null and void.
We sustain the respondent court's jurisdiction. As stated in
Nuguid v. Nuguid, (supra):
We pause to reflect. If the case were to be remanded for
probate of the will, nothing will be gained. On the contrary, this
litigation will be protracted. And for aught that appears in the
record, in the record, in the event of probate or if the court
rejects the will, probability exists that the case will come up
once again before us on the same issue of the intrinsic validity
or nullity of the will. Result, waste of time, effort, expense, plus
added anxiety. These are the practical considerations that
induce us to a belief that we might as well meet head-on the
issue of the validity of the provisions of the will in question.
(Section 2, Rule 1, Rules of Court. Case, et al. v. Jugo, et al.,
77 Phil. 517, 522). After all, there exists a justiciable
controversy crying for solution.
We see no useful purpose that would be served if we remand
the nullified provision to the proper court in a separate action
for that purpose simply because, in the probate of a will, the
court does not ordinarily look into the intrinsic validity of its
provisions.
Article 739 of the Civil Code provides:
The following donations shall be void: chanrobles virtual law
library
(1) Those made between persons who were guilty of adultery
or concubinage at the time of the donation;
(2) Those made between persons found guilty of the same
criminal offense, in consideration thereof;
(3) Those made to a public officer or his wife, descendants and
ascendants, by reason of his office.
In the case referred to in No. 1, the action for declaration of
nullity may be brought by the spouse of the donor or donee;
and the guilt of the donor and donee may be proved by
preponderance of evidence in the same action.
Article 1028 of the Civil Code provides:
The prohibitions mentioned in Article 739, concerning
donations inter vivos shall apply to testamentary provisions.
In Article III of the disputed Will, executed on August 15, 1968,
or almost six years before the testator's death on July 16,
1974, Martin Jugo stated that respondent Rufina Gomez was
his legal wife from whom he had been estranged "for so many
years." He also declared that respondents Carmelita Jugo and
Oscar Jugo were his legitimate children. In Article IV, he stated
that he had been living as man and wife with the petitioner
since 1952. Testator Jugo declared that the petitioner was

28
entitled to his love and affection. He stated that Nepomuceno
represented Jugo as her own husband but "in truth and in fact,
as well as in the eyes of the law, I could not bind her to me in
the holy bonds of matrimony because of my aforementioned
previous marriage.
There is no question from the records about the fact of a prior
existing marriage when Martin Jugo executed his Will. There is
also no dispute that the petitioner and Mr. Jugo lived together
in an ostensible marital relationship for 22 years until his death.
It is also a fact that on December 2, 1952, Martin Jugo and
Sofia J. Nepomuceno contracted a marriage before the Justice
of the Peace of Victoria, Tarlac. The man was then 51 years
old while the woman was 48. Nepomuceno now contends that
she acted in good faith for 22 years in the belief that she was
legally married to the testator.
The records do not sustain a finding of innocence or good faith.
As argued by the private respondents:
First. The last will and testament itself expressly admits
indubitably on its face the meretricious relationship between
the testator and petitioner, the devisee.
Second. Petitioner herself initiated the presentation of
evidence on her alleged ignorance of the true civil status of the
testator, which led private respondents to present contrary
evidence.
In short, the parties themselves dueled on the intrinsic validity
of the legacy given in the will to petitioner by the deceased
testator at the start of the proceedings.
Whether or not petitioner knew that testator Martin Jugo, the
man he had lived with as man and wife, as already married,
was an important and specific issue brought by the parties
before the trial court, and passed upon by the Court of
Appeals.
Instead of limiting herself to proving the extrinsic validity of the
will, it was petitioner who opted to present evidence on her
alleged good faith in marrying the testator. (Testimony of
Petitioner, TSN of August 1, 1982, pp. 56-57 and pp. 62-64).
Private respondents, naturally, presented evidence that would
refute the testimony of petitioner on the point.
Sebastian Jugo, younger brother of the deceased testator,
testified at length on the meretricious relationship of his brother
and petitioner. (TSN of August 18,1975). Clearly, the good faith
of petitioner was by option of the parties made a decisive issue
right at the inception of the case.
Confronted by the situation, the trial court had to make a ruling
on the question.
When the court a quo held that the testator Martin Jugo and
petitioner 'were deemed guilty of adultery or concubinage', it

was a finding that petitioner was not the innocent woman she
pretended to be.
xxx xxx xxx
3. If a review of the evidence must be made nonetheless, then
private respondents respectfully offer the following analysis:
FIRST: The secrecy of the marriage of petitioner with the
deceased testator in a town in Tarlac where neither she nor the
testator ever resided. If there was nothing to hide from, why the
concealment' ? Of course, it maybe argued that the marriage of
the deceased with private respondent Rufina Gomez was
likewise done in secrecy. But it should be remembered that
Rufina Gomez was already in the family way at that time and it
would seem that the parents of Martin Jugo were not in favor of
the marriage so much so that an action in court was brought
concerning the marriage. (Testimony of Sebastian Jugo, TSN
of August 18, 1975, pp. 29-30) chanrobles virtual law library
SECOND: Petitioner was a sweetheart of the deceased
testator when they were still both single. That would be in 1922
as Martin Jugo married respondent Rufina Gomez on
November 29, 1923 (Exh. 3). Petitioner married the testator
only on December 5, 1952. There was a space of about 30
years in between. During those 30 years, could it be believed
that she did not even wonder why Martin Jugo did not marry
her nor contact her anymore after November, 1923 - facts that
should impel her to ask her groom before she married him in
secrecy, especially so when she was already about 50 years
old at the time of marriage.
THIRD: The fact that petitioner broke off from Martin Jugo in
1923 is by itself conclusive demonstration that she new that
the man she had openly lived for 22 years as man and wife
was a married man with already two children.
FOURTH: Having admitted that she knew the children of
respondent Rufina Gomez, is it possible that she would not
have asked Martin Jugo whether or not they were his
illegitimate or legitimate children and by whom? That is unFilipino.
FIFTH: Having often gone to Pasig to the residence of the
parents of the deceased testator, is it possible that she would
not have known that the mother of private respondent Oscar
Jugo and Carmelita Jugo was respondent Rufina Gomez,
considering that the houses of the parents of Martin Jugo
(where he had lived for many years) and that of respondent
Rufina Gomez were just a few meters away?
Such pretentions of petitioner Sofia Nepomuceno are
unbelievable. They are, to say the least, inherently improbable,
for they are against the experience in common life and the
ordinary instincts and promptings of human nature that a
woman would not bother at all to ask the man she was going to
marry whether or not he was already married to another,
knowing that her groom had children. It would be a story that
would strain human credulity to the limit if petitioner did not

29
know that Martin Jugo was already a married man in view of
the irrefutable fact that it was precisely his marriage to
respondent Rufina Gomez that led petitioner to break off with
the deceased during their younger years.
Moreover, the prohibition in Article 739 of the Civil Code is
against the making of a donation between persons who are
living in adultery or concubinage. It is the donation which
becomes void. The giver cannot give even assuming that the
recipient may receive. The very wordings of the Will invalidate
the legacy because the testator admitted he was disposing the
properties to a person with whom he had been living in
concubinage.
WHEREFORE, the petition is DISMISSED for lack of merit.
The decision of the Court of Appeals, now Intermediate
Appellate Court, is AFFIRMED. No costs.
SO ORDERED.
Republic of the Philippines
SUPREME COURT
Manila
EN BANC
G.R. No. L-7188

August 9, 1954

In re: Will and Testament of the deceased REVEREND


SANCHO ABADIA.
SEVERINA A. VDA. DE ENRIQUEZ, ET AL., petitionersappellees,
vs.
MIGUEL ABADIA, ET AL., oppositors-appellants.
Manuel A. Zosa, Luis B. Ladonga, Mariano A. Zosa and B. G.
Advincula for appellants.
C. de la Victoria for appellees.
MONTEMAYOR, J.:
On September 6, 1923, Father Sancho Abadia, parish priest of
Talisay, Cebu, executed a document purporting to be his Last
Will and Testament now marked Exhibit "A". Resident of the
City of Cebu, he died on January 14, 1943, in the municipality
of Aloguinsan, Cebu, where he was an evacuee. He left
properties estimated at P8,000 in value. On October 2, 1946,
one Andres Enriquez, one of the legatees in Exhibit "A", filed a
petition for its probate in the Court of First Instance of Cebu.
Some cousins and nephews who would inherit the estate of the
deceased if he left no will, filed opposition.
During the hearing one of the attesting witnesses, the other
two being dead, testified without contradiction that in his
presence and in the presence of his co-witnesses, Father
Sancho wrote out in longhand Exhibit "A" in Spanish which the
testator spoke and understood; that he (testator) signed on he
left hand margin of the front page of each of the three folios or
sheets of which the document is composed, and numbered the

same with Arabic numerals, and finally signed his name at the
end of his writing at the last page, all this, in the presence of
the three attesting witnesses after telling that it was his last will
and that the said three witnesses signed their names on the
last page after the attestation clause in his presence and in the
presence of each other. The oppositors did not submit any
evidence.
The learned trial court found and declared Exhibit "A" to be a
holographic will; that it was in the handwriting of the testator
and that although at the time it was executed and at the time of
the testator's death, holographic wills were not permitted by
law still, because at the time of the hearing and when the case
was to be decided the new Civil Code was already in force,
which Code permitted the execution of holographic wills, under
a liberal view, and to carry out the intention of the testator
which according to the trial court is the controlling factor and
may override any defect in form, said trial court by order dated
January 24, 1952, admitted to probate Exhibit "A", as the Last
Will and Testament of Father Sancho Abadia. The oppositors
are appealing from that decision; and because only questions
of law are involved in the appeal, the case was certified to us
by the Court of Appeals.
The new Civil Code (Republic Act No. 386) under article 810
thereof provides that a person may execute a holographic will
which must be entirely written, dated and signed by the testator
himself and need not be witnessed. It is a fact, however, that at
the time that Exhibit "A" was executed in 1923 and at the time
that Father Abadia died in 1943, holographic wills were not
permitted, and the law at the time imposed certain
requirements for the execution of wills, such as numbering
correlatively each page (not folio or sheet) in letters and
signing on the left hand margin by the testator and by the three
attesting witnesses, requirements which were not complied
with in Exhibit "A" because the back pages of the first two
folios of the will were not signed by any one, not even by the
testator and were not numbered, and as to the three front
pages, they were signed only by the testator.
Interpreting and applying this requirement this Court in the
case of In re Estate of Saguinsin, 41 Phil., 875, 879, referring
to the failure of the testator and his witnesses to sign on the left
hand margin of every page, said:
. . . . This defect is radical and totally vitiates the testament. It
is not enough that the signatures guaranteeing authenticity
should appear upon two folios or leaves; three pages having
been written on, the authenticity of all three of them should be
guaranteed by the signature of the alleged testatrix and her
witnesses.
And in the case of Aspe vs. Prieto, 46 Phil., 700, referring to
the same requirement, this Court declared:
From an examination of the document in question, it appears
that the left margins of the six pages of the document are
signed only by Ventura Prieto. The noncompliance with section
2 of Act No. 2645 by the attesting witnesses who omitted to

30
sign with the testator at the left margin of each of the five
pages of the document alleged to be the will of Ventura Prieto,
is a fatal defect that constitutes an obstacle to its probate.
What is the law to apply to the probate of Exh. "A"? May we
apply the provisions of the new Civil Code which not allows
holographic wills, like Exhibit "A" which provisions were
invoked by the appellee-petitioner and applied by the lower
court? But article 795 of this same new Civil Code expressly
provides: "The validity of a will as to its form depends upon the
observance of the law in force at the time it is made." The
above provision is but an expression or statement of the weight
of authority to the affect that the validity of a will is to be judged
not by the law enforce at the time of the testator's death or at
the time the supposed will is presented in court for probate or
when the petition is decided by the court but at the time the
instrument was executed. One reason in support of the rule is
that although the will operates upon and after the death of the
testator, the wishes of the testator about the disposition of his
estate among his heirs and among the legatees is given
solemn expression at the time the will is executed, and in
reality, the legacy or bequest then becomes a completed act.
This ruling has been laid down by this court in the case of In re
Will of Riosa, 39 Phil., 23. It is a wholesome doctrine and
should be followed.
Of course, there is the view that the intention of the testator
should be the ruling and controlling factor and that all adequate
remedies and interpretations should be resorted to in order to
carry out said intention, and that when statutes passed after
the execution of the will and after the death of the testator
lessen the formalities required by law for the execution of wills,
said subsequent statutes should be applied so as to validate
wills defectively executed according to the law in force at the
time of execution. However, we should not forget that from the
day of the death of the testator, if he leaves a will, the title of
the legatees and devisees under it becomes a vested right,
protected under the due process clause of the constitution
against a subsequent change in the statute adding new legal
requirements of execution of wills which would invalidate such
a will. By parity of reasoning, when one executes a will which is
invalid for failure to observe and follow the legal requirements
at the time of its execution then upon his death he should be
regarded and declared as having died intestate, and his heirs
will then inherit by intestate succession, and no subsequent
law with more liberal requirements or which dispenses with
such requirements as to execution should be allowed to
validate a defective will and thereby divest the heirs of their
vested rights in the estate by intestate succession. The general
rule is that the Legislature can not validate void wills (57 Am.
Jur., Wills, Sec. 231, pp. 192-193).
In view of the foregoing, the order appealed from is reversed,
and Exhibit "A" is denied probate. With costs.
Paras, C.J., Pablo, Bengzon, Padilla, Reyes, A., Jugo, Bautista
Angelo, Labrador, Concepcion and Reyes J.B.L., JJ., concur.

Republic of the Philippines


SUPREME COURT
Manila
SECOND DIVISION
G.R. No. L-46364 April 6, 1990
SULPICIA JIMENEZ and TORIBIO MATIAS, petitioners,
vs.
VICENTE FERNANDEZ alias HOSPICIO FERNANDEZ and
TEODORA GRADO, respondents.
Antonio E. Bengzon III for petitioners.
Agustin U. Cruz for private respondents.

PARAS, J.:
Before Us is a petition for review on certiorari of the following
Decision 1 and Resolution 2 of the Honorable Court of
Appeals: (1) Decision, dated March 1, 1977 in C.A.-G.R. No.
49178-R entitled "Sulpicia Jimenez, et al., v. Vicente
Fernandez, et al." affirming in toto the judgment of the Court of
First Instance of Pangasinan, Third Judicial District in Civil
Case No. 14802-I between the same parties and (2)
Resolution dated June 3, 1977 denying plaintiffs-appellants'
motion for reconsideration.
As gathered from the records, the factual background of this
case is as follows:
The land in question is the Eastern portion with an area of Four
Hundred Thirty Six (436) square meters of that parcel of
residential land situated in Barrio Dulig (now Magsaysay),
Municipality of Labrador, Pangasinan actually covered by
Transfer Certificate of Title No. 82275 (Exhibit A) issued in the
name of Sulpicia Jimenez.
The entire parcel of land with an area of 2,932 square meters,
formerly belonged to Fermin Jimenez. Fermin Jimenez has two
(2) sons named Fortunato and Carlos Jimenez. This Fortunato
Jimenez who predeceased his father has only one child, the
petitioner Sulpicia Jimenez. After the death of Fermin Jimenez,
the entire parcel of land was registered under Act 496 in the
name of Carlos Jimenez and Sulpicia Jimenez (uncle and
niece) in equal shares pro-indiviso. As a result of the
registration case Original Certificate of Title No. 50933 (Exhibit
8) was issued on February 28, 1933, in the names of Carlos
Jimenez and Sulpicia Jimenez, in equal shares pro-indiviso.
Carlos Jimenez died on July 9, 1936 and his illegitimate
daughter, Melecia Cayabyab, also known as Melecia Jimenez,
took possession of the eastern portion of the property
consisting of 436 square meters.

31
On January 20, 1944, Melecia Jimenez sold said 436 square
meter-portion of the property to Edilberto Cagampan and
defendant Teodora Grado executed a contract entitled
"Exchange of Real Properties" whereby the former transferred
said 436 square meter-portion to the latter, who has been in
occupation since.
On August 29, 1969, plaintiff Sulpicia Jimenez executed an
affidavit adjudicating unto herself the other half of the property
appertaining to Carlos Jimenez, upon manifestation that she is
the only heir of her deceased uncle. Consequently Transfer
Certificate of Title No. 82275 was issued on October 1, 1969 in
petitioner's name alone over the entire 2,932 square meter
property.
On April 1, 1970, Sulpicia Jimenez, joined by her husband,
instituted the present action for the recovery of the eastern
portion of the property consisting of 436 square meters
occupied by defendant Teodora Grado and her son.
After trial on the merits, the lower court rendered judgment, the
dispositive portion of which reads:
WHEREFORE, decision is hereby rendered dismissing the
complaint and holding the defendant, Teodora Grado, the
absolute owner of the land in question; ordering the plaintiffs to
pay to the defendant the amount of P500.00 as damages, as
attorney's fees, and to pay the costs of suit.
SO ORDERED. (Rollo, p. 20)
Petitioner appealed the above judgment to the respondent
Court of Appeals and on March 1, 1977, respondent Court of
Appeals rendered a decision affirming the same in toto. Said
decision was rendered by a special division of five (5) justices,
with the Hon. Lourdes San Diego, dissenting.
Petitioners within the reglementary period granted by the
Honorable Court of Appeals, filed therewith a motion for
reconsideration. But said motion for reconsideration was
denied by the Court of Appeals in its resolution dated June 3,
1977.
In their appeal to the respondent Court of Appeals from the
aforequoted decision of the trial court, herein petitioner raised
the following assignments of error to wit:
ASSIGNMENTS OF ERROR
I
THE LOWER COURT ERRED IN NOT DECLARING THAT
MELECIA CAYABYAB, ALSO KNOWN AS MELECIA
JIMENEZ, IS NOT THE DAUGHTER OF CARLOS JIMENEZ.
II
THE LOWER COURT ERRED IN NOT DECLARING THAT
MELECIA CAYABYAB, ALSO KNOWN AS MELECIA

JIMENEZ, HAS NO RIGHT TO SELL THE LAND IN


QUESTION TO EDILBERTO CAGAMPAN.
III
THE LOWER COURT ERRED IN NOT DECLARING THAT
EDILBERTO CAGAMPAN DID NOT BECOME THE OWNER
OF THE LAND IN QUESTION BY VIRTUE OF THE DEED OF
SALE (EXH. "1") EXECUTED BY MELECIA CAYABYAB,
ALIAS MELECIA JIMENEZ, IN HIS FAVOR.
IV
THE LOWER COURT ERRED IN NOT DECLARING THAT
TEODORA GRADO DID NOT BECOME THE OWNER OF
THE LAND IN QUESTION BY VIRTUE OF THE DEED OF
EXCHANGE (EXH. "7") EXECUTED BY HER AND
EDILBERTO CAGAMPAN.
V
THE LOWER COURT ERRED IN NOT DECLARING THAT
THE TITLE OF APPELLANT SULPICIA JIMENEZ OVER THE
LAND IN QUESTION CAN NOT BE DEFEATED BY THE
ADVERSE OPEN AND NOTORIOUS POSSESSION OF
APPELLEE TEODORA GRADO.
VI
THE LOWER COURT ERRED IN DECLARING THAT THE
APPELLEE TEODORA GRADO IS THE ABSOLUTE OWNER
OF THE LAND IN QUESTION IN THE LIGHT OF THE
DECISION OF THE SUPREME COURT IN THE CASE OF
LOURDES ARCUINO, ET AL., V. RUFINA APARIS AND
CASIANO PURAY, G.R. NO. L-23424, PROMULGATED
JANUARY 31, 1968, WHICH CASE IS NOT APPLICABLE TO
THE CASE AT BAR.
VII
THE LOWER COURT ERRED IN DISMISSING THE
COMPLAINT AND ORDERING THE APPELLANTS TO PAY
THE APPELLEES THE SUM OF P500.00 AS ATTORNEYS
FEES PLUS THE COSTS.
From the foregoing, this petition for review was filed.
We find merit in the petition.
From the start the respondent court erred in not declaring that
Melecia Jimenez Cayabyab also known as Melecia Jimenez, is
not the daughter of Carlos Jimenez and therefore, had no right
over the property in question. Respondents failed to present
concrete evidence to prove that Melecia Cayabyab was really
the daughter of Carlos Jimenez. Nonetheless, assuming for the
sake of argument that Melecia Cayabyab was the illegitimate
daughter of Carlos Jimenez there can be no question that
Melecia Cayabyab had no right to succeed to the estate of
Carlos Jimenez and could not have validly acquired, nor legally

32
transferred to Edilberto Cagampan that portion of the property
subject of this petition.
It is well-settled in this jurisdiction that the rights to the
succession are transmitted from the moment of the death of
the decedent (Art. 777, Civil Code). Moreover, Art. 2263 of the
Civil Code provides as follows:
Rights to the inheritance of a person who died with or without a
will, before the effectivity of this Code, shall be governed by the
Civil Code of 1889, by other previous laws, and by the Rules of
Court . . . (Rollo, p. 17)
Thus, since Carlos Jimenez, owner of one-half pro-indiviso
portion of that parcel of land then covered by Original
Certificate of title No. 50933, died on July 9, 1936 (Exhibit "F")
way before the effectivity of the Civil Code of the Philippines,
the successional rights pertaining to his estate must be
determined in accordance with the Civil Code of 1889.
Citing the case of Cid v. Burnaman (24 SCRA 434) wherein
this Court categorically held that:
To be an heir under the rules of Civil Code of 1889 (which was
the law in force when Carlos Jimenez died and which should
be the governing law in so far as the right to inherit from his
estate was concerned), a child must be either a child
legitimate, legitimated, or adopted, or else an acknowledged
natural child for illegitimate not natural are disqualified to
inherit. (Civil Code of 1889, Art. 807, 935)
Even assuming that Melecia Cayabyab was born out of the
common-law-relationship between her mother (Maria
Cayabyab) and Carlos Jimenez, she could not even be
considered an acknowledged natural child because Carlos
Jimenez was then legally married to Susana Abalos and
therefore not qualified to marry Maria Cayabyab and
consequently Melecia Cayabyab was an illegitimate spurious
child and not entitled to any successional rights in so far as the
estate of Carlos Jimenez was concerned.
Melecia Cayabyab in the absence of any voluntary conveyance
to her by Carlos Jimenez or Sulpicia Jimenez of the litigated
portion of the land could not even legally transfer the parcel of
land to Edilberto Cagampan who accordingly, could not also
legally transfer the same to herein private respondents.
Analyzing the case before Us in this manner, We can
immediately discern another error in the decision of the
respondent court, which is that the said court sustained and
made applicable to the case at bar the ruling in the case of
Arcuino, et al., v. Aparis and Puray, No. L-23424, January 31,
1968, 22 SCRA 407, wherein We held that:
. . . it is true that the lands registered under the Torrens System
may not be acquired by prescription but plaintiffs herein are not
the registered owners. They merely claim to have acquired by
succession, their alleged title or interest in lot No. 355. At any
rate plaintiffs herein are guilty of laches.

The respondent court relying on the Arcuino case, concluded


that respondents had acquired the property under litigation by
prescription. We cannot agree with such conclusion, because
there is one very marked and important difference between the
case at bar and that of the Arcuino case, and that is, that since
1933 petitioner Sulpicia Jimenez was a title holder, the
property then being registered in her and her uncle Carlos
Jimenez' name. In the Arcuino case, this Supreme Court held.
"(I)t is true that lands registered under the Torrens System may
not be acquired by prescription but plaintiffs herein are not the
registered owners." (Rollo, p. 38) Even in the said cited case
the principle of imprescriptibility of Torrens Titles was
respected.
Melecia Cayabyab's possession or of her predecessors-ininterest would be unavailing against the petitioner Sulpicia
Jimenez who was the holder pro-indiviso with Carlos Jimenez
of the Torrens Certificate of Title covering a tract of land which
includes the portion now in question, from February 28, 1933,
when the Original Certificate of Title No. 50933 (Exhibit 8) was
issued.
No possession by any person of any portion of the land
covered by said original certificate of titles, could defeat the
title of the registered owner of the land covered by the
certificate of title. (Benin v. Tuason, L-26127, June 28, 1974,
57 SCRA 531)
Sulpicia's title over her one-half undivided property remained
good and continued to be good when she segregated it into a
new title (T.C.T No. 82275, Exhibit "A") in 1969. Sulpicia's
ownership over her one-half of the land and which is the land
in dispute was always covered by a Torrens title, and therefore,
no amount of possession thereof by the respondents, could
ever defeat her proprietary rights thereon. It is apparent, that
the right of plaintiff (now petitioner) to institute this action to
recover possession of the portion of the land in question based
on the Torrens Title of Sulpicia Jimenez, T.C.T. No. 82275
(Exhibit "A") is imprescriptible and not barred under the
doctrine of laches. (J.M. Tuason & Co. v. Macalindong, L15398, December 29, 1962, Francisco v. Cruz, et al., 43 O.G.
5105) Rollo, p. 39)
The respondent Court of Appeals declared the petitioner
Sulpicia Jimenez guilty of laches and citing the ruling in the
case of Heirs of Lacamen v. Heirs of Laruan (65 SCRA 605),
held that, since petitioner Sulpicia Jimenez executed her
Affidavit of Self-Adjudication only in 1969, she lost the right to
recover possession of the parcel of land subject of the
litigation.
In this instance, again We rule for the petitioner. There is no
absolute rule as to what constitutes laches or staleness of
demand; each case is to be determined according to its
particular circumstances. The question of laches is addressed
to the sound discretion of the court and since laches is an
equitable doctrine, its application is controlled by equitable
considerations. It cannot be worked to defeat justice or to

33
perpetrate fraud and injustice. It would be rank injustice and
patently inequitous to deprive the lawful heirs of their rightful
inheritance.
Petitioner Sulpicia Jimenez is entitled to the relief prayed for,
declaring her to be the sole and absolute owner of the land in
question with right to its possession and enjoyment. Since her
uncle Carlos Jimenez died in 1936, his pro-indiviso share in
the properties then owned in co-ownership with his niece
Sulpicia descended by intestacy to Sulpicia Jimenez alone
because Carlos died without any issue or other heirs.
After all, the professed objective of Act No. 496, otherwise
known as the Land Registration Act or the law which
established the Torrens System of Land Registration in the
Philippines is that the stability of the landholding system in the
Philippines depends on the confidence of the people in the
titles covering the properties. And to this end, this Court has
invariably upheld the indefeasibility of the Torrens Title and in,
among others, J.M. Tuason and Co., Inc. v. Macalindong (6
SCRA 938), held that "the right of the appellee to file an action
to recover possession based on its Torrens Title is
imprescriptible and not barred under the doctrine of laches.
WHEREFORE, the Petition for Review is hereby GRANTED.
The Decision and Resolution dated March 1, 1977 and June 3,
1977 in CA G.R. No. L-49178-R are SET ASIDE.
SO ORDERED
Republic of the Philippines
SUPREME COURT
Manila
EN BANC
G.R. No. L-16749

January 31, 1963

IN THE MATTER OF THE TESTATE ESTATE OF EDWARD E.


CHRISTENSEN, DECEASED.
ADOLFO C. AZNAR, Executor and LUCY CHRISTENSEN,
Heir of the deceased, Executor and Heir-appellees,
vs.
HELEN CHRISTENSEN GARCIA, oppositor-appellant.
M. R. Sotelo for executor and heir-appellees.
Leopoldo M. Abellera and Jovito Salonga for oppositorappellant.
LABRADOR, J.:
This is an appeal from a decision of the Court of First Instance
of Davao, Hon. Vicente N. Cusi, Jr., presiding, in Special
Proceeding No. 622 of said court, dated September 14, 1949,
approving among things the final accounts of the executor,
directing the executor to reimburse Maria Lucy Christensen the
amount of P3,600 paid by her to Helen Christensen Garcia as
her legacy, and declaring Maria Lucy Christensen entitled to
the residue of the property to be enjoyed during her lifetime,

and in case of death without issue, one-half of said residue to


be payable to Mrs. Carrie Louise C. Borton, etc., in accordance
with the provisions of the will of the testator Edward E.
Christensen. The will was executed in Manila on March 5,
1951 and contains the following provisions:
3. I declare ... that I have but ONE (1) child, named MARIA
LUCY CHRISTENSEN (now Mrs. Bernard Daney), who was
born in the Philippines about twenty-eight years ago, and who
is now residing at No. 665 Rodger Young Village, Los Angeles,
California, U.S.A.
4. I further declare that I now have no living ascendants, and
no descendants except my above named daughter, MARIA
LUCY CHRISTENSEN DANEY.
xxx

xxx

xxx

7. I give, devise and bequeath unto MARIA HELEN


CHRISTENSEN, now married to Eduardo Garcia, about
eighteen years of age and who, notwithstanding the fact that
she was baptized Christensen, is not in any way related to me,
nor has she been at any time adopted by me, and who, from all
information I have now resides in Egpit, Digos, Davao,
Philippines, the sum of THREE THOUSAND SIX HUNDRED
PESOS (P3,600.00), Philippine Currency the same to be
deposited in trust for the said Maria Helen Christensen with the
Davao Branch of the Philippine National Bank, and paid to her
at the rate of One Hundred Pesos (P100.00), Philippine
Currency per month until the principal thereof as well as any
interest which may have accrued thereon, is exhausted..
xxx

xxx

xxx

12. I hereby give, devise and bequeath, unto my well-beloved


daughter, the said MARIA LUCY CHRISTENSEN DANEY (Mrs.
Bernard Daney), now residing as aforesaid at No. 665 Rodger
Young Village, Los Angeles, California, U.S.A., all the income
from the rest, remainder, and residue of my property and
estate, real, personal and/or mixed, of whatsoever kind or
character, and wheresoever situated, of which I may be
possessed at my death and which may have come to me from
any source whatsoever, during her lifetime: ....
It is in accordance with the above-quoted provisions that the
executor in his final account and project of partition ratified the
payment of only P3,600 to Helen Christensen Garcia and
proposed that the residue of the estate be transferred to his
daughter, Maria Lucy Christensen.
Opposition to the approval of the project of partition was filed
by Helen Christensen Garcia, insofar as it deprives her (Helen)
of her legitime as an acknowledged natural child, she having
been declared by Us in G.R. Nos. L-11483-84 an
acknowledged natural child of the deceased Edward E.
Christensen. The legal grounds of opposition are (a) that the
distribution should be governed by the laws of the Philippines,
and (b) that said order of distribution is contrary thereto insofar
as it denies to Helen Christensen, one of two acknowledged

34
natural children, one-half of the estate in full ownership. In
amplification of the above grounds it was alleged that the law
that should govern the estate of the deceased Christensen
should not be the internal law of California alone, but the entire
law thereof because several foreign elements are involved,
that the forum is the Philippines and even if the case were
decided in California, Section 946 of the California Civil Code,
which requires that the domicile of the decedent should apply,
should be applicable. It was also alleged that Maria Helen
Christensen having been declared an acknowledged natural
child of the decedent, she is deemed for all purposes legitimate
from the time of her birth.
The court below ruled that as Edward E. Christensen was a
citizen of the United States and of the State of California at the
time of his death, the successional rights and intrinsic validity
of the provisions in his will are to be governed by the law of
California, in accordance with which a testator has the right to
dispose of his property in the way he desires, because the right
of absolute dominion over his property is sacred and inviolable
(In re McDaniel's Estate, 77 Cal. Appl. 2d 877, 176 P. 2d 952,
and In re Kaufman, 117 Cal. 286, 49 Pac. 192, cited in page
179, Record on Appeal). Oppositor Maria Helen Christensen,
through counsel, filed various motions for reconsideration, but
these were denied. Hence, this appeal.
The most important assignments of error are as follows:
I
THE LOWER COURT ERRED IN IGNORING THE DECISION
OF THE HONORABLE SUPREME COURT THAT HELEN IS
THE ACKNOWLEDGED NATURAL CHILD OF EDWARD E.
CHRISTENSEN AND, CONSEQUENTLY, IN DEPRIVING HER
OF HER JUST SHARE IN THE INHERITANCE.
II
THE LOWER COURT ERRED IN ENTIRELY IGNORING
AND/OR FAILING TO RECOGNIZE THE EXISTENCE OF
SEVERAL FACTORS, ELEMENTS AND CIRCUMSTANCES
CALLING FOR THE APPLICATION OF INTERNAL LAW.
III
THE LOWER COURT ERRED IN FAILING TO RECOGNIZE
THAT UNDER INTERNATIONAL LAW, PARTICULARLY
UNDER THE RENVOI DOCTRINE, THE INTRINSIC VALIDITY
OF THE TESTAMENTARY DISPOSITION OF THE
DISTRIBUTION OF THE ESTATE OF THE DECEASED
EDWARD E. CHRISTENSEN SHOULD BE GOVERNED BY
THE LAWS OF THE PHILIPPINES.
IV
THE LOWER COURT ERRED IN NOT DECLARING THAT
THE SCHEDULE OF DISTRIBUTION SUBMITTED BY THE
EXECUTOR IS CONTRARY TO THE PHILIPPINE LAWS.

V
THE LOWER COURT ERRED IN NOT DECLARING THAT
UNDER THE PHILIPPINE LAWS HELEN CHRISTENSEN
GARCIA IS ENTITLED TO ONE-HALF (1/2) OF THE ESTATE
IN FULL OWNERSHIP.
There is no question that Edward E. Christensen was a citizen
of the United States and of the State of California at the time of
his death. But there is also no question that at the time of his
death he was domiciled in the Philippines, as witness the
following facts admitted by the executor himself in appellee's
brief:
In the proceedings for admission of the will to probate, the
facts of record show that the deceased Edward E. Christensen
was born on November 29, 1875 in New York City, N.Y.,
U.S.A.; his first arrival in the Philippines, as an appointed
school teacher, was on July 1, 1901, on board the U.S. Army
Transport "Sheridan" with Port of Embarkation as the City of
San Francisco, in the State of California, U.S.A. He stayed in
the Philippines until 1904.
In December, 1904, Mr. Christensen returned to the United
States and stayed there for the following nine years until 1913,
during which time he resided in, and was teaching school in
Sacramento, California.
Mr. Christensen's next arrival in the Philippines was in July of
the year 1913. However, in 1928, he again departed the
Philippines for the United States and came back here the
following year, 1929. Some nine years later, in 1938, he again
returned to his own country, and came back to the Philippines
the following year, 1939.
Wherefore, the parties respectfully pray that the foregoing
stipulation of facts be admitted and approved by this Honorable
Court, without prejudice to the parties adducing other evidence
to prove their case not covered by this stipulation of facts.
1wph1.t
Being an American citizen, Mr. Christensen was interned by
the Japanese Military Forces in the Philippines during World
War II. Upon liberation, in April 1945, he left for the United
States but returned to the Philippines in December, 1945.
Appellees Collective Exhibits "6", CFI Davao, Sp. Proc. 622, as
Exhibits "AA", "BB" and "CC-Daney"; Exhs. "MM", "MM-l",
"MM-2-Daney" and p. 473, t.s.n., July 21, 1953.)
In April, 1951, Edward E. Christensen returned once more to
California shortly after the making of his last will and testament
(now in question herein) which he executed at his lawyers'
offices in Manila on March 5, 1951. He died at the St. Luke's
Hospital in the City of Manila on April 30, 1953. (pp. 2-3)
In arriving at the conclusion that the domicile of the deceased
is the Philippines, we are persuaded by the fact that he was
born in New York, migrated to California and resided there for
nine years, and since he came to the Philippines in 1913 he

35
returned to California very rarely and only for short visits
(perhaps to relatives), and considering that he appears never
to have owned or acquired a home or properties in that state,
which would indicate that he would ultimately abandon the
Philippines and make home in the State of California.
Sec. 16. Residence is a term used with many shades of
meaning from mere temporary presence to the most
permanent abode. Generally, however, it is used to denote
something more than mere physical presence. (Goodrich on
Conflict of Laws, p. 29)
As to his citizenship, however, We find that the citizenship that
he acquired in California when he resided in Sacramento,
California from 1904 to 1913, was never lost by his stay in the
Philippines, for the latter was a territory of the United States
(not a state) until 1946 and the deceased appears to have
considered himself as a citizen of California by the fact that
when he executed his will in 1951 he declared that he was a
citizen of that State; so that he appears never to have intended
to abandon his California citizenship by acquiring another. This
conclusion is in accordance with the following principle
expounded by Goodrich in his Conflict of Laws.
The terms "'residence" and "domicile" might well be taken to
mean the same thing, a place of permanent abode. But
domicile, as has been shown, has acquired a technical
meaning. Thus one may be domiciled in a place where he has
never been. And he may reside in a place where he has no
domicile. The man with two homes, between which he divides
his time, certainly resides in each one, while living in it. But if
he went on business which would require his presence for
several weeks or months, he might properly be said to have
sufficient connection with the place to be called a resident. It is
clear, however, that, if he treated his settlement as continuing
only for the particular business in hand, not giving up his
former "home," he could not be a domiciled New Yorker.
Acquisition of a domicile of choice requires the exercise of
intention as well as physical presence. "Residence simply
requires bodily presence of an inhabitant in a given place,
while domicile requires bodily presence in that place and also
an intention to make it one's domicile." Residence, however, is
a term used with many shades of meaning, from the merest
temporary presence to the most permanent abode, and it is not
safe to insist that any one use et the only proper one.
(Goodrich, p. 29)
The law that governs the validity of his testamentary
dispositions is defined in Article 16 of the Civil Code of the
Philippines, which is as follows:

the nature of the property and regardless of the country where


said property may be found.
The application of this article in the case at bar requires the
determination of the meaning of the term "national law" is used
therein.
There is no single American law governing the validity of
testamentary provisions in the United States, each state of the
Union having its own private law applicable to its citizens only
and in force only within the state. The "national law" indicated
in Article 16 of the Civil Code above quoted can not, therefore,
possibly mean or apply to any general American law. So it can
refer to no other than the private law of the State of California.
The next question is: What is the law in California governing
the disposition of personal property? The decision of the court
below, sustains the contention of the executor-appellee that
under the California Probate Code, a testator may dispose of
his property by will in the form and manner he desires, citing
the case of Estate of McDaniel, 77 Cal. Appl. 2d 877, 176 P. 2d
952. But appellant invokes the provisions of Article 946 of the
Civil Code of California, which is as follows:
If there is no law to the contrary, in the place where personal
property is situated, it is deemed to follow the person of its
owner, and is governed by the law of his domicile.
The existence of this provision is alleged in appellant's
opposition and is not denied. We have checked it in the
California Civil Code and it is there. Appellee, on the other
hand, relies on the case cited in the decision and testified to by
a witness. (Only the case of Kaufman is correctly cited.) It is
argued on executor's behalf that as the deceased Christensen
was a citizen of the State of California, the internal law thereof,
which is that given in the abovecited case, should govern the
determination of the validity of the testamentary provisions of
Christensen's will, such law being in force in the State of
California of which Christensen was a citizen. Appellant, on the
other hand, insists that Article 946 should be applicable, and in
accordance therewith and following the doctrine of the renvoi,
the question of the validity of the testamentary provision in
question should be referred back to the law of the decedent's
domicile, which is the Philippines.
The theory of doctrine of renvoi has been defined by various
authors, thus:

ART. 16. Real property as well as personal property is subject


to the law of the country where it is situated.

The problem has been stated in this way: "When the Conflict of
Laws rule of the forum refers a jural matter to a foreign law for
decision, is the reference to the purely internal rules of law of
the foreign system; i.e., to the totality of the foreign law minus
its Conflict of Laws rules?"

However, intestate and testamentary successions, both with


respect to the order of succession and to the amount of
successional rights and to the intrinsic validity of testamentary
provisions, shall be regulated by the national law of the person
whose succession is under consideration, whatever may be

On logic, the solution is not an easy one. The Michigan court


chose to accept the renvoi, that is, applied the Conflict of Laws
rule of Illinois which referred the matter back to Michigan law.
But once having determined the the Conflict of Laws principle
is the rule looked to, it is difficult to see why the reference back

36
should not have been to Michigan Conflict of Laws. This would
have resulted in the "endless chain of references" which has
so often been criticized be legal writers. The opponents of the
renvoi would have looked merely to the internal law of Illinois,
thus rejecting the renvoi or the reference back. Yet there
seems no compelling logical reason why the original reference
should be the internal law rather than to the Conflict of Laws
rule. It is true that such a solution avoids going on a merry-goround, but those who have accepted the renvoi theory avoid
this inextricabilis circulas by getting off at the second reference
and at that point applying internal law. Perhaps the opponents
of the renvoi are a bit more consistent for they look always to
internal law as the rule of reference.
Strangely enough, both the advocates for and the objectors to
the renvoi plead that greater uniformity will result from adoption
of their respective views. And still more strange is the fact that
the only way to achieve uniformity in this choice-of-law problem
is if in the dispute the two states whose laws form the legal
basis of the litigation disagree as to whether the renvoi should
be accepted. If both reject, or both accept the doctrine, the
result of the litigation will vary with the choice of the forum. In
the case stated above, had the Michigan court rejected the
renvoi, judgment would have been against the woman; if the
suit had been brought in the Illinois courts, and they too
rejected the renvoi, judgment would be for the woman. The
same result would happen, though the courts would switch with
respect to which would hold liability, if both courts accepted the
renvoi.
The Restatement accepts the renvoi theory in two instances:
where the title to land is in question, and where the validity of a
decree of divorce is challenged. In these cases the Conflict of
Laws rule of the situs of the land, or the domicile of the parties
in the divorce case, is applied by the forum, but any further
reference goes only to the internal law. Thus, a person's title to
land, recognized by the situs, will be recognized by every
court; and every divorce, valid by the domicile of the parties,
will be valid everywhere. (Goodrich, Conflict of Laws, Sec. 7,
pp. 13-14.)
X, a citizen of Massachusetts, dies intestate, domiciled in
France, leaving movable property in Massachusetts, England,
and France. The question arises as to how this property is to
be distributed among X's next of kin.
Assume (1) that this question arises in a Massachusetts court.
There the rule of the conflict of laws as to intestate succession
to movables calls for an application of the law of the
deceased's last domicile. Since by hypothesis X's last domicile
was France, the natural thing for the Massachusetts court to do
would be to turn to French statute of distributions, or whatever
corresponds thereto in French law, and decree a distribution
accordingly. An examination of French law, however, would
show that if a French court were called upon to determine how
this property should be distributed, it would refer the
distribution to the national law of the deceased, thus applying
the Massachusetts statute of distributions. So on the surface of

things the Massachusetts court has open to it alternative


course of action: (a) either to apply the French law is to
intestate succession, or (b) to resolve itself into a French court
and apply the Massachusetts statute of distributions, on the
assumption that this is what a French court would do. If it
accepts the so-called renvoi doctrine, it will follow the latter
course, thus applying its own law.
This is one type of renvoi. A jural matter is presented which the
conflict-of-laws rule of the forum refers to a foreign law, the
conflict-of-laws rule of which, in turn, refers the matter back
again to the law of the forum. This is renvoi in the narrower
sense. The German term for this judicial process is
'Ruckverweisung.'" (Harvard Law Review, Vol. 31, pp. 523571.)
After a decision has been arrived at that a foreign law is to be
resorted to as governing a particular case, the further question
may arise: Are the rules as to the conflict of laws contained in
such foreign law also to be resorted to? This is a question
which, while it has been considered by the courts in but a few
instances, has been the subject of frequent discussion by
textwriters and essayists; and the doctrine involved has been
descriptively designated by them as the "Renvoyer" to send
back, or the "Ruchversweisung", or the "Weiterverweisung",
since an affirmative answer to the question postulated and the
operation of the adoption of the foreign law in toto would in
many cases result in returning the main controversy to be
decided according to the law of the forum. ... (16 C.J.S. 872.)
Another theory, known as the "doctrine of renvoi", has been
advanced. The theory of the doctrine of renvoi is that the court
of the forum, in determining the question before it, must take
into account the whole law of the other jurisdiction, but also its
rules as to conflict of laws, and then apply the law to the actual
question which the rules of the other jurisdiction prescribe. This
may be the law of the forum. The doctrine of the renvoi has
generally been repudiated by the American authorities. (2 Am.
Jur. 296)
The scope of the theory of renvoi has also been defined and
the reasons for its application in a country explained by Prof.
Lorenzen in an article in the Yale Law Journal, Vol. 27, 19171918, pp. 529-531. The pertinent parts of the article are quoted
herein below:
The recognition of the renvoi theory implies that the rules of the
conflict of laws are to be understood as incorporating not only
the ordinary or internal law of the foreign state or country, but
its rules of the conflict of laws as well. According to this theory
'the law of a country' means the whole of its law.
xxx

xxx

xxx

Von Bar presented his views at the meeting of the Institute of


International Law, at Neuchatel, in 1900, in the form of the
following theses:

37
(1) Every court shall observe the law of its country as regards
the application of foreign laws.
(2) Provided that no express provision to the contrary exists,
the court shall respect:
(a) The provisions of a foreign law which disclaims the right to
bind its nationals abroad as regards their personal statute, and
desires that said personal statute shall be determined by the
law of the domicile, or even by the law of the place where the
act in question occurred.
(b) The decision of two or more foreign systems of law,
provided it be certain that one of them is necessarily
competent, which agree in attributing the determination of a
question to the same system of law.
xxx

xxx

xxx

If, for example, the English law directs its judge to distribute the
personal estate of an Englishman who has died domiciled in
Belgium in accordance with the law of his domicile, he must
first inquire whether the law of Belgium would distribute
personal property upon death in accordance with the law of
domicile, and if he finds that the Belgian law would make the
distribution in accordance with the law of nationality that is
the English law he must accept this reference back to his
own law.
We note that Article 946 of the California Civil Code is its
conflict of laws rule, while the rule applied in In re Kaufman,
Supra, its internal law. If the law on succession and the conflict
of laws rules of California are to be enforced jointly, each in its
own intended and appropriate sphere, the principle cited In re
Kaufman should apply to citizens living in the State, but Article
946 should apply to such of its citizens as are not domiciled in
California but in other jurisdictions. The rule laid down of
resorting to the law of the domicile in the determination of
matters with foreign element involved is in accord with the
general principle of American law that the domiciliary law
should govern in most matters or rights which follow the person
of the owner.
When a man dies leaving personal property in one or more
states, and leaves a will directing the manner of distribution of
the property, the law of the state where he was domiciled at the
time of his death will be looked to in deciding legal questions
about the will, almost as completely as the law of situs is
consulted in questions about the devise of land. It is logical
that, since the domiciliary rules control devolution of the
personal estate in case of intestate succession, the same rules
should determine the validity of an attempted testamentary
dispostion of the property. Here, also, it is not that the
domiciliary has effect beyond the borders of the domiciliary
state. The rules of the domicile are recognized as controlling
by the Conflict of Laws rules at the situs property, and the
reason for the recognition as in the case of intestate
succession, is the general convenience of the doctrine. The
New York court has said on the point: 'The general principle

that a dispostiton of a personal property, valid at the domicile of


the owner, is valid anywhere, is one of the universal
application. It had its origin in that international comity which
was one of the first fruits of civilization, and it this age, when
business intercourse and the process of accumulating property
take but little notice of boundary lines, the practical wisdom
and justice of the rule is more apparent than ever. (Goodrich,
Conflict of Laws, Sec. 164, pp. 442-443.)
Appellees argue that what Article 16 of the Civil Code of the
Philippines pointed out as the national law is the internal law of
California. But as above explained the laws of California have
prescribed two sets of laws for its citizens, one for residents
therein and another for those domiciled in other jurisdictions.
Reason demands that We should enforce the California
internal law prescribed for its citizens residing therein, and
enforce the conflict of laws rules for the citizens domiciled
abroad. If we must enforce the law of California as in comity
we are bound to go, as so declared in Article 16 of our Civil
Code, then we must enforce the law of California in
accordance with the express mandate thereof and as above
explained, i.e., apply the internal law for residents therein, and
its conflict-of-laws rule for those domiciled abroad.
It is argued on appellees' behalf that the clause "if there is no
law to the contrary in the place where the property is situated"
in Sec. 946 of the California Civil Code refers to Article 16 of
the Civil Code of the Philippines and that the law to the
contrary in the Philippines is the provision in said Article 16 that
the national law of the deceased should govern. This
contention can not be sustained. As explained in the various
authorities cited above the national law mentioned in Article 16
of our Civil Code is the law on conflict of laws in the California
Civil Code, i.e., Article 946, which authorizes the reference or
return of the question to the law of the testator's domicile. The
conflict of laws rule in California, Article 946, Civil Code,
precisely refers back the case, when a decedent is not
domiciled in California, to the law of his domicile, the
Philippines in the case at bar. The court of the domicile can not
and should not refer the case back to California; such action
would leave the issue incapable of determination because the
case will then be like a football, tossed back and forth between
the two states, between the country of which the decedent was
a citizen and the country of his domicile. The Philippine court
must apply its own law as directed in the conflict of laws rule of
the state of the decedent, if the question has to be decided,
especially as the application of the internal law of California
provides no legitime for children while the Philippine law, Arts.
887(4) and 894, Civil Code of the Philippines, makes natural
children legally acknowledged forced heirs of the parent
recognizing them.
The Philippine cases (In re Estate of Johnson, 39 Phil. 156;
Riera vs. Palmaroli, 40 Phil. 105; Miciano vs. Brimo, 50 Phil.
867; Babcock Templeton vs. Rider Babcock, 52 Phil. 130; and
Gibbs vs. Government, 59 Phil. 293.) cited by appellees to
support the decision can not possibly apply in the case at bar,
for two important reasons, i.e., the subject in each case does

38
not appear to be a citizen of a state in the United States but
with domicile in the Philippines, and it does not appear in each
case that there exists in the state of which the subject is a
citizen, a law similar to or identical with Art. 946 of the
California Civil Code.
We therefore find that as the domicile of the deceased
Christensen, a citizen of California, is the Philippines, the
validity of the provisions of his will depriving his acknowledged
natural child, the appellant, should be governed by the
Philippine Law, the domicile, pursuant to Art. 946 of the Civil
Code of California, not by the internal law of California..
WHEREFORE, the decision appealed from is hereby reversed
and the case returned to the lower court with instructions that
the partition be made as the Philippine law on succession
provides. Judgment reversed, with costs against appellees.

Republic of the Philippines


SUPREME COURT
Manila
EN BANC
G.R. No. L-22595

November 1, 1927

Testate Estate of Joseph G. Brimo, JUAN MICIANO,


administrator, petitioner-appellee,
vs.
ANDRE BRIMO, opponent-appellant.
Ross, Lawrence and Selph for appellant.
Camus and Delgado for appellee.

ROMUALDEZ, J.:
The partition of the estate left by the deceased Joseph
G. Brimo is in question in this case.
The judicial administrator of this estate filed a scheme of
partition. Andre Brimo, one of the brothers of the deceased,
opposed it. The court, however, approved it.
The errors which the oppositor-appellant assigns are:
(1) The approval of said scheme of partition; (2) denial of
his participation in the inheritance; (3) the denial of the motion
for reconsideration of the order approving the partition; (4) the
approval of the purchase made by the Pietro Lana of the
deceased's business and the deed of transfer of said business;
and (5) the declaration that the Turkish laws are impertinent to
this cause, and the failure not to postpone the approval of the
scheme of partition and the delivery of the deceased's
business to Pietro Lanza until the receipt of the depositions
requested in reference to the Turkish laws.

The appellant's opposition is based on the fact that the


partition in question puts into effect the provisions of Joseph G.
Brimo's will which are not in accordance with the laws of his
Turkish nationality, for which reason they are void as being in
violation or article 10 of the Civil Code which, among other
things, provides the following:
Nevertheless, legal and testamentary successions, in
respect to the order of succession as well as to the amount of
the successional rights and the intrinsic validity of their
provisions, shall be regulated by the national law of the person
whose succession is in question, whatever may be the nature
of the property or the country in which it may be situated.
But the fact is that the oppositor did not prove that said
testimentary dispositions are not in accordance with the
Turkish laws, inasmuch as he did not present any evidence
showing what the Turkish laws are on the matter, and in the
absence of evidence on such laws, they are presumed to be
the same as those of the Philippines. (Lim and Lim vs.
Collector of Customs, 36 Phil., 472.)
It has not been proved in these proceedings what the
Turkish laws are. He, himself, acknowledges it when he
desires to be given an opportunity to present evidence on this
point; so much so that he assigns as an error of the court in not
having deferred the approval of the scheme of partition until
the receipt of certain testimony requested regarding the
Turkish laws on the matter.
The refusal to give the oppositor another opportunity to
prove such laws does not constitute an error. It is discretionary
with the trial court, and, taking into consideration that the
oppositor was granted ample opportunity to introduce
competent evidence, we find no abuse of discretion on the part
of the court in this particular. There is, therefore, no evidence in
the record that the national law of the testator Joseph G. Brimo
was violated in the testamentary dispositions in question
which, not being contrary to our laws in force, must be
complied with and executed. lawphil.net
Therefore, the approval of the scheme of partition in this
respect was not erroneous.
In regard to the first assignment of error which deals with
the exclusion of the herein appellant as a legatee, inasmuch as
he is one of the persons designated as such in will, it must be
taken into consideration that such exclusion is based on the
last part of the second clause of the will, which says:
Second. I like desire to state that although by law, I am a
Turkish citizen, this citizenship having been conferred upon me
by conquest and not by free choice, nor by nationality and, on
the other hand, having resided for a considerable length of
time in the Philippine Islands where I succeeded in acquiring
all of the property that I now possess, it is my wish that the
distribution of my property and everything in connection with
this, my will, be made and disposed of in accordance with the
laws in force in the Philippine islands, requesting all of my

39
relatives to respect this wish, otherwise, I annul and cancel
beforehand whatever disposition found in this will favorable to
the person or persons who fail to comply with this request.
The institution of legatees in this will is conditional, and
the condition is that the instituted legatees must respect the
testator's will to distribute his property, not in accordance with
the laws of his nationality, but in accordance with the laws of
the Philippines.
If this condition as it is expressed were legal and valid,
any legatee who fails to comply with it, as the herein oppositor
who, by his attitude in these proceedings has not respected the
will of the testator, as expressed, is prevented from receiving
his legacy.
The fact is, however, that the said condition is void,
being contrary to law, for article 792 of the civil Code provides
the following:
Impossible conditions and those contrary to law or good
morals shall be considered as not imposed and shall not
prejudice the heir or legatee in any manner whatsoever, even
should the testator otherwise provide.
And said condition is contrary to law because it
expressly ignores the testator's national law when, according to
article 10 of the civil Code above quoted, such national law of
the testator is the one to govern his testamentary dispositions.
Said condition then, in the light of the legal provisions
above cited, is considered unwritten, and the institution of
legatees in said will is unconditional and consequently valid
and effective even as to the herein oppositor.
It results from all this that the second clause of the will
regarding the law which shall govern it, and to the condition
imposed upon the legatees, is null and void, being contrary to
law.
All of the remaining clauses of said will with all their
dispositions and requests are perfectly valid and effective it not
appearing that said clauses are contrary to the testator's
national law.
Therefore, the orders appealed from are modified and it
is directed that the distribution of this estate be made in such a
manner as to include the herein appellant Andre Brimo as one
of the legatees, and the scheme of partition submitted by the
judicial administrator is approved in all other respects, without
any pronouncement as to costs.
So ordered.
Street, Malcolm, Avancea, Villamor and Ostrand, JJ., concur.

FIRST DIVISION
[G.R. No. 124371. November 23, 2000]

PAULA T. LLORENTE, petitioner, vs. COURT OF APPEALS


and ALICIA F. LLORENTE, respondents.
DECISION
PARDO, J.:
The Case
The case raises a conflict of laws issue.
What is before us is an appeal from the decision of the Court of
Appeals modifying that of the Regional Trial Court, Camarines
Sur, Branch 35, Iriga City declaring respondent Alicia F.
Llorente (herinafter referred to as Alicia), as co-owners of
whatever property she and the deceased Lorenzo N. Llorente
(hereinafter referred to as Lorenzo) may have acquired
during the twenty-five (25) years that they lived together as
husband and wife.
The Facts
The deceased Lorenzo N. Llorente was an enlisted serviceman
of the United States Navy from March 10, 1927 to September
30, 1957.
On February 22, 1937, Lorenzo and petitioner Paula Llorente
(hereinafter referred to as Paula) were married before a
parish priest, Roman Catholic Church, in Nabua, Camarines
Sur.
Before the outbreak of the Pacific War, Lorenzo departed for
the United States and Paula stayed in the conjugal home in
barrio Antipolo, Nabua, Camarines Sur.
On November 30, 1943, Lorenzo was admitted to United
States citizenship and Certificate of Naturalization No.
5579816 was issued in his favor by the United States District
Court, Southern District of New York.
Upon the liberation of the Philippines by the American Forces
in 1945, Lorenzo was granted an accrued leave by the U. S.
Navy, to visit his wife and he visited the Philippines. He
discovered that his wife Paula was pregnant and was living in
and having an adulterous relationship with his brother, Ceferino
Llorente.
On December 4, 1945, Paula gave birth to a boy registered in
the Office of the Registrar of Nabua as Crisologo Llorente,
with the certificate stating that the child was not legitimate and
the line for the fathers name was left blank.
Lorenzo refused to forgive Paula and live with her. In fact, on
February 2, 1946, the couple drew a written agreement to the
effect that (1) all the family allowances allotted by the United
States Navy as part of Lorenzos salary and all other
obligations for Paulas daily maintenance and support would be
suspended; (2) they would dissolve their marital union in
accordance with judicial proceedings; (3) they would make a
separate agreement regarding their conjugal property acquired

40
during their marital life; and (4) Lorenzo would not prosecute
Paula for her adulterous act since she voluntarily admitted her
fault and agreed to separate from Lorenzo peacefully. The
agreement was signed by both Lorenzo and Paula and was
witnessed by Paulas father and stepmother. The agreement
was notarized by Notary Public Pedro Osabel.
Lorenzo returned to the United States and on November 16,
1951 filed for divorce with the Superior Court of the State of
California in and for the County of San Diego. Paula was
represented by counsel, John Riley, and actively participated in
the proceedings. On November 27, 1951, the Superior Court
of the State of California, for the County of San Diego found all
factual allegations to be true and issued an interlocutory
judgment of divorce.
On December 4, 1952, the divorce decree became final.
In the meantime, Lorenzo returned to the Philippines.
On January 16, 1958, Lorenzo married Alicia F. Llorente in
Manila. Apparently, Alicia had no knowledge of the first
marriage even if they resided in the same town as Paula, who
did not oppose the marriage or cohabitation.
From 1958 to 1985, Lorenzo and Alicia lived together as
husband and wife. Their twenty-five (25) year union produced
three children, Raul, Luz and Beverly, all surnamed Llorente.
On March 13, 1981, Lorenzo executed a Last Will and
Testament. The will was notarized by Notary Public Salvador
M. Occiano, duly signed by Lorenzo with attesting witnesses
Francisco Hugo, Francisco Neibres and Tito Trajano. In the
will, Lorenzo bequeathed all his property to Alicia and their
three children, to wit:
(1) I give and bequeath to my wife ALICIA R. FORTUNO
exclusively my residential house and lot, located at San
Francisco, Nabua, Camarines Sur, Philippines, including ALL
the personal properties and other movables or belongings that
may be found or existing therein;
(2) I give and bequeath exclusively to my wife Alicia R.
Fortuno and to my children, Raul F. Llorente, Luz F. Llorente
and Beverly F. Llorente, in equal shares, all my real properties
whatsoever and wheresoever located, specifically my real
properties located at Barangay Aro-Aldao, Nabua, Camarines
Sur; Barangay Paloyon, Nabua, Camarines Sur; Barangay
Baras, Sitio Puga, Nabua, Camarines Sur; and Barangay
Paloyon, Sitio Nalilidong, Nabua, Camarines Sur;
(3) I likewise give and bequeath exclusively unto my wife
Alicia R. Fortuno and unto my children, Raul F. Llorente, Luz F.
Llorente and Beverly F. Llorente, in equal shares, my real
properties located in Quezon City Philippines, and covered by
Transfer Certificate of Title No. 188652; and my lands in
Antipolo, Rizal, Philippines, covered by Transfer Certificate of
Title Nos. 124196 and 165188, both of the Registry of Deeds of
the province of Rizal, Philippines;

(4) That their respective shares in the above-mentioned


properties, whether real or personal properties, shall not be
disposed of, ceded, sold and conveyed to any other persons,
but could only be sold, ceded, conveyed and disposed of by
and among themselves;
(5) I designate my wife ALICIA R. FORTUNO to be the sole
executor of this my Last Will and Testament, and in her default
or incapacity of the latter to act, any of my children in the order
of age, if of age;
(6) I hereby direct that the executor named herein or her
lawful substitute should served (sic) without bond;
(7) I hereby revoke any and all my other wills, codicils, or
testamentary dispositions heretofore executed, signed, or
published, by me;
(8) It is my final wish and desire that if I die, no relatives of
mine in any degree in the Llorentes Side should ever bother
and disturb in any manner whatsoever my wife Alicia R.
Fortunato and my children with respect to any real or personal
properties I gave and bequeathed respectively to each one of
them by virtue of this Last Will and Testament.
On December 14, 1983, Lorenzo filed with the Regional Trial
Court, Iriga, Camarines Sur, a petition for the probate and
allowance of his last will and testament wherein Lorenzo
moved that Alicia be appointed Special Administratrix of his
estate.
On January 18, 1984, the trial court denied the motion for the
reason that the testator Lorenzo was still alive.
On January 24, 1984, finding that the will was duly executed,
the trial court admitted the will to probate.
On June 11, 1985, before the proceedings could be
terminated, Lorenzo died.
On September 4, 1985, Paula filed with the same court a
petition for letters of administration over Lorenzos estate in
her favor. Paula contended (1) that she was Lorenzos
surviving spouse, (2) that the various property were acquired
during their marriage, (3) that Lorenzos will disposed of all his
property in favor of Alicia and her children, encroaching on her
legitime and 1/2 share in the conjugal property.
On December 13, 1985, Alicia filed in the testate proceeding
(Sp. Proc. No. IR-755), a petition for the issuance of letters
testamentary.
On October 14, 1985, without terminating the testate
proceedings, the trial court gave due course to Paulas petition
in Sp. Proc. No. IR-888.
On November 6, 13 and 20, 1985, the order was published in
the newspaper Bicol Star.

41
On May 18, 1987, the Regional Trial Court issued a joint
decision, thus:
Wherefore, considering that this court has so found that the
divorce decree granted to the late Lorenzo Llorente is void and
inapplicable in the Philippines, therefore the marriage he
contracted with Alicia Fortunato on January 16, 1958 at Manila
is likewise void. This being so the petition of Alicia F. Llorente
for the issuance of letters testamentary is denied. Likewise,
she is not entitled to receive any share from the estate even if
the will especially said so her relationship with Lorenzo having
gained the status of paramour which is under Art. 739 (1).
On the other hand, the court finds the petition of Paula Titular
Llorente, meritorious, and so declares the intrinsic disposition
of the will of Lorenzo Llorente dated March 13, 1981 as void
and declares her entitled as conjugal partner and entitled to
one-half of their conjugal properties, and as primary
compulsory heir, Paula T. Llorente is also entitled to one-third
of the estate and then one-third should go to the illegitimate
children, Raul, Luz and Beverly, all surname (sic) Llorente, for
them to partition in equal shares and also entitled to the
remaining free portion in equal shares.
Petitioner, Paula Llorente is appointed legal administrator of
the estate of the deceased, Lorenzo Llorente. As such let the
corresponding letters of administration issue in her favor upon
her filing a bond in the amount (sic) of P100,000.00
conditioned for her to make a return to the court within three
(3) months a true and complete inventory of all goods, chattels,
rights, and credits, and estate which shall at any time come to
her possession or to the possession of any other person for
her, and from the proceeds to pay and discharge all debts,
legacies and charges on the same, or such dividends thereon
as shall be decreed or required by this court; to render a true
and just account of her administration to the court within one
(1) year, and at any other time when required by the court and
to perform all orders of this court by her to be performed.
On the other matters prayed for in respective petitions for
want of evidence could not be granted.

On July 31, 1995, the Court of Appeals promulgated its


decision, affirming with modification the decision of the trial
court in this wise:
WHEREFORE, the decision appealed from is hereby
AFFIRMED with the MODIFICATION that Alicia is declared as
co-owner of whatever properties she and the deceased may
have acquired during the twenty-five (25) years of cohabitation.
SO ORDERED.
On August 25, 1995, petitioner filed with the Court of Appeals a
motion for reconsideration of the decision.
On March 21, 1996, the Court of Appeals, denied the motion
for lack of merit.
Hence, this petition.
The Issue
Stripping the petition of its legalese and sorting through the
various arguments raised, the issue is simple. Who are
entitled to inherit from the late Lorenzo N. Llorente?
We do not agree with the decision of the Court of Appeals. We
remand the case to the trial court for ruling on the intrinsic
validity of the will of the deceased.
The Applicable Law
The fact that the late Lorenzo N. Llorente became an American
citizen long before and at the time of: (1) his divorce from
Paula; (2) marriage to Alicia; (3) execution of his will; and (4)
death, is duly established, admitted and undisputed.
Thus, as a rule, issues arising from these incidents are
necessarily governed by foreign law.
The Civil Code clearly provides:

SO ORDERED.

Art. 15. Laws relating to family rights and duties, or to the


status, condition and legal capacity of persons are binding
upon citizens of the Philippines, even though living abroad.

In time, Alicia filed with the trial court a motion for


reconsideration of the aforequoted decision.

Art. 16. Real property as well as personal property is subject


to the law of the country where it is situated.

On September 14, 1987, the trial court denied Alicias motion


for reconsideration but modified its earlier decision, stating that
Raul and Luz Llorente are not children legitimate or otherwise
of Lorenzo since they were not legally adopted by him.
Amending its decision of May 18, 1987, the trial court declared
Beverly Llorente as the only illegitimate child of Lorenzo,
entitling her to one-third (1/3) of the estate and one-third (1/3)
of the free portion of the estate.

However, intestate and testamentary succession, both with


respect to the order of succession and to the amount of
successional rights and to the intrinsic validity of testamentary
provisions, shall be regulated by the national law of the person
whose succession is under consideration, whatever may be
the nature of the property and regardless of the country
wherein said property may be found. (emphasis ours)

On September 28, 1987, respondent appealed to the Court of


Appeals.

True, foreign laws do not prove themselves in our jurisdiction


and our courts are not authorized to take judicial notice of
them. Like any other fact, they must be alleged and proved.

42
While the substance of the foreign law was pleaded, the Court
of Appeals did not admit the foreign law. The Court of Appeals
and the trial court called to the fore the renvoi doctrine, where
the case was referred back to the law of the decedents
domicile, in this case, Philippine law.
We note that while the trial court stated that the law of New
York was not sufficiently proven, in the same breath it made
the categorical, albeit equally unproven statement that
American law follows the domiciliary theory hence, Philippine
law applies when determining the validity of Lorenzos will.
First, there is no such thing as one American law. The
"national law" indicated in Article 16 of the Civil Code cannot
possibly apply to general American law. There is no such law
governing the validity of testamentary provisions in the United
States. Each State of the union has its own law applicable to
its citizens and in force only within the State. It can therefore
refer to no other than the law of the State of which the
decedent was a resident. Second, there is no showing that the
application of the renvoi doctrine is called for or required by
New York State law.
The trial court held that the will was intrinsically invalid since it
contained dispositions in favor of Alice, who in the trial courts
opinion was a mere paramour. The trial court threw the will
out, leaving Alice, and her two children, Raul and Luz, with
nothing.
The Court of Appeals also disregarded the will. It declared
Alice entitled to one half (1/2) of whatever property she and
Lorenzo acquired during their cohabitation, applying Article 144
of the Civil Code of the Philippines.
The hasty application of Philippine law and the complete
disregard of the will, already probated as duly executed in
accordance with the formalities of Philippine law, is fatal,
especially in light of the factual and legal circumstances here
obtaining.
Validity of the Foreign Divorce
In Van Dorn v. Romillo, Jr. we held that owing to the nationality
principle embodied in Article 15 of the Civil Code, only
Philippine nationals are covered by the policy against absolute
divorces, the same being considered contrary to our concept of
public policy and morality. In the same case, the Court ruled
that aliens may obtain divorces abroad, provided they are valid
according to their national law.
Citing this landmark case, the Court held in Quita v. Court of
Appeals, that once proven that respondent was no longer a
Filipino citizen when he obtained the divorce from petitioner,
the ruling in Van Dorn would become applicable and petitioner
could very well lose her right to inherit from him.
In Pilapil v. Ibay-Somera, we recognized the divorce obtained
by the respondent in his country, the Federal Republic of
Germany. There, we stated that divorce and its legal effects

may be recognized in the Philippines insofar as respondent is


concerned in view of the nationality principle in our civil law on
the status of persons.
For failing to apply these doctrines, the decision of the Court of
Appeals must be reversed. We hold that the divorce obtained
by Lorenzo H. Llorente from his first wife Paula was valid and
recognized in this jurisdiction as a matter of comity. Now, the
effects of this divorce (as to the succession to the estate of the
decedent) are matters best left to the determination of the trial
court.
Validity of the Will
The Civil Code provides:
Art. 17. The forms and solemnities of contracts, wills, and
other public instruments shall be governed by the laws of the
country in which they are executed.
When the acts referred to are executed before the diplomatic
or consular officials of the Republic of the Philippines in a
foreign country, the solemnities established by Philippine laws
shall be observed in their execution. (underscoring ours)
The clear intent of Lorenzo to bequeath his property to his
second wife and children by her is glaringly shown in the will
he executed. We do not wish to frustrate his wishes, since he
was a foreigner, not covered by our laws on family rights and
duties, status, condition and legal capacity.
Whether the will is intrinsically valid and who shall inherit from
Lorenzo are issues best proved by foreign law which must be
pleaded and proved. Whether the will was executed in
accordance with the formalities required is answered by
referring to Philippine law. In fact, the will was duly probated.
As a guide however, the trial court should note that whatever
public policy or good customs may be involved in our system of
legitimes, Congress did not intend to extend the same to the
succession of foreign nationals. Congress specifically left the
amount of successional rights to the decedent's national law.
Having thus ruled, we find it unnecessary to pass upon the
other issues raised.
The Fallo
WHEREFORE, the petition is GRANTED. The decision of the
Court of Appeals in CA-G. R. SP No. 17446 promulgated on
July 31, 1995 is SET ASIDE.
In lieu thereof, the Court REVERSES the decision of the
Regional Trial Court and RECOGNIZES as VALID the decree
of divorce granted in favor of the deceased Lorenzo N.
Llorente by the Superior Court of the State of California in and
for the County of San Diego, made final on December 4, 1952.
Further, the Court REMANDS the cases to the court of origin
for determination of the intrinsic validity of Lorenzo N.

43
Llorentes will and determination of the parties successional
rights allowing proof of foreign law with instructions that the
trial court shall proceed with all deliberate dispatch to settle the
estate of the deceased within the framework of the Rules of
Court.
No costs.
SO ORDERED.
Davide, Jr., C.J., (Chairman), Puno, Kapunan, and YnaresSantiago, JJ., concur.
FIRST DIVISION

[G.R. No. 108581. December 8, 1999]

LOURDES L. DOROTHEO, petitioner, vs. COURT OF


APPEALS, NILDA D. QUINTANA, for Herself and
as Attorney-in-Fact of VICENTE DOROTHEO and
JOSE DOROTHEO, respondents.
DECISION
YNARES-SANTIAGO, J.:
May a last will and testament admitted to probate but
declared intrinsically void in an order that has become final and
executory still be given effect? This is the issue that arose from
the following antecedents:
Private respondents were the legitimate children of
Alejandro Dorotheo and Aniceta Reyes. The latter died in
1969 without her estate being settled. Alejandro died
thereafter. Sometime in 1977, after Alejandros death,
petitioner, who claims to have taken care of Alejandro before
he died, filed a special proceeding for the probate of the latters
last will and testament. In 1981, the court issued an order
admitting Alejandros will to probate. Private respondents did
not appeal from said order. In 1983, they filed a Motion To
Declare The Will Intrinsically Void. The trial court granted the
motion and issued an order, the dispositive portion of which
reads:
WHEREFORE, in view of the foregoing, Order is hereby
issued declaring Lourdes Legaspi not the wife of the late
Alejandro Dorotheo, the provisions of the last will and
testament of Alejandro Dorotheo as intrinsically void, and
declaring the oppositors Vicente Dorotheo, Jose Dorotheo and
Nilda Dorotheo Quintana as the only heirs of the late spouses
Alejandro Dorotheo and Aniceta Reyes, whose respective
estates shall be liquidated and distributed according to the
laws on intestacy upon payment of estate and other taxes due
to the government.[1]
Petitioner moved for reconsideration arguing that she is
entitled to some compensation since she took care of
Alejandro prior to his death although she admitted that they

were not married to each other. Upon denial of her motion for
reconsideration, petitioner appealed to the Court of Appeals,
but the same was dismissed for failure to file appellants brief
within the extended period granted.[2] This dismissal became
final and executory on February 3, 1989 and a corresponding
entry of judgment was forthwith issued by the Court of Appeals
on May 16, 1989. A writ of execution was issued by the lower
court
to
implement
the
final
and
executory
Order. Consequently, private respondents filed several
motions including a motion to compel petitioner to surrender to
them the Transfer Certificates of Titles (TCT) covering the
properties of the late Alejandro. When petitioner refused to
surrender the TCTs, private respondents filed a motion for
cancellation of said titles and for issuance of new titles in their
names. Petitioner opposed the motion.
An Order was issued on November 29, 1990 by Judge
Zain B. Angas setting aside the final and executory Order
dated January 30, 1986, as well as the Order directing the
issuance of the writ of execution, on the ground that the order
was merely interlocutory, hence not final in character. The
court added that the dispositive portion of the said Order even
directs the distribution of the estate of the deceased
spouses. Private
respondents
filed
a
motion
for
reconsideration which was denied in an Order dated February
1, 1991. Thus, private respondents filed a petition before the
Court of Appeals, which nullified the two assailed Orders dated
November 29, 1990 and February 1, 1991.
Aggrieved, petitioner instituted a petition for review
arguing that the case filed by private respondents before the
Court of Appeals was a petition under Rule 65 on the ground of
grave abuse of discretion or lack of jurisdiction. Petitioner
contends that in issuing the two assailed orders, Judge Angas
cannot be said to have no jurisdiction because he was
particularly designated to hear the case. Petitioner likewise
assails the Order of the Court of Appeals upholding the validity
of the January 30, 1986 Order which declared the intrinsic
invalidity of Alejandros will that was earlier admitted to
probate.
Petitioner also filed a motion to reinstate her as executrix
of the estate of the late Alejandro and to maintain the status
quo or lease of the premises thereon to third parties. [3] Private
respondents opposed the motion on the ground that petitioner
has no interest in the estate since she is not the lawful wife of
the late Alejandro.
The petition is without merit. A final and executory
decision or order can no longer be disturbed or reopened no
matter how erroneous it may be. In setting aside the January
30, 1986 Order that has attained finality, the trial court in effect
nullified the entry of judgment made by the Court of Appeals. It
is well settled that a lower court cannot reverse or set aside
decisions or orders of a superior court, for to do so would be to
negate the hierarchy of courts and nullify the essence of
review. It has been ruled that a final judgment on probated will,
albeit erroneous, is binding on the whole world.[4]
It has been consistently held that if no appeal is taken in
due time from a judgment or order of the trial court, the same
attains finality by mere lapse of time. Thus, the order allowing
the will became final and the question determined by the court
in such order can no longer be raised anew, either in the same
proceedings or in a different motion. The matters of due
execution of the will and the capacity of the testator acquired
the character of res judicata and cannot again be brought into
question, all juridical questions in connection therewith being

44
for once and forever closed.[5] Such final order makes the will
conclusive against the whole world as to its extrinsic validity
and due execution.[6]
It should be noted that probate proceedings deals
generally with the extrinsic validity of the will sought to be
probated,[7] particularly on three aspects:
whether the will submitted is indeed, the decedents last will
and testament;
compliance with the prescribed formalities for the execution
of wills;
the testamentary capacity of the testator;[8]
and the due execution of the last will and testament.[9]
Under the Civil Code, due execution includes a
determination of whether the testator was of sound and
disposing mind at the time of its execution, that he had freely
executed the will and was not acting under duress, fraud,
menace or undue influence and that the will is genuine and not
a forgery,[10] that he was of the proper testamentary age and
that he is a person not expressly prohibited by law from making
a will.[11]
The intrinsic validity is another matter and questions
regarding the same may still be raised even after the will has
been authenticated.[12] Thus, it does not necessarily follow that
an extrinsically valid last will and testament is always
intrinsically valid. Even if the will was validly executed, if the
testator provides for dispositions that deprives or impairs the
lawful heirs of their legitime or rightful inheritance according to
the laws on succession,[13] the unlawful provisions/dispositions
thereof cannot be given effect. This is specially so when the
courts had already determined in a final and executory
decision that the will is intrinsically void. Such determination
having attained that character of finality is binding on this Court
which will no longer be disturbed. Not that this Court finds the
will to be intrinsically valid, but that a final and executory
decision of which the party had the opportunity to challenge
before the higher tribunals must stand and should no longer be
reevaluated. Failure to avail of the remedies provided by law
constitutes waiver. And if the party does not avail of other
remedies despite its belief that it was aggrieved by a decision
or court action, then it is deemed to have fully agreed and is
satisfied with the decision or order. As early as 1918, it has
been declared that public policy and sound practice demand
that, at the risk of occasional errors, judgments of courts must
at some point of time fixed by law [14] become final otherwise
there will be no end to litigation. Interes rei publicae ut finis sit
litium - the very object of which the courts were constituted was
to put an end to controversies.[15] To fulfill this purpose and to
do so speedily, certain time limits, more or less arbitrary, have
to be set up to spur on the slothful.[16] The only instance where
a party interested in a probate proceeding may have a final
liquidation set aside is when he is left out by reason of
circumstances beyond his control or through mistake or
inadvertence
not
imputable
to
negligence,[17] which
circumstances do not concur herein.
Petitioner was privy to the suit calling for the declaration
of the intrinsic invalidity of the will, as she precisely appealed
from an unfavorable order therefrom. Although the final and

executory Order of January 30, 1986 wherein private


respondents were declared as the only heirs do not bind those
who are not parties thereto such as the alleged illegitimate son
of the testator, the same constitutes res judicata with respect to
those who were parties to the probate proceedings. Petitioner
cannot again raise those matters anew for relitigation
otherwise that would amount to forum-shopping. It should be
remembered that forum shopping also occurs when the same
issue had already been resolved adversely by some other
court.[18] It is clear from the executory order that the estates of
Alejandro and his spouse should be distributed according to
the laws of intestate succession.
Petitioner posits that the January 30, 1986 Order is
merely interlocutory, hence it can still be set aside by the trial
court. In support thereof, petitioner argues that an order
merely declaring who are heirs and the shares to which set of
heirs is entitled cannot be the basis of execution to require
delivery of shares from one person to another particularly when
no project of partition has been filed.[19] The trial court declared
in the January 30, 1986 Order that petitioner is not the legal
wife of Alejandro, whose only heirs are his three legitimate
children (petitioners herein), and at the same time it nullified
the will. But it should be noted that in the same Order, the trial
court also said that the estate of the late spouses be
distributed according to the laws of intestacy. Accordingly, it
has no option but to implement that order of intestate
distribution and not to reopen and again re-examine the
intrinsic provisions of the same will.
It can be clearly inferred from Article 960 of the Civil
Code, on the law of successional rights that testacy is
preferred to intestacy.[20] But before there could be testate
distribution, the will must pass the scrutinizing test and
safeguards provided by law considering that the deceased
testator is no longer available to prove the voluntariness of his
actions, aside from the fact that the transfer of the estate is
usually onerous in nature and that no one is presumed to give
- Nemo praesumitur donare.[21] No intestate distribution of the
estate can be done until and unless the will had failed to pass
both its extrinsic and intrinsic validity. If the will is extrinsically
void, the rules of intestacy apply regardless of the intrinsic
validity thereof. If it is extrinsically valid, the next test is to
determine its intrinsic validity that is whether the provisions of
the will are valid according to the laws of succession. In this
case, the court had ruled that the will of Alejandro was
extrinsically valid but the intrinsic provisions thereof were
void. Thus, the rules of intestacy apply as correctly held by the
trial court.
Furthermore, Alejandros disposition in his will of the
alleged share in the conjugal properties of his late spouse,
whom he described as his only beloved wife, is not a valid
reason to reverse a final and executory order. Testamentary
dispositions of properties not belonging exclusively to the
testator or properties which are part of the conjugal regime
cannot be given effect. Matters with respect to who owns the
properties that were disposed of by Alejandro in the void will
may still be properly ventilated and determined in the intestate
proceedings for the settlement of his and that of his late
spouses estate.
Petitioners motion for appointment as administratrix is
rendered moot considering that she was not married to the late
Alejandro and, therefore, is not an heir.
WHEREFORE, the petition is DENIED and the decision
appealed from is AFFIRMED.

45
SO ORDERED.
Davide,
Jr.,
C.J.,
Kapunan, and Pardo, JJ., concur.

(Chairman),

Puno,

Republic of the Philippines


SUPREME COURT
Manila
EN BANC
G.R. No. L-6801

March 14, 1912

JULIANA BAGTAS, plaintiffs-appellee,


vs.
ISIDRO PAGUIO, ET AL., defendants-appellants.
Salas and Kalaw for appellants.
Jose Santiago for appellee.
TRENT, J.:
This is an appeal from an order of the Court of First Instance of
the Province of Bataan, admitting to probate a document which
was offered as the last will and testament of Pioquinto Paguio
y Pizarro. The will purports to have been executed in the
pueblo of Pilar, Province of Bataan, on the 19th day of April,
1908. The testator died on the 28th of September, 1909, a year
and five months following the date of the execution of the will.
The will was propounded by the executrix, Juliana Bagtas,
widow of the decedent, and the opponents are a son and
several grandchildren by a former marriage, the latter being the
children of a deceased daughter.
The basis of the opposition to the probation of the will is that
the same was not executed according to the formalities and
requirements of the law touching wills, and further that the
testator was not in the full of enjoyment and use of his mental
faculties and was without the mental capacity necessary to
execute a valid will.
The record shows that the testator, Pioquinto Paguio, for some
fourteen of fifteen years prior to the time of his death suffered
from a paralysis of the left side of his body; that a few years
prior to his death his hearing became impaired and that he lost
the power of speech. Owing to the paralysis of certain muscles
his head fell to one side, and saliva ran from his mouth. He
retained the use of his right hand, however, and was able to
write fairly well. Through the medium of signs he was able to
indicate his wishes to his wife and to other members of his
family.
At the time of the execution of the will there were present the
four testamentary witnesses, Agustin Paguio, Anacleto Paguio,
and Pedro Paguio, and attorney, Seor Marco, and one
Florentino Ramos. Anacleto Paguio and the attorney have
since died, and consequently their testimony was not available
upon the trial of the case in the lower court. The other three
testamentary witnesses and the witness Florentino Ramos
testified as to the manner in which the will was executed.
According to the uncontroverted testimony of these witnesses
the will was executed in the following manner:

Pioquinto Paguio, the testator, wrote out on pieces of paper


notes and items relating to the disposition of his property, and
these notes were in turn delivered to Seor Marco, who
transcribed them and put them in form. The witnesses testify
that the pieces of paper upon which the notes were written are
delivered to attorney by the testator; that the attorney read
them to the testator asking if they were his testamentary
dispositions; that the testator assented each time with an
affirmative movement of his head; that after the will as a whole
had been thus written by the attorney, it was read in a loud
voice in the presence of the testator and the witnesses; that
Seor Marco gave the document to the testator; that the latter,
after looking over it, signed it in the presence of the four
subscribing witnesses; and that they in turn signed it in the
presence of the testator and each other.
These are the facts of record with reference to the execution of
the will and we are in perfect accord with the judgment of the
lower court that the formalities of the Code of Civil Procedure
have been fully complied with.
This brings us now to a consideration of appellants' second
assignment of error, viz, the testator's alleged mental
incapacity at the time of the execution of the will. Upon this
point considerable evidence was adduced at the trial. One of
the attesting witnesses testified that at the time of the
execution of the will the testator was in his right mind, and that
although he was seriously ill, he indicated by movements of his
head what his wishes were. Another of the attesting witnesses
stated that he was not able to say whether decedent had the
full use of his mental faculties or not, because he had been ill
for some years, and that he (the witnesses) was not a
physician. The other subscribing witness, Pedro Paguio,
testified in the lower court as a witness for the opponents. He
was unable to state whether or not the will was the wish of the
testator. The only reasons he gave for his statement were the
infirmity and advanced age of the testator and the fact that he
was unable to speak. The witness stated that the testator
signed the will, and he verified his own signature as a
subscribing witness.
Florentino Ramos, although not an attesting witness, stated
that he was present when the will was executed and his
testimony was cumulative in corroboration of the manner in
which the will was executed and as to the fact that the testator
signed the will. This witness also stated that he had frequently
transacted matters of business for the decedent and had
written letters and made inventories of his property at his
request, and that immediately before and after the execution of
the will he had performed offices of his character. He stated
that the decedent was able to communicate his thoughts by
writing. The testimony of this witness clearly indicates the
presence of mental capacity on the part of the testator. Among
other witnesses for the opponents were two physician, Doctor
Basa and Doctor Viado. Doctor Basa testified that he had
attended the testator some four or five years prior to his death
and that the latter had suffered from a cerebral congestion
from which the paralysis resulted. The following question was
propounded to Doctor Basa:
Q.
Referring to mental condition in which you
found him the last time you attended him, do you think
he was in his right mind?

46
A.
I can not say exactly whether he was in his
right mind, but I noted some mental disorder, because
when I spoke to him he did not answer me.
Doctor Basa testified at more length, but the substance of his
testimony is that the testator had suffered a paralysis and that
he had noticed some mental disorder. He does not say that the
testator was not in his right mind at the time of the execution of
the will, nor does he give it at his opinion that he was without
the necessary mental capacity to make a valid will. He did not
state in what way this mental disorder had manifested itself
other than that he had noticed that the testator did not reply to
him on one occasion when he visited him.
Doctor Viado, the other physician, have never seen the
testator, but his answer was in reply to a hypothetical question
as to what be the mental condition of a person who was 79
years old and who had suffered from a malady such as the
testator was supposed to have had according to the testimony
of Doctor Basa, whose testimony Doctor Viado had heard. He
replied and discussed at some length the symptoms and
consequences of the decease from which the testator had
suffered; he read in support of his statements from a work by a
German Physician, Dr. Herman Eichost. In answer, however, to
a direct question, he stated that he would be unable to certify
to the mental condition of a person who was suffering from
such a disease.
We do not think that the testimony of these two physicians in
any way strengthens the contention of the appellants. Their
testimony only confirms the fact that the testator had been for a
number of years prior to his death afflicted with paralysis, in
consequence of which his physician and mental strength was
greatly impaired. Neither of them attempted to state what was
the mental condition of the testator at the time he executed the
will in question. There can be no doubt that the testator's
infirmities were of a very serious character, and it is quite
evident that his mind was not as active as it had been in the
earlier years of his life. However, we can not include from this
that he wanting in the necessary mental capacity to dispose of
his property by will.
The courts have been called upon frequently to nullify wills
executed under such circumstances, but the weight of the
authority is in support if the principle that it is only when those
seeking to overthrow the will have clearly established the
charge of mental incapacity that the courts will intervene to set
aside a testamentary document of this character. In the case
of Bugnao vs. Ubag (14 Phil. Rep., 163), the question of
testamentary capacity was discussed by this court. The
numerous citations there given from the decisions of the United
States courts are especially applicable to the case at bar and
have our approval. In this jurisdiction the presumption of law is
in favor of the mental capacity of the testator and the burden is
upon the contestants of the will to prove the lack of
testamentary capacity. (In the matter of the will of Cabigting, 14
Phil. Rep., 463; in the matter of the will of Butalid, 10 Phil.
Rep., 27; Hernaez vs. Hernaez, 1 Phil. Rep., 689.)
The rule of law relating to the presumption of mental
soundness is well established, and the testator in the case at
bar never having been adjudged insane by a court of
competent jurisdiction, this presumption continues, and it is
therefore incumbent upon the opponents to overcome this

legal presumption by proper evidence. This we think they have


failed to do. There are many cases and authorities which we
might cite to show that the courts have repeatedly held that
mere weakness of mind and body, induced by age and disease
do not render a person incapable of making a will. The law
does not require that a person shall continue in the full
enjoyment and use of his pristine physical and mental powers
in order to execute a valid will. If such were the legal standard,
few indeed would be the number of wills that could meet such
exacting requirements. The authorities, both medical and legal,
are universal in statement that the question of mental capacity
is one of degree, and that there are many gradations from the
highest degree of mental soundness to the lowest conditions of
diseased mentality which are denominated as insanity and
idiocy.
The right to dispose of property by testamentary disposition is
as sacred as any other right which a person may exercise and
this right should not be nullified unless mental incapacity is
established in a positive and conclusive manner. In discussing
the question of testamentary capacity, it is stated in volume 28,
70, of the American and English Encyclopedia of Law, that
Contrary to the very prevalent lay impression, perfect
soundness of mind is not essential to testamentary
capacity. A testator may be afflicted with a variety of
mental weaknesses, disorders, or peculiarities and
still be capable in law of executing a valid will. (See
the numerous cases there cited in support of this
statement.)
The rule relating to testamentary capacity is stated in Buswell
on Insanity, section 365, and quoted with approval inCampbell
vs. Campbell (130 Ill., 466), as follows:
To constitute a sound and disposing mind, it is not
necessary that the mind shall be wholly unbroken,
unimpaired, or unshattered by disease or otherwise,
or that the testator should be in the full possession of
his reasoning faculties.
In note, 1 Jarman on Wills, 38, the rule is thus stated:
The question is not so much, that was the degree of
memory possessed by the testator, as, had he a
disposing memory? Was he able to remember the
property he was about to bequeath, the manner of
disturbing it, and the objects of his bounty? In a word,
were his mind and memory sufficiently sound to
enable him to know and understand the business in
which he was engaged at the time when he executed
his will. (See authorities there cited.)
In Wilson vs. Mitchell (101 Penn., 495), the following facts
appeared upon the trial of the case: The testator died at the
age of nearly 102 years. In his early years he was an intelligent
and well informed man. About seven years prior to his death he
suffered a paralytic stroke and from that time his mind and
memory were mush enfeebled. He became very dull of hearing
and in consequence of the shrinking of his brain he was
affected with senile cataract causing total blindness. He
became filthy and obscene in his habits, although formerly he
was observant of the properties of life. The court, in
commenting upon the case, said:

47
Neither age, nor sickness, nor extreme distress, nor
debility of body will affect the capacity to make a will,
if sufficient intelligence remains. The failure of
memory is not sufficient to create the incapacity,
unless it be total, or extend to his immediate family or
property. . . .

VIRGILIO REGALA, JR.,

CORONA

and RAFAEL TITCO,

LEONAR
Petitioners,

DEL CAS
- versus -

xxx

xxx

LORENZO LAXA,

Promulga
Respondent.

x------------------------------------------------------------------x
DECISION
DEL CASTILLO, J.:
It is incumbent upon those who oppose the probate of a will
to clearly establish that the decedent was not of sound and disposing
mind at the time of the execution of said will. Otherwise, the state is

In the above case the will was sustained. In the case at bar we
might draw the same contrast as was pictured by the court in
the case just quoted. The striking change in the physical and
mental vigor of the testator during the last years of his life may
have led some of those who knew him in his earlier days to
entertain doubts as to his mental capacity to make a will, yet
we think that the statements of the witnesses to the execution
of the will and statements of the conduct of the testator at that
time all indicate that he unquestionably had mental capacity
and that he exercised it on this occasion. At the time of the
execution of the will it does not appear that his conduct was
irrational in any particular. He seems to have comprehended
clearly what the nature of the business was in which he was
engaged. The evidence show that the writing and execution of
the will occupied a period several hours and that the testator
was present during all this time, taking an active part in all the
proceedings. Again, the will in the case at bar is perfectly
reasonable and its dispositions are those of a rational person.

duty-bound to give full effect to the wishes of the testator to distribute


his estate in the manner provided in his will so long as it is legally
tenable.[1]

Before us is a Petition for Review on Certiorari[2] of the June


15, 2006 Decision[3] of the Court of Appeals (CA) in CA-G.R. CV No.
80979 which reversed the September 30, 2003 Decision [4] of the
Regional Trial Court (RTC), Branch 52, Guagua, Pampanga in
Special Proceedings No. G-1186. The assailed CA Decision granted
the petition for probate of the notarial will of Paciencia Regala
(Paciencia), to wit:
WHEREFORE, premises considered,
finding the appeal to be impressed with merit, the
decision in SP. PROC. NO. G-1186 dated 30
September 2003, is hereby SET ASIDE and a
new one entered GRANTING the petition for the
probate of the will of PACIENCIA REGALA.

For the reasons above stated, the order probating the will
should be and the same is hereby affirmed, with costs of this
instance against the appellants.
Arellano, C.J., Torres, Mapa, Johnson, Carson and Moreland,
JJ., concur.

SO ORDERED.[5]
Republic of the Philippines
Supreme Court
Baguio City

Also assailed herein is the August 31, 2006 CA

FIRST DIVISION

Resolution[6] which denied the Motion for Reconsideration thereto.


G.R. No. 174489

SEBASTIAN M. BALTAZAR,

Petitioners call us to reverse the CAs assailed Decision and

ANTONIO L. MANGALINDAN,
ROSIE M. MATEO,
NENITA A. PACHECO,

VILLARA

xxx

Dougal (the testator) had lived over one hundred


years before he made the will, and his physical and
mental weakness and defective memory were in
striking contrast with their strength in the meridian of
his life. He was blind; not deaf, but hearing impaired;
his mind acted slowly, he was forgetful or recent
events, especially of names, and repeated questions
in conversation; and sometimes, when aroused for
sleep or slumber, would seem bewildered. It is not
singular that some of those who had known him when
he was remarkable for vigor and intelligence, are of
the opinion that his reason was so far gone that he
was incapable of making a will, although they never
heard him utter an irrational expression.

ANTONIO B. BALTAZAR,

BERSAM

instead affirm the Decision of the RTC which disallowed the notarial
Present:

will of Paciencia.

April 11, 2

48
spouses Lorenzo R. Laxa and Corazon F. Laxa
and their two children and I also command them
to offer masses yearly for the repose of my soul
and that of D[]a Nicomeda Regala, Epifania
Regala and their spouses and with respect to the
fishpond situated at San Antonio, I likewise
command to fulfill the wishes of D[]a Nicomeda
Regala in accordance with her testament as
stated in my testament. x x x[12]

Factual Antecedents

Paciencia was a 78 year old spinster when she made her last
will and testament entitled Tauli Nang Bilin o Testamento Miss
Paciencia Regala[7] (Will) in the Pampango dialect on September 13,
1981. The Will, executed in the house of retired Judge Ernestino G.
Limpin (Judge Limpin), was read to Paciencia twice. After which,
Paciencia expressed in the presence of the instrumental witnesses
that the document is her last will and testament. She thereafter
affixed her signature at the end of the said document on page 3[8] and
then on the left margin of pages 1, 2 and 4 thereof.

The filial relationship of Lorenzo with Paciencia remains


undisputed. Lorenzo is Paciencias nephew whom she treated as her
own son. Conversely, Lorenzo came to know and treated Paciencia
as his own mother.[13] Paciencia lived with Lorenzos family in

[9]

Sasmuan, Pampanga and it was she who raised and cared for
Lorenzo since his birth. Six days after the execution of the Will or on

The witnesses to the Will were Dra. Maria Lioba A. Limpin


(Dra. Limpin), Francisco Garcia (Francisco) and Faustino R. Mercado
(Faustino). The three attested to the Wills due execution by affixing

September 19, 1981, Paciencia left for the United States of America
(USA). There, she resided with Lorenzo and his family until her death
on January 4, 1996.

their signatures below its attestation clause[10] and on the left margin of
pages 1, 2 and 4 thereof,[11] in the presence of Paciencia and of one
another and of Judge Limpin who acted as notary public.

In the interim, the Will remained in the custody of Judge


Limpin.

Childless and without any brothers or sisters, Paciencia


bequeathed all her properties to respondent Lorenzo R. Laxa
(Lorenzo) and his wife Corazon F. Laxa and their children Luna
Lorella Laxa and Katherine Ross Laxa, thus:
xxxx
Fourth - In consideration of their
valuable services to me since then up to the
present by the spouses LORENZO LAXA and
CORAZON F. LAXA, I hereby BEQUEATH,
CONVEY and GIVE all my properties
enumerated in parcels 1 to 5 unto the spouses
LORENZO R. LAXA and CORAZON F.
LAXA and their children, LUNA LORELLA LAXA
and KATHERINE LAXA, and the spouses
Lorenzo R. Laxa and Corazon F. Laxa both of
legal age, Filipinos, presently residing at Barrio
Sta. Monica, [Sasmuan], Pampanga and their
children, LUNA LORELLA and KATHERINE
ROSS LAXA, who are still not of legal age and
living with their parents who would decide to
bequeath since they are the children of the
spouses;
xxxx
[Sixth] - Should other properties of mine
may be discovered aside from the properties
mentioned in this last will and testament, I am
also bequeathing and giving the same to the

More than four years after the death of Paciencia or on April


27, 2000, Lorenzo filed a petition [14] with the RTC of Guagua,
Pampanga for the probate of the Will of Paciencia and for the
issuance of Letters of Administration in his favor, docketed as Special
Proceedings No. G-1186.

There being no opposition to the petition after its due


publication, the RTC issued an Order on June 13, 2000[15] allowing
Lorenzo to present evidence on June 22, 2000. On said date, Dra.
Limpin testified that she was one of the instrumental witnesses in the
execution of the last will and testament of Paciencia on September
13, 1981.[16] The Will was executed in her fathers (Judge Limpin)
home office, in her presence and of two other witnesses, Francisco
and Faustino.[17] Dra. Limpin positively identified the Will and her
signatures on all its four pages.[18] She likewise positively identified the
signature of her father appearing thereon.[19] Questioned by the
prosecutor regarding Judge Limpins present mental fitness, Dra.
Limpin testified that her father had a stroke in 1991 and had to

49
undergo brain surgery.[20] The judge can walk but can no longer talk
On January 29, 2001, the RTC issued an Order [30] denying

and remember her name. Because of this, Dra. Limpin stated that
her father can no longer testify in court.[21]

the requests of both Lorenzo and Antonio to be appointed


administrator since the former is a citizen and resident of

The following day or on June 23, 2000, petitioner Antonio


Baltazar (Antonio) filed an opposition[22] to Lorenzos petition. Antonio

the USA while the latters claim as a co-owner of the properties


subject of the Will has not yet been established.

averred that the properties subject of Paciencias Will belong to


Nicomeda Regala Mangalindan, his predecessor-in-interest; hence,
Paciencia had no right to bequeath them to Lorenzo.

[23]

Meanwhile, proceedings on the petition for the probate of


the Will continued. Dra. Limpin was recalled for cross-examination by
the petitioners. She testified as to the age of her father at the time the

Barely a month after or on July 20, 2000, Antonio, now

latter notarized the Will of Paciencia; the living arrangements of

joined by petitioners Sebastian M. Baltazar, Virgilio Regala, Jr., Nenita

Paciencia at the time of the execution of the Will; and the lack of

A. Pacheco, Felix B. Flores, Rafael Titco, Rosie M. Mateo (Rosie) and

photographs when the event took place. [31]

Antonio

L.

Opposition

[24]

Mangalindan

filed

Supplemental

contending that Paciencias Will was null and void

Aside from Dra. Limpin, Lorenzo and Monico Mercado

because ownership of the properties had not been transferred and/or

(Monico) also took the witness stand. Monico, son of Faustino,

titled to Paciencia before her death pursuant to Article 1049,

testified on his fathers condition. According to him his father can no

paragraph 3 of the Civil Code. [25] Petitioners also opposed the

longer talk and express himself due to brain damage. A medical

issuance of Letters of Administration in Lorenzos favor arguing that

certificate was presented to the court to support this allegation. [32]

Lorenzo was disqualified to be appointed as such, he being a citizen


and resident of the USA.[26] Petitioners prayed that Letters of
Administration be instead issued in favor of Antonio.[27]

For his part, Lorenzo testified that: from 1944 until his
departure for the USA in April 1980, he lived in Sasmuan, Pampanga
with his family and his aunt, Paciencia; in 1981 Paciencia went to the

Later still on September 26, 2000, petitioners filed an


Amended Opposition

[28]

USA and lived with him and his family until her death in January 1996;

asking the RTC to deny the probate of

the relationship between him and Paciencia was like that of a mother

Paciencias Will on the following grounds: the Will was not executed

and child since Paciencia took care of him since birth and took him in

and attested to in accordance with the requirements of the law; that

as an adopted son; Paciencia was a spinster without children, and

Paciencia was mentally incapable to make a Will at the time of its

without brothers and sisters; at the time of Paciencias death, she did

execution; that she was forced to execute the Will under duress or

not suffer from any mental disorder and was of sound mind, was not

influence of fear or threats; that the execution of the Will had been

blind, deaf or mute; the Will was in the custody of Judge Limpin and

procured by undue and improper pressure and influence by Lorenzo

was only given to him after Paciencias death through Faustino; and

or by some other persons for his benefit; that the signature of

he was already residing in the USA when the Will was executed.

Paciencia on the Will was forged; that assuming the signature to be

[33]

genuine, it was obtained through fraud or trickery; and, that Paciencia

different documents and in the Will itself and stated that he was

did not intend the document to be her Will. Simultaneously,

familiar with Paciencias signature because he accompanied her in

petitioners filed an Opposition and Recommendation [29] reiterating

her transactions.[34] Further, Lorenzo belied and denied having used

their opposition to the appointment of Lorenzo as administrator of the

force, intimidation, violence, coercion or trickery upon Paciencia to

properties and requesting for the appointment of Antonio in his stead.

execute the Will as he was not in the Philippines when the same was

Lorenzo positively identified the signature of Paciencia in three

50
executed.[35] On cross-examination, Lorenzo clarified that Paciencia

him, Paciencia thought that the documents pertained to a lease of

informed him about the Will shortly after her arrival in the USA but

one of her rice lands,[51] and it was he who explained that the

that he saw a copy of the Will only after her death.[36]

documents were actually a special power of attorney to lease and sell


her fishpond and other properties upon her departure for the USA,

As to Francisco, he could no longer be presented in court


as he already died on May 21, 2000.

and a Will which would transfer her properties to Lorenzo and his
family upon her death.[52] Upon hearing this, Paciencia allegedly
uttered the following words: Why will I never [return], why will I sell all

For petitioners, Rosie testified that her mother and


Paciencia were first cousins.

[37]

my properties? Who is Lorenzo? Is he the only [son] of God? I

She claimed to have helped in the

have other relatives [who should] benefit from my properties. Why

household chores in the house of Paciencia thereby allowing her to

should I die already?[53] Thereafter, Antonio advised Paciencia not to

stay therein from morning until evening and that during the period of

sign the documents if she does not want to, to which the latter

her service in the said household, Lorenzos wife and his children

purportedly replied, I know nothing about those, throw them away or

She served in the said household

it is up to you. The more I will not sign them. [54] After which,

from 1980 until Paciencias departure for the USA on September 19,

Paciencia left the documents with Antonio. Antonio kept the unsigned

were staying in the same house.

1981.

[38]

[39]

documents
and eventually turned them over to Faustino on September 18, 1981.
On September 13, 1981, Rosie claimed that she saw

[55]

Faustino bring something for Paciencia to sign at the latters house.


[40]

Rosie admitted, though, that she did not see what that something

Ruling of the Regional Trial Court

was as same was placed inside an envelope.[41] However, she


remembered Paciencia instructing Faustino to first look for money
before she signs them.[42] A few days after or on September 16,

On September 30, 2003, the RTC rendered its


Decision[56] denying the petition thus:

1981, Paciencia went to the house of Antonios mother and brought


with her the said envelope.[43] Upon going home, however, the
envelope was no longer with Paciencia.[44] Rosie further testified that
Paciencia was referred to as magulyan or forgetful because she
would sometimes leave her wallet in the kitchen then start looking for

WHEREFORE, this court hereby (a)


denies the petition dated April 24, 2000; and (b)
disallows the notarized will dated September 13,
1981 of Paciencia Regala.
SO ORDERED.[57]

it moments later.[45] On cross examination, it was established that


Rosie was neither a doctor nor a psychiatrist, that her conclusion that
Paciencia was magulyan was based on her personal assessment,
[46]

and that it was Antonio who requested her to testify in court.[47]

The trial court gave considerable weight to the testimony of


Rosie and concluded that at the time Paciencia signed the Will, she
was no longer possessed of sufficient reason or strength of mind to
have testamentary capacity.[58]

In his direct examination, Antonio stated that Paciencia was


his aunt.[48] He identified the Will and testified that he had seen the

Ruling of the Court of Appeals

said document before because Paciencia brought the same to his


mothers house and showed it to him along with another document on
September 16, 1981.[49] Antonio alleged that when the documents
were shown to him, the same were still unsigned. [50] According to

On appeal, the CA reversed the RTC Decision and granted


the probate of the Will of Paciencia. The appellate court did not agree
with the RTCs conclusion that Paciencia was of unsound mind when

51
she executed the Will. It ratiocinated that the state of being
magulyandoes not make a person mentally unsound so [as] to
render [Paciencia] unfit for executing a Will.[59] Moreover, the
oppositors in the probate proceedings were not able to overcome the
presumption that every person is of sound mind. Further, no concrete
circumstances or events were given to prove the allegation that
Paciencia was tricked or forced into signing the Will.[60]

Faithful
complian
ce with
the
formalitie
s
laid down
by law is
apparent
from the
face of
the Will.

Petitioners moved for reconsideration[61] but the motion was


denied by the CA in its Resolution[62] dated August 31, 2006.

Courts are tasked to determine nothing more than the


extrinsic validity of a

Hence, this petition.

Will in probate proceedings.[64] This is expressly provided for in Rule


75, Section 1 of the Rules of Court, which states:
Issues
Rule 75
PRODUCTION OF WILL.
ALLOWANCE OF WILL
NECESSARY.

Petitioners come before this Court by way of Petition for


Review on Certiorari ascribing upon the CA the following errors:

Section
1. Allowance
necessary. Conclusive as to execution.
No will shall pass either real or
personal estate unless it is proved and
allowed in the proper court. Subject to
the right of appeal, such allowance of
the will shall be conclusive as to its due
execution.

I.
THE HONORABLE COURT OF APPEALS
SERIOUSLY ERRED WHEN IT ALLOWED THE
PROBATE OF PACIENCIAS WILL DESPITE
RESPONDENTS UTTER FAILURE TO
COMPLY WITH SECTION 11, RULE 76 OF THE
RULES OF COURT;
II.
THE HONORABLE COURT OF APPEALS
GRAVELY ERRED IN MAKING CONCLUSIONS
NOT IN ACCORDANCE WITH THE EVIDENCE
ON RECORD;
III.
THE HONORABLE COURT OF APPEALS
GRAVELY ERRED IN RULING THAT
PETITIONERS FAILED TO PROVE THAT
PACIENCIA WAS NOT OF SOUND MIND AT
THE TIME THE WILL WAS ALLEGEDLY
EXECUTED[63]

The pivotal issue is whether the authenticity and due


execution of the notarial Will was sufficiently established to warrant its
allowance for probate.

Our Ruling

We deny the petition.

Due execution of the will or its extrinsic validity pertains to


whether the testator, being of sound mind, freely executed the will in
accordance with the formalities prescribed by law.[65] These
formalities are enshrined in Articles 805 and 806 of the New Civil
Code, to wit:
Art. 805. Every will, other than a
holographic will, must be subscribed at the end
thereof by the testator himself or by the testator's
name written by some other person in his
presence, and by his express direction, and
attested and subscribed by three or more credible
witnesses in the presence of the testator and of
one another.
The testator or the person requested by
him to write his name and the instrumental
witnesses of the will, shall also sign, as aforesaid,
each and every page thereof, except the last, on
the left margin, and all the pages shall be
numbered correlatively in letters placed on the
upper part of each page.

52
The attestation shall state the number
of pages used upon which the will is written, and
the fact that the testator signed the will and every
page thereof, or caused some other person to
write his name, under his express direction, in the
presence of the instrumental witnesses, and that
the latter witnessed and signed the will and all the
pages thereof in the presence of the testator and
of one another.
If the attestation clause is in a language
not known to the witnesses, it shall be interpreted
to them.
Art. 806. Every will must be
acknowledged before a notary public by the
testator and the witnesses. The notary public shall
not be required to retain a copy of the will, or file
another with the Office of the Clerk of Court.

stripped her of testamentary capacity. They likewise claimed in their


Motion for Reconsideration[66] filed with the CA that Paciencia was not
only magulyan but was actually suffering from paranoia.[67]

We are not convinced.

We agree with the position of the CA that the state of being


forgetful does not necessarily make a person mentally unsound so as
to render him unfit to execute a Will.[68] Forgetfulness is not equivalent
to being of unsound mind. Besides, Article 799 of the New Civil Code
states:

Here, a careful examination of the face of the Will shows


faithful compliance with the formalities laid down by law. The
signatures of the testatrix, Paciencia, her instrumental witnesses and
the notary public, are all present and evident on the Will. Further, the
attestation clause explicitly states the critical requirement that the
testatrix and her instrumental witnesses signed the Will in the
presence of one another and that the witnesses attested and

Art. 799. To be of sound mind, it is


not necessary that the testator be in full
possession of all his reasoning faculties, or that
his mind be wholly unbroken, unimpaired, or
unshattered by disease, injury or other cause.
It shall be sufficient if the testator was
able at the time of making the will to know the
nature of the estate to be disposed of, the proper
objects of his bounty, and the character of the
testamentary act.

subscribed to the Will in the presence of the testator and of one


another. In fact, even the petitioners acceded that the signature of

In this case, apart from the testimony of Rosie pertaining to

Paciencia in the Will may be authentic although they question her

Paciencias forgetfulness, there is no substantial evidence, medical or

state of mind when she signed the same as well as the voluntary

otherwise, that would show that Paciencia was of unsound mind at

nature of said act.

the time of the execution of the Will. On the other hand, we find more
worthy of credence Dra. Limpins testimony as to the soundness of

The
burden to
prove
that
Pacienci
a was of
unsound
mind at
the time
of
the
executio
n of the
will lies
on
the
shoulder
s of the
petitioner
s.

mind of Paciencia when the latter went to Judge Limpins house and
voluntarily executed the Will. The testimony of subscribing witnesses
to a Will concerning the testators mental condition is entitled to great
weight where they are truthful and intelligent. [69] More importantly, a
testator is presumed to be of sound mind at the time of the execution
of the Will and the burden to prove otherwise lies on the
oppositor. Article 800 of the New Civil Code states:
Art. 800. The law presumes that every
person is of sound mind, in the absence of proof
to the contrary.

Petitioners, through their witness Rosie, claim that


Paciencia was magulyan or forgetful so much so that it effectively

The burden of proof that the testator


was not of sound mind at the time of making his
dispositions is on the person who opposes the
probate of the will; but if the testator, one month,
or less, before making his will was publicly known
to be insane, the person who maintains the

53
validity of the will must prove that the testator
made it during a lucid interval.

An essential element of the validity of the Will is the


willingness of the testator or testatrix to execute the document that will

Here, there was no showing that Paciencia was publicly


known to be insane one month or less before the making of the
Will. Clearly, thus, the burden to prove that Paciencia was of
unsound mind lies upon the shoulders of petitioners. However and
as earlier mentioned, no substantial evidence was presented by them
to prove the same, thereby warranting the CAs finding that petitioners
failed to discharge such burden.

distribute his/her earthly possessions upon his/her death. Petitioners


claim that Paciencia was forced to execute the Will under duress or
influence of fear or threats; that the execution of the Will had been
procured by undue and improper pressure and influence by Lorenzo
or by some other persons for his benefit; and that assuming
Paciencias signature to be genuine, it was obtained through fraud or
trickery. These are grounded on the alleged conversation between
Paciencia and Antonio on September 16, 1981 wherein the former

Furthermore, we are convinced that Paciencia was aware of

purportedly repudiated the Will and left it unsigned.

the nature of her estate to be disposed of, the proper objects of her
bounty and the character of the testamentary act. As aptly pointed
out by the CA:

We are not persuaded.


We take into consideration the unrebutted fact that
Paciencia loved and treated Lorenzo as her own son and that love

A scrutiny of the Will discloses that


[Paciencia] was aware of the nature of the
document
she
executed. She
specially
requested that the customs of her faith be
observed upon her death. She was well aware of
how she acquired the properties from her parents
and the properties she is bequeathing to
LORENZO, to his wife CORAZON and to his two
(2) children. A third child was born after the
execution of the will and was not included therein
as devisee.[70]
Bare
allegation
s
of
duress or
influence
of fear or
threats,
undue
and
improper
influence
and
pressure,
fraud and
trickery
cannot
be used
as basis
to deny
the
probate
of a will.

even extended to Lorenzos wife and children. This kind of


relationship is not unusual. It is in fact not unheard of in our culture for
old maids or spinsters to care for and raise their nephews and nieces
and treat them as their own children. Such is a prevalent and
accepted cultural practice that has resulted in many family discords
between those favored by the testamentary disposition of a testator
and those who stand to benefit in case of intestacy.

In this case, evidence shows the acknowledged fact that


Paciencias relationship with Lorenzo and his family is different from
her relationship with petitioners. The very fact that she cared for and
raised Lorenzo and lived with him both here and abroad, even if the
latter was already married and already has children, highlights the
special bond between them. This unquestioned relationship between
Paciencia and the devisees tends to support the authenticity of the
said document as against petitioners allegations of duress, influence
of fear or threats, undue and improper influence, pressure, fraud, and
trickery which, aside from being factual in nature, are not supported by
concrete, substantial and credible evidence on record. It is worth
stressing that bare arguments, no matter how forceful, if not based on
concrete and substantial evidence cannot suffice to move the Court to
uphold said allegations.[71] Furthermore, a purported will is not [to be]
denied legalization on dubious grounds. Otherwise, the very

54
institution of testamentary succession will be shaken to its foundation,

They insist that all subscribing witnesses and the notary

for even if a will has been duly executed in fact, whether x x x it will be

public should have been presented in court since all but one witness,

probated would have to depend largely on the attitude of those

Francisco, are still living.

interested in [the estate of the deceased].[72]


We cannot agree with petitioners.
Court
should
be
convince
d by the
evidence
presente
d before
it that the
Will was
duly
executed
.

We note that the inability of Faustino and Judge Limpin to


appear and testify before the court was satisfactorily explained during
the probate proceedings. As testified to by his son, Faustino had a
heart attack, was already bedridden and could no longer talk and
express himself due to brain damage. To prove this, said witness
presented the corresponding medical certificate. For her part, Dra.
Limpin testified that her father, Judge Limpin, suffered a stroke in 1991
and had to undergo brain surgery. At that time, Judge Limpin could
Petitioners dispute the authenticity of Paciencias Will on the

ground that Section 11 of Rule 76 of the Rules of Court was not


complied with. It provides:

no longer talk and could not even remember his daughters name so
that Dra. Limpin stated that given such condition, her father could no
longer testify. It is well to note that at that point, despite ample

RULE 76
ALLOWANCE OR DISALLOWANCE OF WILL
Section 11. Subscribing witnesses
produced or accounted for where will contested.
If the will is contested, all the subscribing
witnesses, and the notary in the case of wills
executed under the Civil Code of the Philippines,
if present in the Philippines and not insane, must
be produced and examined, and the death,
absence, or insanity of any of them must be
satisfactorily shown to the court. If all or some of
such witnesses are present in the Philippines but
outside the province where the will has been filed,
their deposition must be taken. If any or all of
them testify against the due execution of the will,
or do not remember having attested to it, or are
otherwise of doubtful credibility, the will may
nevertheless, be allowed if the court is satisfied
from the testimony of other witnesses and from all
the evidence presented that the will was executed
and attested in the manner required by law.

opportunity, petitioners neither interposed any objections to the

If a holographic will is contested, the


same shall be allowed if at least three (3)
witnesses who know the handwriting of the
testator explicitly declare that the will and the
signature are in the handwriting of the testator; in
the absence of any competent witnesses, and if
the court deem it necessary, expert testimony
may be resorted to. (Emphasis supplied.)

from the attesting witnesses, although they must testify, that the will

testimonies of said witnesses nor challenged the same on cross


examination. We thus hold that for all intents and purposes, Lorenzo
was able to satisfactorily account for the incapacity and failure of the
said subscribing witness and of the notary public to testify in
court. Because of this the probate of Paciencias Will may be allowed
on the basis of Dra. Limpins testimony proving her sanity and the due
execution of the Will, as well as on the proof of her handwriting. It is
an established rule that [a] testament may not be disallowed just
because the attesting witnesses declare against its due execution;
neither does it have to be necessarily allowed just because all the
attesting witnesses declare in favor of its legalization; what is decisive
is that the court is convinced by evidence before it, not necessarily

was or was not duly executed in the manner required by law.[73]

Moreover, it bears stressing that [i]rrespective x x x of the


posture of any of the parties as regards the authenticity and due
execution of the will x x x in question, it is the mandate of the law that it
is the evidence before the court and/or [evidence that] ought to be
before it that is controlling.[74] The very existence of [the Will] is in

55
itself prima facie proof that the supposed [testatrix] has willed that
[her] estate be distributed in the manner therein provided, and it is
incumbent upon the state that, if legally tenable, such desire be given
full effect independent of the attitude of the parties affected
thereby.[75] This, coupled with Lorenzos established relationship with
Paciencia, the evidence and the testimonies of disinterested

There is a distinct and consequential reason the Civil Code


provides a comprehensive catalog of imperatives for the proper
execution of a notarial will. Full and faithful compliance with all
the detailed requisites under Article 805 of the Code leave little
room for doubt as to the validity in the due execution of the
notarial will. Article 806 likewise imposes another safeguard to
the validity of notarial wills that they be acknowledged
before a notary public by the testator and the witnesses. A
notarial will executed with indifference to these two codal
provisions opens itself to nagging questions as to its
legitimacy.

witnesses, as opposed to the total lack of evidence presented by


petitioners apart from their self-serving testimonies, constrain us to tilt
the balance in favor of the authenticity of the Will and its allowance for
probate.

WHEREFORE, the petition is DENIED. The Decision


dated June 15, 2006 and the Resolution dated August 31, 2006 of the
Court of Appeals in CA-G.R. CV No. 80979 are AFFIRMED.
SO ORDERED.

The will, consisting of two (2) pages and written in the


vernacular Pilipino, read in full:
HULING HABILIN NI EUGENIA E. IGSOLO
SA NGALAN NG MAYKAPAL, AMEN:

Republic of the Philippines


SUPREME COURT
Manila
THIRD DIVISION
G.R. No. 122880

The case stems from a petition for probate filed on 10 April


1984 with the Regional Trial Court (RTC) of Manila. The
petition filed by petitioner Felix Azuela sought to admit to
probate the notarial will of Eugenia E. Igsolo, which was
notarized on 10 June 1981. Petitioner is the son of the cousin
of the decedent.

April 12, 2006

FELIX AZUELA, Petitioner,


vs.
COURT OF APPEALS, GERALDA AIDA CASTILLO
substituted by ERNESTO G. CASTILLO, Respondents.
DECISION
TINGA, J.:
The core of this petition is a highly defective notarial will,
purportedly executed by Eugenia E. Igsolo (decedent), who
died on 16 December 1982 at the age of 80. In refusing to give
legal recognition to the due execution of this document, the
Court is provided the opportunity to assert a few important
doctrinal rules in the execution of notarial wills, all self-evident
in view of Articles 805 and 806 of the Civil Code.
A will whose attestation clause does not contain the
number of pages on which the will is written is fatally
defective. A will whose attestation clause is not signed by
the instrumental witnesses is fatally defective. And
perhaps most importantly, a will which does not contain
an acknowledgment, but a mere jurat, is fatally defective.
Any one of these defects is sufficient to deny probate. A
notarial will with all three defects is just aching for judicial
rejection.

AKO, si EUGENIA E. IGSOLO, nakatira sa 500 San Diego St.,


Sampaloc, Manila, pitongput siyam (79) na gulang, nasa
hustong pagi-isip, pag-unawa at memoria ay nag-hahayag na
ito na ang aking huling habilin at testamento, at binabali wala
ko lahat ang naunang ginawang habilin o testamento:
Una-Hinihiling ko na ako ay mailibing sa Sementerio del Norte,
La Loma sang-ayong sa kaugalian at patakaran ng simbahang
katoliko at ang taga-pag-ingat (Executor) ng habiling ito ay
magtatayo ng bantayog upang silbing ala-ala sa akin ng aking
pamilya at kaibigan;
Pangalawa-Aking ipinagkakaloob at isinasalin ang lahat ng
karapatan sa aking pamangkin na si Felix Azuela, na siyang
nag-alaga sa akin sa mahabang panahon, yaong mga bahay
na nakatirik sa lote numero 28, Block 24 at nakapangalan sa
Pechaten Korporasyon, ganoon din ibinibigay ko ang lahat ng
karapatan sa bahay na nakatirik sa inoopahan kong lote,
numero 43, Block 24 na pag-aari ng Pechaten Corporation.
Ipinagkakaloob kong buong buo ang lahat ng karapatan sa
bahay at lupa na nasa 500 San Diego St., Lot 42, Block 24,
Sampaloc, Manila kay Felix Azuela at ang pagkakaloob kong
ito ay walang pasubalit at kondiciones;
Pangatlo- Na ninunumbrahan ko si VART PAGUE na siyang
nagpapatupad ng huling habiling ito at kagustuhan ko rin na
hindi na kailanman siyang mag-lagak ng piyansiya.
Aking nilagdaan ang Huling Habilin na ito dito sa Maynila ika
10 ng Hunyo, 1981.
(Sgd.)
EUGENIA E. IGSOLO
(Tagapagmana)
PATUNAY NG MGA SAKSI

56
Ang kasulatang ito, na binubuo ng ____ dahon pati ang huling
dahong ito, na ipinahayag sa amin ni Eugenia E. Igsolo,
tagapagmana na siya niyang Huling Habilin, ngayon ika-10 ng
Hunyo 1981, ay nilagdaan ng nasabing tagapagmana sa ilalim
ng kasulatang nabanggit at sa kaliwang panig ng lahat at
bawat dahon, sa harap ng lahat at bawat sa amin, at kami
namang mga saksi ay lumagda sa harap ng nasabing
tagapagmana at sa harap ng lahat at bawat isa sa amin, sa
ilalim ng nasabing kasulatan at sa kaliwang panig ng lahat at
bawat dahon ng kasulatan ito.
EUGENIA E. IGSOLO
address: 500 San Diego St.
Sampaloc, Manila Res. Cert. No. A-7717-37
Issued at Manila on March 10, 1981.
QUIRINO AGRAVA
address: 1228-Int. 3, Kahilum
Pandacan, Manila Res. Cert. No. A-458365
Issued at Manila on Jan. 21, 1981
LAMBERTO C. LEAO
address: Avenue 2, Blcok 7,
Lot 61, San Gabriel, G.MA., Cavite Res.
Cert. No. A-768277 issued at Carmona, Cavite on Feb. 7, 1981
JUANITO ESTRERA
address: City Court Compound,
City of Manila Res. Cert. No. A574829
Issued at Manila on March 2, 1981.
Nilagdaan ko at ninotario ko ngayong 10 ng Hunyo 10, 1981
dito sa Lungsod ng Maynila.
(Sgd.)
PETRONIO Y. BAUTISTA
Doc. No. 1232 ; NOTARIO PUBLIKO
Page No. 86 ; Until Dec. 31, 1981
Book No. 43 ; PTR-152041-1/2/81-Manila
Series of 1981 TAN # 1437-977-81
The three named witnesses to the will affixed their signatures
on the left-hand margin of both pages of the will, but not at the
bottom of the attestation clause.
The probate petition adverted to only two (2) heirs, legatees
and devisees of the decedent, namely: petitioner himself, and
one Irene Lynn Igsolo, who was alleged to have resided
abroad. Petitioner prayed that the will be allowed, and that
letters testamentary be issued to the designated executor, Vart
Prague.
The petition was opposed by Geralda Aida Castillo (Geralda
Castillo), who represented herself as the attorney-in-fact of "the
12 legitimate heirs" of the decedent.2 Geralda Castillo claimed
that the will is a forgery, and that the true purpose of its
emergence was so it could be utilized as a defense in several
court cases filed by oppositor against petitioner, particularly for
forcible entry and usurpation of real property, all centering on
petitioners right to occupy the properties of the decedent.3 It
also asserted that contrary to the representations of petitioner,
the decedent was actually survived by 12 legitimate heirs,

namely her grandchildren, who were then residing abroad. Per


records, it was subsequently alleged that decedent was the
widow of Bonifacio Igsolo, who died in 1965,4 and the mother
of a legitimate child, Asuncion E. Igsolo, who predeceased her
mother by three (3) months.5
Oppositor Geralda Castillo also argued that the will was not
executed and attested to in accordance with law. She pointed
out that decedents signature did not appear on the second
page of the will, and the will was not properly acknowledged.
These twin arguments are among the central matters to this
petition.
After due trial, the RTC admitted the will to probate, in an
Order dated 10 August 1992.6 The RTC favorably took into
account the testimony of the three (3) witnesses to the will,
Quirino Agrava, Lamberto Leano, and Juanito Estrada. The
RTC also called to fore "the modern tendency in respect to the
formalities in the execution of a will x x x with the end in view of
giving the testator more freedom in expressing his last
wishes;"7 and from this perspective, rebutted oppositors
arguments that the will was not properly executed and attested
to in accordance with law.
After a careful examination of the will and consideration of the
testimonies of the subscribing and attesting witnesses, and
having in mind the modern tendency in respect to the
formalities in the execution of a will, i.e., the liberalization of the
interpretation of the law on the formal requirements of a will
with the end in view of giving the testator more freedom in
expressing his last wishes, this Court is persuaded to rule that
the will in question is authentic and had been executed by the
testatrix in accordance with law.
On the issue of lack of acknowledgement, this Court has noted
that at the end of the will after the signature of the testatrix, the
following statement is made under the sub-title, "Patunay Ng
Mga Saksi":
"Ang kasulatang ito, na binubuo ng _____ dahon pati ang
huling dahong ito, na ipinahayag sa amin ni Eugenia N. Igsolo,
tagapagmana na siya niyang Huling Habilin, ngayong ika-10
ng Hunyo 1981, ay nilagdaan ng nasabing tagapagmana sa
ilalim ng kasulatang nabanggit at sa kaliwang panig ng lahat at
bawat dahon, sa harap ng lahat at bawat sa amin, at kami
namang mga saksi ay lumagda sa harap ng nasabing
tagapagmana at sa harap ng lahat at bawat isa sa amin, sa
ilalim ng nasabing kasulatan at sa kaliwang panig ng lahat at
bawat dahon ng kasulatan ito."
The aforequoted declaration comprises the attestation clause
and the acknowledgement and is considered by this Court as a
substantial compliance with the requirements of the law.
On the oppositors contention that the attestation clause was
not signed by the subscribing witnesses at the bottom thereof,
this Court is of the view that the signing by the subscribing
witnesses on the left margin of the second page of the will
containing the attestation clause and acknowledgment, instead
of at the bottom thereof, substantially satisfies the purpose of
identification and attestation of the will.

57
With regard to the oppositors argument that the will was not
numbered correlatively in letters placed on upper part of each
page and that the attestation did not state the number of pages
thereof, it is worthy to note that the will is composed of only two
pages. The first page contains the entire text of the
testamentary dispositions, and the second page contains the
last portion of the attestation clause and acknowledgement.
Such being so, the defects are not of a serious nature as to
invalidate the will. For the same reason, the failure of the
testatrix to affix her signature on the left margin of the second
page, which contains only the last portion of the attestation
clause and acknowledgment is not a fatal defect.
As regards the oppositors assertion that the signature of the
testatrix on the will is a forgery, the testimonies of the three
subscribing witnesses to the will are convincing enough to
establish the genuineness of the signature of the testatrix and
the due execution of the will.8
The Order was appealed to the Court of Appeals by Ernesto
Castillo, who had substituted his since deceased mother-inlaw, Geralda Castillo. In a Decision dated 17 August 1995, the
Court of Appeals reversed the trial court and ordered the
dismissal of the petition for probate.9 The Court of Appeals
noted that the attestation clause failed to state the number of
pages used in the will, thus rendering the will void and
undeserving of probate.10
Hence, the present petition.
Petitioner argues that the requirement under Article 805 of the
Civil Code that "the number of pages used in a notarial will be
stated in the attestation clause" is merely directory, rather than
mandatory, and thus susceptible to what he termed as "the
substantial compliance rule."11
The solution to this case calls for the application of Articles 805
and 806 of the Civil Code, which we replicate in full.
Art. 805. Every will, other than a holographic will, must be
subscribed at the end thereof by the testator himself or by the
testator's name written by some other person in his presence,
and by his express direction, and attested and subscribed by
three or more credible witnesses in the presence of the testator
and of one another.
The testator or the person requested by him to write his name
and the instrumental witnesses of the will, shall also sign, as
aforesaid, each and every page thereof, except the last, on the
left margin, and all the pages shall be numbered correlatively
in letters placed on the upper part of each page.
The attestation shall state the number of pages used upon
which the will is written, and the fact that the testator signed
the will and every page thereof, or caused some other person
to write his name, under his express direction, in the presence
of the instrumental witnesses, and that the latter witnessed and
signed the will and all the pages thereof in the presence of the
testator and of one another.
If the attestation clause is in a language not known to the
witnesses, it shall be interpreted to them.

Art. 806. Every will must be acknowledged before a notary


public by the testator and the witnesses. The notary public
shall not be required to retain a copy of the will, or file another
with the office of the Clerk of Court.
The appellate court, in its Decision, considered only one
defect, the failure of the attestation clause to state the number
of pages of the will. But an examination of the will itself reveals
several more deficiencies.
As admitted by petitioner himself, the attestation clause fails to
state the number of pages of the will. 12 There was an
incomplete attempt to comply with this requisite, a space
having been allotted for the insertion of the number of pages in
the attestation clause. Yet the blank was never filled in; hence,
the requisite was left uncomplied with.
The Court of Appeals pounced on this defect in reversing the
trial court, citing in the process Uy Coque v. Navas L.
Sioca13 and In re: Will of Andrada.14 In Uy Coque, the Court
noted that among the defects of the will in question was the
failure of the attestation clause to state the number of pages
contained in the will.15 In ruling that the will could not be
admitted to probate, the Court made the following
consideration which remains highly relevant to this day: "The
purpose of requiring the number of sheets to be stated in the
attestation clause is obvious; the document might easily be
so prepared that the removal of a sheet would completely
change the testamentary dispositions of the will and in the
absence of a statement of the total number of sheets such
removal might be effected by taking out the sheet and
changing the numbers at the top of the following sheets or
pages. If, on the other hand, the total number of sheets is
stated in the attestation clause the falsification of the document
will involve the inserting of new pages and the forging of the
signatures of the testator and witnesses in the margin, a matter
attended with much greater difficulty."16
The case of In re Will of Andrada concerned a will the
attestation clause of which failed to state the number of sheets
or pages used. This consideration alone was sufficient for the
Court to declare "unanim[ity] upon the point that the defect
pointed out in the attesting clause is fatal."17 It was further
observed that "it cannot be denied that the x x x requirement
affords additional security against the danger that the will may
be tampered with; and as the Legislature has seen fit to
prescribe this requirement, it must be considered material."18
Against these cited cases, petitioner cites Singson v.
Florentino19 and Taboada v. Hon. Rosal,20 wherein the Court
allowed probate to the wills concerned therein despite the fact
that the attestation clause did not state the number of pages of
the will. Yet the appellate court itself considered the import of
these two cases, and made the following distinction which
petitioner is unable to rebut, and which we adopt with approval:
Even a cursory examination of the Will (Exhibit "D"), will readily
show that the attestation does not state the number of pages
used upon which the will is written. Hence, the Will is void and
undeserving of probate.
We are not impervious of the Decisions of the Supreme Court
in "Manuel Singson versus Emilia Florentino, et al., 92 Phil.
161 and Apolonio [Taboada] versus Hon. Avelino Rosal, et al.,

58
118 SCRA 195," to the effect that a will may still be valid even if
the attestation does not contain the number of pages used
upon which the Will is written. However, the Decisions of the
Supreme Court are not applicable in the aforementioned
appeal at bench. This is so because, in the case of "Manuel
Singson versus Emilia Florentino, et al., supra," although the
attestation in the subject Will did not state the number of pages
used in the will, however, the same was found in the last part
of the body of the Will:
"x x x
The law referred to is article 618 of the Code of Civil
Procedure, as amended by Act No. 2645, which requires that
the attestation clause shall state the number of pages or
sheets upon which the will is written, which requirement has
been held to be mandatory as an effective safeguard against
the possibility of interpolation or omission of some of the pages
of the will to the prejudice of the heirs to whom the property is
intended to be bequeathed (In re Will of Andrada, 42 Phil. 180;
Uy Coque vs. Navas L. Sioca, 43 Phil., 405; Gumban vs.
Gorcho, 50 Phil. 30; Quinto vs. Morata, 54 Phil. 481;
Echevarria vs. Sarmiento, 66 Phil. 611). The ratio decidendi of
these cases seems to be that the attestation clause must
contain a statement of the number of sheets or pages
composing the will and that if this is missing or is omitted, it will
have the effect of invalidating the will if the deficiency cannot
be supplied, not by evidence aliunde, but by a consideration or
examination of the will itself. But here the situation is different.
While the attestation clause does not state the number of
sheets or pages upon which the will is written, however, the
last part of the body of the will contains a statement that it is
composed of eight pages, which circumstance in our opinion
takes this case out of the rigid rule of construction and places it
within the realm of similar cases where a broad and more
liberal view has been adopted to prevent the will of the testator
from being defeated by purely technical considerations." (page
165-165, supra) (Underscoring supplied)
In "Apolonio Tabaoda versus Hon. Avelino Rosal, et al." supra,
the notarial acknowledgement in the Will states the number of
pages used in the:
"x x x
We have examined the will in question and noticed that the
attestation clause failed to state the number of pages used in
writing the will. This would have been a fatal defect were it not
for the fact that, in this case, it is discernible from the entire will
that it is really and actually composed of only two pages duly
signed by the testatrix and her instrumental witnesses. As
earlier stated, the first page which contains the entirety of the
testamentary dispositions is signed by the testatrix at the end
or at the bottom while the instrumental witnesses signed at the
left margin. The other page which is marked as "Pagina dos"
comprises the attestation clause and the acknowledgment. The
acknowledgment itself states that "this Last Will and Testament
consists of two pages including this page" (pages 200-201,
supra) (Underscoring supplied).
However, in the appeal at bench, the number of pages used in
the will is not stated in any part of the Will. The will does not
even contain any notarial acknowledgment wherein the
number of pages of the will should be stated.21

Both Uy Coque and Andrada were decided prior to the


enactment of the Civil Code in 1950, at a time when the
statutory provision governing the formal requirement of wills
was Section
618 of the Code of Civil Procedure.22 Reliance on these cases
remains apropos, considering that the requirement that the
attestation state the number of pages of the will is extant from
Section 618.23 However, the enactment of the Civil Code in
1950 did put in force a rule of interpretation of the requirements
of wills, at least insofar as the attestation clause is concerned,
that may vary from the philosophy that governed these two
cases. Article 809 of the Civil Code states: "In the absence of
bad faith, forgery, or fraud, or undue and improper pressure
and influence, defects and imperfections in the form of
attestation or in the language used therein shall not render the
will invalid if it is proved that the will was in fact executed and
attested in substantial compliance with all the requirements of
article 805."
In the same vein, petitioner cites the report of the Civil Code
Commission, which stated that "the underlying and
fundamental objective permeating the provisions on the [law]
on [wills] in this project consists in the [liberalization] of the
manner of their execution with the end in view of giving the
testator more [freedom] in [expressing] his last wishes. This
objective is in accord with the [modern tendency] in respect to
the formalities in the execution of wills."24 However, petitioner
conveniently omits the qualification offered by the Code
Commission in the very same paragraph he cites from their
report, that such liberalization be "but with sufficient safeguards
and restrictions to prevent the commission of fraud and the
exercise of undue and improper pressure and influence upon
the testator."25
Caneda v. Court of Appeals26 features an extensive discussion
made by Justice Regalado, speaking for the Court on the
conflicting views on the manner of interpretation of the legal
formalities required in the execution of the attestation clause in
wills.27 Uy Coque and Andrada are cited therein, along with
several other cases, as examples of the application of the rule
of strict construction.28 However, the Code Commission opted
to recommend a more liberal construction through the
"substantial compliance rule" under Article 809. A cautionary
note was struck though by Justice J.B.L. Reyes as to how
Article 809 should be applied:
x x x The rule must be limited to disregarding those defects
that can be supplied by an examination of the will itself:
whether all the pages are consecutively numbered; whether
the signatures appear in each and every page; whether the
subscribing witnesses are three or the will was notarized. All
these are facts that the will itself can reveal, and defects or
even omissions concerning them in the attestation clause can
be safely disregarded. But the total number of pages, and
whether all persons required to sign did so in the
presence of each other must substantially appear in the
attestation clause, being the only check against perjury in
the probate proceedings.29 (Emphasis supplied.)
The Court of Appeals did cite these comments by Justice
J.B.L. Reyes in its assailed decision, considering that the
failure to state the number of pages of the will in the attestation
clause is one of the defects which cannot be simply

59
disregarded. In Caneda itself, the Court refused to allow the
probate of a will whose attestation clause failed to state that
the witnesses subscribed their respective signatures to the will
in the presence of the testator and of each other,30 the other
omission cited by Justice J.B.L. Reyes which to his estimation
cannot be lightly disregarded.

For one, the attestation clause was not signed by the


instrumental witnesses. While the signatures of the
instrumental witnesses appear on the left-hand margin of the
will, they do not appear at the bottom of the attestation clause
which after all consists of their averments before the notary
public.

Caneda suggested: "[I]t may thus be stated that the rule, as it


now stands, is that omission which can be supplied by an
examination of the will itself, without the need of resorting to
extrinsic evidence, will not be fatal and, correspondingly, would
not obstruct the allowance to probate of the will being assailed.
However, those omissions which cannot be supplied except by
evidence aliunde would result in the invalidation of the
attestation clause and ultimately, of the will itself."31 Thus, a
failure by the attestation clause to state that the testator signed
every page can be liberally construed, since that fact can be
checked by a visual examination; while a failure by the
attestation clause to state that the witnesses signed in one
anothers presence should be considered a fatal flaw since the
attestation is the only textual guarantee of compliance.32

Cagro v. Cagro36 is material on this point. As in this case, "the


signatures of the three witnesses to the will do not appear at
the bottom of the attestation clause, although the page
containing the same is signed by the witnesses on the lefthand margin."37 While three (3) Justices38 considered the
signature requirement had been substantially complied with, a
majority of six (6), speaking through Chief Justice Paras, ruled
that the attestation clause had not been duly signed, rendering
the will fatally defective.

The failure of the attestation clause to state the number of


pages on which the will was written remains a fatal flaw,
despite Article 809. The purpose of the law in requiring the
clause to state the number of pages on which the will is written
is to safeguard against possible interpolation or omission of
one or some of its pages and to prevent any increase or
decrease in the pages.33 The failure to state the number of
pages equates with the absence of an averment on the part of
the instrumental witnesses as to how many pages consisted
the will, the execution of which they had ostensibly just
witnessed and subscribed to. Following Caneda, there is
substantial compliance with this requirement if the will states
elsewhere in it how many pages it is comprised of, as was the
situation in Singson andTaboada. However, in this case, there
could have been no substantial compliance with the
requirements under Article 805 since there is no statement in
the attestation clause or anywhere in the will itself as to the
number of pages which comprise the will.

We are of the opinion that the position taken by the appellant is


correct. The attestation clause is "a memorandum of the facts
attending the execution of the will" required by law to be made
by the attesting witnesses, and it must necessarily bear their
signatures. An unsigned attestation clause cannot be
considered as an act of the witnesses, since the omission of
their signatures at the bottom thereof negatives their
participation.

At the same time, Article 809 should not deviate from the need
to comply with the formal requirements as enumerated under
Article 805. Whatever the inclinations of the members of the
Code Commission in incorporating Article 805, the fact remains
that they saw fit to prescribe substantially the same formal
requisites as enumerated in Section 618 of the Code of Civil
Procedure, convinced that these remained effective
safeguards against the forgery or intercalation of notarial
wills.34 Compliance with these requirements, however picayune
in impression, affords the public a high degree of comfort that
the testator himself or herself had decided to convey
property post mortem in the manner established in the
will.35 The transcendent legislative intent, even as
expressed in the cited comments of the Code
Commission, is for the fruition of the testators
incontestable desires, and not for the indulgent admission
of wills to probate.
The Court could thus end here and affirm the Court of Appeals.
However, an examination of the will itself reveals a couple of
even more critical defects that should necessarily lead to its
rejection.

There is no question that the signatures of the three witnesses


to the will do not appear at the bottom of the attestation clause,
although the page containing the same is signed by the
witnesses on the left-hand margin.

The petitioner and appellee contends that signatures of the


three witnesses on the left-hand margin conform substantially
to the law and may be deemed as their signatures to the
attestation clause. This is untenable, because said signatures
are in compliance with the legal mandate that the will be
signed on the left-hand margin of all its pages. If an attestation
clause not signed by the three witnesses at the bottom thereof,
be admitted as sufficient, it would be easy to add such clause
to a will on a subsequent occasion and in the absence of the
testator and any or all of the witnesses.39
The Court today reiterates the continued efficacy of Cagro.
Article 805 particularly segregates the requirement that the
instrumental witnesses sign each page of the will, from the
requisite that the will be "attested and subscribed by [the
instrumental witnesses]." The respective intents behind these
two classes of signature are distinct from each other. The
signatures on the left-hand corner of every page signify, among
others, that the witnesses are aware that the page they are
signing forms part of the will. On the other hand, the signatures
to the attestation clause establish that the witnesses are
referring to the statements contained in the attestation clause
itself. Indeed, the attestation clause is separate and apart from
the disposition of the will. An unsigned attestation clause
results in an unattested will. Even if the instrumental witnesses
signed the left-hand margin of the page containing the
unsigned attestation clause, such signatures cannot
demonstrate these witnesses undertakings in the clause, since
the signatures that do appear on the page were directed
towards a wholly different avowal.
The Court may be more charitably disposed had the witnesses
in this case signed the attestation clause itself, but not the left-

60
hand margin of the page containing such clause. Without
diminishing the value of the instrumental witnesses signatures
on each and every page, the fact must be noted that it is the
attestation clause which contains the utterances reduced into
writing of the testamentary witnesses themselves. It is the
witnesses, and not the testator, who are required under Article
805 to state the number of pages used upon which the will is
written; the fact that the testator had signed the will and every
page thereof; and that they witnessed and signed the will and
all the pages thereof in the presence of the testator and of one
another. The only proof in the will that the witnesses have
stated these elemental facts would be their signatures on the
attestation clause.
Thus, the subject will cannot be considered to have been
validly attested to by the instrumental witnesses, as they failed
to sign the attestation clause.
Yet, there is another fatal defect to the will on which the denial
of this petition should also hinge. The requirement under Article
806 that "every will must be acknowledged before a notary
public by the testator and the witnesses" has also not been
complied with. The importance of this requirement is
highlighted by the fact that it had been segregated from the
other requirements under Article 805 and entrusted into a
separate provision, Article 806. The non-observance of Article
806 in this case is equally as critical as the other cited flaws in
compliance with Article 805, and should be treated as of
equivalent import.
In lieu of an acknowledgment, the notary public, Petronio Y.
Bautista, wrote "Nilagdaan ko at ninotario ko ngayong10 ng
Hunyo 10 (sic), 1981 dito sa Lungsod ng Maynila."40 By no
manner of contemplation can those words be construed as an
acknowledgment. An acknowledgment is the act of one who
has executed a deed in going before some competent officer
or court and declaring it to be his act or deed.41 It involves an
extra step undertaken whereby the signor actually declares to
the notary that the executor of a document has attested to the
notary that the same is his/her own free act and deed.
It might be possible to construe the averment as a jurat, even
though it does not hew to the usual language thereof. A jurat is
that part of an affidavit where the notary certifies that before
him/her, the document was subscribed and sworn to by the
executor.42 Ordinarily, the language of the jurat should avow
that the document was subscribed and sworn before the notary
public, while in this case, the notary public averred that he
himself "signed and notarized" the document. Possibly though,
the word "ninotario" or "notarized" encompasses the signing of
and swearing in of the executors of the document, which in this
case would involve the decedent and the instrumental
witnesses.
Yet even if we consider what was affixed by the notary public
as a jurat, the will would nonetheless remain invalid, as the
express requirement of Article 806 is that the will be
"acknowledged", and not merely subscribed and sworn to. The
will does not present any textual proof, much less one under
oath, that the decedent and the instrumental witnesses
executed or signed the will as their own free act or deed. The
acknowledgment made in a will provides for another allimportant legal safeguard against spurious wills or those made
beyond the free consent of the testator. An acknowledgement

is not an empty meaningless act.43 The acknowledgment


coerces the testator and the instrumental witnesses to declare
before an officer of the law that they had executed and
subscribed to the will as their own free act or deed. Such
declaration is under oath and under pain of perjury, thus
allowing for the criminal prosecution of persons who participate
in the execution of spurious wills, or those executed without the
free consent of the testator. It also provides a further degree of
assurance that the testator is of certain mindset in making the
testamentary dispositions to those persons he/she had
designated in the will.
It may not have been said before, but we can assert the rule,
self-evident as it is under Article 806. A notarial will that is
not acknowledged before a notary public by the testator
and the witnesses is fatally defective, even if it is
subscribed and sworn to before a notary public.
There are two other requirements under Article 805 which were
not fully satisfied by the will in question. We need not discuss
them at length, as they are no longer material to the
disposition of this case. The provision requires that the testator
and the instrumental witnesses sign each and every page of
the will on the left margin, except the last; and that all the
pages shall be numbered correlatively in letters placed on the
upper part of each page. In this case, the decedent, unlike the
witnesses, failed to sign both pages of the will on the left
margin, her only signature appearing at the so-called "logical
end"44 of the will on its first page. Also, the will itself is not
numbered correlatively in letters on each page, but instead
numbered with Arabic numerals. There is a line of thought that
has disabused the notion that these two requirements be
construed as mandatory.45 Taken in isolation, these omissions,
by themselves, may not be sufficient to deny probate to a will.
Yet even as these omissions are not decisive to the
adjudication of this case, they need not be dwelt on, though
indicative as they may be of a general lack of due regard for
the requirements under Article 805 by whoever executed the
will.
All told, the string of mortal defects which the will in question
suffers from makes the probate denial inexorable.
WHEREFORE, the petition is DENIED. Costs against
petitioner.
SO ORDERED.
DANTE O. TINGA
Associate Justice

THIRD DIVISION
LETICIA VALMONTE ORTEGA,
Petitioner,
- versus -

G.R. No. 157451

61
JOSEFINA C. VALMONTE,

Promulgated:

Respondent.
December 16, 2005
x -- -- -- -- -- -- -- -- -- -- -- -- -- -- -- -- -- -- -- -- -- -- -- -- -- -- -- -- x
DECISION
PANGANIBAN, J.:
The law favors the probate of a will. Upon those who
oppose it rests the burden of showing why it should not be
allowed. In the present case, petitioner has failed to
discharge this burden satisfactorily. For this reason, the
Court cannot attribute any reversible error on the part of the
appellate tribunal that allowed the probate of the will.
The Case

Before the Court is a Petition for Review[1] under Rule


45 of the Rules of Court, seeking to reverse and set aside the
December 12, 2002 Decision[2] and the March 7, 2003
Resolution[3] of the Court of Appeals (CA) in CA-GR CV No.
44296. The assailed Decision disposed as follows:
WHEREFORE,
the
appeal
is GRANTED, and the Decision appealed
from is REVERSED and SET ASIDE. In its
place judgment is rendered approving and
allowing probate to the said last will and
testament of Placido Valmonte and ordering
the issuance of letters testamentary to the
petitioner Josefina Valmonte. Let this case
be remanded to the court a quo for further
and concomitant proceedings.[4]

The assailed Resolution denied petitioners Motion for


Reconsideration.

The Facts
The facts were summarized in the assailed Decision
of the CA, as follows:
x x x: Like so many others before
him, Placido toiled and lived for a long time
in the United States until he finally reached
retirement. In 1980, Placido finally came
home to stay in the Philippines, and he lived
in the house and lot located at #9200
Catmon St., San Antonio Village, Makati,

which he owned in common with his sister


Ciriaca Valmonte and titled in their names in
TCT 123468. Two years after his arrival
from the United States and at the age of 80
he wed Josefina who was then 28 years old,
in a ceremony solemnized by Judge Perfecto
Laguio, Jr. on February 5, 1982. But in a
little more than two years of wedded bliss,
Placido died on October 8, 1984 of a cause
written down as COR PULMONALE.

Placido executed a notarial last will


and testament written in English and
consisting of two (2) pages, and dated June
15, 1983 but acknowledged only on August
9, 1983. The first page contains the entire
testamentary dispositions and a part of the
attestation clause, and was signed at the
end or bottom of that page by the testator
and on the left hand margin by the three
instrumental witnesses. The second page
contains the continuation of the attestation
clause and the acknowledgment, and was
signed by the witnesses at the end of the
attestation clause and again on the left hand
margin. It provides in the body that:

LAST WILL AND TESTAMENT OF PLACIDO


VALMONTE IN THE NAME OF THE LORD
AMEN:

I, PLACIDO VALMONTE, of legal age,


married to Josefina Cabansag Valmonte, and a
resident of 9200 Catmon Street, Makati, Metro
Manila, 83 years of age and being of sound
and disposing mind and memory, do hereby
declare this to be my last will and testament:
1. It is my will that I be buried in the Catholic
Cemetery, under the auspices of the Catholic
Church in accordance with the rites and said
Church and that a suitable monument to be
erected and provided my by executrix (wife) to
perpetuate my memory in the minds of my
family and friends;
2.
I give, devise and bequeath unto my
loving wife, JOSEFINA C. VALMONTE, one
half (1/2) portion of the follow-described
properties, which belongs to me as [co-owner]:
a.
Lot 4-A, Block 13 described on plan
Psd-28575, LRC, (GLRO), situated in Makati,

62
Metro Manila, described and covered by TCT
No. 123468 of the Register of Deeds of Pasig,
Metro-Manila registered jointly as co-owners
with my deceased sister (Ciriaca Valmonte),
having share and share alike;
b.
2-storey building standing on the abovedescribed property, made of strong and mixed
materials used as my residence and my wife
and located at No. 9200 Catmon Street,
Makati, Metro Manila also covered by Tax
Declaration No. A-025-00482, Makati, MetroManila, jointly in the name of my deceased
sister, Ciriaca Valmonte and myself as coowners, share and share alike or equal coowners thereof;
3.
All the rest, residue and remainder
of my real and personal properties,
including my savings account bank book
in USA which is in the possession of my
nephew, and all others whatsoever and
wherever found, I give, devise and
bequeath to my said wife, Josefina C.
Valmonte;
4.
I hereby appoint my wife, Josefina
C. Valmonte as sole executrix of my last
will and testament, and it is my will that
said executrix be exempt from filing a
bond;
IN WITNESS WHEREOF, I have
hereunto set my hand this 15th day of June
1983 in Quezon City, Philippines.

The allowance to probate of this will was


opposed by Leticia on the grounds that:
1. Petitioner failed to allege all assets
of the testator, especially those found
in the USA;
2. Petitioner failed to state the names,
ages, and residences of the heirs of
the testator; or to give them proper
notice pursuant to law;
3. Will was not executed and attested
as required by law and legal
solemnities and formalities were not
complied with;
4. Testator was mentally incapable to
make a will at the time of the alleged
execution he being in an advance sate
of senility;

5.
Will was executed under duress,
or the influence of fear or threats;
6.
Will was procured by undue and
improper influence and pressure on the
part of the petitioner and/or her agents
and/or assistants; and/or
7.
Signature
of
testator
was
procured by fraud, or trick, and he did
not intend that the instrument should
be his will at the time of affixing his
signature thereto;

and she also opposed the appointment as


Executrix of Josefina alleging her want of
understanding and integrity.
At the hearing, the petitioner
Josefina testified and called as witnesses the
notary public Atty. Floro Sarmiento who
prepared and notarized the will, and the
instrumental witnesses spouses Eugenio
Gomez, Jr. and Feliza Gomez and Josie
Collado. For the opposition, the oppositor
Leticia and her daughter Mary Jane Ortega
testified.
According to Josefina after her
marriage with the testator they lived in her
parents house at Salingcob, Bacnotan, La
Union but they came to Manila every month
to get his $366.00 monthly pension and
stayed at the said Makati residence. There
were times though when to shave off on
expenses, the testator would travel alone.
And it was in one of his travels by his
lonesome self when the notarial will was
made. The will was witnessed by the
spouses Eugenio and Feliza Gomez, who
were their wedding sponsors, and by Josie
Collado. Josefina said she had no
knowledge of the existence of the last will
and testament of her husband, but just
serendipitously found it in his attache case
after his death. It was only then that she
learned that the testator bequeathed to her
his properties and she was named the
executrix in the said will. To her estimate,
the value of property both real and personal
left by the testator is worth more or less
P100,000.00. Josefina declared too that the
testator never suffered mental infirmity
because despite his old age he went alone
to the market which is two to three

63
kilometers from their home cooked and
cleaned the kitchen and sometimes if she
could not accompany him, even traveled to
Manila alone to claim his monthly pension.
Josefina also asserts that her husband was
in good health and that he was hospitalized
only because of a cold but which eventually
resulted in his death.
Notary Public Floro Sarmiento, the
notary public who notarized the testators
will, testified that it was in the first week of
June 1983 when the testator together with
the three witnesses of the will went to his
house cum law office and requested him to
prepare his last will and testament. After the
testator instructed him on the terms and
dispositions he wanted on the will, the notary
public told them to come back on June 15,
1983 to give him time to prepare it. After he
had prepared the will the notary public kept it
safely hidden and locked in his drawer. The
testator and his witnesses returned on the
appointed date but the notary public was out
of town so they were instructed by his wife to
come back on August 9, 1983, and which
they did. Before the testator and his
witnesses signed the prepared will, the
notary public explained to them each and
every term thereof in Ilocano, a dialect which
the testator spoke and understood. He
likewise explained that though it appears that
the will was signed by the testator and his
witnesses on June 15, 1983, the day when it
should have been executed had he not gone
out of town, the formal execution was
actually on August 9, 1983. He reasoned
that he no longer changed the typewritten
date of June 15, 1983 because he did not
like the document to appear dirty. The
notary public also testified that to his
observation the testator was physically and
mentally capable at the time he affixed his
signature on the will.
The attesting witnesses to the will
corroborated the testimony of the notary
public, and testified that the testator went
alone to the house of spouses Eugenio and
Feliza Gomez at GSIS Village, Quezon City
and requested them to accompany him to
the house of Atty. Floro Sarmiento purposely
for his intended will; that after giving his
instructions to Atty. Floro Sarmiento, they
were told to return on June 15, 1983; that
they returned on June 15, 1983 for the
execution of the will but were asked to come
back instead on August 9, 1983 because of

the absence of the notary public; that the


testator executed the will in question in their
presence while he was of sound and
disposing mind and that he was strong and
in good health; that the contents of the will
was explained by the notary public in the
Ilocano and Tagalog dialect and that all of
them as witnesses attested and signed the
will in the presence of the testator and of
each other. And that during the execution,
the testators wife, Josefina was not with
them.
The oppositor Leticia declared that
Josefina should not inherit alone because
aside from her there are other children from
the siblings of Placido who are just as
entitled to inherit from him. She attacked the
mental capacity of the testator, declaring that
at the time of the execution of the notarial
will the testator was already 83 years old and
was no longer of sound mind. She knew
whereof she spoke because in 1983 Placido
lived in the Makati residence and asked
Leticias family to live with him and they took
care of him. During that time, the testators
physical and mental condition showed
deterioration, aberrations and senility. This
was corroborated by her daughter Mary Jane
Ortega for whom Placido took a fancy and
wanted to marry.
Sifting through the evidence, the
court a
quo held
that [t]he
evidence
adduced, reduces the opposition to two
grounds, namely:
1.

Non-compliance with the legal


solemnities and formalities in
the execution and attestation of
the will; and

2.

Mental incapacity of the


testator at the time of the
execution of the will as he was
then in an advanced state of
senility

It then found these grounds extant


and proven, and accordingly disallowed
probate.[5]

64

Ruling of the Court of Appeals

Reversing the trial court, the appellate court admitted the


will of Placido Valmonte to probate. The CA upheld the
credibility of the notary public and the subscribing witnesses
who had acknowledged the due execution of the will.
Moreover, it held that the testator had testamentary capacity at
the time of the execution of the will. It added that his sexual
exhibitionism and unhygienic, crude and impolite ways[6] did
not make him a person of unsound mind.
Hence, this Petition.[7]

Issues
Petitioner raises the following issues for our
consideration:
I.

Whether or not the findings of the probate


court are entitled to great respect.

II.

Whether or not the signature of Placido


Valmonte in the subject will was procured by
fraud or trickery, and that Placido Valmonte
never intended that the instrument should be
his last will and testament.

III.

Whether or not Placido Valmonte has


testamentary capacity at the time he
allegedly executed the subject will.[8]

In short, petitioner assails the CAs allowance of the


probate of the will of Placido Valmonte.
This Courts Ruling

The Petition has no merit.

Main Issue:

Probate of a Will
At the outset, we stress that only questions of law may be
raised in a Petition for Review under Section 1 of Rule 45 of
the Rules of Court. As an exception, however, the evidence
presented during the trial may be examined and the factual
matters resolved by this Court when, as in the instant case, the
findings of fact of the appellate court differ from those of the
trial court.[9]
The fact that public policy favors the probate of a will
does not necessarily mean that every will presented for
probate should be allowed. The law lays down the procedures
and requisites that must be satisfied for the probate of a will. [10]
Verily, Article 839 of the Civil Code states the instances when a
will may be disallowed, as follows:
Article 839. The will shall be
disallowed in any of the following cases:

(1)
If the formalities required by
law have not been complied with;

(2)
If the testator was insane,
or otherwise mentally incapable of making a
will, at the time of its execution;

(3)
If it was executed through
force or under duress, or the influence of
fear, or threats;

(4)
If it was procured by undue
and improper pressure and influence, on the
part of the beneficiary or of some other
person;

(5)
If the signature
testator was procured by fraud;

of

the

65
(6)
If the testator acted by
mistake or did not intend that the instrument
he signed should be his will at the time of
affixing his signature thereto.
In the present case, petitioner assails the validity of
Placido Valmontes will by imputing fraud in its execution and
challenging the testators state of mind at the time.
Existence of Fraud in the

more credible witnesses who must also attest to it in the


presence of the testator and of one another.[19] Furthermore,
the testator and the witnesses must acknowledge the will
before a notary public.[20] In any event, we agree with the CA
that the variance in the dates of the will as to its supposed
execution and attestation was satisfactorily and persuasively
explained by the notary public and the instrumental
witnesses.[21]
The pertinent transcript of stenographic notes taken
on June 11, 1985, November 25, 1985, October 13, 1986, and
October 21, 1987 -- as quoted by the CA -- are reproduced
respectively as follows:

Execution of a Will

Atty. Floro Sarmiento:

Petitioner does not dispute the due observance of the


formalities in the execution of the will, but maintains that the
circumstances surrounding it are indicative of the existence of
fraud. Particularly, she alleges that respondent, who is the
testators wife and sole beneficiary, conspired with the notary
public and the three attesting witnesses in deceiving Placido to
sign it. Deception is allegedly reflected in the varying dates of
the execution and the attestation of the will.

Petitioner contends that it was highly dubious for a woman at


the prime of her young life [to] almost immediately plunge into
marriage with a man who [was] thrice her age x x x and who
happened to be [a] Fil-American pensionado,[11] thus casting
doubt on the intention of respondent in seeking the probate of
the will. Moreover, it supposedly defies human reason, logic
and common experience[12] for an old man with a severe
psychological condition to have willingly signed a last will and
testament.
We are not convinced. Fraud is a trick, secret device,
false statement, or pretense, by which the subject of it is
cheated. It may be of such character that the testator is misled
or deceived as to the nature or contents of the document which
he executes, or it may relate to some extrinsic fact, in
consequence of the deception regarding which the testator is
led to make a certain will which, but for the fraud, he would not
have made.[13]
We stress that the party challenging the will bears the
burden of proving the existence of fraud at the time of its
execution.[14] The burden to show otherwise shifts to the
proponent of the will only upon a showing of credible evidence
of fraud.[15] Unfortunately in this case, other than the selfserving allegations of petitioner, no evidence of fraud was ever
presented.
It is a settled doctrine that the omission of some
relatives does not affect the due execution of a will. [16] That the
testator was tricked into signing it was not sufficiently established
by the fact that he had instituted his wife, who was more than
fifty years his junior, as the sole beneficiary; and disregarded
petitioner and her family, who were the ones who had taken the
cudgels of taking care of [the testator] in his twilight years.[17]
Moreover, as correctly ruled by the appellate court,
the conflict between the dates appearing on the will does not
invalidate the document, because the law does not even
require that a [notarial] will x x x be executed and
acknowledged on the same occasion.[18] More important, the
will must be subscribed by the testator, as well as by three or

You typed this document exhibit C,


specifying the date June 15 when
the testator and his witnesses were
supposed to be in your office?

Yes sir.

On June 15, 1983, did the testator and


his witnesses come to your house?

They did as of agreement but


unfortunately, I was out of town.

xxx
Q

xxx

xxx

The
document
has
been
acknowledged on August 9, 1983
as per acknowledgement appearing
therein. Was this the actual date
when
the
document
was
acknowledged?

Yes sir.

What about the date when the testator


and the three witnesses affixed their
respective signature on the first and
second pages of exhibit C?

On that particular date when it was


acknowledged, August 9, 1983.

Why did you not make the necessary


correction on the date appearing on
the body of the document as well as
the attestation clause?

Because I do not like anymore to make


some alterations so I put it in my
own handwriting August 9, 1983 on
the acknowledgement. (tsn, June
11, 1985, pp. 8-10)

66
Eugenio Gomez:
Q

It appears on the first page Mr.


Witness that it is dated June 15,
1983,
whereas
in
the
acknowledgement it is dated August
9, 1983, will you look at this
document
and
tell
us
this
discrepancy in the date?
We went to Atty. Sarmiento together
with Placido Valmonte and the two
witnesses; that was first week of
June and Atty. Sarmiento told us to
return on the 15th of June but when
we returned, Atty. Sarmiento was
not there.

When you did not find Atty. Sarmiento


on June 15, 1983, did you again go
back?

We returned on the 9th of August and


there we signed.

This August 9, 1983 where you said it


is there where you signed, who
were your companions?
The two witnesses, me and Placido
Valmonte. (tsn, November 25, 1985,
pp. 7-8)

Felisa Gomez on cross-examination:


Q

xxx
A

Why did you have to go to the office of


Atty. Floro Sarmiento, three times?

xxx

signature. (tsn, October 13, 1986,


pp. 4-6)
Josie Collado:
Q

When you did not find Atty. Sarmiento


in his house on June 15, 1983,
what transpired?

The wife of Atty. Sarmiento told us that


we will be back on August 9, 1983.

And on August 9, 1983 did you go


back to the house of Atty.
Sarmiento?

Yes, Sir.

For what purpose?

Our purpose is just to sign the will.

Were you able to sign the will you


mentioned?

Yes sir. (tsn, October 21, 1987, pp. 45)[22]

Notably, petitioner failed to substantiate her claim of a


grand conspiracy in the commission of a fraud. There was no
showing that the witnesses of the proponent stood to receive
any benefit from the allowance of the will. The testimonies of
the three subscribing witnesses and the notary are credible
evidence of its due execution.[23] Their testimony favoring it
and the finding that it was executed in accordance with the
formalities required by law should be affirmed, absent any
showing of ill motives.[24]

xxx

The reason why we went there three


times is that, the first week of June
was out first time. We went there to
talk to Atty. Sarmiento and Placido
Valmonte about the last will and
testament. After that what they
have talked what will be placed in
the testament, what Atty. Sarmiento
said was that he will go back on the
15th of June. When we returned on
June 15, Atty. Sarmiento was not
there so we were not able to sign it,
the will. That is why, for the third
time we went there on August 9 and
that was the time we affixed our

Capacity to Make a Will

In determining the capacity of the testator to make a


will, the Civil Code gives the following guidelines:
Article 798. In order to make a will
it is essential that the testator be of sound
mind at the time of its execution.
Article 799. To be of sound mind, it
is not necessary that the testator be in full
possession of all his reasoning faculties, or
that his mind be wholly unbroken,

67
unimpaired, or shattered by disease, injury
or other cause.

a person incapable of making a will; a weak


or feebleminded person may make a valid
will, provided he has understanding and
memory sufficient to enable him to know
what he is about to do and how or to whom
he is disposing of his property. To constitute
a sound and disposing mind, it is not
necessary that the mind be unbroken or
unimpaired or unshattered by disease or
otherwise. It has been held that testamentary
incapacity does not necessarily require that
a person shall actually be insane or of
unsound mind."[26]

It shall be sufficient if the testator


was able at the time of making the will to
know the nature of the estate to be disposed
of, the proper objects of his bounty, and the
character of the testamentary act.
Article 800. The law presumes that
every person is of sound mind, in the
absence of proof to the contrary.
The burden of proof that the
testator was not of sound mind at the time of
making his dispositions is on the person who
opposes the probate of the will; but if the
testator, one month, or less, before making
his will was publicly known to be insane, the
person who maintains the validity of the will
must prove that the testator made it during a
lucid interval.

WHEREFORE, the Petition is DENIED, and the


assailed Decision and Resolution of the Court of Appeals are
AFFIRMED. Costs against petitioner.

According to Article 799, the three things that the


testator must have the ability to know to be considered of
sound mind are as follows: (1) the nature of the estate to be
disposed of, (2) the proper objects of the testators bounty, and
(3) the character of the testamentary act. Applying this test to
the present case, we find that the appellate court was correct
in holding that Placido had testamentary capacity at the time of
the execution of his will.

SO ORDERED.
Republic of the Philippines
SUPREME COURT
Manila
EN BANC
January 19, 1906

It must be noted that despite his advanced age, he


was still able to identify accurately the kinds of property he
owned, the extent of his shares in them and even their
locations. As regards the proper objects of his bounty, it was
sufficient that he identified his wife as sole beneficiary. As we
have stated earlier, the omission of some relatives from the will
did not affect its formal validity. There being no showing of
fraud in its execution, intent in its disposition becomes
irrelevant.

G.R. No. 1641


GERMAN JABONETA, plaintiff-appellant,
vs.
RICARDO GUSTILO, ET AL., defendants-appellees.
Ledesma, Sumulong and Quintos for appellant.
Del-Pan, Ortigas and Fisher for appellees.
CARSON, J.:

Worth reiterating in determining soundness of mind


is Alsua-Betts v. CA,[25] which held thus:

"Between the highest degree of


soundness of mind and memory which
unquestionably
carries
with
it
full
testamentary capacity, and that degrees of
mental aberration generally known as
insanity or idiocy, there are numberless
degrees of mental capacity or incapacity and
while on one hand it has been held that mere
weakness of mind, or partial imbecility from
disease of body, or from age, will not render

In these proceedings probate was denied the last will and


testament of Macario Jaboneta, deceased, because the lower
court was of the opinion from the evidence adduced at the
hearing that Julio Javellana, one of the witnesses, did not
attach his signature thereto in the presence of Isabelo Jena,
another of the witnesses, as required by the provisions of
section 618 of the Code of Civil Procedure.
The following is a copy of the evidence which appears of
record on this particular point, being a part of the testimony of
the said Isabeo Jena:
Q.

1641

A.
1641
others.

Who first signed the will?


I signed it first, and afterwards Aniceto and the

68
Q.
1641
referred?

Who were those others to whom you have just

A.
1641 After the witness Aniceto signed the will I left the
house, because I was in a hurry, and at the moment when I
was leaving I saw Julio Javellana with the pen in his hand in
position ready to sign (en actitud de firmar). I believe he
signed, because he was at the table. . . .
Q.
1641 State positively whether Julio Javellana did or
did not sign as a witness to the will.
A.
1641 I can't say certainly, because as I was leaving
the house I saw Julio Javellana with the pen in his hand, in
position ready to sign. I believe he signed.
Q.

1641

Why do you believe Julio Javellana signed?

A.
1641 Because he had the pen in his hand, which was
resting on the paper, though I did not actually see him sign.
Q.

1641

Explain this contradictory statement.

A.
1641 After I signed I asked permission to leave,
because I was in a hurry, and while I was leaving Julio had
already taken the pen in his hand, as it appeared, for the
purpose of signing, and when I was near the door I happened
to turn my face and I saw that he had his hand with the pen
resting on the will, moving it as if for the purpose of signing.
Q.
1641 State positively whether Julio moved his hand
with the pen as if for the purpose of signing, or whether he was
signing
A.

I believe he was signing.

The truth and accuracy of the testimony of this witness does


not seem to have been questioned by any of the parties to the
proceedings, but the court, nevertheless, found the following
facts:
On the 26th day of December, 1901, Macario Jaboneta
executed under the following circumstances the document in
question, which has been presented for probate as his will:
Being in the house of Arcadio Jarandilla, in Jaro, in this
province, he ordered that the document in question be written,
and calling Julio Javellana, Aniceto Jalbuena, and Isabelo Jena
as witnesses, executed the said document as his will. They
were all together, and were in the room where Jaboneta was,
and were present when he signed the document, Isabelo Jena
signing afterwards as a witness, at his request, and in his
presence and in the presence of the other two witnesses.
Aniceto Jalbuena then signed as a witness in the presence of
the testator, and in the presence of the other two persons who
signed as witnesses. At that moment Isabelo Jena, being in a
hurry to leave, took his hat and left the room. As he was
leaving the house Julio Javellana took the pen in his hand and
put himself in position to sign the will as a witness, but did not

sign in the presence of Isabelo Jena; but nevertheless, after


Jena had left the room the said Julio Javellana signed as a
witness in the presence of the testator and of the witness
Aniceto Jalbuena.
We can not agree with so much of the above finding of facts as
holds that the signature of Javellana was not signed in the
presence of Jena, in compliance with the provisions of section
618 of the Code of Civil Procedure. The fact that Jena was still
in the room when he saw Javellana moving his hand and pen
in the act of affixing his signature to the will, taken together
with the testimony of the remaining witnesses which shows
that Javellana did in fact there and then sign his name to the
will, convinces us that the signature was affixed in the
presence of Jena. The fact that he was in the act of leaving,
and that his back was turned while a portion of the name of the
witness was being written, is of no importance. He, with the
other witnesses and the testator, had assembled for the
purpose of executing the testament, and were together in the
same room for that purpose, and at the moment when the
witness Javellana signed the document he was actually and
physically present and in such position with relation to
Javellana that he could see everything which took place by
merely casting his eyes in the proper direction, and without any
physical obstruction to prevent his doing so, therefore we are
of opinion that the document was in fact signed before he
finally left the room.
The purpose of a statutory requirement that the witness sign in
the presence of the testator is said to be that the testator may
have ocular evidence of the identity of the instrument
subscribed by the witness and himself, and the generally
accepted tests of presence are vision and mental
apprehension. (See Am. & Eng. Enc. of Law, vol. 30, p. 599,
and cases there cited.)
In the matter of Bedell (2 Connoly (N.Y.), 328) it was held that it
is sufficient if the witnesses are together for the purpose of
witnessing the execution of the will, and in a position to actually
see the testator write, if they choose to do so; and there are
many cases which lay down the rule that the true test of vision
is not whether the testator actually saw the witness sign, but
whether he might have seen him sign, considering his mental
and physical condition and position at the time of the
subscription. (Spoonemore vs. Cables, 66 Mo., 579.)
The principles on which these cases rest and the tests of
presence as between the testator and the witnesses are
equally applicable in determining whether the witnesses signed
the instrument in the presence of each other, as required by
the statute, and applying them to the facts proven in these
proceedings we are of opinion that the statutory requisites as
to the execution of the instrument were complied with, and that
the lower court erred in denying probate to the will on the
ground stated in the ruling appealed from.
We are of opinion from the evidence of record that the
instrument propounded in these proceedings was satisfactorily

69
proven to be the last will and testament of Macario Jaboneta,
deceased, and that it should therefore be admitted to probate.
The judgment of the trial court is reversed, without especial
condemnation of costs, and after twenty days the record will be
returned to the court form whence it came, where the proper
orders will be entered in conformance herewith. So ordered.
Arellano, C.J., Torres, Mapa, and Johnson, JJ., concur.

Republic of the Philippines


SUPREME COURT
Manila
SECOND DIVISION
G.R. No. 103554 May 28, 1993
TEODORO CANEDA, LORENZA CANEDA, TERESA
CANEDA, JUAN CABALLERO, AUREA CABALLERO,
OSCAR LAROSA, HELEN CABALLERO, SANTOS
CABALLERO, PABLO CABALLERO, VICTOR RAGA,
MAURICIA RAGA, QUIRICA RAGA, RUPERTO ABAPO,
represented herein by his Attorney-in-Fact, ARMSTICIA *
ABAPO VELANO, and CONSESO CANEDA, represented
herein by his heirs, JESUS CANEDA, NATIVIDAD CANEDA
and ARTURO CANEDA, petitioners,
vs.
HON. COURT OF APPEALS and WILLIAM CABRERA, as
Special Administrator of the Estate of Mateo
Caballero, respondents.
REGALADO, J.:
Presented for resolution by this Court in the present petition for
review on certiorari is the issue of whether or not the
attestation clause contained in the last will and testament of
the late Mateo Caballero complies with the requirements of
Article 805, in relation to Article 809, of the Civil Code.
The records show that on December 5, 1978, Mateo Caballero,
a widower without any children and already in the twilight years
of his life, executed a last will and testament at his residence in
Talisay, Cebu before three attesting witnesses, namely,
Cipriano Labuca, Gregorio Cabando and Flaviano Toregosa.
The said testator was duly assisted by his lawyer, Atty. Emilio
Lumontad, and a notary public, Atty. Filoteo Manigos, in the
preparation of that last will. 1 It was declared therein, among
other things, that the testator was leaving by way of legacies
and devises his real and personal properties to Presentacion
Gaviola, Angel Abatayo, Rogelio Abatayo, Isabelito Abatayo,
Benoni G. Cabrera and Marcosa Alcantara, all of whom do not
appear to be related to the testator. 2
Four months later, or on April 4, 1979, Mateo Caballero himself
filed a petition docketed as Special Proceeding No. 3899-R
before Branch II of the then Court of First Instance of Cebu
seeking the probate of his last will and testament. The probate
court set the petition for hearing on August 20, 1979 but the
same and subsequent scheduled hearings were postponed for

one reason to another. On May 29, 1980, the testator passed


away before his petition could finally be heard by the probate
court. 3 On February 25, 1981, Benoni Cabrera, on of the
legatees named in the will, sough his appointment as special
administrator of the testator's estate, the estimated value of
which was P24,000.00, and he was so appointed by the
probate court in its order of March 6, 1981. 4
Thereafter, herein petitioners, claiming to be nephews and
nieces of the testator, instituted a second petition, entitled "In
the Matter of the Intestate Estate of Mateo Caballero" and
docketed as Special Proceeding No. 3965-R, before Branch IX
of the aforesaid Court of First Instance of Cebu. On October
18, 1982, herein petitioners had their said petition intestate
proceeding consolidated with Special Proceeding No. 3899-R
in Branch II of the Court of First Instance of Cebu and opposed
thereat the probate of the Testator's will and the appointment of
a special administrator for his estate. 5
Benoni Cabrera died on February 8, 1982 hence the probate
court, now known as Branch XV of the Regional Trial Court of
Cebu, appointed William Cabrera as special administrator on
June 21, 1983. Thereafter, on July 20, 1983, it issued an order
for the return of the records of Special Proceeding No. 3965-R
to the archives since the testate proceeding for the probate of
the will had to be heard and resolved first. On March 26, 1984
the case was reraffled and eventually assigned to Branch XII of
the Regional Trial Court of Cebu where it remained until the
conclusion of the probate proceedings. 6
In the course of the hearing in Special Proceeding No. 3899-R,
herein petitioners appeared as oppositors and objected to the
allowance of the testator's will on the ground that on the
alleged date of its execution, the testator was already in the
poor state of health such that he could not have possibly
executed the same. Petitioners likewise reiterated the issue as
to the genuineness of the signature of the testator therein. 7
On the other hand, one of the attesting witnesses, Cipriano
Labuca, and the notary public Atty. Filoteo Manigos, testified
that the testator executed the will in question in their presence
while he was of sound and disposing mind and that, contrary to
the assertions of the oppositors, Mateo Caballero was in good
health and was not unduly influenced in any way in the
execution of his will. Labuca also testified that he and the other
witnesses attested and signed the will in the presence of the
testator and of each other. The other two attesting witnesses
were not presented in the probate hearing as the had died by
then. 8
On April 5, 1988, the probate court rendered a decision
declaring the will in question as the last will and testament of
the late Mateo Caballero, on the ratiocination that:
. . . The self-serving testimony of the two
witnesses of the oppositors cannot
overcome the positive testimonies of Atty.
Filoteo Manigos and Cipriano Labuca who
clearly told the Court that indeed Mateo
Caballero executed the Last Will and
Testament now marked Exhibit "C" on
December 5, 1978. Moreover, the fact that it
was Mateo Caballero who initiated the
probate of his Will during his lifetime when

70
he caused the filing of the original petition
now marked Exhibit "D" clearly underscores
the fact that this was indeed his Last Will. At
the start, counsel for the oppositors
manifested that he would want the signature
of Mateo Caballero in Exhibit "C" examined
by a handwriting expert of the NBI but it
would seem that despite their avowal and
intention for the examination of this signature
of Mateo Caballero in Exhibit "C", nothing
came out of it because they abandoned the
idea and instead presented Aurea Caballero
and Helen Caballero Campo as witnesses
for the oppositors.
All told, it is the finding of this Court that
Exhibit "C" is the Last Will and Testament of
Mateo Caballero and that it was executed in
accordance with all the requisites of the law. 9
Undaunted by the said judgment of the probate court,
petitioners elevated the case in the Court of Appeals in CAG.R. CV No. 19669. They asserted therein that the will in
question is null and void for the reason that its attestation
clause is fatally defective since it fails to specifically state that
the instrumental witnesses to the will witnessed the testator
signing the will in their presence and that they also signed the
will and all the pages thereof in the presence of the testator
and of one another.
On October 15, 1991, respondent court promulgated its
decision 10 affirming that of the trial court, and ruling that the
attestation clause in the last will of Mateo Caballero
substantially complies with Article 805 of the Civil Code, thus:
The question therefore is whether the
attestation clause in question may be
considered as having substantialy complied
with the requirements of Art. 805 of the Civil
Code. What appears in the attestation clause
which the oppositors claim to be defective is
"we do certify that the testament was read by
him and the attestator, Mateo Caballero, has
published unto us the foregoing will
consisting of THREE PAGES, including the
acknowledgment, each page numbered
correlatively in letters of the upper part of
each page, as his Last Will and
Testament, and he has signed the same and
every page thereof, on the spaces provided
for his signature and on the left hand margin
in the presence of the said testator and in
the presence of each and all of us (emphasis
supplied).
To our thinking, this is sufficient compliance
and no evidence need be presented to
indicate the meaning that the said will was
signed by the testator and by them (the
witnesses) in the presence of all of them and
of one another. Or as the language of the
law would have it that the testator signed the
will "in the presence of the instrumental
witnesses, and that the latter witnessed and

signed the will and all the pages thereof in


the presence of the testator and of one
another." If not completely or ideally perfect
in accordance with the wordings of Art. 805
but (sic) the phrase as formulated is in
substantial compliance with the requirement
of the law." 11
Petitioners moved for the reconsideration of the said ruling of
respondent court, but the same was denied in the latter's
resolution of January 14, 1992, 12 hence this appeal now
before us. Petitioners assert that respondent court has ruled
upon said issue in a manner not in accord with the law and
settled jurisprudence on the matter and are now questioning
once more, on the same ground as that raised before
respondent court, the validity of the attestation clause in the
last will of Mateo Caballero.
We find the present petition to be meritorious, as we shall
shortly hereafter, after some prefatory observations which we
feel should be made in aid of the rationale for our resolution of
the controversy.
1. A will has been defined as a species of conveyance whereby
a person is permitted, with the formalities prescribed by law, to
control to a certain degree the disposition of his estate after his
death. 13 Under the Civil Code, there are two kinds of wills
which a testator may execute. 14 the first kind is the ordinary or
attested will, the execution of which is governed by Articles 804
to 809 of the Code. Article 805 requires that:
Art. 805. Every will, other than a holographic
will, must be subscribed at the end thereof
by the testator himself or by the testator's
name written by some other person in his
presence, and by his express direction, and
attested and subscribed by three or more
credible witnesses in the presence of the
testator and of one another.
The testator or the person requested by him
to write his name and the instrumental
witnesses of the will, shall also sign, as
aforesaid, each and every page thereof,
except the last, on the left margin, and all the
pages shall be numbered correlatively in
letters placed on the upper part of each
page.
The attestation should state the number of
pages used upon which the will is written,
and the fact that the testator signed the will
and every page thereof, or caused some
other person to write his name, under his
express direction, in the presence of the
instrumental witnesses, and that the latter
witnessed and signed the will and all the
pages thereof in the presence of the testator
and of one another.
If the attestation clause is in a language not
known to the witness, it shall be interpreted
to them.

71
In addition, the ordinary will must be acknowledged before a
notary public by a testator and the attesting witness.15 hence it
is likewise known as notarial will. Where the attestator is deaf
or deaf-mute, Article 807 requires that he must personally read
the will, if able to do so. Otherwise, he should designate two
persons who would read the will and communicate its contents
to him in a practicable manner. On the other hand, if the
testator is blind, the will should be read to him twice; once, by
anyone of the witnesses thereto, and then again, by the notary
public before whom it is acknowledged. 16
The other kind of will is the holographic will, which Article 810
defines as one that is entirely written, dated, and signed by the
testator himself. This kind of will, unlike the ordinary type,
requires no attestation by witnesses. A common requirement in
both kinds of will is that they should be in writing and must
have been executed in a language or dialect known to the
testator. 17
However, in the case of an ordinary or attested will, its
attestation clause need not be written in a language or dialect
known to the testator since it does not form part of the
testamentary disposition. Furthermore, the language used in
the attestation clause likewise need not even be known to the
attesting witnesses. 18 The last paragraph of Article 805 merely
requires that, in such a case, the attestation clause shall be
interpreted to said witnesses.
An attestation clause refers to that part of an ordinary will
whereby the attesting witnesses certify that the instrument has
been executed before them and to the manner of the execution
the same. 19 It is a separate memorandum or record of the
facts surrounding the conduct of execution and once signed by
the witnesses, it gives affirmation to the fact that compliance
with the essential formalities required by law has been
observed. 20 It is made for the purpose of preserving in a
permanent form a record of the facts that attended the
execution of a particular will, so that in case of failure of the
memory of the attesting witnesses, or other casualty, such
facts may still be proved. 21
Under the third paragraph of Article 805, such a clause, the
complete lack of which would result in the invalidity of the
will, 22 should state (1) the number of the pages used upon
which the will is written; (2) that the testator signed, or
expressly caused another to sign, the will and every page
thereof in the presence of the attesting witnesses; and (3) that
theattesting witnesses witnessed the signing by the testator of
the will and all its pages, and that said witnesses also signed
the will and every page thereof in the presence of the testator
and of one another.
The purpose of the law in requiring the clause to state the
number of pages on which the will is written is to safeguard
against possible interpolation or omission of one or some of its
pages and to prevent any increase or decrease in the
pages; 23 whereas the subscription of the signature of the
testator and the attesting witnesses is made for the purpose of
authentication and identification, and thus indicates that the will
is the very same instrument executed by the testator and
attested to by the witnesses. 24
Further, by attesting and subscribing to the will, the witnesses
thereby declare the due execution of the will as embodied in

the attestation clause. 25 The attestation clause, therefore,


provide strong legal guaranties for the due execution of a will
and to insure the authenticity thereof. 26 As it appertains only to
the witnesses and not to the testator, it need be signed only by
them. 27 Where it is left unsigned, it would result in the
invalidation of the will as it would be possible and easy to add
the clause on a subsequent occasion in the absence of the
testator and its witnesses. 28
In its report, the Code Commission commented on the reasons
of the law for requiring the formalities to be followed in the
execution of wills, in the following manner:
The underlying and fundamental objectives
permeating the provisions on the law on wills
in this Project consists in the liberalization of
the manner of their execution with the end in
view of giving the testator more freedom in
expressing his last wishes, but with sufficient
safeguards and restrictions to prevent the
commission of fraud and the exercise of
undue and improper pressure and influence
upon the testator.
This objective is in accord with the modern
tendency with respect to the formalities in
the execution of wills. . . . 29
2. An examination of the last will and testament of Mateo
Caballero shows that it is comprised of three sheets all of
which have been numbered correlatively, with the left margin of
each page thereof bearing the respective signatures of the
testator and the three attesting witnesses. The part of the will
containing the testamentary dispositions is expressed in the
Cebuano-Visayan dialect and is signed at the foot thereof by
the testator. The attestation clause in question, on the other
hand, is recited in the English language and is likewise signed
at the end thereof by the three attesting witnesses
hereto. 30 Since it is the proverbial bone of contention, we
reproduce it again for facility of reference:
We, the undersigned attesting Witnesses,
whose Residences and postal addresses
appear on the Opposite of our respective
names, we do hereby certify that the
Testament was read by him and the testator,
MATEO CABALLERO; has published unto
us the foregoing Will consisting of THREE
PAGES, including the Acknowledgment,
each page numbered correlatively in the
letters on the upper part of each page, as his
Last Will and Testament and he has the
same and every page thereof, on the spaces
provided for his signature and on the left
hand margin, in the presence of the said
testator and in the presence of each and all
of us.
It will be noted that Article 805 requires that the witness should
both attest and subscribe to the will in the presence of the
testator and of one another. "Attestation" and "subscription"
differ in meaning. Attestation is the act of senses, while
subscription is the act of the hand. The former is mental, the
latter mechanical, and to attest a will is to know that it was

72
published as such, and to certify the facts required to constitute
an actual and legal publication; but to subscribe a paper
published as a will is only to write on the same paper the
names of the witnesses, for the sole purpose of identification. 31
In Taboada vs. Rizal, 32 we clarified that attestation consists in
witnessing the testator's execution of the will in order to see
and take note mentally that those things are done which the
statute requires for the execution of a will and that the
signature of the testator exists as a fact. On the other hand,
subscription is the signing of the witnesses' names upon the
same paper for the purpose of identification of such paper as
the will which was executed by the testator. As it involves a
mental act, there would be no means, therefore, of
ascertaining by a physical examination of the will whether the
witnesses had indeed signed in the presence of the testator
and of each other unless this is substantially expressed in the
attestation.
It is contended by petitioners that the aforequoted attestation
clause, in contravention of the express requirements of the
third paragraph of Article 805 of the Civil Code for attestation
clauses, fails to specifically state the fact that the attesting
witnesses the testator sign the will and all its pages in their
presence and that they, the witnesses, likewise signed the will
and every page thereof in the presence of the testator and of
each other. We agree.
What is fairly apparent upon a careful reading of the attestation
clause herein assailed is the fact that while it recites that the
testator indeed signed the will and all its pages in the presence
of the three attesting witnesses and states as well the number
of pages that were used, the same does not expressly state
therein the circumstance that said witnesses subscribed their
respective signatures to the will in the presence of the testator
and of each other.
The phrase "and he has signed the same and every page
thereof, on the spaces provided for his signature and on the
left hand margin," obviously refers to the testator and not the
instrumental witnesses as it is immediately preceded by the
words "as his Last Will and Testament." On the other hand,
although the words "in the presence of the testator and in the
presence of each and all of us" may, at first blush, appear to
likewise signify and refer to the witnesses, it must, however, be
interpreted as referring only to the testator signing in the
presence of the witnesses since said phrase immediately
follows the words "he has signed the same and every page
thereof, on the spaces provided for his signature and on the
left hand margin." What is then clearly lacking, in the final
logical analysis , is the statement that the witnesses signed the
will and every page thereof in the presence of the testator and
of one another.
It is our considered view that the absence of that statement
required by law is a fatal defect or imperfection which must
necessarily result in the disallowance of the will that is here
sought to be admitted to probate. Petitioners are correct in
pointing out that the aforestated defect in the attestation clause
obviously cannot be characterized as merely involving the form
of the will or the language used therein which would warrant
the application of the substantial compliance rule, as
contemplated in the pertinent provision thereon in the Civil
Code, to wit:

Art. 809. In the absence of bad faith, forgery,


or fraud, or undue and improper pressure
and influence, defects and imperfections in
the form of attestation or in the
language used therein shall not render the
will invalid if it is not proved that the will was
in fact executed and attested in substantial
compliance with all the requirements of
article 805" (Emphasis supplied.)
While it may be true that the attestation clause is indeed
subscribed at the end thereof and at the left margin of each
page by the three attesting witnesses, it certainly cannot be
conclusively inferred therefrom that the said witness affixed
their respective signatures in the presence of the testator and
of each other since, as petitioners correctly observed, the
presence of said signatures only establishes the fact that it was
indeed signed, but it does not prove that the attesting
witnesses did subscribe to the will in the presence of the
testator and of each other. The execution of a will is supposed
to be one act so that where the testator and the witnesses sign
on various days or occasions and in various combinations, the
will cannot be stamped with the imprimatur of effectivity. 33
We believe that the further comment of former Justice J.B.L.
Reyes 34 regarding Article 809, wherein he urged caution in the
application of the substantial compliance rule therein, is correct
and should be applied in the case under consideration, as well
as to future cases with similar questions:
. . . The rule must be limited to disregarding
those defects that can be supplied by an
examination of the will itself: whether all the
pages are consecutively numbered; whether
the signatures appear in each and every
page; whether the subscribing witnesses are
three or the will was notarized. All theses are
facts that the will itself can reveal, and
defects or even omissions concerning them
in the attestation clause can be safely
disregarded. But the total number of
pages, and whether all persons required to
sign did so in the presence of each other
must substantially appear in the attestation
clause, being the only check against perjury
in the probate proceedings. (Emphasis ours.)
3. We stress once more that under Article 809, the defects and
imperfections must only be with respect to the form of the
attestation or the language employed therein. Such defects or
imperfections would not render a will invalid should it be
proved that the will was really executed and attested in
compliance with Article 805. In this regard, however, the
manner of proving the due execution and attestation has been
held to be limited to merely an examination of the will itself
without resorting to evidence aliunde, whether oral or written.
The foregoing considerations do not apply where the
attestation clause totally omits the fact that the attesting
witnesses signed each and every page of the will in the
presence of the testator and of each other. 35 In such a
situation, the defect is not only in the form or language of the
attestation clause but the total absence of a specific element
required by Article 805 to be specifically stated in the

73
attestation clause of a will. That is precisely the defect
complained of in the present case since there is no plausible
way by which we can read into the questioned attestation
clause statement, or an implication thereof, that the attesting
witness did actually bear witness to the signing by the testator
of the will and all of its pages and that said instrumental
witnesses also signed the will and every page thereof in the
presence of the testator and of one another.
Furthermore, the rule on substantial compliance in Article 809
cannot be revoked or relied on by respondents since it
presupposes that the defects in the attestation clause can be
cured or supplied by the text of the will or a consideration of
matters apparent therefrom which would provide the data not
expressed in the attestation clause or from which it may
necessarily be gleaned or clearly inferred that the acts not
stated in the omitted textual requirements were actually
complied within the execution of the will. In other words,
defects must be remedied by intrinsic evidence supplied by the
will itself.
In the case at bar, contrarily, proof of the acts required to have
been performed by the attesting witnesses can be supplied by
only extrinsic evidence thereof, since an overall appreciation of
the contents of the will yields no basis whatsoever from with
such facts may be plausibly deduced. What private respondent
insists on are the testimonies of his witnesses alleging that
they saw the compliance with such requirements by the
instrumental witnesses, oblivious of the fact that he is thereby
resorting to extrinsic evidence to prove the same and would
accordingly be doing by the indirection what in law he cannot
do directly.
4. Prior to the advent of the Civil Code on August 30, 1950,
there was a divergence of views as to which manner of
interpretation should be followed in resolving issues centering
on compliance with the legal formalities required in the
execution of wills. The formal requirements were at that time
embodied primarily in Section 618 of Act No. 190, the Code of
Civil Procedure. Said section was later amended by Act No.
2645, but the provisions respecting said formalities found in
Act. No. 190 and the amendment thereto were practically
reproduced and adopted in the Civil Code.
One view advance the liberal or substantial compliance rule.
This was first laid down in the case of Abangan vs.
Abangan, 36 where it was held that the object of the solemnities
surrounding the execution of wills is to close the door against
bad faith and fraud, to avoid substitution of wills and
testaments and to guarantee their truth and authenticity.
Therefore, the laws on this subject should be interpreted in
such a way as to attain these primordial ends. Nonetheless, it
was also emphasized that one must not lose sight of the fact
that it is not the object of the law to restrain and curtail the
exercise of the right to make a will, hence when an
interpretation already given assures such ends, any other
interpretation whatsoever that adds nothing but demands more
requisites entirely unnecessary, useless and frustrative of the
testator's last will, must be disregarded. The subsequent cases
of Avera vs. Garcia, 37 Aldaba vs. Roque, 38 Unson vs.
Abella, 39 Pecson vs. Coronel, 40 Fernandez vs. Vergel de Dios,
et al., 41 and Nayve vs. Mojal, et al. 42 all adhered to this
position.

The other view which advocated the rule that statutes which
prescribe the formalities that should be observed in the
execution of wills are mandatory in nature and are to be strictly
construed was followed in the subsequent cases of In the
Matter of the Estate of Saguinsin, 43 In re Will of Andrada, 44 Uy
Coque vs. Sioca, 45 In re Estate of Neumark, 46 and Sano vs.
Quintana. 47
Gumban vs. Gorecho, et al., 48 provided the Court with the
occasion to clarify the seemingly conflicting decisions in the
aforementioned cases. In said case of Gumban, the attestation
clause had failed to state that the witnesses signed the will and
each and every page thereof on the left margin in the presence
of the testator. The will in question was disallowed, with these
reasons therefor:
In support of their argument on the
assignment of error above-mentioned,
appellants rely on a series of cases of this
court beginning with (I)n the Matter of the
(E)state of Saguinsin ([1920], 41 Phil., 875),
continuing with In re Will of Andrada [1921],
42 Phil., 180), Uy Coque vs. Navas L. Sioca
[1922], 43 Phil., 405), and In re Estate of
Neumark ([1923], 46 Phil., 841), and ending
with Sano vs. Quintana([1925], 48 Phil.,
506). Appellee counters with the citation of a
series of cases beginning withAbangan vs.
Abangan ([1919], 40 Phil., 476), continuing
through Aldaba vs. Roque ([1922], 43 Phil.,
378), and Fernandez vs. Vergel de
Dios ([1924], 46 Phil., 922), and culminating
in Nayve vs. Mojal and Aguilar ([1924], 47
Phil., 152). In its last analysis, our task is to
contrast and, if possible, conciliate the last
two decisions cited by opposing counsel,
namely, those of Sano vs. Quintana,supra,
and Nayve vs. Mojal and Aguilar, supra.
In the case of Sano vs. Quintana, supra, it
was decided that an attestation clause which
does not recite that the witnesses signed the
will and each and every page thereof on the
left margin in the presence of the testator is
defective, and such a defect annuls the will.
The case of Uy Coque vs. Sioca, supra, was
cited, but the case of Nayve vs. Mojal and
Aguilar, supra, was not mentioned. In
contrast, is the decision in Nayve vs. Mojal
and Aguilar, supra, wherein it was held that
the attestation clause must estate the fact
that the testator and the witnesses
reciprocally saw the signing of the will, for
such an act cannot be proved by the mere
exhibition of the will, if it is not stated therein.
It was also held that the fact that the testator
and the witnesses signed each and every
page of the will can be proved also by the
mere examination of the signatures
appearing on the document itself, and the
omission to state such evident facts does not
invalidate the will.
It is a habit of courts to reaffirm or distinguish
previous cases; seldom do they admit

74
inconsistency in doctrine. Yet here, unless
aided impossible to reconcile the Mojal and
Quintana decisions. They are fundamentally
at variance. If we rely on one, we affirm. If
we rely on the other, we reverse.
In resolving this puzzling question of
authority, three outstanding points may be
mentioned. In the first place, the Mojal,
decision was concurred in by only four
members of the court, less than a majority,
with two strong dissenting opinions; the
Quintana decision was concurred in by
seven members of the court, a clear majority,
with one formal dissent. In the second place,
the Mojal decision was promulgated in
December, 1924, while the Quintana
decision was promulgated in December,
1925; the Quintana decision was thus
subsequent in point of time. And in the third
place, the Quintana decision is believed
more nearly to conform to the applicable
provisions of the law.
The right to dispose of property by will is
governed entirely by statute. The law of the
case is here found in section 61 of the Code
of Civil Procedure as amended by Act No.
2645, and in section 634 of the same Code,
as unamended. It is in part provided in
section 61, as amended that
"No will . . . shall be valid . . . unless . . .." It is
further provided in the same section that
"The attestation shall state the number of
sheets or pages used, upon which the will is
written, and the fact that the testator signed
the will and every page thereof, or caused
some other person to write his name, under
his express direction, in the presence of
three witnesses, and the latter witnessed
and signed the will and all pages thereof in
the presence of the testator and of each
other." Codal section 634 provides that "The
will shall be disallowed in either of the
following case: 1. If not executed
and attested as in this Act provided." The law
not alone carefully makes use of the
imperative, but cautiously goes further and
makes use of the negative, to enforce
legislative intention. It is not within the
province of the courts to disregard the
legislative purpose so emphatically and
clearly expressed.
We adopt and reaffirm the decision in the
case of Sano vs. Quintana, supra, and, to
the extent necessary, modify the decision in
the case of Nayve vs. Mojal and
Aguilar, supra. (Emphases in the original
text).
But after the Gumban clarificatory pronouncement, there were
decisions of the Court that once more appeared to revive the
seeming diversity of views that was earlier threshed out
therein. The cases of Quinto vs. Morata, 49Rodriguez vs.

Alcala, 50 Enchevarria vs. Sarmiento, 51 and Testate Estate of


Toray 52 went the way of the ruling as restated in Gumban.
But De Gala vs. Gonzales, et al., 53 Rey vs. Cartagena, 54 De
Ticson vs. De Gorostiza, 55 Sebastian vs.
Panganiban, 56 Rodriguez vs. Yap, 57 Grey vs. Fabia, 58 Leynez
vs. Leynez, 59 Martir vs. Martir, 60 Alcala vs. De Villa, 61Sabado
vs.
Fernandez, 62 Mendoza vs. Pilapil, 63 and Lopez vs.
Liboro, 64 veered away from the strict interpretation rule and
established a trend toward an application of the liberal view.
The Code Commission, cognizant of such a conflicting welter
of views and of the undeniable inclination towards a liberal
construction, recommended the codification of the substantial
compliance rule, as it believed this rule to be in accord with the
modern tendency to give a liberal approach to the
interpretation of wills. Said rule thus became what is now
Article 809 of the Civil Code, with this explanation of the Code
Commission:
The present law provides for only one form
of executing a will, and that is, in accordance
with the formalities prescribed by Section
618 of the Code of Civil Procedure as
amended by Act No. 2645. The Supreme
Court of the Philippines had previously
upheld the strict compliance with the legal
formalities and had even said that the
provisions of Section 618 of the Code of Civil
Procedure, as amended regarding the
contents of the attestation clause were
mandatory, and non-compliance therewith
invalidated the will (Uy Coque vs. Sioca, 43
Phil. 405). These decisions necessarily
restrained the freedom of the testator in
disposing of his property.
However, in recent years the Supreme Court
changed its attitude and has become more
liberal in the interpretation of the formalities
in the execution of wills. This liberal view is
enunciated in the cases ofRodriguez vs. Yap,
G.R. No. 45924, May 18, 1939; Leynez vs.
Leynez, G.R. No. 46097, October 18,
1939; Martir vs. Martir, G.R. No. 46995, June
21, 1940; and Alcala vs. Villa, G.R. No.
47351, April 18, 1941.
In the above mentioned decisions of our
Supreme Court, it has practically gone back
to the original provisions of Section 618 of
the Code of Civil Procedure before its
amendment by Act No. 2645 in the year
1916. To turn this attitude into a legislative
declaration and to attain the main objective
of the proposed Code in the liberalization of
the manner of executing wills, article 829 of
the Project is recommended, which reads:
"Art. 829. In the absence of bad faith,
forgery, or fraud, or undue and improper
pressure and influence, defects and
imperfections in the form of attestation or
in the language used therein shall not

75
render the will invalid if it is proved that
the will was in fact executed and attested
in substantial compliance with all the
requirements of article 829."65
The so-called liberal rule, the Court said in Gil vs.
Murciano, 66 "does not offer any puzzle or difficulty, nor does it
open the door to serious consequences. The later decisions do
tell us when and where to stop; they draw the dividing line with
precision. They do not allow evidence aliunde to fill a void in
any part of the document or supply missing details that should
appear in the will itself. They only permit a probe into the will,
an exploration into its confines, to ascertain its meaning or to
determine the existence or absence of the requisite formalities
of law. This clear, sharp limitation eliminates uncertainty and
ought to banish any fear of dire results."
It may thus be stated that the rule, as it now stands, is that
omissions which can be supplied by an examination of the will
itself, without the need of resorting to extrinsic evidence, will
not be fatal and, correspondingly, would not obstruct the
allowance to probate of the will being assailed. However, those
omissions which cannot be supplied except by
evidence aliunde would result in the invalidation of the
attestation clause and ultimately, of the will itself.67
WHEREFORE, the petition is hereby GRANTED and the
impugned decision of respondent court is hereby REVERSED
and SET ASIDE. The court a quo is accordingly directed to
forthwith DISMISS its Special Proceeding No. 3899-R (Petition
for the Probate of the Last Will and Testament of Mateo
Caballero) and to REVIVE Special Proceeding No. 3965-R (In
the matter of the Intestate Estate of Mateo Caballero) as an
active case and thereafter duly proceed with the settlement of
the estate of the said decedent.
SO ORDERED.
Narvasa, C.J., Padilla, Regalado, and Nocon, JJ., concur.
Republic of the Philippines
SUPREME COURT
Manila
THIRD DIVISION
G.R. No. 192916

October 11, 2010

MANUEL A. ECHAVEZ, Petitioner,


vs.
DOZEN CONSTRUCTION AND DEVELOPMENT
CORPORATION and THE REGISTER OF DEEDS OF CEBU
CITY, Respondents.

a Deed of Donation Mortis Causa.1 Manuel accepted the


donation.
In March 1986, Vicente executed a Contract to Sell over the
same lots in favor of Dozen Construction and Development
Corporation (Dozen Corporation). In October 1986, they
executed two Deeds of Absolute Sale over the same properties
covered by the previous Contract to Sell.
On November 6, 1986, Vicente died. Emiliano Cabanig,
Vicentes nephew, filed a petition for the settlement of Vicentes
intestate estate. On the other hand, Manuel filed a petition to
approve Vicentes donation mortis causa in his favor and an
action to annul the contracts of sale Vicente executed in favor
of Dozen Corporation. These cases were jointly heard.
The Regional Trial Court (RTC) dismissed Manuels petition to
approve the donation and his action for annulment of the
contracts of sale.2 The RTC found that the execution of a
Contract to Sell in favor of Dozen Corporation, after Vicente
had donated the lots to Manuel, was an equivocal act that
revoked the donation. The Court of Appeals (CA) affirmed the
RTCs decision.3 The CA held that since the donation in favor
of Manuel was a donation mortis causa, compliance with the
formalities for the validity of wills should have been observed.
The CA found that the deed of donation did not contain an
attestation clause and was therefore void.
The Petition for Review on Certiorari
Manuel claims that the CA should have applied the rule on
substantial compliance in the construction of a will to Vicentes
donation mortis causa. He insists that the strict construction of
a will was not warranted in the absence of any indication of
bad faith, fraud, or substitution in the execution of the Deed of
Donation Mortis Causa. He argues that the CA ignored the
Acknowledgment portion of the deed of donation, which
contains the "import and purpose" of the attestation clause
required in the execution of wills. The Acknowledgment reads:
BEFORE ME, Notary Public, this 7th day of September 1985 at
Talisay, Cebu, personally appeared VICENTE S. Echavez with
Res. Cert. No. 16866094 issued on April 10, 1985 at [sic]
Talisay, Cebu known to me to be the same person who
executed the foregoing instrument of Deed of Donartion Mortis
Causa before the Notary Public and in the presence of the
foregoing three (3) witnesses who signed this instrument
before and in the presence of each other and of the Notary
Public and all of them acknowledge to me that the same is
their voluntary act and deed. [Emphasis in the original.]
THE COURTS RULING

RESOLUTION
BRION, J.:

The CA correctly declared that a donation mortis causa must


comply with the formalities prescribed by law for the validity of
wills,4 "otherwise, the donation is void and would produce no
effect." 5 Articles 805 and 806 of the Civil Code should have
been applied.

Vicente Echavez (Vicente) was the absolute owner of several


lots in Cebu City, which includes Lot No. 1956-A and Lot No.
1959 (subject lots). On September 7, 1985, Vicente donated
the subject lots to petitioner Manuel Echavez (Manuel) through

As the CA correctly found, the purported attestation clause


embodied in the Acknowledgment portion does not contain the
number of pages on which the deed was written.lavvphilThe
exception to this rule in Singson v. Florentino6and Taboada v.

76
Hon. Rosal,7 cannot be applied to the present case, as the
facts of this case are not similar with those of Singson and
Taboada. In those cases, the Court found that although the
attestation clause failed to state the number of pages upon
which the will was written, the number of pages was stated in
one portion of the will. This is not the factual situation in the
present case.
Even granting that the Acknowledgment embodies what the
attestation clause requires, we are not prepared to hold that an
attestation clause and an acknowledgment can be merged in
one statement.
That the requirements of attestation and acknowledgment are
embodied in two separate provisions of the Civil Code (Articles
805 and 806, respectively) indicates that the law contemplates
two distinct acts that serve different purposes. An
acknowledgment is made by one executing a deed, declaring
before a competent officer or court that the deed or act is his
own. On the other hand, the attestation of a will refers to the
act of the instrumental witnesses themselves who certify to the
execution of the instrument before them and to the manner of
its execution.81avvphi1
Although the witnesses in the present case acknowledged the
execution of the Deed of Donation Mortis Causa before the
notary public, this is not the avowal the law requires from the
instrumental witnesses to the execution of a decedents will. An
attestation must state all the details the third paragraph of
Article 805 requires. In the absence of the required avowal by
the witnesses themselves, no attestation clause can be
deemed embodied in the Acknowledgement of the Deed of
Donation Mortis Causa.
Finding no reversible error committed by the CA, the Court
hereby DENIES Manuels petition for review on certiorari.
SO ORDERED.
Republic of the Philippines
SUPREME COURT
Manila
EN BANC
G.R. No. L-20357

November 25, 1967

IN THE MATTER OF THE PETITION FOR THE ALLOWANCE


OF THE WILL OF GREGORIO GATCHALIAN, deceased.
PEDRO REYES GARCIA, petitioner-appellant,
vs.
FELIPE GATCHALIAN, AURORA G. CAMINS, ANGELES G.
COSCA, FEDERICO G. TUBOG, VIRGINIA G. TALANAY and
ANGELES G. TALANAY, oppositors-appellees.
E. Debuque for petitioner-appellant.
E. L. Segovia for oppositors-appellees.

Proceedings No. 2623 denying the allowance of the will of the


late Gregorio Gatchalian, on the ground that the attesting
witnesses did not acknowledge it before a notary public, as
required by law.
On March 15, 1967, Gregorio Gatchalian, a widower of 71
years of age, died in the municipality of Pasig, Province of
Rizal, leaving no forced heirs. On April 2 of the same year,
appellant filed a petition with the above named court for the
probate of said alleged will (Exhibit "C") wherein he was
instituted as sole heir. Felipe Gatchalian, Aurora G. Camins,
Angeles G. Cosca, Federico G. Tubog, Virginia G. Talanay and
Angeles G. Talanay, appellees herein, opposed the petition on
the ground, among others, that the will was procured by fraud;
that the deceased did not intend the instrument signed by him
to be as his will; and that the deceased was physically and
mentally incapable of making a will at the time of the alleged
execution of said will.
After due trial, the court rendered the appealed decision finding
the document Exhibit "C" to be the authentic last will of the
deceased but disallowing it for failure to comply with the
mandatory requirement of Article 806 of the New Civil Code
that the will must be acknowledged before a notary public by
the testator and the witnesses.
An examination of the document (Exhibit "C") shows that the
same was acknowledged before a notary public by the testator
but not by the instrumental witnesses.
Article 806 of the New Civil Code reads as follows:
Every will must be acknowledged before a notary
public by the testator and the witnesses. The notary
public shall not be required to retain a copy of the will,
or file another with the office of the Clerk of Court.
We have held heretofore that compliance with the requirement
contained in the above legal provision to the effect that a will
must be acknowledged before a notary public by the testator
and also by the witnesses is indispensable for its validity (In re:
Testate Estate of Alberto, G. R. No. L-11948, April 29, 1959).
As the document under consideration does not comply with
this requirement, it is obvious that the same may not be
probated.
WHEREFORE, the decision appealed from is affirmed, with
costs.
Makalintal, Bengzon, J.P., Zaldivar, Sanchez, Castro, Angeles
and Fernando, JJ., concur.
Concepcion, C.J., and Reyes, J.B.L., J., took no part.

Republic of the Philippines


SUPREME COURT
Manila

DIZON, J.:
FIRST DIVISION
This is an appeal taken by Pedro Reyes Garcia from the
decision of the Court of First Instance of Rizal in Special

77
G.R. No. L-32213 November 26, 1973
AGAPITA N. CRUZ, petitioner,
vs.
HON. JUDGE GUILLERMO P. VILLASOR, Presiding Judge
of Branch I, Court of First Instance of Cebu, and MANUEL
B. LUGAY, respondents.
Paul G. Gorrez for petitioner.
Mario D. Ortiz for respondent Manuel B. Lugay.
ESGUERRA, J.:
Petition to review on certiorari the judgment of the Court First
Instance of Cebu allowing the probate of the last will a
testament of the late Valente Z. Cruz. Petitioner-appellant
Agapita N. Cruz, the surviving spouse of the said decease
opposed the allowance of the will (Exhibit "E"), alleging the will
was executed through fraud, deceit, misrepresentation and
undue influence; that the said instrument was execute without
the testator having been fully informed of the content thereof,
particularly as to what properties he was disposing and that the
supposed last will and testament was not executed in
accordance with law. Notwithstanding her objection, the Court
allowed the probate of the said last will and testament Hence
this appeal by certiorari which was given due course.
The only question presented for determination, on which the
decision of the case hinges, is whether the supposed last will
and testament of Valente Z. Cruz (Exhibit "E") was executed in
accordance with law, particularly Articles 805 and 806 of the
new Civil Code, the first requiring at least three credible
witnesses to attest and subscribe to the will, and the second
requiring the testator and the witnesses to acknowledge the
will before a notary public.
Of the three instrumental witnesses thereto, namely
Deogracias T. Jamaloas Jr., Dr. Francisco Paares and Atty.
Angel H. Teves, Jr., one of them, the last named, is at the
same time the Notary Public before whom the will was
supposed to have been acknowledged. Reduced to simpler
terms, the question was attested and subscribed by at least
three credible witnesses in the presence of the testator and of
each other, considering that the three attesting witnesses must
appear before the notary public to acknowledge the same. As
the third witness is the notary public himself, petitioner argues
that the result is that only two witnesses appeared before the
notary public to acknowledge the will. On the other hand,
private respondent-appellee, Manuel B. Lugay, who is the
supposed executor of the will, following the reasoning of the
trial court, maintains that there is substantial compliance with
the legal requirement of having at least three attesting
witnesses even if the notary public acted as one of them,
bolstering up his stand with 57 American Jurisprudence, p. 227
which, insofar as pertinent, reads as follows:
It is said that there are, practical reasons for
upholding a will as against the purely technical
reason that one of the witnesses required by law
signed as certifying to an acknowledgment of the
testator's signature under oath rather than as
attesting the execution of the instrument.

After weighing the merits of the conflicting claims of the parties,


We are inclined to sustain that of the appellant that the last will
and testament in question was not executed in accordance
with law. The notary public before whom the will was
acknowledged cannot be considered as the third instrumental
witness since he cannot acknowledge before himself his
having signed the will. To acknowledge before means to avow
(Javellana v. Ledesma, 97 Phil. 258, 262; Castro v. Castro, 100
Phil. 239, 247); to own as genuine, to assent, to admit; and
"before" means in front or preceding in space or ahead of. (The
New Webster Encyclopedic Dictionary of the English
Language, p. 72; Funk & Wagnalls New Standard Dictionary of
the English Language, p. 252; Webster's New International
Dictionary 2d. p. 245.) Consequently, if the third witness were
the notary public himself, he would have to avow assent, or
admit his having signed the will in front of himself. This cannot
be done because he cannot split his personality into two so
that one will appear before the other to acknowledge his
participation in the making of the will. To permit such a situation
to obtain would be sanctioning a sheer absurdity.
Furthermore, the function of a notary public is, among others,
to guard against any illegal or immoral arrangement Balinon v.
De Leon, 50 0. G. 583.) That function would defeated if the
notary public were one of the attesting instrumental witnesses.
For them he would be interested sustaining the validity of the
will as it directly involves him and the validity of his own act. It
would place him in inconsistent position and the very purpose
of acknowledgment, which is to minimize fraud (Report of
Code Commission p. 106-107), would be thwarted.
Admittedly, there are American precedents holding that notary
public may, in addition, act as a witness to the executive of the
document he has notarized. (Mahilum v. Court Appeals, 64 0.
G. 4017; 17 SCRA 482; Sawyer v. Cox, 43 Ill. 130). There are
others holding that his signing merely as notary in a will
nonetheless makes him a witness thereon (Ferguson v.
Ferguson, 47 S. E. 2d. 346; In Re Douglas Will, N. Y. S. 2d.
641; Ragsdal v. Hill, 269 S. W. 2d. 911, Tyson Utterback, 122
So. 496; In Re Baybee's Estate 160 N. 900; W. Merill v. Boal,
132 A. 721;See also Trenwith v. Smallwood, 15 So. 1030). But
these authorities do not serve the purpose of the law in this
jurisdiction or are not decisive of the issue herein because the
notaries public and witnesses referred to aforecited cases
merely acted as instrumental, subscribing attesting witnesses,
and not as acknowledging witnesses. He the notary public
acted not only as attesting witness but also acknowledging
witness, a situation not envisaged by Article 805 of the Civil
Code which reads:
ART. 806. Every will must be acknowledged
before a notary public by the testator and the
witnesses. The notary public shall not be required
to retain a copy of the will or file another with the
office of the Clerk of Court. [Emphasis supplied]
To allow the notary public to act as third witness, or one the
attesting and acknowledging witnesses, would have the effect
of having only two attesting witnesses to the will which would
be in contravention of the provisions of Article 80 be requiring
at least three credible witnesses to act as such and of Article
806 which requires that the testator and the required number of
witnesses must appear before the notary public to
acknowledge the will. The result would be, as has been said,
that only two witnesses appeared before the notary public for

78
or that purpose. In the circumstances, the law would not be
duly in observed.
FOR ALL THE FOREGOING, the judgment appealed from is
hereby reversed and the probate of the last will and testament
of Valente Z. Cruz (Exhibit "E") is declared not valid and hereby
set aside.
Cost against the appellee.
Makalintal, C.J., Castro, Teehankee, Makasiar and Muoz
Palma, JJ., concur.

Republic of the Philippines


SUPREME COURT
Manila
FIRST DIVISION
G.R. No. 74695 September 14, 1993
In the Matter of the Probate of the Last Will and Testament
of the Deceased Brigido Alvarado, CESAR
ALVARADO, petitioner,
vs.
HON. RAMON G. GAVIOLA, JR., Presiding Justice, HON.
MA. ROSARIO QUETULIO LOSA and HON. LEONOR INES
LUCIANO, Associate Justices, Intermediate Appellate
Court, First Division (Civil Cases), and BAYANI MA.
RINO, respondents.
Vicente R. Redor for petitioner.
Bayani Ma. Rino for and in his own behalf.
BELLOSILLO, J.:
Before us is an appeal from the Decision dated 11 April
1986 1 of the First Civil Cases Division of the then Intermediate
Appellate Court, now Court of Appeals, which affirmed the
Order dated 27 June 1983 2 of the Regional Trial Court of Sta.
Cruz, Laguna, admitting to probate the last will and
testament 3 with codicil 4 of the late Brigido Alvarado.
On 5 November 1977, the 79-year old Brigido Alvarado
executed a notarial will entitled "Huling Habilin" wherein he
disinherited an illegitimate son (petitioner) and expressly
revoked a previously executed holographic will at the time
awaiting probate before Branch 4 of the Regional Trial Court of
sta. Cruz, Laguna.
As testified to by the three instrumental witnesses, the notary
public and by private respondent who were present at the
execution, the testator did not read the final draft of the will
himself. Instead, private respondent, as the lawyer who drafted
the eight-paged document, read the same aloud in the
presence of the testator, the three instrumental witnesses and
the notary public. The latter four followed the reading with their
own respective copies previously furnished them.

Meanwhile, Brigido's holographic will was subsequently


admitted to probate on 9 December 1977. On the 29th day of
the same month, a codicil entitled "Kasulatan ng Pagbabago
sa Ilang Pagpapasiya na Nasasaad sa Huling Habilin na may
Petsa Nobiembre 5, 1977 ni Brigido Alvarado" was executed
changing some dispositions in the notarial will to generate cash
for the testator's eye operation. Brigido was then suffering from
glaucoma. But the disinheritance and revocatory clauses were
unchanged. As in the case of the notarial will, the testator did
not personally read the final draft of the codicil. Instead, it was
private respondent who read it aloud in his presence and in the
presence of the three instrumental witnesses (same as those
of the notarial will) and the notary public who followed the
reading using their own copies.
A petition for the probate of the notarial will and codicil was
filed upon the testator's death on 3 January 1979 by private
respondent as executor with the Court of First Instance, now
Regional Trial Court, of Siniloan, Laguna. 5Petitioner, in turn,
filed an Opposition on the following grounds: that the will
sought to be probated was not executed and attested as
required by law; that the testator was insane or otherwise
mentally incapacitated to make a will at the time of its
execution due to senility and old age; that the will was
executed under duress, or influence of fear and threats; that it
was procured by undue and improper pressure and influence
on the part of the beneficiary who stands to get the lion's share
of the testator's estate; and lastly, that the signature of the
testator was procured by fraud or trick.
When the oppositor (petitioner) failed to substantiate the
grounds relied upon in the Opposition, a Probate Order was
issued on 27 June 1983 from which an appeal was made to
respondent court. The main thrust of the appeal was that the
deceased was blind within the meaning of the law at the time
his "Huling Habilin" and the codicil attached thereto was
executed; that since the reading required by Art. 808 of the
Civil Code was admittedly not complied with, probate of the
deceased's last will and codicil should have been denied.
On 11 April 1986, the Court of Appeals rendered the decision
under review with the following findings: that Brigido Alvarado
was not blind at the time his last will and codicil were executed;
that assuming his blindness, the reading requirement of Art.
808 was substantially complied with when both documents
were read aloud to the testator with each of the three
instrumental witnesses and the notary public following the
reading with their respective copies of the instruments. The
appellate court then concluded that although Art. 808 was not
followed to the letter, there was substantial compliance since
its purpose of making known to the testator the contents of the
drafted will was served.
The issues now before us can be stated thus: Was Brigido
Alvarado blind for purpose of Art, 808 at the time his "Huling
Habilin" and its codicil were executed? If so, was the doublereading requirement of said article complied with?
Regarding the first issue, there is no dispute on the following
facts: Brigido Alvarado was not totally blind at the time the will
and codicil were executed. However, his vision on both eyes
was only of "counting fingers at three (3) feet" by reason of the
glaucoma which he had been suffering from for several years

79
and even prior to his first consultation with an eye specialist on
14 December 1977.
The point of dispute is whether the foregoing circumstances
would qualify Brigido as a "blind" testator under Art. 808 which
reads:
Art. 808. If the testator is blind, the will shall be
read to him twice; once, by one of the subscribing
witnesses, and again, by the notary public before
whom the will is acknowledged.
Petitioner contends that although his father was not totally
blind when the will and codicil were executed, he can be so
considered within the scope of the term as it is used in Art.
808. To support his stand, petitioner presented before the trial
court a medical certificate issued by Dr. Salvador R. Salceda,
Director of the Institute of Opthalmology (Philippine Eye
Research Institute), 6 the contents of which were interpreted in
layman's terms by Dr. Ruperto Roasa, whose expertise was
admitted by private respondent. 7 Dr. Roasa explained that
although the testator could visualize fingers at three (3) feet, he
could no longer read either printed or handwritten matters as of
14 December 1977, the day of his first consultation. 8
On the other hand, the Court of Appeals, contrary to the
medical testimony, held that the testator could still read on the
day the will and the codicil were executed but chose not to do
so because of "poor eyesight." 9 Since the testator was still
capable of reading at that time, the court a quo concluded that
Art. 808 need not be complied with.

ascertaining whether or not the lawyer who drafted the will and
codicil did so confortably with his instructions. Hence, to
consider his will as validly executed and entitled to probate, it
is essential that we ascertain whether Art. 808 had been
complied with.
Article 808 requires that in case of testators like Brigido
Alvarado, the will shall be read twice; once, by one of the
instrumental witnesses and, again, by the notary public before
whom the will was acknowledged. The purpose is to make
known to the incapacitated testator the contents of the
document before signing and to give him an opportunity to
object if anything is contrary to his instructions.
That Art. 808 was not followed strictly is beyond cavil. Instead
of the notary public and an instrumental witness, it was the
lawyer (private respondent) who drafted the eight-paged will
and the five-paged codicil who read the same aloud to the
testator, and read them only once, not twice as Art. 808
requires.
Private respondent however insists that there was substantial
compliance and that the single reading suffices for purposes of
the law. On the other hand, petitioner maintains that the only
valid compliance or compliance to the letter and since it is
admitted that neither the notary public nor an instrumental
witness read the contents of the will and codicil to Brigido,
probate of the latter's will and codicil should have been
disallowed.
We sustain private respondent's stand and necessarily, the
petition must be denied.

We agree with petitioner in this respect.


Regardless of respondent's staunch contention that the
testator was still capable of reading at the time his will and
codicil were prepared, the fact remains and this was testified to
by his witnesses, that Brigido did not do so because of his
"poor," 10 "defective," 11 or "blurred" 12 vision making it
necessary for private respondent to do the actual reading for
him.
The following pronouncement in Garcia
vs. Vasquez 13 provides an insight into the scope of the term
"blindness" as used in Art. 808, to wit:
The rationale behind the requirement of
reading the will to the testator if he is blind or
incapable of reading the will himself (as
when he is illiterate), is to make the
provisions thereof known to him, so that he
may be able to object if they are not in
accordance with his wishes . . .
Clear from the foregoing is that Art. 808 applies not only to
blind testators but also to those who, for one reason or
another, are "incapable of reading the(ir) will(s)." Since Brigido
Alvarado was incapable of reading the final drafts of his will
and codicil on the separate occasions of their execution due to
his "poor," "defective," or "blurred" vision, there can be no other
course for us but to conclude that Brigido Alvarado comes
within the scope of the term "blind" as it is used in Art. 808.
Unless the contents were read to him, he had no way of

This Court has held in a number of occasions that substantial


compliance is acceptable where the purpose of the law has
been satisfied, the reason being that the solemnities
surrounding the execution of wills are intended to protect the
testator from all kinds of fraud and trickery but are never
intended to be so rigid and inflexible as to destroy the
testamentary privilege. 14
In the case at bar, private respondent read the testator's will
and codicil aloud in the presence of the testator, his three
instrumental witnesses, and the notary public. Prior and
subsequent thereto, the testator affirmed, upon being asked,
that the contents read corresponded with his instructions. Only
then did the signing and acknowledgement take place. There is
no evidence, and petitioner does not so allege, that the
contents of the will and codicil were not sufficiently made
known and communicated to the testator. On the contrary, with
respect to the "Huling Habilin," the day of the execution was
not the first time that Brigido had affirmed the truth and
authenticity of the contents of the draft. The uncontradicted
testimony of Atty. Rino is that Brigido Alvarado already
acknowledged that the will was drafted in accordance with his
expressed wishes even prior to 5 November 1977 when Atty.
Rino went to the testator's residence precisely for the purpose
of securing his conformity to the draft. 15
Moreover, it was not only Atty. Rino who read the documents
on 5 November and 29 December 1977. The notary public
and the three instrumental witnesses likewise read the will and
codicil, albeit silently. Afterwards, Atty. Nonia de la Pena (the
notary public) and Dr. Crescente O. Evidente (one of the three

80
instrumental witnesses and the testator's physician) asked the
testator whether the contents of the document were of his own
free will. Brigido answered in the affirmative. 16 With four
persons following the reading word for word with their own
copies, it can be safely concluded that the testator was
reasonably assured that what was read to him (those which he
affirmed were in accordance with his instructions), were the
terms actually appearing on the typewritten documents. This is
especially true when we consider the fact that the three
instrumental witnesses were persons known to the testator,
one being his physician (Dr. Evidente) and another
(Potenciano C. Ranieses) being known to him since childhood.
The spirit behind the law was served though the letter was not.
Although there should be strict compliance with the substantial
requirements of the law in order to insure the authenticity of the
will, the formal imperfections should be brushed aside when
they do not affect its purpose and which, when taken into
account, may only defeat the testator's will. 17
As a final word to convince petitioner of the propriety of the trial
court's Probate Order and its affirmance by the Court of
Appeals, we quote the following pronouncement in Abangan
v. Abangan, 18 to wit:
The object of the solemnities surrounding the
execution of wills is to close the door against
bad faith and fraud, to avoid the substitution
of wills and testaments and to guaranty their
truth and authenticity. Therefore the laws on
the subject should be interpreted in such a
way as to attain these primordial ends. But,
on the other hand, also one must not lose
sight of the fact that it is not the object of the
law to restrain and curtail the exercise of the
right to make a will. So when an
interpretation already given assures such
ends, any other interpretation whatsoever,
that adds nothing but demands more
requisites entirely unnecessary, useless and
frustrative of the testator's will, must be
disregarded (emphasis supplied).
Brigido Alvarado had expressed his last wishes in clear and
unmistakable terms in his "Huling Habilin" and the codicil
attached thereto. We are unwilling to cast these aside fro the
mere reason that a legal requirement intended for his
protection was not followed strictly when such compliance had
been rendered unnecessary by the fact that the purpose of the
law, i.e., to make known to the incapacitated testator the
contents of the draft of his will, had already been
accomplished. To reiterate, substantial compliance suffices
where the purpose has been served.
WHEREFORE, the petition is DENIED and the assailed
Decision of respondent Court of Appeals dated 11 April 1986 is
AFFIRMED. Considering the length of time that this case has
remained pending, this decision is immediately executory.
Costs against petitioner.
SO ORDERED.
Cruz, Grio-Aquino, Davide, Jr. and Quiason, JJ., concur.

Republic of the Philippines


SUPREME COURT
Manila
SECOND DIVISION

G.R. No. 106720 September 15, 1994


SPOUSES ROBERTO AND THELMA AJERO, petitioners,
vs.
THE COURT OF APPEALS AND CLEMENTE
SAND, respondents.
Miguel D. Larida for petitioners.
Montilla Law Office for private respondent.

PUNO, J.:
This is an appeal by certiorari from the Decision of the Court
of
Appeals 1 in CA-G.R. CV No. 22840, dated March 30, 1992,
the dispositive portion of which reads;
PREMISES CONSIDERED, the questioned
decision of November 19, 1988 of the trial
court is hereby REVERSED and SET ASIDE,
and the petition for probate is hereby
DISMISSED. No costs.
The earlier Decision was rendered by the RTC of
Quezon City, Branch 94, 2 in Sp. Proc. No. Q-37171,
and the instrument submitted for probate is the
holographic will of the late Annie Sand, who died on
November 25, 1982.
In the will, decedent named as devisees, the following:
petitioners Roberto and Thelma Ajero, private respondent
Clemente Sand, Meriam S. Arong, Leah Sand, Lilia Sand,
Edgar Sand, Fe Sand, Lisa S. Sand, and Dr. Jose Ajero, Sr.,
and their children.
On January 20, 1983, petitioners instituted Sp. Proc. No. Q37171, for allowance of decedent's holographic will. They
alleged that at the time of its execution, she was of sound and
disposing mind, not acting under duress, fraud or undue
influence, and was in every respect capacitated to dispose of
her estate by will.
Private respondent opposed the petition on the grounds that:
neither the testament's body nor the signature therein was in
decedent's handwriting; it contained alterations and corrections
which were not duly signed by decedent; and, the will was
procured by petitioners through improper pressure and undue
influence. The petition was likewise opposed by Dr. Jose Ajero.

81
He contested the disposition in the will of a house and lot
located in Cabadbaran, Agusan Del Norte. He claimed that
said property could not be conveyed by decedent in its entirety,
as she was not its sole owner.
Notwithstanding the oppositions, the trial court admitted the
decedent's holographic will to probate. It found, inter alia:
Considering then that the probate
proceedings herein must decide only the
question of identity of the will, its due
execution and the testamentary capacity of
the testatrix, this probate court finds no
reason at all for the disallowance of the will
for its failure to comply with the formalities
prescribed by law nor for lack of
testamentary capacity of the testatrix.
For one, no evidence was presented to show
that the will in question is different from the
will actually executed by the testatrix. The
only objections raised by the oppositors . . .
are that the will was not written in the
handwriting of the testatrix which properly
refers to the question of its due execution,
and not to the question of identity of will. No
other will was alleged to have been executed
by the testatrix other than the will herein
presented. Hence, in the light of the
evidence adduced, the identity of the will
presented for probate must be accepted, i.e.,
the will submitted in Court must be deemed
to be the will actually executed by the
testatrix.
xxx xxx xxx
While the fact that it was entirely written,
dated and signed in the handwriting of the
testatrix has been disputed, the petitioners,
however, have satisfactorily shown in Court
that the holographic will in question was
indeed written entirely, dated and signed in
the handwriting of the testatrix. Three (3)
witnesses who have convincingly shown
knowledge of the handwriting of the testatrix
have been presented and have explicitly and
categorically identified the handwriting with
which the holographic will in question was
written to be the genuine handwriting and
signature of the testatrix. Given then the
aforesaid evidence, the requirement of the
law that the holographic will be entirely
written, dated and signed in the handwriting
of the testatrix has been complied with.
xxx xxx xxx
As to the question of the testamentary
capacity of the testratix, (private respondent)
Clemente Sand himself has testified in Court
that the testatrix was completely in her
sound mind when he visited her during her
birthday celebration in 1981, at or around

which time the holographic will in question


was executed by the testatrix. To be of sound
mind, it is sufficient that the testatrix, at the
time of making the will, knew the value of the
estate to be disposed of, the proper object of
her bounty, and thecharacter of the
testamentary act . . . The will itself shows
that the testatrix even had detailed
knowledge of the nature of her estate. She
even identified the lot number and square
meters of the lots she had conveyed by will.
The objects of her bounty were likewise
identified explicitly. And considering that she
had even written a nursing book which
contained the law and jurisprudence on will
and succession, there is more than sufficient
showing that she knows the character of the
testamentary act.
In this wise, the question of identity of the
will, its due execution and the testamentary
capacity of the testatrix has to be resolved in
favor of the allowance of probate of the will
submitted herein.
Likewise, no evidence was presented to
show sufficient reason for the disallowance
of herein holographic will. While it was
alleged that the said will was procured by
undue and improper pressure and influence
on the part of the beneficiary or of some
other person, the evidence adduced have
not shown any instance where improper
pressure or influence was exerted on the
testatrix. (Private respondent) Clemente
Sand has testified that the testatrix was still
alert at the time of the execution of the
will, i.e., at or around the time of her birth
anniversary celebration in 1981. It was also
established that she is a very intelligent
person and has a mind of her own. Her
independence of character and to some
extent, her sense of superiority, which has
been testified to in Court, all show the
unlikelihood of her being unduly influenced
or improperly pressured to make the
aforesaid will. It must be noted that the
undue influence or improper pressure in
question herein only refer to the making of a
will and not as to the specific testamentary
provisions therein which is the proper subject
of another proceeding. Hence, under the
circumstances, this Court cannot find
convincing reason for the disallowance of the
will herein.
Considering then that it is a well-established
doctrine in the law on succession that in
case of doubt, testate succession should be
preferred over intestate succession, and the
fact that no convincing grounds were
presented and proven for the disallowance of
the holographic will of the late Annie Sand,
the aforesaid will submitted herein must be
admitted to probate. 3 (Citations omitted.)

82
On appeal, said Decision was reversed, and the petition for
probate of decedent's will was dismissed. The Court of Appeals
found that, "the holographic will fails to meet the requirements
for its validity." 4 It held that the decedent did not comply with
Articles 813 and 814 of the New Civil Code, which read, as
follows:
Art. 813: When a number of dispositions
appearing in a holographic will are signed
without being dated, and the last disposition
has a signature and date, such date
validates the dispositions preceding it,
whatever be the time of prior dispositions.
Art. 814: In case of insertion, cancellation,
erasure or alteration in a holographic will, the
testator must authenticate the same by his
full signature.
It alluded to certain dispositions in the will which were either
unsigned and undated, or signed but not dated. It also found
that the erasures, alterations and cancellations made thereon
had not been authenticated by decedent.
Thus, this appeal which is impressed with merit.
Section 9, Rule 76 of the Rules of Court provides that will shall
be disallowed in any of the following cases:
(a) If not executed and attested as required
by law;
(b) If the testator was insane, or otherwise
mentally incapable to make a will, at the time
of its execution;
(c) If it was executed under duress, or the
influence of fear, or threats;
(d) If it was procured by undue and improper
pressure and influence, on the part of the
beneficiary, or of some other person for his
benefit;
(e) If the signature of the testator was
procured by fraud or trick, and he did not
intend that the instrument should be his will
at the time of fixing his signature thereto.
In the same vein, Article 839 of the New Civil Code
reads:
Art. 839: The will shall be disallowed in any
of the following cases;
(1) If the formalities required by law
have not been complied with;
(2) If the testator was insane, or
otherwise mentally incapable of making
a will, at the time of its execution;

(3) If it was executed through force or


under duress, or the influence of fear, or
threats;
(4) If it was procured by undue and
improper pressure and influence, on the
part of the beneficiary or of some other
person;
(5) If the signature of the testator was
procured by fraud;
(6) If the testator acted by mistake or did
not intend that the instrument he signed
should be his will at the time of affixing
his signature thereto.
These lists are exclusive; no other grounds can serve to
disallow a will. 5 Thus, in a petition to admit a holographic will
to probate, the only issues to be resolved are: (1) whether the
instrument submitted is, indeed, the decedent's last will and
testament; (2) whether said will was executed in accordance
with the formalities prescribed by law; (3) whether the
decedent had the necessary testamentary capacity at the time
the will was executed; and, (4) whether the execution of the will
and its signing were the voluntary acts of the decedent. 6
In the case at bench, respondent court held that the
holographic will of Anne Sand was not executed in accordance
with the formalities prescribed by law. It held that Articles 813
and 814 of the New Civil Code, ante, were not complied with,
hence, it disallowed the probate of said will. This is erroneous.
We reiterate what we held in Abangan vs. Abangan, 40 Phil.
476, 479 (1919), that:
The object of the solemnities surrounding the
execution of wills is to close the door against
bad faith and fraud, to avoid substitution of
wills and testaments and to guaranty their
truth and authenticity. Therefore, the laws on
this subject should be interpreted in such a
way as to attain these primordial ends. But,
on the other hand, also one must not lose
sight of the fact that it is not the object of the
law to restrain and curtail the exercise of the
right to make a will. So when an
interpretation already given assures such
ends, any other interpretation whatsoever,
that adds nothing but demands more
requisites entirely unnecessary, useless and
frustrative of the testator's last will, must be
disregarded.
For purposes of probating non-holographic wills, these formal
solemnities include the subscription, attestation, and
acknowledgment requirements under Articles 805 and 806 of
the New Civil Code.
In the case of holographic wills, on the other hand, what
assures authenticity is the requirement that they be totally
autographic or handwritten by the testator himself, 7 as
provided under Article 810 of the New Civil Code, thus:

83
A person may execute a holographic will
which must be entirely written, dated, and
signed by the hand of the testator himself. It
is subject to no other form, and may be
made in or out of the Philippines, and need
not be witnessed. (Emphasis supplied.)
Failure to strictly observe other formalities will not
result in the disallowance of a holographic will that is
unquestionably handwritten by the testator.
A reading of Article 813 of the New Civil Code shows that its
requirement affects the validity of the dispositions contained in
the holographic will, but not its probate. If the testator fails to
sign and date some of the dispositions, the result is that these
dispositions cannot be effectuated. Such failure, however, does
not render the whole testament void.
Likewise, a holographic will can still be admitted to probate,
notwithstanding non-compliance with the provisions of Article
814. In the case of Kalaw vs. Relova 132 SCRA 237
242 (1984), this Court held:
Ordinarily, when a number of erasures,
corrections, and interlineations made by the
testator in a holographic Will have not been
noted under his signature, . . . the Will is not
thereby invalidated as a whole, but at most
only as respects the particular words erased,
corrected or interlined. Manresa gave an
identical commentary when he said "la
omission de la salvedad no anula el
testamento, segun la regla de jurisprudencia
establecida en la sentencia de 4 de Abril de
1985." 8 (Citations omitted.)
Thus, unless the unauthenticated alterations, cancellations or
insertions were made on the date of the holographic will or on
testator's signature, 9 their presence does not invalidate the will
itself. 10 The lack of authentication will only result in
disallowance of such changes.

If it should contain any erased, corrected, or


interlined words, the testator must identify them
over his signature.
Foreigners may execute holographic wills in their
own language.
This separation and distinction adds support to the
interpretation that only the requirements of Article 810 of the
New Civil Code and not those found in Articles 813 and 814
of the same Code are essential to the probate of a
holographic will.
The Court of Appeals further held that decedent Annie Sand
could not validly dispose of the house and lot located in
Cabadbaran, Agusan del Norte, in its entirety. This is correct
and must be affirmed.
As a general rule, courts in probate proceedings are limited to
pass only upon the extrinsic validity of the will sought to be
probated. However, in exceptional instances, courts are not
powerless to do what the situation constrains them to do, and
pass upon certain provisions of the will. 11 In the case at bench,
decedent herself indubitably stated in her holographic will that
the Cabadbaran property is in the name of her late father, John
H. Sand (which led oppositor Dr. Jose Ajero to question her
conveyance of the same in its entirety). Thus, as correctly held
by respondent court, she cannot validly dispose of the whole
property, which she shares with her father's other heirs.
IN VIEW WHEREOF, the instant petition is GRANTED. The
Decision of the Court of Appeals in CA-G.R. CV No. 22840,
dated March 30, 1992, is REVERSED and SET ASIDE, except
with respect to the invalidity of the disposition of the entire
house and lot in Cabadbaran, Agusan del Norte. The Decision
of the Regional Trial Court of Quezon City, Branch 94 in Sp.
Proc. No. Q-37171, dated November 19, 1988, admitting to
probate the holographic will of decedent Annie Sand, is hereby
REINSTATED, with the above qualification as regards the
Cabadbaran property. No costs.
SO ORDERED.

It is also proper to note that the requirements of authentication


of changes and signing and dating of dispositions appear in
provisions (Articles 813 and 814) separate from that which
provides for the necessary conditions for the validity of the
holographic will (Article 810). The distinction can be traced to
Articles 678 and 688 of the Spanish Civil Code, from which the
present provisions covering holographic wills are taken. They
read as follows:
Art. 678: A will is called holographic when the
testator writes it himself in the form and with the
requisites required in Article 688.
Art. 688: Holographic wills may be executed only by
persons of full age.
In order that the will be valid it must be drawn on
stamped paper corresponding to the year of its
execution, written in its entirety by the testator and
signed by him, and must contain a statement of the
year, month and day of its execution.

Narvasa, C.J., Padilla, Regalado and Mendoza, JJ.,


concur.
Republic of the Philippines
SUPREME COURT
Manila
FIRST DIVISION
G.R. No. L-40207 September 28, 1984
ROSA K. KALAW, petitioner,
vs.
HON. JUDGE BENJAMIN RELOVA, Presiding Judge of the
CFI of Batangas, Branch VI, Lipa City, and GREGORIO K.
KALAW, respondents.
Leandro H. Fernandez for petitioner.

84
Antonio Quintos and Jose M. Yacat for respondents.
MELENCIO-HERRERA, J.:
On September 1, 1971, private respondent GREGORIO K.
KALAW, claiming to be the sole heir of his deceased sister,
Natividad K. Kalaw, filed a petition before the Court of First
Instance of Batangas, Branch VI, Lipa City, for the probate of
her holographic Will executed on December 24, 1968.
The holographic Will reads in full as follows:
My Last will and Testament
In the name of God, Amen.
I Natividad K. Kalaw Filipino 63years of age, single, and a
resident of Lipa City, being of sound and disposing mind and
memory, do hereby declare thus to be my last will and
testament.
1. It is my will that I'll be burried in the cemetery of the catholic
church of Lipa City. In accordance with the rights of said
Church, and that my executrix hereinafter named provide and
erect at the expose of my state a suitable monument to
perpetuate my memory.
xxx xxx xxx
The holographic Will, as first written, named ROSA K. Kalaw, a
sister of the testatrix as her sole heir. Hence, on November 10,
1971, petitioner ROSA K. Kalaw opposed probate alleging, in
substance, that the holographic Will contained alterations,
corrections, and insertions without the proper authentication by
the full signature of the testatrix as required by Article 814 of
the Civil Code reading:
Art. 814. In case of any insertion,
cancellation, erasure or alteration in a
holographic will the testator must
authenticate the same by his full signature.
ROSA's position was that the holographic Will, as first written,
should be given effect and probated so that she could be the
sole heir thereunder.
After trial, respondent Judge denied probate in an Order, dated
September 3, 197 3, reading in part:
The document Exhibit "C" was submitted to
the National Bureau of Investigation for
examination. The NBI reported that the
handwriting, the signature, the insertions
and/or additions and the initial were made by
one and the same person. Consequently,
Exhibit "C" was the handwriting of the
decedent, Natividad K. Kalaw. The only
question is whether the win, Exhibit 'C',
should be admitted to probate although the
alterations and/or insertions or additions
above-mentioned were not authenticated by
the full signature of the testatrix pursuant to

Art. 814 of the Civil Code. The petitioner


contends that the oppositors are estopped to
assert the provision of Art. 814 on the ground
that they themselves agreed thru their
counsel to submit the Document to the NBI
FOR EXAMINATIONS. This is untenable.
The parties did not agree, nor was it
impliedly understood, that the oppositors
would be in estoppel.
The Court finds, therefore, that the provision
of Article 814 of the Civil Code is applicable
to Exhibit "C". Finding the insertions,
alterations and/or additions in Exhibit "C" not
to be authenticated by the full signature of
the testatrix Natividad K. Kalaw, the Court
will deny the admission to probate of Exhibit
"C".
WHEREFORE, the petition to probate
Exhibit "C" as the holographic will of
Natividad K. Kalaw is hereby denied.
SO ORDERED.
From that Order, GREGORIO moved for reconsideration
arguing that since the alterations and/or insertions were the
testatrix, the denial to probate of her holographic Will would be
contrary to her right of testamentary disposition.
Reconsideration was denied in an Order, dated November 2,
1973, on the ground that "Article 814 of the Civil Code being ,
clear and explicit, (it) requires no necessity for interpretation."
From that Order, dated September 3, 1973, denying probate,
and the Order dated November 2, 1973 denying
reconsideration, ROSA filed this Petition for Review on
certiorari on the sole legal question of whether or not
theoriginal unaltered text after subsequent alterations and
insertions were voided by the Trial Court for lack of
authentication by the full signature of the testatrix, should be
probated or not, with her as sole heir.
Ordinarily, when a number of erasures, corrections, and
interlineations made by the testator in a holographic Will litem
not been noted under his signature, ... the Will is not thereby
invalidated as a whole, but at most only as respects the
particular words erased, corrected or interlined.1 Manresa
gave an Identical commentary when he said "la omision de la
salvedad no anula el testamento, segun la regla de
jurisprudencia establecida en la sentencia de 4 de Abril de
1895." 2
However, when as in this case, the holographic Will in dispute
had only one substantial provision, which was altered by
substituting the original heir with another, but which alteration
did not carry the requisite of full authentication by the full
signature of the testator, the effect must be that the entire Will
is voided or revoked for the simple reason that nothing remains
in the Will after that which could remain valid. To state that the
Will as first written should be given efficacy is to disregard the
seeming change of mind of the testatrix. But that change of
mind can neither be given effect because she failed to
authenticate it in the manner required by law by affixing her full
signature,

85
The ruling in Velasco, supra, must be held confined to such
insertions, cancellations, erasures or alterations in a
holographic Will, which affect only the efficacy of the altered
words themselves but not the essence and validity of the Will
itself. As it is, with the erasures, cancellations and alterations
made by the testatrix herein, her real intention cannot be
determined with certitude. As Manresa had stated in his
commentary on Article 688 of the Spanish Civil Code, whence
Article 814 of the new Civil Code was derived:
... No infringe lo dispuesto en este articulo
del Codigo (el 688) la sentencia que no
declara la nulidad de un testamento olografo
que contenga palabras tachadas,
enmendadas o entre renglones no salvadas
por el testador bajo su firnia segun previene
el parrafo tercero del mismo, porque, en
realidad, tal omision solo puede afectar a la
validez o eficacia de tales palabras, y nunca
al testamento mismo, ya por estar esa
disposicion en parrafo aparte de aquel que
determine las condiciones necesarias para
la validez del testamento olografo, ya
porque, de admitir lo contrario, se Ilegaria al
absurdo de que pequefias enmiendas no
salvadas, que en nada afectasen a la parte
esencial y respectiva del testamento,
vinieran a anular este, y ya porque el
precepto contenido en dicho parrafo ha de
entenderse en perfecta armonia y
congruencia con el art. 26 de la ley del
Notariado que declara nulas las adiciones
apostillas entrerrenglonados, raspaduras y
tachados en las escrituras matrices, siempre
que no se salven en la forma prevenida,
paro no el documento que las contenga, y
con mayor motivo cuando las palabras
enmendadas, tachadas, o
entrerrenglonadas no tengan importancia ni
susciten duda alguna acerca del
pensamiento del testador, o constituyan
meros accidentes de ortografia o de purez
escrituraria, sin trascendencia alguna(l).
Mas para que sea aplicable la doctrina de
excepcion contenida en este ultimo fallo, es
preciso que las tachaduras, enmiendas o
entrerrenglonados sin salvar saan de pala
bras que no afecter4 alteren ni uarien de
modo substancial la express voluntad del
testador manifiesta en el documento. Asi lo
advierte la sentencia de 29 de Noviembre de
1916, que declara nulo un testamento
olografo por no estar salvada por el testador
la enmienda del guarismo ultimo del ao en
que fue extendido 3(Emphasis ours).
WHEREFORE, this Petition is hereby dismissed and the
Decision of respondent Judge, dated September 3, 1973, is
hereby affirmed in toto. No costs.

Relova, J., took no part.


Separate Opinions
TEEHANKEE, J., concurring:
I concur. Rosa, having appealed to this Court on a sole
question of law, is bound by the trial court's factual finding that
the peculiar alterations in the holographic will crossing out
Rosa's name and instead inserting her brother Gregorio's
name as sole heir and "sole executrix" were made by the
testatrix in her own handwriting. (I find it peculiar that the
testatrix who was obviously an educated person would
unthinkingly make such crude alterations instead of consulting
her lawyer and writing an entirely new holographic wig in order
to avoid any doubts as to her change of heir. It should be noted
that the first alteration crossing out "sister Rosa K. Kalaw" and
inserting "brother Gregorio Kalaw" as sole heir is not even
initialed by the testatrix. Only the second alteration crossing
out "sister Rosa K. Kalaw" and inserting "brother Gregorio
Kalaw" as "sole executrix" is initialed.) Probate of the radically
altered will replacing Gregorio for Rosa as sole heir is properly
denied, since the same was not duly authenticated by the full
signature of the executrix as mandatorily required by Article
814 of the Civil Code. The original unaltered will naming Rosa
as sole heir cannot, however, be given effect in view of the trial
court's factual finding that the testatrix had by her own
handwriting substituted Gregorio for Rosa, so that there is no
longer any will naming Rosa as sole heir. The net result is that
the testatrix left no valid will and both Rosa and Gregorio as
her next of kill succeed to her intestate estate.
Republic of the Philippines
SUPREME COURT
Manila
FIRST DIVISION
G.R. No. L-58509 December 7, 1982
IN THE MATTER OF THE PETITION TO APPROVE THE
WILL OF RICARDO B. BONILLA deceased, MARCELA
RODELAS, petitioner-appellant,
vs.
AMPARO ARANZA, ET AL., oppositors-appellees, ATTY.
LORENZO SUMULONG, intervenor.
Luciano A. Joson for petitioner-appellant.
Cesar Paralejo for oppositor-appellee.
RELOVA, J.:
This case was certified to this Tribunal by the Court of Appeals
for final determination pursuant to Section 3, Rule 50 of the
Rules of Court.
As found by the Court of Appeals:

SO ORDERED.
Plana, Gutierrez, Jr. and De la Fuente, JJ., concur.

... On January 11, 1977, appellant filed a petition with


the Court of First Instance of Rizal for the probate of
the holographic will of Ricardo B. Bonilla and the

86
issuance of letters testamentary in her favor. The
petition, docketed as Sp. Proc. No. 8432, was opposed
by the appellees Amparo Aranza Bonilla, Wilferine
Bonilla Treyes Expedita Bonilla Frias and Ephraim
Bonilla on the following grounds:

In the case of Gam vs. Yap, 104 Phil. 509, 522,


the Supreme Court held that 'in the matter of
holographic wills the law, it is reasonable to
suppose, regards the document itself as the
material proof of authenticity of said wills.

(1) Appellant was estopped from claiming that the


deceased left a will by failing to produce the will within
twenty days of the death of the testator as required by
Rule 75, section 2 of the Rules of Court;

MOREOVER, this Court notes that the alleged


holographic will was executed on January 25,
1962 while Ricardo B. Bonilla died on May 13,
1976. In view of the lapse of more than 14 years
from the time of the execution of the will to the
death of the decedent, the fact that the original of
the will could not be located shows to our mind
that the decedent had discarded before his death
his allegedly missing Holographic Will.

(2) The alleged copy of the alleged holographic will


did not contain a disposition of property after death and
was not intended to take effect after death, and
therefore it was not a will
(3) The alleged hollographic will itself,and not an
alleged copy thereof, must be produced, otherwise it
would produce no effect, as held in Gam v. Yap, 104
Phil. 509; and
(4 ) The deceased did not leave any will,
holographic or otherwise, executed and attested
as required by law.
The appellees likewise moved for the
consolidation of the case with another case Sp.
Proc. No, 8275). Their motion was granted by the
court in an order dated April 4, 1977.
On November 13, 1978, following the
consolidation of the cases, the appellees moved
again to dismiss the petition for the probate of the
will. They argued that:
(1) The alleged holographic was not a last will but
merely an instruction as to the management and
improvement of the schools and colleges founded
by decedent Ricardo B. Bonilla; and
(2) Lost or destroyed holographic wills cannot be
proved by secondary evidence unlike ordinary
wills.
Upon opposition of the appellant, the motion to
dismiss was denied by the court in its order of
February 23, 1979.
The appellees then filed a motion for
reconsideration on the ground that the order was
contrary to law and settled pronouncements and
rulings of the Supreme Court, to which the
appellant in turn filed an opposition. On July 23,
1979, the court set aside its order of February 23,
1979 and dismissed the petition for the probate of
the will of Ricardo B. Bonilla. The court said:
... It is our considered opinion that once the
original copy of the holographic will is lost, a copy
thereof cannot stand in lieu of the original.

Appellant's motion for reconsideration was denied. Hence, an


appeal to the Court of Appeals in which it is contended that the
dismissal of appellant's petition is contrary to law and wellsettled jurisprudence.
On July 7, 1980, appellees moved to forward the case to this
Court on the ground that the appeal does not involve question
of fact and alleged that the trial court committed the following
assigned errors:
I. THE LOWER COURT ERRED IN HOLDING THAT
A LOST HOLOGRAPHIC WILL MAY NOT BE
PROVED BY A COPY THEREOF;
II. THE LOWER COURT ERRED IN HOLDING THAT
THE DECEDENT HAS DISCARDED BEFORE HIS
DEATH THE MISSING HOLOGRAPHIC WILL;
III. THE LOWER COURT ERRED IN DISMISSING
APPELLANT'S WILL.
The only question here is whether a holographic will which was
lost or cannot be found can be proved by means of a
photostatic copy. Pursuant to Article 811 of the Civil Code,
probate of holographic wills is the allowance of the will by the
court after its due execution has been proved. The probate
may be uncontested or not. If uncontested, at least one
Identifying witness is required and, if no witness is available,
experts may be resorted to. If contested, at least three
Identifying witnesses are required. However, if the holographic
will has been lost or destroyed and no other copy is available,
the will can not be probated because the best and only
evidence is the handwriting of the testator in said will. It is
necessary that there be a comparison between sample
handwritten statements of the testator and the handwritten will.
But, a photostatic copy or xerox copy of the holographic will
may be allowed because comparison can be made with the
standard writings of the testator. In the case of Gam vs. Yap,
104 PHIL. 509, the Court ruled that "the execution and the
contents of a lost or destroyed holographic will may not be
proved by the bare testimony of witnesses who have seen
and/or read such will. The will itself must be presented;
otherwise, it shall produce no effect. The law regards the
document itself as material proof of authenticity." But, in
Footnote 8 of said decision, it says that "Perhaps it may be
proved by a photographic or photostatic copy. Even a
mimeographed or carbon copy; or by other similar means, if
any, whereby the authenticity of the handwriting of the

87
deceased may be exhibited and tested before the probate
court," Evidently, the photostatic or xerox copy of the lost or
destroyed holographic will may be admitted because then the
authenticity of the handwriting of the deceased can be
determined by the probate court.
WHEREFORE, the order of the lower court dated October 3,
1979, denying appellant's motion for reconsideration dated
August 9, 1979, of the Order dated July 23, 1979, dismissing
her petition to approve the will of the late Ricardo B. Bonilla, is
hereby SET ASIDE.

deceased Bibiana Roxas de Jesus. On May 26, 1973,


respondent Judge Jose Colayco set the hearing of the probate
of the holographic Win on July 21, 1973.
Petitioner Simeon R. Roxas testified that after his appointment
as administrator, he found a notebook belonging to the
deceased Bibiana R. de Jesus and that on pages 21, 22, 23
and 24 thereof, a letter-win addressed to her children and
entirely written and signed in the handwriting of the deceased
Bibiana R. de Jesus was found. The will is dated "FEB./61 "
and states: "This is my win which I want to be respected
although it is not written by a lawyer. ...

SO ORDERED.
Teehankee, Actg. C.J., Melencio-Herrera, Plana, Vasquez and
Gutierrez, Jr., JJ., concur.

Republic of the Philippines


SUPREME COURT
Manila
FIRST DIVISION
G.R. No. L-38338 January 28, 1985
IN THE MATTER OF THE INTESTATE ESTATE OF ANDRES
G. DE JESUS AND BIBIANA ROXAS DE JESUS, SIMEON R.
ROXAS & PEDRO ROXAS DE JESUS, petitioners,
vs.
ANDRES R. DE JESUS, JR., respondent.
Raul S. Sison Law Office for petitioners.
Rafael Dinglasan, Jr. for heir M. Roxas.
Ledesma, Guytingco Velasco and Associates for Ledesa and
A. R. de Jesus.
GUTIERREZ, JR., J.:
This is a petition for certiorari to set aside the order of
respondent Hon. Jose C. Colayco, Presiding Judge Court of
First Instance of Manila, Branch XXI disallowing the probate of
the holographic Will of the deceased Bibiana Roxas de Jesus.
The antecedent facts which led to the filing of this petition are
undisputed.
After the death of spouses Andres G. de Jesus and Bibiana
Roxas de Jesus, Special Proceeding No. 81503 entitled "In the
Matter of the Intestate Estate of Andres G. de Jesus and
Bibiana Roxas de Jesus" was filed by petitioner Simeon R.
Roxas, the brother of the deceased Bibiana Roxas de Jesus.
On March 26, 1973, petitioner Simeon R. Roxas was
appointed administrator. After Letters of Administration had
been granted to the petitioner, he delivered to the lower court a
document purporting to be the holographic Will of the

The testimony of Simeon R. Roxas was corroborated by the


testimonies of Pedro Roxas de Jesus and Manuel Roxas de
Jesus who likewise testified that the letter dated "FEB./61 " is
the holographic Will of their deceased mother, Bibiana R. de
Jesus. Both recognized the handwriting of their mother and
positively Identified her signature. They further testified that
their deceased mother understood English, the language in
which the holographic Will is written, and that the date
"FEB./61 " was the date when said Will was executed by their
mother.
Respondent Luz R. Henson, another compulsory heir filed an
"opposition to probate" assailing the purported holographic Will
of Bibiana R. de Jesus because a it was not executed in
accordance with law, (b) it was executed through force,
intimidation and/or under duress, undue influence and
improper pressure, and (c) the alleged testatrix acted by
mistake and/or did not intend, nor could have intended the said
Will to be her last Will and testament at the time of its
execution.
On August 24, 1973, respondent Judge Jose C. Colayco
issued an order allowing the probate of the holographic Will
which he found to have been duly executed in accordance with
law.
Respondent Luz Roxas de Jesus filed a motion for
reconsideration alleging inter alia that the alleged holographic
Will of the deceased Bibiana R. de Jesus was not dated as
required by Article 810 of the Civil Code. She contends that the
law requires that the Will should contain the day, month and
year of its execution and that this should be strictly complied
with.
On December 10, 1973, respondent Judge Colayco
reconsidered his earlier order and disallowed the probate of
the holographic Will on the ground that the word "dated" has
generally been held to include the month, day, and year. The
dispositive portion of the order reads:
WHEREFORE, the document purporting to be the
holographic Will of Bibiana Roxas de Jesus, is hereby
disallowed for not having been executed as required
by the law. The order of August 24, 1973 is hereby set
aside.
The only issue is whether or not the date "FEB./61 " appearing
on the holographic Will of the deceased Bibiana Roxas de
Jesus is a valid compliance with the Article 810 of the Civil
Code which reads:

88
ART. 810. A person may execute a holographic will
which must be entirely written, dated, and signed by
the hand of the testator himself. It is subject to no
other form, and may be made in or out of the
Philippines, and need not be witnessed.
The petitioners contend that while Article 685 of the Spanish
Civil Code and Article 688 of the Old Civil Code require the
testator to state in his holographic Win the "year, month, and
day of its execution," the present Civil Code omitted the phrase
Ao mes y dia and simply requires that the holographic Will
should be dated. The petitioners submit that the liberal
construction of the holographic Will should prevail.

succession is nothing more than a


disposition based upon the presumed will of
the decedent.
Thus, the prevailing policy is to require satisfaction of the legal
requirements in order to guard against fraud and bad faith but
without undue or unnecessary curtailment of testamentary
privilege Icasiano v. Icasiano, 11 SCRA 422). If a Will has been
executed in substantial compliance with the formalities of the
law, and the possibility of bad faith and fraud in the exercise
thereof is obviated, said Win should be admitted to probate
(Rey v. Cartagena 56 Phil. 282). Thus,
xxx xxx xxx

Respondent Luz Henson on the other hand submits that the


purported holographic Will is void for non-compliance with
Article 810 of the New Civil Code in that the date must contain
the year, month, and day of its execution. The respondent
contends that Article 810 of the Civil Code was patterned after
Section 1277 of the California Code and Section 1588 of the
Louisiana Code whose Supreme Courts had consistently ruled
that the required date includes the year, month, and day, and
that if any of these is wanting, the holographic Will is invalid.
The respondent further contends that the petitioner cannot
plead liberal construction of Article 810 of the Civil Code
because statutes prescribing the formalities to be observed in
the execution of holographic Wills are strictly construed.
We agree with the petitioner.
This will not be the first time that this Court departs from a strict
and literal application of the statutory requirements regarding
the due execution of Wills. We should not overlook the liberal
trend of the Civil Code in the manner of execution of Wills, the
purpose of which, in case of doubt is to prevent intestacy
The underlying and fundamental objectives
permeating the provisions of the law on wigs
in this Project consists in the liberalization of
the manner of their execution with the end in
view of giving the testator more freedom in
expressing his last wishes, but with sufficien
safeguards and restrictions to prevent the
commission of fraud and the exercise of
undue and improper pressure and influence
upon the testator.
This objective is in accord with the modem
tendency with respect to the formalities in
the execution of wills. (Report of the Code
Commission, p. 103)
In Justice Capistrano's concurring opinion in Heirs
of Raymundo Castro v. Bustos (27 SCRA 327) he emphasized
that:
xxx xxx xxx
... The law has a tender regard for the will of
the testator expressed in his last will and
testament on the ground that any disposition
made by the testator is better than that which
the law can make. For this reason, intestate

... More than anything else, the facts and


circumstances of record are to be
considered in the application of any given
rule. If the surrounding circumstances point
to a regular execution of the wilt and the
instrument appears to have been executed
substantially in accordance with the
requirements of the law, the inclination
should, in the absence of any suggestion of
bad faith, forgery or fraud, lean towards its
admission to probate, although the
document may suffer from some
imperfection of language, or other nonessential defect. ... (Leynez v. Leynez 68
Phil. 745).
If the testator, in executing his Will, attempts to comply with all
the requisites, although compliance is not literal, it is sufficient
if the objective or purpose sought to be accomplished by such
requisite is actually attained by the form followed by the
testator.
The purpose of the solemnities surrounding the execution of
Wills has been expounded by this Court in Abangan v.
Abanga 40 Phil. 476, where we ruled that:
The object of the solemnities surrounding the
execution of wills is to close the door against
bad faith and fraud, to avoid substitution of
wills and testaments and to guaranty their
truth and authenticity. ...
In particular, a complete date is required to provide against
such contingencies as that of two competing Wills executed on
the same day, or of a testator becoming insane on the day on
which a Will was executed (Velasco v. Lopez, 1 Phil. 720).
There is no such contingency in this case.
We have carefully reviewed the records of this case and found
no evidence of bad faith and fraud in its execution nor was
there any substitution of Wins and Testaments. There is no
question that the holographic Will of the deceased Bibiana
Roxas de Jesus was entirely written, dated, and signed by the
testatrix herself and in a language known to her. There is also
no question as to its genuineness and due execution. All the
children of the testatrix agree on the genuineness of the
holographic Will of their mother and that she had the
testamentary capacity at the time of the execution of said Will.

89
The objection interposed by the oppositor-respondent Luz
Henson is that the holographic Will is fatally defective because
the date "FEB./61 " appearing on the holographic Will is not
sufficient compliance with Article 810 of the Civil Code. This
objection is too technical to be entertained.
As a general rule, the "date" in a holographic Will should
include the day, month, and year of its execution. However,
when as in the case at bar, there is no appearance of fraud,
bad faith, undue influence and pressure and the authenticity of
the Will is established and the only issue is whether or not the
date "FEB./61" appearing on the holographic Will is a valid
compliance with Article 810 of the Civil Code, probate of the
holographic Will should be allowed under the principle of
substantial compliance.
WHEREFORE, the instant petition is GRANTED. The order
appealed from is REVERSED and SET ASIDE and the order
allowing the probate of the holographic Will of the deceased
Bibiana Roxas de Jesus is reinstated.
SO ORDERED.
Teehankee (Chairman), Melencio-Herrera, Plana, Relova and
De la Fuente, JJ., concur.
THIRD DIVISION
UY KIAO ENG,
Petitioner,
- versus NIXON LEE,
Respondent
DECISION
NACHURA, J.:
Before the Court is a petition for review
on certiorari under Rule 45 of the Rules of Court, assailing the
August 23, 2006 Amended Decision[1] of the Court of Appeals
(CA) in CA-G.R. SP No. 91725 and the February 23, 2007
Resolution,[2] denying the motion for reconsideration thereof.
The relevant facts and proceedings follow.
Alleging that his father passed away on June 22, 1992 in
Manila and left a holographic will, which is now in the custody
of petitioner Uy Kiao Eng, his mother, respondent Nixon Lee
filed, on May 28, 2001, a petition for mandamus with damages,
docketed as Civil Case No. 01100939, before the Regional
Trial Court (RTC) of Manila, to compel petitioner to produce the
will so that probate proceedings for the allowance thereof could
be instituted. Allegedly, respondent had already requested his
mother to settle and liquidate the patriarchs estate and to
deliver to the legal heirs their respective inheritance, but
petitioner refused to do so without any justifiable reason.[3]
In her answer with counterclaim, petitioner traversed the
allegations in the complaint and posited that the same be
dismissed for failure to state a cause of action, for lack of
cause of action, and for non-compliance with a condition

precedent for the filing thereof. Petitioner denied that she was
in custody of the original holographic will and that she knew of
its whereabouts. She, moreover, asserted that photocopies of
the will were given to respondent and to his siblings. As a
matter of fact, respondent was able to introduce, as an exhibit,
a copy of the will in Civil Case No. 224-V-00 before the RTC of
Valenzuela City. Petitioner further contended that respondent
should have first exerted earnest efforts to amicably settle the
controversy with her before he filed the suit.[4]
The RTC heard the case. After the presentation and
formal offer of respondents evidence, petitioner demurred,
contending that her son failed to prove that she had in her
custody the original holographic will. Importantly, she asserted
that the pieces of documentary evidence presented, aside from
being hearsay, were all immaterial and irrelevant to the issue
involved in the petitionthey did not prove or disprove that she
unlawfully neglected the performance of an act which the law
specifically enjoined as a duty resulting from an office, trust or
station, for the court to issue the writ of mandamus.[5]
The RTC, at first, denied the demurrer to evidence. [6] In
its February 4, 2005 Order,[7] however, it granted the same on
petitioners motion for reconsideration. Respondents motion
for reconsideration of this latter order was denied on
September 20, 2005.[8] Hence, the petition was dismissed.
Aggrieved, respondent sought review from the appellate
court. On April 26, 2006, the CA initially denied the appeal for
lack of merit. It ruled that the writ of mandamus would issue
only in instances when no other remedy would be available
and sufficient to afford redress. Under Rule 76, in an action for
the settlement of the estate of his deceased father, respondent
could ask for the presentation or production and for the
approval or probate of the holographic will. The CA further
ruled that respondent, in the proceedings before the trial court,
failed to present sufficient evidence to prove that his mother
had in her custody the original copy of the will.[9]
Respondent moved for reconsideration. The appellate
court, in the assailed August 23, 2006 Amended Decision,
[10]
granted the motion, set aside its earlier ruling, issued the
writ, and ordered the production of the will and the payment of
attorneys fees. It ruled this time that respondent was able to
show by testimonial evidence that his mother had in her
possession the holographic will.
Dissatisfied with this turn of events, petitioner filed a
motion for reconsideration. The appellate court denied this
motion in the further assailed February 23, 2007 Resolution.[11]
Left with no other recourse, petitioner brought the matter
before this Court, contending in the main that the petition for
mandamus is not the proper remedy and that the testimonial
evidence used by the appellate court as basis for its ruling is
inadmissible.[12]
The Court cannot sustain the CAs issuance of the writ.
The first paragraph of Section 3 of Rule 65 of the Rules
of Court pertinently provides that
SEC. 3. Petition for mandamus.
When any tribunal, corporation, board,
officer or person unlawfully neglects the
performance of an act which the law
specifically enjoins as a duty resulting from
an office, trust, or station, or unlawfully

90
excludes another from the use and
enjoyment of a right or office to which such
other is entitled, and there is no other plain,
speedy and adequate remedy in the ordinary
course of law, the person aggrieved thereby
may file a verified petition in the proper court,
alleging the facts with certainty and praying
that judgment be rendered commanding the
respondent, immediately or at some other
time to be specified by the court, to do the
act required to be done to protect the rights
of the petitioner, and to pay the damages
sustained by the petitioner by reason of the
wrongful acts of the respondent.[13]

the redress of private wrongs, but only in matters relating to the


public.[26]

Mandamus is a command issuing from a court of law


of competent jurisdiction, in the name of the state or the
sovereign, directed to some inferior court, tribunal, or board, or
to some corporation or person requiring the performance of a
particular duty therein specified, which duty results from the
official station of the party to whom the writ is directed or from
operation of law.[14]This definition recognizes the public
character of the remedy, and clearly excludes the idea that it
may be resorted to for the purpose of enforcing the
performance of duties in which the public has no interest.
[15]
The writ is a proper recourse for citizens who seek to
enforce a public right and to compel the performance of a
public duty, most especially when the public right involved is
mandated by the Constitution.[16] As the quoted provision
instructs, mandamus will lie if the tribunal, corporation, board,
officer, or person unlawfully neglects the performance of an act
which the law enjoins as a duty resulting from an office, trust or
station.[17]

In the instant case, the Court, without unnecessarily


ascertaining whether the obligation involved herethe
production of the original holographic willis in the nature of a
public or a private duty, rules that the remedy of mandamus
cannot be availed of by respondent Lee because there lies
another plain, speedy and adequate remedy in the ordinary
course of law. Let it be noted that respondent has a photocopy
of the will and that he seeks the production of the original for
purposes of probate. The Rules of Court, however, does not
prevent him from instituting probate proceedings for the
allowance of the will whether the same is in his possession or
not. Rule 76, Section 1 relevantly provides:

The writ of mandamus, however, will not issue to compel


an official to do anything which is not his duty to do or which it
is his duty not to do, or to give to the applicant anything to
which he is not entitled by law.[18] Nor will mandamus issue to
enforce a right which is in substantial dispute or as to which a
substantial doubt exists, although objection raising a mere
technical question will be disregarded if the right is clear and
the case is meritorious.[19] As a rule, mandamus will not lie in
the absence of any of the following grounds: [a] that the court,
officer, board, or person against whom the action is taken
unlawfully neglected the performance of an act which the law
specifically enjoins as a duty resulting from office, trust, or
station; or [b] that such court, officer, board, or person has
unlawfully excluded petitioner/relator from the use and
enjoyment of a right or office to which he is entitled.[20] On the
part of the relator, it is essential to the issuance of a writ of
mandamus that he should have a clear legal right to the thing
demanded and it must be the imperative duty of respondent to
perform the act required.[21]
Recognized further in this jurisdiction is the principle that
mandamus cannot be used to enforce contractual obligations.
[22]
Generally, mandamus will not lie to enforce purely private
contract
rights,
and
will
not
lie
against
an individual unless some obligation in the nature of
a
public or quasi-public duty is imposed. [23] The writ is not
appropriate to enforce a private right against an individual.
[24]
The writ of mandamus lies to enforce the execution of an
act, when, otherwise, justice would be obstructed; and,
regularly, issues only in cases relating to the public and to the
government; hence, it is called a prerogative writ.[25] To
preserve its prerogative character, mandamus is not used for

Moreover, an important principle followed in the issuance


of the writ is that there should be no plain, speedy and
adequate remedy in the ordinary course of law other than the
remedy of mandamus being invoked.[27] In other words,
mandamus can be issued only in cases where the usual
modes of procedure and forms of remedy are powerless to
afford relief.[28] Although classified as a legal remedy,
mandamus is equitable in its nature and its issuance is
generally controlled by equitable principles.[29] Indeed, the grant
of the writ of mandamus lies in the sound discretion of the
court.

Section 1. Who may petition for the


allowance of will.Any executor, devisee, or
legatee named in a will, or any other person
interested in the estate, may, at any time,
after the death of the testator, petition the
court having jurisdiction to have the will
allowed, whether the same be in his
possession or not, or is lost or destroyed.
An adequate remedy is further provided by Rule 75, Sections 2
to 5, for the production of the original holographic will. Thus
SEC. 2. Custodian of will to deliver.
The person who has custody of a will shall,
within twenty(20) days after he knows of
the death of the testator, deliver the will to
the court having jurisdiction, or to the
executor named in the will.
SEC. 3. Executor to present will and
accept or refuse trust.A person named as
executor in a will shall within twenty (20)
days after he knows of the death of the
testator, or within twenty (20) days after he
knows that he is named executor if he
obtained such knowledge after the death of
the testator, present such will to the court
having jurisdiction, unless the will has
reached the court in any other manner, and
shall, within such period, signify to the court
in writing his acceptance of the trust or his
refusal to accept it.
SEC. 4. Custodian and executor
subject to fine for neglect.A person who
neglects any of the duties required in the two
last preceding sections without excuse
satisfactory to the court shall be fined not
exceeding two thousand pesos.

91
SEC. 5. Person retaining will may be
committed.A person having custody of a
will after the death of the testator who
neglects without reasonable cause to deliver
the same, when ordered so to do, to the
court having jurisdiction, may be committed
to prison and there kept until he delivers the
will.[30]
There being a plain, speedy and adequate remedy in
the ordinary course of law for the production of the subject will,
the remedy of mandamus cannot be availed of. Suffice it to
state that respondent Lee lacks a cause of action in his
petition. Thus, the Court grants the demurrer.
WHEREFORE, premises considered, the petition for
review on certiorari is GRANTED. The August 23, 2006
Amended Decision and the February 23, 2007 Resolution of
the Court of Appeals in CA-G.R. SP No. 91725
are REVERSED and SET ASIDE. Civil Case No. 01100939
before the Regional Trial Court of Manila is DISMISSED.
SO ORDERED.

FIRST DIVISION

[G.R. No. 123486. August 12, 1999]

EUGENIA
RAMONAL
CODOY,
and
MANUEL
RAMONAL, petitioners, vs. EVANGELINE
R.
CALUGAY, JOSEPHINE SALCEDO, and EUFEMIA
PATIGAS, respondents.
DECISION
PARDO, J.:
Before us is a petition for review on certiorari of the
decision of the Court of Appeals[1] and its resolution denying
reconsideration, ruling:
Upon the unrebutted testimony of appellant Evangeline
Calugay and witness Matilde Ramonal Binanay, the
authenticity of testators holographic will has been established
and the handwriting and signature therein (exhibit S) are hers,
enough to probate said will. Reversal of the
judgment appealed from and the probate of the holographic will
in question be called for. The rule is that after plaintiff has
completed presentation of his evidence and the defendant files
a motion for judgment on demurrer to evidence on the ground
that upon the facts and the law plaintiff has shown no right to
relief, if the motion is granted and the order to dismissal is
reversed on appeal, the movant loses his right to present
evidence in his behalf (Sec. 1 Rule 35 Revised Rules of
Court). Judgment may, therefore, be rendered for appellant in
the instant case.

Wherefore, the order appealed from is REVERSED and


judgment rendered allowing the probate of the holographic will
of the testator Matilde Seo Vda. de Ramonal.[2]
The facts are as follows:
On April 6, 1990, Evangeline Calugay, Josephine Salcedo
and Eufemia Patigas, devisees and legatees of the holographic
will of the deceased Matilde Seo Vda. de Ramonal, filed with
the Regional Trial Court, Misamis Oriental, Branch 18, a
petition[3] for probate of the holographic will of the deceased,
who died on January 16, 1990.
In the petition, respondents claimed that the deceased
Matilde Seo Vda. de Ramonal, was of sound and disposing
mind when she executed the will on August 30, 1978, that
there was no fraud, undue influence, and duress employed in
the person of the testator, and the will was written voluntarily.
The assessed value of the decedents property, including
all real and personal property was about P400,000.00, at the
time of her death.[4]
On June 28, 1990, Eugenia Ramonal Codoy and Manuel
Ramonal filed an opposition[5] to the petition for probate,
alleging that the holographic will was a forgery and that the
same is even illegible. This gives an impression that a third
hand of an interested party other than the true hand of
Matilde Seo Vda. de Ramonal executed the holographic will.
Petitioners argued that the repeated dates incorporated
or appearing on the will after every disposition is out of the
ordinary. If the deceased was the one who executed the will,
and was not forced, the dates and the signature should appear
at the bottom after the dispositions, as regularly done and not
after every disposition. And assuming that the holographic will
is in the handwriting of the deceased, it was procured by undue
and improper pressure and influence on the part of the
beneficiaries, or through fraud and trickery.
Respondents presented six (6) witnesses and various
documentary evidence. Petitioners instead of presenting their
evidence, filed a demurrer[6] to evidence, claiming that
respondents failed to establish sufficient factual and legal basis
for the probate of the holographic will of the deceased Matilde
Seo Vda. de Ramonal.
On November 26, 1990, the lower Court issued an order,
the dispositive portion of which reads:
WHEREFORE, in view of the foregoing consideration, the
Demurrer to Evidence having being well taken, same is
granted, and the petition for probate of the document (Exhibit
S) on the purported Holographic Will of the late Matilde Seo
Vda. de Ramonal, is denied for insufficiency of evidence and
lack of merits.[7]
On December 12, 1990, respondents filed a notice of
appeal,[8] and in support of their appeal, the respondents once
again reiterated the testimony of the following witnesses,
namely: (1) Augusto Neri; (2) Generosa Senon; (3) Matilde
Ramonal Binanay; (4) Teresita Vedad; (5) Fiscal Rodolfo
Waga; and (6) Evangeline Calugay.
To have a clear understanding of the testimonies of the
witnesses, we recite an account of their testimonies.

92
Augusto Neri, Clerk of Court, Court of First Instance of
Misamis Oriental, where the special proceedings for the
probate of the holographic will of the deceased was filed. He
produced and identified the. records of the case. The
documents presented bear the signature of the deceased,
Matilde Seo Vda. de Ramonal, for the purpose of laying the
basis for comparison of the handwriting of the testatrix, with the
writing treated or admitted as genuine by the party against
whom the evidence is offered.
Generosa Senon, election registrar of Cagayan de Oro,
was presented to produce and identify the voters affidavit of
the decedent. However, the voters affidavit was not produced
for the same was already destroyed and no longer available.
Matilde Ramonal Binanay, testified that the deceased
Matilde Seo Vda. de Ramonal was her aunt, and that after the
death of Matildes husband, the latter lived with her in her
parents house for eleven (11) years, from 1958 to
1969. During those eleven (11) years of close association with
the deceased, she acquired familiarity with her signature and
handwriting as she used to accompany her (deceased Matilde
Seo Vda. de Ramonal) in collecting rentals from her various
tenants of commercial buildings, and the deceased always
issued receipts. In addition to this, she (witness Matilde
Binanay) assisted the deceased in posting the records of the
accounts, and carried personal letters of the deceased to her
creditors.
Matilde Ramonal Binanay further testified that at the
time of the death of Matilde Vda. de Ramonal, she left a
holographic will dated August 30, 1978, which was personally
and entirely written, dated and signed, by the deceased and
that all the dispositions therein, the dates, and the signatures in
said will, were that of the deceased.
Fiscal Rodolfo Waga testified that before he was
appointed City Fiscal of Cagayan de Oro, he was a practicing
lawyer, and handled all the pleadings and documents signed
by the deceased in connection with the intestate proceedings
of her late husband, as a result of which he is familiar with the
handwriting of the latter. He testified that the signature
appearing in the holographic will was similar to that of the
deceased, Matilde Seo Vda. de Ramonal, but he can not be
sure.
The fifth witness presented was Mrs. Teresita Vedad, an
employee of the Department of Environment and Natural
Resources, Region 10. She testified that she processed the
application of the deceased for pasture permit and was familiar
with the signature of the deceased, since the deceased signed
documents in her presence, when the latter was applying for
pasture permit.
Finally, Evangeline Calugay, one of the respondents,
testified that she had lived with the deceased since birth, and
was in fact adopted by the latter. That after a long period of
time she became familiar with the signature of the
deceased. She testified that the signature appearing in the
holographic will is the true and genuine signature of Matilde
Seo Vda. de Ramonal.
The holographic will which was written in Visayan, is
translated in English as follows:
Instruction
August 30, 1978

1. My share at Cogon, Raminal Street, for Evangeline


Calugay.
(Sgd) Matilde Vda de Ramonal
August 30, 1978
2. Josefina Salcedo must be given 1,500 square meters at
Pinikitan Street.
(Sgd) Matilde Vda de Ramonal
August 30, 1978
3. My jewelrys shall be divided among:
1. Eufemia Patigas
2. Josefina Salcedo
3. Evangeline Calugay
(Sgd)Matilde Vda de Ramonal
August 30, 1978
4. I bequeath my one (1) hectare land at Mandumol, Indahag
to Evangeline R. Calugay
(Sgd) Matilde Vda de Ramonal
"August 30, 1978
5. Give the 2,500 Square Meters at Sta. Cruz Ramonal
Village in favor of Evangeline R. Calugay, Helen must continue
with the Sta. Cruz, once I am no longer around.
(Sgd) Matilde Vda de Ramonal
August 30, 1978
6. Bury me where my husband Justo is ever buried.
(Sgd) Matilde Vda de Ramonal
"August 30,1978
Gene and Manuel:
"Follow my instruction in order that I will rest peacefully.
Mama
Matilde Vda de Ramonal
On October 9, 1995, the Court of Appeals, rendered
decision[9] ruling that the appeal was meritorious. Citing the

93
decision in the case of Azaola vs. Singson, 109 Phil. 102,
penned by Mr. Justice J. B. L. Reyes, a recognized authority in
civil law, the Court of Appeals held:
x x x even if the genuineness of the holographic will
were contested, we are of the opinion that Article 811 of our
present civil code can not be interpreted as to require the
compulsory presentation of three witnesses to identify the
handwriting of the testator, under penalty of having the probate
denied. Since no witness may have been present at the
execution of the holographic will, none being required by law
(art. 810, new civil code), it becomes obvious that the
existence of witnesses possessing the requisite qualifications
is a matter beyond the control of the proponent. For it is not
merely a question of finding and producing any three
witnesses; they must be witnesses who know the handwriting
and signature of the testator and who can declare (truthfully,
of course, even if the law does not express) that the will and
the signature are in the handwriting of the testator. There may
be no available witness acquainted with the testators hand; or
even if so familiarized, the witness may be unwilling to give a
positive opinion. Compliance with the rule of paragraph 1 of
article 811 may thus become an impossibility. That is evidently
the reason why the second paragraph of article 811 prescribes
that
in the absence of any competent witness referred to in the
preceding paragraph, and if the court deems it necessary,
expert testimony may be resorted to.
As can be seen, the law foresees the possibility that no
qualified witness may be found (or what amounts to the same
thing, that no competent witness may be willing to testify to the
authenticity of the will), and provides for resort to expert
evidence to supply the deficiency.
It may be true that the rule of this article (requiring that three
witnesses be presented if the will is contested and only one if
no contest is had) was derived from the rule established for
ordinary testaments (CF Cabang vs. Delfinado, 45 PHIL 291;
Tolentino v. Francisco, 57 PHIL 742). But it can not be ignored
that the requirement can be considered mandatory only in case
of ordinary testaments, precisely because the presence of at
least three witnesses at the execution of ordinary wills is made
by law essential to their validity (Art. 805). Where the will is
holographic, no witness need be present (art.10), and the
rule requiring production of three witnesses must be
deemed merely permissive if absurd results are to be
avoided.
Again, under Art.811, the resort to expert evidence is
conditioned by the words if the court deem it necessary,
which reveal that what the law deems essential is that the court
should be convinced of the wills authenticity. Where the
prescribed number of witnesses is produced and the court is
convinced by their testimony that the will is genuine, it may
consider it unnecessary to call for expert evidence. On the
other hand, if no competent witness is available, or none of
those produced is convincing, the court may still, and in fact it
should resort to handwriting experts. The duty of the court, in
fine, is to exhaust all available lines of inquiry, for the state is
as much interested as the proponent that the true intention of
the testator be carried into effect.

Paraphrasing Azaola vs. Singson, even if the genuineness of


the holographic will were contested, Article 811 of the civil code
cannot be interpreted as to require the compulsory
presentation of three witnesses to identify the handwriting of
the testator, under penalty of the having the probate
denied. No witness need be present in the execution of the
holographic will. And the rule requiring the production of
three witnesses is merely permissive. What the law deems
essential is that the court is convinced of the authenticity of the
will. Its duty is to exhaust all available lines of inquiry, for the
state is as much interested in the proponent that the true
intention of the testator be carried into effect. And because the
law leaves it to the trial court to decide if experts are still
needed, no unfavorable inference can be drawn from a partys
failure to offer expert evidence, until and unless the court
expresses dissatisfaction with the testimony of the lay
witnesses.[10]
According to the Court of Appeals, Evangeline Calugay,
Matilde Ramonal Binanay and other witnesses definitely and in
no uncertain terms testified that the handwriting and signature
in the holographic will were those of the testator herself.
Thus, upon the unrebutted testimony of appellant
Evangeline Calugay and witness Matilde Ramonal Binanay,
the Court of Appeals sustained the authenticity of the
holographic will and the handwriting and signature therein, and
allowed the will to probate.
Hence, this petition.
The petitioners raise the following issues:
(1) Whether or not the ruling of the case of Azaola
vs. Singson, 109 Phil. 102, relied upon by the
respondent Court of Appeals, was applicable to
the case.
(2) Whether or not the Court of Appeals erred in
holding that private respondents had been able
to present credible evidence to prove that the
date, text, and signature on the holographic will
were written entirely in the hand of the testatrix.
(3) Whether or not the Court of Appeals erred in not
analyzing the signatures in the holographic will
of Matilde Seo Vda. de Ramonal.
In this petition, the petitioners ask whether the provisions
of Article 811 of the Civil Code are permissive or
mandatory. The article provides, as a requirement for the
probate of a contested holographic will, that at least three
witnesses explicitly declare that the signature in the will is the
genuine signature of the testator.
We are convinced, based on the language used, that
Article 811 of the Civil Code is mandatory. The word shall
connotes a mandatory order. We have ruled that shall in a
statute commonly denotes an imperative obligation and is
inconsistent with the idea of discretion and that the
presumption is that the word shall, when used in a statute is
mandatory.[11]
Laws are enacted to achieve a goal intended and to
guide against an evil or mischief that aims to prevent. In the
case at bar, the goal to achieve is to give effect to the wishes
of the deceased and the evil to be prevented is the possibility

94
that unscrupulous individuals who for their benefit will employ
means to defeat the wishes of the testator.
So, we believe that the paramount consideration in
present petition is to determine the true intent of
deceased. An exhaustive and objective consideration of
evidence is imperative to establish the true intent of
testator.

the
the
the
the

It will be noted that not all the witnesses presented by the


respondents testified explicitly that they were familiar with the
handwriting of the testator. In the case of Augusto Neri, clerk
of court, Court of First Instance, Misamis Oriental, he merely
identified the record of Special Proceedings No. 427 before
said court. He was not presented to declare explicitly that the
signature appearing in the holographic was that of the
deceased.
Generosa E. Senon, the election registrar of Cagayan de
Oro City, was presented to identify the signature of the
deceased in the voters affidavit, which was not even produced
as it was no longer available.
Matilde Ramonal Binanay, on the other hand, testified
that:
Q. And you said for eleven (11) years Matilde Vda de
Ramonal resided with your parents at Pinikitan,
Cagayan de Oro City. Would you tell the court what
was your occupation or how did Matilde Vda de
Ramonal keep herself busy that time?
A. Collecting rentals.
Q. From where?

Q. Why do you say so?


A. Because we sometimes post a record of accounts in
behalf of Matilde Vda. De Ramonal.
Q. How is this record of accounts made? How is this
reflected?
A. In handwritten.[14]
xxx
Q. In addition to collection of rentals, posting records of
accounts of tenants and deed of sale which you said
what else did you do to acquire familiarity of the
signature of Matilde Vda De Ramonal?
A. Posting records.
Q. Aside from that?
A. Carrying letters.
Q. Letters of whom?
A. Matilde
Q. To whom?
A. To her creditors.[15]
xxx
Q. You testified that at the time of her death she left a
will. I am showing to you a document with its title
tugon is this the document you are referring to?
A. Yes, sir.

A. From the land rentals and commercial buildings at


Pabayo-Gomez streets.[12]
xxx
Q. Who sometime accompany her?

Q. Showing to you this exhibit S, there is that


handwritten tugon, whose handwriting is this?
A. My aunt.

A. I sometimes accompany her

Q. Why do you say this is the handwriting of your


aunt?

Q. In collecting rentals does she issue receipts?

A. Because I am familiar with her signature.[16]

A. Yes, sir.[13]
xxx
Q. Showing to you the receipt dated 23 October 1979, is
this the one you are referring to as one of the receipts
which she issued to them?
A. Yes, sir.
Q. Now there is that signature of Matilde vda. De
Ramonal, whose signature is that Mrs. Binanay?
A. Matilde vda. De Ramonal.
Q. Why do you say that that is a signature of Matilde
vda. De Ramonal?
A. I am familiar with her signature.
Q. Now, you tell the court Mrs. Binanay, whether you know
Matilde vda de Ramonal kept records of the accounts
of her tenants?
A. Yes, sir.

What Ms. Binanay saw were pre-prepared receipts and


letters of the deceased, which she either mailed or gave to her
tenants. She did not declare that she saw the deceased sign a
document or write a note.
Further, during the cross-examination, the counsel for
petitioners elicited the fact that the will was not found in the
personal belongings of the deceased but was in the
possession of Ms. Binanay. She testified that:
Q. Mrs. Binanay, when you were asked by counsel for the
petitioners if the late Matilde Seno vda de Ramonal
left a will you said, yes?
A. Yes, sir.
Q. Who was in possession of that will?
A. I.
Q. Since when did you have the possession of the will?
A. It was in my mothers possession.
Q. So, it was not in your possession?

95
A. Sorry, yes.
Q. And when did you come into possession since as you
said this was originally in the possession of your
mother?
A. 1985.

[17]

xxx
Q. Now, Mrs. Binanay was there any particular reason
why your mother left that will to you and therefore you
have that in your possession?

8,1978 which is only about eight months from August


30,1978. Do you notice that the signature Matilde
Vda de Ramonal is beautifully written and legible?
A. Yes, sir the handwriting shows that she was very
exhausted.
Q. You just say that she was very exhausted while that in
1978 she was healthy was not sickly and she was
agile. Now, you said she was exhausted?
A. In writing.

A. It was not given to me by my mother, I took that in the


aparador when she died.

Q. How did you know that she was exhausted when you
were not present and you just tried to explain yourself
out because of the apparent inconsistencies?

Q. After taking that document you kept it with you?

A. That was I think. (sic)

A. I presented it to the fiscal.

Q. Now, you already observed this signature dated 1978,


the same year as the alleged holographic will. In
exhibit I, you will notice that there is no retracing;
there is no hesitancy and the signature was written on
a fluid movement. x x x And in fact , the name
Eufemia R. Patigas here refers to one of the
petitioners?

Q. For what purpose?


A. Just to seek advice.
Q. Advice of what?
A. About the will.[18]
In her testimony it was also evident that Ms. Binanay kept
the fact about the will from petitioners, the legally adopted
children of the deceased. Such actions put in issue her motive
of keeping the will a secret to petitioners and revealing it only
after the death of Matilde Seo Vda. de Ramonal.
In the testimony of Ms. Binanay, the following were
established:
Q. Now, in 1978 Matilde Seno Vda de Ramonal was not
yet a sickly person is that correct?
A. Yes, sir.
Q. She was up and about and was still uprightly and she
could walk agilely and she could go to her building to
collect rentals, is that correct?
A. Yes, sir.[19]
xxx
Q. Now, let us go to the third signature of Matilde
Ramonal. Do you know that there are retracings in
the word Vda.?
A. Yes, a little. The letter L is continuous.
Q. And also in Matilde the letter L is continued to letter D?
A. Yes, sir.

A. Yes, sir.
Q. You will also notice Mrs. Binanay that it is not only with
the questioned signature appearing in the alleged
holographic will marked as Exhibit X but in the
handwriting themselves, here you will notice the
hesitancy and tremors, do you notice that?
A. Yes, sir.[21]
Evangeline Calugay declared that the holographic will
was written, dated and signed in the handwriting of the
testator. She testified that:
Q. You testified that you stayed with the house of the
spouses Matilde and Justo Ramonal for the period of
22 years. Could you tell the court the services if any
which you rendered to Matilde Ramonal?
A. During my stay I used to go with her to the church, to
the market and then to her transactions.
Q. What else? What services that you rendered?
A. After my college days I assisted her in going to the
bank, paying taxes and to her lawyer.
Q. What was your purpose of going to her lawyer?
A. I used to be her personal driver.

Q. Again the third signature of Matilde Vda de Ramonal


the letter L in Matilde is continued towards letter D.

Q. In the course of your stay for 22 years did you acquire


familiarity of the handwriting of Matilde Vda de
Ramonal?

A. Yes, sir.

A. Yes, sir.

Q. And there is a retracing in the word Vda.?

Q. How come that you acquired familiarity?

A. Yes, sir.[20]

A. Because I lived with her since birth.[22]


xxx

Q. Now, that was 1979, remember one year after the


alleged holographic will. Now, you identified a
document marked as Exhibit R. This is dated January

xxx
Q. Now, I am showing to you Exhibit S which is captioned
tugon dated Agosto 30, 1978 there is a signature

96
here below item No. 1, will you tell this court whose
signature is this?
A. Yes, sir, that is her signature.

A. I can not remember if I have assisted her in other


matters but if there are documents to show that I have
assisted then I can recall.[28]
xxx

Q. Why do you say that is her signature?


A. I am familiar with her signature.[23]
So, the only reason that Evangeline can give as to why
she was familiar with the handwriting of the deceased was
because she lived with her since birth. She never declared
that she saw the deceased write a note or sign a document.
The former lawyer of the deceased, Fiscal Waga, testified
that:
Q. Do you know Matilde Vda de Ramonal?
A. Yes, sir I know her because she is my godmother the
husband is my godfather. Actually I am related to the
husband by consanguinity.
Q. Can you tell the name of the husband?
A. The late husband is Justo Ramonal.[24]
xxx
Q. Can you tell this court whether the spouses Justo
Ramonal and Matilde Ramonal have legitimate
children?
A. As far as I know they have no legitimate children.[25]
xxx

Q. Now, I am showing to you exhibit S which is titled


tugon, kindly go over this document, Fiscal Waga
and tell the court whether you are familiar with the
handwriting contained in that document marked as
exhibit S?
A. I am not familiar with the handwriting.
Q. This one, Matilde Vda de Ramonal, whose signature is
this?
A. I think this signature here it seems to be the signature
of Mrs. Matilde vda de Ramonal.
Q. Now, in item No. 2 there is that signature here of
Matilde Vda de Ramonal, can you tell the court whose
signature is this?
A. Well, that is similar to that signature appearing in the
project of partition.
Q. Also in item no. 3 there is that signature Matilde Vda
de Ramonal, can you tell the court whose signature is
that?
A. As I said, this signature also seems to be the signature
of Matilde vda de Ramonal.
Q. Why do you say that?

Q. You said after becoming a lawyer you practice your


profession? Where?

A. Because there is a similarity in the way it is being


written.

A. Here in Cagayan de Oro City.

Q. How about this signature in item no. 4, can you tell the
court whose signature is this?

Q. Do you have services rendered with the deceased


Matilde vda de Ramonal?
A. I assisted her in terminating the partition, of properties.
Q. When you said assisted, you acted as her counsel?
Any sort of counsel as in what case is that, Fiscal?
A. It is about the project partition to terminate the property,
which was under the court before.[26]
xxx

A. The same is true with the signature in item no. 4. It


seems that they are similar.[29]
xxx
Q. Mr. Prosecutor, I heard you when you said that the
signature of Matilde Vda de Ramonal Appearing in
exhibit S seems to be the signature of Matilde vda de
Ramonal?
A. Yes, it is similar to the project of partition.

Q. Appearing in special proceeding no. 427 is the


amended inventory which is marked as exhibit N of
the estate of Justo Ramonal and there appears a
signature over the type written word Matilde vda de
Ramonal, whose signature is this?

Q. So you are not definite that this is the signature of


Matilde vda de Ramonal. You are merely
supposing that it seems to be her signature
because it is similar to the signature of the project
of partition which you have made?

A. That is the signature of Matilde Vda de Ramonal.

A. That is true.[30]

Q. Also in exhibit n-3, whose signature is this?


A. This one here that is the signature of Mrs. Matilde vda
de Ramonal.[27]
xxx
Q. Aside from attending as counsel in that Special
Proceeding Case No. 427 what were the other
assistance wherein you were rendering professional
service to the deceased Matilde Vda de Ramonal?

From the testimonies of these witnesses, the Court of


Appeals allowed the will to probate and disregard the
requirement of three witnesses in case of contested
holographic will, citing the decision in Azaola vs. Singson,
[31]
ruling that the requirement is merely directory and not
mandatory.
In the case of Ajero vs. Court of Appeals, [32] we said that
the object of the solemnities surrounding the execution of wills
is to close the door against bad faith and fraud, to avoid

97
substitution of wills and testaments and to guaranty their truth
and authenticity. Therefore, the laws on this subject should be
interpreted in such a way as to attain these primordial
ends. But, on the other hand, also one must not lose sight of
the fact that it is not the object of the law to restrain and curtail
the exercise of the right to make a will.
However, we cannot eliminate the possibility of a false
document being adjudged as the will of the testator, which is
why if the holographic will is contested, that law requires three
witnesses to declare that the will was in the handwriting of the
deceased.
The will was found not in the personal belongings of the
deceased but with one of the respondents, who kept it even
before the death of the deceased. In the testimony of Ms.
Binanay, she revealed that the will was in her possession as
early as 1985, or five years before the death of the deceased.
There was no opportunity for an expert to compare the
signature and the handwriting of the deceased with other
documents signed and executed by her during her
lifetime. The only chance at comparison was during the crossexamination of Ms. Binanay when the lawyer of petitioners
asked Ms. Binanay to compare the documents which
contained the signature of the deceased with that of the
holographic will and she is not a handwriting expert. Even the
former lawyer of the deceased expressed doubts as to the
authenticity of the signature in the holographic will.
A visual examination of the holographic will convince us
that the strokes are different when compared with other
documents written by the testator. The signature of the
testator in some of the disposition is not readable. There were
uneven strokes, retracing and erasures on the will.
Comparing the signature in the holographic will dated
August 30, 1978,[33] and the signatures in several documents
such as the application letter for pasture permit dated
December 30, 1980,[34] and a letter dated June 16, 1978,[35] the
strokes are different. In the letters, there are continuous flows
of the strokes, evidencing that there is no hesitation in writing
unlike that of the holographic will. We, therefore, cannot be
certain that the holographic will was in the handwriting by the
deceased.
IN VIEW WHEREOF, the decision appealed from is SET
ASIDE. The records are ordered remanded to the court of
origin with instructions to allow petitioners to adduce evidence
in support of their opposition to the probate of the holographic
will of the deceased Matilde Seo Vda. de Ramonal.
No costs.
SO ORDERED.
Davide
Jr.,
C.J.,
(Chairman),
Kapunan, and Ynares-Santiago, JJ., concur.

Republic of the Philippines


SUPREME COURT
Manila
EN BANC

Puno,

G.R. No. L-20234

December 23, 1964

PAULA DE LA CERNA, ET AL., petitioners,


vs.
MANUELA REBACA POTOT, ET AL., and THE
HONORABLE COURT OF APPEALS, respondents.
Philip M. Alo and Crispin M. Menchavez for petitioners.
Nicolas Jumapao for respondents.
REYES, J.B.L., J.:
Appeal by Paula de la Cerna and others from a decision of the
Court of Appeals, Sixth Division (C.A.-G.R. No. 23763-R)
reversing that of the Court of First Instance of Cebu (Civ. Case
No. R-3819) and ordering the dismissal of an action for
partition.
The factual background appears in the following portion of the
decision of the Court of Appeals (Petition, Annex A, pp. 2-4):
It appears that on May 9, 1939, the spouses, Bernabe
de la Serna and Gervasia Rebaca, executed a joint
last will and testament in the local dialect whereby
they willed that "our two parcels of land acquired
during our marriage together with all improvements
thereon shall be given to Manuela Rebaca, our niece,
whom we have nurtured since childhood, because
God did not give us any child in our union, Manuela
Rebaca being married to Nicolas Potot", and that
"while each of the testators is yet living, he or she will
continue to enjoy the fruits of the two lands
aforementioned", the said two parcels of land being
covered by Tax No. 4676 and Tax No. 6677, both
situated in sitio Bucao, barrio Lugo, municipality of
Borbon, province of Cebu. Bernabe dela Serna died
on August 30, 1939, and the aforesaid will was
submitted to probate by said Gervasia and Manuela
before the Court of First Instance of Cebu which, after
due publication as required by law and there being no
opposition, heard the evidence, and, by Order of
October 31, 1939; in Special Proceedings No. 499,
"declara legalizado el documento Exhibit A como el
testamento y ultima voluntad del finado Bernabe de la
Serna con derecho por parte du su viuda superstite
Gervasia Rebaca y otra testadora al propio tiempo
segun el Exhibit A de gozar de los frutos de los
terranos descritos en dicho documents; y habido
consideracion de la cuantia de dichos bienes, se
decreta la distribucion sumaria de los mismos en
favor de la logataria universal Manuela Rebaca de
Potot previa prestacion por parte de la misma de una
fianza en la sum de P500.00 para responder de
cualesquiera reclamaciones que se presentare contra
los bienes del finado Bernabe de la Serna de los aos
desde esta fecha" (Act Esp. 499, Testamentaria
Finado Bernabe de la Serna) Upon the death of
Gervasia Rebaca on October 14, 1952, another
petition for the probate of the same will insofar as
Gervasia was concerned was filed on November 6,
1952, being Special Proceedings No. 1016-R of the
same Court of First Instance of Cebu, but for failure of
the petitioner, Manuela R. Potot and her attorney,
Manuel Potot to appear, for the hearing of said

98
petition, the case was dismissed on March 30, 1954
Spec. Proc. No. 1016-R, In the matter of the Probate
of the Will of Gervasia Rebaca).
The Court of First Instance ordered the petition heard and
declared the testament null and void, for being executed
contrary to the prohibition of joint wills in the Civil Code (Art.
669, Civil Code of 1889 and Art. 818, Civil Code of the
Philippines); but on appeal by the testamentary heir, the Court
of Appeals reversed, on the ground that the decree of probate
in 1939 was issued by a court of probate jurisdiction and
conclusive on the due execution of the testament. Further, the
Court of Appeals declared that:
... . It is true the law (Art. 669, old Civil Code; Art. 818,
new Civil Code). prohibits the making of a will jointly
by two or more persons either for their reciprocal
benefit or for the benefit of a third person. However,
this form of will has long been sanctioned by use, and
the same has continued to be used; and when, as in
the present case, one such joint last will and
testament has been admitted to probate by final order
of a Court of competent jurisdiction, there seems to
be no alternative except to give effect to the
provisions thereof that are not contrary to law, as was
done in the case of Macrohon vs. Saavedra, 51 Phil.
267, wherein our Supreme Court gave effect to the
provisions of the joint will therein mentioned, saying,
"assuming that the joint will in question is valid."
Whence this appeal by the heirs intestate of the deceased
husband, Bernabe de la Cerna.
The appealed decision correctly held that the final decree of
probate, entered in 1939 by the Court of First Instance of Cebu
(when the testator, Bernabe de la Cerna, died), has conclusive
effect as to his last will and testament despite the fact that
even then the Civil Code already decreed the invalidity of joint
wills, whether in favor of the joint testators, reciprocally, or in
favor of a third party (Art. 669, old Civil Code). The error thus
committed by the probate court was an error of law, that should
have been corrected by appeal, but which did not affect the
jurisdiction of the probate court, nor the conclusive effect of its
final decision, however erroneous. A final judgment rendered
on a petition for the probate of a will is binding upon the whole
world (Manalo vs. Paredes, 47 Phil. 938; In re Estates of
Johnson, 39 Phil. 156); and public policy and sound practice
demand that at the risk of occasional errors judgment of courts
should become final at some definite date fixed by law. Interest
rei publicae ut finis set litium (Dy Cay vs. Crossfield, 38 Phil,
521, and other cases cited in 2 Moran, Comments on the
Rules of Court (1963 Ed., p. 322).
Petitioners, as heirs and successors of the late Bernabe de la
Cerna, are concluded by the 1939 decree admitting his will to
probate. The contention that being void the will cannot be
validated, overlooks that the ultimate decision on Whether an
act is valid or void rests with the courts, and here they have
spoken with finality when the will was probated in 1939. On this
court, the dismissal of their action for partition was correct.
But the Court of Appeals should have taken into account also,
to avoid future misunderstanding, that the probate decree in
1989 could only affect the share of the deceased husband,

Bernabe de la Cerna. It could not include the disposition of the


share of the wife, Gervasia Rebaca, who was then still alive,
and over whose interest in the conjugal properties the probate
court acquired no jurisdiction, precisely because her estate
could not then be in issue. Be it remembered that prior to the
new Civil Code, a will could not be probated during the
testator's lifetime.
It follows that the validity of the joint will, in so far as the estate
of the wife was concerned, must be, on her death, reexamined
and adjudicated de novo, since a joint will is considered a
separate will of each testator. Thus regarded, the holding of the
court of First Instance of Cebu that the joint will is one
prohibited by law was correct as to the participation of the
deceased Gervasia Rebaca in the properties in question, for
the reasons extensively discussed in our decision in Bilbao vs.
Bilbao, 87 Phil. 144, that explained the previous holding
in Macrohon vs. Saavedra, 51 Phil. 267.
Therefore, the undivided interest of Gervasia Rebaca should
pass upon her death to her heirs intestate, and not exclusively
to the testamentary heir, unless some other valid will in her
favor is shown to exist, or unless she be the only heir intestate
of said Gervasia.
It is unnecessary to emphasize that the fact that joint wills
should be in common usage could not make them valid when
our Civil Codes consistently invalidated them, because laws
are only repealed by other subsequent laws, and no usage to
the contrary may prevail against their observance (Art. 5, Civ.
Code of 1889; Art. 7, Civil Code of the Philippines of 1950).
WITH THE FOREGOING MODIFICATION, the judgment of the
Court of Appeals in CA-G.R. No. 23763-R is affirmed. No
Costs.
Bengzon, C.J., Bautista, Angelo, Concepcion, Barrera,
Paredes, Dizon Regala, Makalintal, Bengzon, J.P., and
Zaldivar, JJ., concur.

Republic of the Philippines


SUPREME COURT
Manila
FIRST DIVISION
G.R. No. L-37453 May 25, 1979
RIZALINA GABRIEL GONZALES, petitioner,
vs.
HONORABLE COURT OF APPEALS and LUTGARDA
SANTIAGO, respondents.
Francisco D. Rilloraza, Jr. for petitioners.
Angel A. Sison for private respondent.

99
GUERRERO, J.:
This is a petition for review of the decision of the Court of
Appeals, First Division, 1 promulgated on May 4, 1973 in CA
G.R. No. 36523-R which reversed the decision of the Court of
First Instance of Rizal dated December 15, 1964 and allowed
the probate of the last will and testament of the deceased
Isabel Gabriel. *
It appears that on June 24, 1961, herein private respondent
Lutgarda Santiago filed a petition with the Court of First
Instance of Rizal docketed as Special Proceedings No. 3617,
for the probate of a will alleged to have been executed by the
deceased Isabel Gabriel and designating therein petitioner as
the principal beneficiary and executrix.
There is no dispute in the records that the late Isabel Andres
Gabriel died as a widow and without issue in the municipality of
Navotas, province of Rizal her place of residence, on June 7,
1961 at the age of eighty-five (85), having been born in 1876. It
is likewise not controverted that herein private respondent
Lutgarda Santiago and petitioner Rizalina Gabriel Gonzales
are nieces of the deceased, and that private respondent, with
her husband and children, lived with the deceased at the
latters residence prior an- d up to the time of her death.
The will submitted for probate, Exhibit "F", which is typewritten
and in Tagalog, appears to have been executed in Manila on
the 15th day of April, 1961, or barely two (2) months prior to
the death of Isabel Gabriel. It consists of five (5) pages,
including the pages whereon the attestation clause and the
acknowledgment of the notary public were written. The
signatures of the deceased Isabel Gabriel appear at the end of
the will on page four and at the left margin of all the pages. The
attestation clause, which is found on page four, reads as
follows:
PATUNAY NG MGA SAKSI
Kaming mga nakalagdang mga saksi o testigo na ang
aming mga tinitirahan ay nakasulat sa gawing kanan
at kahilira ng aming mga pangalan sa ibaba nito, ay
pagpapatutuo na ipinakilala ipinaalam at ipinahayag
sa amin ni Isabel Gabriel na ang kasulatang ito na
binubuo ng Limang Dahon (Five Pages) pati na ang
dahong ito, na siya niyang TESTAMENTO AT
HULING HABILIN, ngayong ika 15 ng Abril, 1961, ay
nilagdaan ng nasabing testadora na si Isabel Gabriel
ang nasabing testamento sa ibaba o ilalim ng
kasulatan na nasa ika apat na dahon (page four) at
nasa itaas ng patunay naming ito, at sa kaliwang
panig ng lahat at bawat dahon (and on the left hand
margin of each and every page), sa harap ng lahat at
bawat isa sa amin, at kami namang mga saksi ay
lumagda sa harap ng nasabing testadora, at sa harap
ng lahat at bawat isa sa amin, sa ilalim ng patunay ng
mga saksi at sa kaliwang panig ng lahat at bawa't
dahon ng testamentong ito.
At the bottom thereof, under the heading "Pangalan", are
written the signatures of Matilde D. Orobia, Celso D. Gimpaya
and Maria R. Gimpaya, and opposite the same, under the
heading "Tirahan", are their respective places of residence,
961 Highway 54, Philamlife, for Miss Orobia, and 12 Dagala

St., Navotas, Rizal, for the two Gimpayas. Their signatures


also appear on the left margin of all the other pages. The WW
is paged by typewritten words as follows: "Unang Dahon" and
underneath "(Page One)", "Ikalawang Dahon" and underneath
"(Page Two)", etc., appearing at the top of each page.
The will itself provides that the testatrix desired to be buried in
the Catholic Cemetery of Navotas, Rizal in accordance with the
rites of the Roman Catholic Church, all expenses to be paid
from her estate; that all her obligations, if any, be paid; that
legacies in specified amounts be given to her sister, Praxides
Gabriel Vda. de Santiago, her brother Santiago Gabriel, and
her nephews and nieces, Benjamin, Salud, Rizalina (herein
petitioner), Victoria, Ester, Andres, all surnamed Gabriel, and
Evangeline, Rudyardo Rosa, Andrea, Marcial, Numancia,
Verena an surnamed Santiago. To herein private respondent
Lutgarda Santiago, who was described in the will by the
testatrix as "aking mahal na pamangkin na aking pinalaki,
inalagaan at minahal na katulad ng isang tunay na anak" and
named as universal heir and executor, were bequeathed all
properties and estate, real or personal already acquired, or to
be acquired, in her testatrix name, after satisfying the
expenses, debts and legacies as aforementioned.
The petition was opposed by Rizalina Gabriel Gonzales, herein
petitioner, assailing the document purporting to be the will of
the deceased on the following grounds:
1. that the same is not genuine; and in the
alternative
2. that the same was not executed and
attested as required by law;
3. that, at the time of the alleged execution of
the purported wilt the decedent lacked
testamentary capacity due to old age and
sickness; and in the second alternative
4. That the purported WW was procured
through undue and improper pressure and
influence on the part of the principal
beneficiary, and/or of some other person for
her benefit.
Lutgarda Santiago filed her Answer to the Opposition on
February 1, 1962. After trial, the court a quo rendered
judgment, the summary and dispositive portions of which read:
Passing in summary upon the grounds
advanced by the oppositor, this Court finds:
1. That there is no iota of evidence to
support the contentio that the purported will
of the deceased was procured through
undue and improper pressure and influence
on the part of the petitioner, or of some other
person for her benefit;
2. That there is insufficient evidence to
sustain the contention that at the time of the
alleged execution of the purported will, the

100
deceased lacked testamentary capacity due
to old age and sickness;
3. That sufficient and abundant evidence
warrants conclusively the fact that the
purported will of the deceased was not
executed and attested as required by law;
4. That the evidence is likewise conclusive
that the document presented for probate,
Exhibit 'F' is not the purported win allegedly
dictated by the deceased, executed and
signed by her, and attested by her three
attesting witnesses on April 15, 1961.
WHEREFORE, Exhibit "F", the document
presented for probate as the last wig and
testament of the deceased Isabel Gabriel is
here by DISALLOWED.
From this judgment of disallowance, Lutgarda Santiago
appealed to respondent Court, hence, the only issue decided
on appeal was whether or not the will in question was executed
and attested as required by law. The Court of Appeals, upon
consideration of the evidence adduced by both parties,
rendered the decision now under review, holding that the will in
question was signed and executed by the deceased Isabel
Gabriel on April 15, 1961 in the presence of the three attesting
witnesses, Matilde Orobia, Celso Gimpaya and Maria
Gimpaya, signing and witnessing the document in the
presence of the deceased and of each other as required by
law, hence allow ed probate.
Oppositor Rizalina Gabriel Gonzales moved for
reconsideration 3 of the aforesaid decision and such motion
was opposed 4 by petitioner-appellant Lutgarda Santiago.
Thereafter. parties submitted their respective
Memoranda, 5 and on August 28, 1973, respondent Court,
Former Special First Division, by Resolution 6 denied the
motion for reconsideration stating that:
The oppositor-appellee contends that the
preponderance of evidence shows that the
supposed last wig and testament of Isabel
Gabriel was not executed in accordance with
law because the same was signed on
several occasions, that the testatrix did not
sign the will in the presence of all the
instrumental witnesses did not sign the will in
the presence of each other.
The resolution of the factual issue raised in
the motion for reconsideration hinges on the
appreciation of the evidence. We have
carefully re-examined the oral and
documentary evidence of record, There is no
reason to alter the findings of fact in the
decision of this Court sought to be set
aside. 7
In her petition before this Court, oppositor Rizalina Gabriel
Gonzales contends that respondent Court abused its discretion
and/or acted without or in excess of its jurisdiction in reverssing

the findings of fact and conclusions of the trial court. The


Court, after deliberating on the petition but without giving due
course resolved, in the Resolution dated Oct. 11, 1973 to
require the respondents to comment thereon, which comment
was filed on Nov. 14, 1973. Upon consideration of the
allegations, the issues raised and the arguments adduced in
the petition, as well as the Comment 8 of private respondent
thereon, We denied the petition by Resolution on November
26, 1973, 9 the question raised being factual and for insufficient
showing that the findings of fact by respondent Court were
unsupported by substantial evidence.
Subsequently, or on December 17, 1973, petitioner Rim
Gabriel Goes fried a Motion for Reconsideration 10 which
private respondent answered by way of her Comment or
Opposition 11 filed on January 15, 1974. A Reply and Rejoinder
to Reply followed. Finally, on March 27, 1974, We resolved to
give due course to the petition.
The petitioner in her brief makes the following assignment of
errors:
I. The respondent Court of Appeals erred in holding that the
document, Exhibit "F" was executed and attested as required
by law when there was absolutely no proof that the three
instrumental witnesses were credible witness
II. The Court of Appeals erred in reversing the finding of the
lower court that the preparation and execution of the win
Exhibit "F", was unexpected and coincidental.
III. The Court of Appeals erred in finding that Atty, Paraiso was
not previously furnished with the names and residence
certificates of the witnesses as to enable him to type such data
into the document Exhibit "F".
IV. The Court of Appeals erred in holding that the fact that the
three typewritten lines under the typewritten words "Pangalan"
and "Tinitirahan" were left blank shows beyond cavil that the
three attesting witnesses were all present in the same
occasion.
V. The Court of Appeals erred in reversing the trial court's
finding that it was incredible that Isabel Gabriel could have
dictated the wilt Exhibit "F , without any note or document, to
Atty. Paraiso.
VI. The Court of Appeals erred in reversing the finding of the
trial court that Matilde Orobia was not physically present when
the Will Exhibit "F" was allegedly signed on April 15, 1961 by
the deceased Isabel Gabriel and the other witnesses Celso
Gimpaya and Maria Gimpaya.
VII. The Court of Appeals erred in holding that the trial court
gave undue importance to the picture takings as proof that the
win was improperly executed.
VIII. The Court of Appeals erred in holding that the grave
contradictions, evasions, and misrepresentations of witnesses
(subscribing and notary) presented by the petitioner had been
explained away, and that the trial court erred in rejecting said
testimonies.

101
IX. The Court of Appeals acted in excess of its appellate
jurisdiction or has so far departed from the accepted and usual
course of judicial proceedings, as to call for an exercise of the
power of supervision.
X. The Court of Appeals erred in reversing the decision of the
trial court and admitting to probate Exhibit "F", the alleged last
will and testament of the deceased Isabel Gabriel.
It will be noted from the above assignments of errors that the
same are substantially factual in character and content. Hence,
at the very outset, We must again state the oft-repeated and
well-established rule that in this jurisdiction, the factual findings
of the Court of Appeals are not reviewable, the same being
binding and conclusive on this Court. This rule has been stated
and reiterated in a long line of cases enumerated in Chan vs.
CA (L-27488, June 30, 1970, 33 SCRA 737, 743) 12 and Tapas
vs. CA (L-22202, February 27; 1976, 69 SCRA 393), 13 and in
the more recent cases of Baptisia vs. Carillo and CA (L32192,
July 30, 1976, 72 SCRA 214, 217) and Vda. de Catindig vs.
Heirs of Catalina Roque (L-25777, November 26, 1976, 74
SCRA 83, 88). In the case of Chan vs. CA, this Court said:
... from Guico v. Mayuga, a 1936 decision, the opinion being
penned by the then Justice Recto, it has been well-settled that
the jurisdiction of tills Court in cases brought to us from the
Court of Appeals is limited to reviewing and revising the errors
of law imputed to it, its findings of fact being conclusive. More
specifically, in a decision exactly a month later, this Court,
speaking through the then Justice Laurel, it was held that the
same principle is applicable, even if the Court of Appeals was
in disagreement with the lower court as to the weight of the
evidence with a consequent reversal of its findings of fact ...
Stated otherwise, findings of facts by the Court of Appeals,
when supported by substantive evidence are not reviewable on
appeal by certiorari. Said findings of the appellate court are
final and cannot be disturbed by Us particularly because its
premises are borne out by the record or based upon
substantial evidence and what is more, when such findings are
correct. Assignments of errors involving factual issues cannot
be ventilated in a review of the decision of the Court of Appeals
because only legal questions may be raised. The Supreme
Court is not at liberty to alter or modify the facts as set forth in
the decision of the Court of Appeals sought to be reversed.
Where the findings of the Court of Appeals are contrary to
those of the trial court, a minute scrutiny by the Supreme Court
is in order, and resort to duly-proven evidence becomes
necessary. The general rule We have thus stated above is not
without some recognized exceptions.
Having laid down the above legal precepts as Our foundation,
We now proceed to consider petitioner's assignments of errors.
Petitioner, in her first assignment, contends that the
respondent Court of Appeals erred in holding that the
document, Exhibit "F", was executed and attested as required
by law when there was absolutely no proof that the three
instrumental witnesses were credible witnesses. She argues
that the require. ment in Article 806, Civil Code, that the
witnesses must be credible is an absolute requirement which
must be complied with before an alleged last will and
testament may be admitted to probate and that to be a credible
witness, there must be evidence on record that the witness has

a good standing in his community, or that he is honest and


upright, or reputed to be trustworthy and reliable. According to
petitioner, unless the qualifications of the witness are first
established, his testimony may not be favorably considered.
Petitioner contends that the term "credible" is not synonymous
with "competent" for a witness may be competent under Article
820 and 821 of the Civil Code and still not be credible as
required by Article 805 of the same Code. It is further urged
that the term "credible" as used in the Civil Code should
receive the same settled and well- known meaning it has under
the Naturalization Law, the latter being a kindred legislation
with the Civil Code provisions on wigs with respect to the
qualifications of witnesses.
We find no merit to petitioner's first assignment of error. Article
820 of the Civil Code provides the qualifications of a witness to
the execution of wills while Article 821 sets forth the
disqualification from being a witness to a win. These Articles
state:
Art. 820. Any person of sound mind and of
the age of eighteen years or more, and not
blind, deaf or dumb, and able to read and
write, may be a witness to the execution of a
will mentioned in article 806 of this Code.
"Art. 821. The following are disqualified from
being witnesses to a will:
(1) Any person not domiciled in the
Philippines,
(2) Those who have been convicted of
falsification of a document, perjury or false
testimony.
Under the law, there is no mandatory requirement that the
witness testify initially or at any time during the trial as to his
good standing in the community, his reputation for
trustworthythiness and reliableness, his honesty and
uprightness in order that his testimony may be believed and
accepted by the trial court. It is enough that the qualifications
enumerated in Article 820 of the Civil Code are complied with,
such that the soundness of his mind can be shown by or
deduced from his answers to the questions propounded to him,
that his age (18 years or more) is shown from his appearance,
testimony , or competently proved otherwise, as well as the
fact that he is not blind, deaf or dumb and that he is able to
read and write to the satisfaction of the Court, and that he has
none of the disqualifications under Article 821 of the Civil
Code. We reject petitioner's contention that it must first be
established in the record the good standing of the witness in
the community, his reputation for trustworthiness and
reliableness, his honesty and uprightness, because such
attributes are presumed of the witness unless the contrary is
proved otherwise by the opposing party.
We also reject as without merit petitioner's contention that the
term "credible" as used in the Civil Code should be given the
same meaning it has under the Naturalization Law where the
law is mandatory that the petition for naturalization must be
supported by two character witnesses who must prove their
good standing in the community, reputation for trustworthiness
and reliableness, their honesty and uprightness. The two
witnesses in a petition for naturalization are character

102
witnesses in that being citizens of the Philippines, they
personally know the petitioner to be a resident of the
Philippines for the period of time required by the Act and a
person of good repute and morally irreproachable and that said
petitioner has in their opinion all the qualifications necessary to
become a citizen of the Philippines and is not in any way
disqualified under the provisions of the Naturalization Law
(Section 7, Commonwealth Act No. 473 as amended).
In probate proceedings, the instrumental witnesses are not
character witnesses for they merely attest the execution of a
will or testament and affirm the formalities attendant to said
execution. And We agree with the respondent that the rulings
laid down in the cases cited by petitioner concerning character
witnesses in naturalization proceedings are not applicable to
instrumental witnesses to wills executed under the Civil Code
of the Philippines.
In the case at bar, the finding that each and everyone of the
three instrumental witnesses, namely, Matilde Orobia, Celso
Gimpaya and Maria Gimpaya, are competent and credible is
satisfactorily supported by the evidence as found by the
respondent Court of Appeals, which findings of fact this
Tribunal is bound to accept and rely upon. Moreover, petitioner
has not pointed to any disqualification of any of the said
witnesses, much less has it been shown that anyone of them is
below 18 years of age, of unsound mind, deaf or dumb, or
cannot read or write.
It is true that under Article 805 of the New Civil Code, every
will, other than a holographic will, must be subscribed at the
end thereof by the testator himself or by the testator's name
written by some other person in his presence, and by his
express direction, and attested and subscribed by three or
more credible witnesses in the presence of the testator and of
one another, While the petitioner submits that Article 820 and
821 of the New Civil Code speak of the competency of a
witness due to his qualifications under the first Article and none
of the disqualifications under the second Article, whereas
Article 805 requires the attestation of three or more credible
witnesses, petitioner concludes that the term credible requires
something more than just being competent and, therefore, a
witness in addition to being competent under Articles 820 and
821 must also be a credible witness under Article 805.
Petitioner cites American authorities that competency and
credibility of a witness are not synonymous terms and one may
be a competent witness and yet not a credible one. She
exacerbates that there is no evidence on record to show that
the instrumental witnesses are credible in themselves, that is,
that they are of good standing in the community since one was
a family driver by profession and the second the wife of the
driver, a housekeeper. It is true that Celso Gimpaya was the
driver of the testatrix and his wife Maria Gimpaya, merely a
housekeeper, and that Matilde Orobia was a piano teacher to a
grandchild of the testatrix But the relation of employer and
employee much less the humble or financial position of a
person do not disqualify him to be a competent testamentary
witness. (Molo Pekson and Perez Nable vs. Tanchuco, et al.,
100 Phil. 344; Testate Estate of Raymundo, Off. Gaz., March
18,1941, p. 788).
Private respondent maintains that the qualifications of the three
or more credible witnesses mentioned in Article 805 of the Civil

Code are those mentioned in Article 820 of the same Code,


this being obvious from that portion of Article 820 which says
"may be Q witness to the execution of a will mentioned in
Article 805 of this Code," and cites authorities that the word
"credible" insofar as witnesses to a will are concerned simply
means " competent." Thus, in the case of Suntay vs. Suntay,
95 Phil. 500, the Supreme Court held that "Granting that a will
was duly executed and that it was in existence at the time of,
and not revoked before, the death of the testator, still the
provisions of the lost wig must be clearly and distinctly proved
by at least two credible witnesses. 'Credible witnesses' mean
competent witnesses and not those who testify to facts from or
upon hearsay. " emphasis supplied).
In Molo Pekson and Perez Nable vs. Tanchuco, et al., 100 Phil.
344, the Supreme Court held that "Section 620 of the same
Code of Civil Procedure provides that any person of sound
mind, and of the age of eighteen years or more, and not blind,
deaf, or dumb and able to read and write, may be a witness to
the execution of a will. This same provision is reproduced in
our New Civil Code of 1950, under Art. 820. The relation of
employer and employee, or being a relative to the beneficiary
in a win, does not disqualify one to be a witness to a will. The
main qualification of a witness in the attestation of wills, if other
qualifications as to age, mental capacity and literacy are
present, is that said witness must be credible, that is to say, his
testimony may be entitled to credence. There is a long line of
authorities on this point, a few of which we may cite:
A 'credible witness is one who is not is not to
testify by mental incapacity, crime, or other
cause. Historical Soc of Dauphin County vs.
Kelker 74 A. 619, 226 Pix 16, 134 Am. St.
Rep. 1010. (Words and Phrases, Vol. 10, p.
340).
As construed by the common law, a 'credible
witness' to a will means a 'competent
witness.' Appeal of Clark, 95 A. 517, 114 Me.
105, Ann. Cas. 1917A, 837. (lbid, p. 341).
Expression 'credible witness' in relation to
attestation of wins means 'competent
witness that is, one competent under the law
to testify to fact of execution of will. Vernon's
Ann. Civ St. art. 8283. Moos vs. First State
Bank of Uvalde, Tex . Civ. App. 60 S.W. 2nd
888, 889. (Ibid, p. 342)
The term 'credible', used in the statute of
wills requiring that a will shall be attested by
two credible witnesses means competent;
witnesses who, at the time of attesting the
will, are legally competent to testify, in a
court of justice, to the facts attested by
subscribing the will, the competency being
determined as of the date of the execution of
the will and not of the timr it is offered for
probate, Smith vs. Goodell 101 N.E. 255,
256, 258 111. 145. (Ibid.)
Credible witnesses as used in the statute
relating to wills, means competent witnesses
that is, such persons as are not legally

103
disqualified from testifying in courts of
justice, by reason of mental incapacity,
interest, or the commission of crimes, or
other cause excluding them from testifying
generally, or rendering them incompetent in
respect of the particular subject matter or in
the particular suit. Hill vs. Chicago Title &
Trust co 152 N.E. 545, 546, 322 111. 42.
(Ibid. p, 343)
In the strict sense, the competency of a person to be an
instrumental witness to a will is determined by the statute, that
is Art. 820 and 821, Civil Code, whereas his credibility depends
On the appreciation of his testimony and arises from the belief
and conclusion of the Court that said witness is telling the truth.
Thus, in the case of Vda. de Aroyo v. El Beaterio del
Santissimo Rosario de Molo, No. L-22005, May 3, 1968, the
Supreme Court held and ruled that: "Competency as a witness
is one thing, and it is another to be a credible witness, so
credible that the Court must accept what he says. Trial courts
may allow a person to testify as a witness upon a given matter
because he is competent, but may thereafter decide whether to
believe or not to believe his testimony." In fine, We state the
rule that the instrumental witnesses in Order to be competent
must be shown to have the qualifications under Article 820 of
the Civil Code and none of the disqualifications under Article
821 and for their testimony to be credible, that is worthy of
belief and entitled to credence, it is not mandatory that
evidence be first established on record that the witnesses have
a good standing in the community or that they are honest and
upright or reputed to be trustworthy and reliable, for a person is
presumed to be such unless the contrary is established
otherwise. In other words, the instrumental witnesses must be
competent and their testimonies must be credible before the
court allows the probate of the will they have attested. We,
therefore, reject petitioner's position that it was fatal for
respondent not to have introduced prior and independent proof
of the fact that the witnesses were "credible witnesses that is,
that they have a good standing in the community and reputed
to be trustworthy and reliable.
Under the second, third, fourth, fifth, sixth, seventh and eighth
assignments of errors, petitioner disputes the findings of fact of
the respondent court in finding that the preparation and
execution of the will was expected and not coincidental, in
finding that Atty. Paraiso was not previously furnished with the
names and residence certificates of the witnesses as to enable
him to type such data into the document Exhibit "F", in holding
that the fact that the three typewritten lines under the
typewritten words "pangalan" and "tinitirahan" were left blank
shows beyond cavil that the three attesting witnesses were all
present in the same occasion, in holding credible that Isabel
Gabriel could have dictated the will without note or document
to Atty. Paraiso, in holding that Matilde Orobia was physically
present when the will was signed on April 15, 1961 by the
deceased Isabel Gabriel and the other witnesses Celso
Gimpaya and Maria Gimpaya, in holding that the trial court
gave undue importance to the picture takings as proof that the
will was improperly executed, and in holding that the grave
contradictions, evasions and misrepresentations of the
witnesses (subscribing and notary) presented by the petitioner
had been explained away.
Since the above errors are factual We must repeat what We
have previously laid down that the findings of fact of the

appellate court are binding and controlling which We cannot


review, subject to certain exceptions which We win consider
and discuss hereinafter. We are convinced that the appellate
court's findings are sufficiently justified and supported by the
evidence on record. Thus, the alleged unnaturalness
characterizing the trip of the testatrix to the office of Atty.
Paraiso and bringing all the witnesses without previous
appointment for the preparation and execution of the win and
that it was coincidental that Atty. Paraiso was available at the
moment impugns the finding of the Court of Appeals that
although Atty. Paraiso admitted the visit of Isabel Gabriel and
of her companions to his office on April 15, 1961 was
unexpected as there was no prior appointment with him, but he
explained that he was available for any business transaction
on that day and that Isabel Gabriel had earlier requested him
to help her prepare her will. The finding of the appellate court is
amply based on the testimony of Celso Gimpaya that he was
not only informed on the morning of the day that he witnessed
the will but that it was the third time when Isabel Gabriel told
him that he was going to witness the making of her will, as well
as the testimony of Maria Gimpaya that she was called by her
husband Celso Gimpaya to proceed to Isabel Gabriel's house
which was nearby and from said house, they left in a car to the
lawyer's office, which testimonies are recited in the respondent
Court's decision.
The respondent Court further found the following facts: that
Celso Gimpaya and his wife Maria Gimpaya obtained
residence certificates a few days before Exhibit "F" was
executed. Celso Gimpaya's residence certificate No. A5114942 was issued at Navotas, Rizal on April 13, 1961 while
Maria Gimpaya's residence certificate No. A-5114974 was
issued also at Navotas, Rizal on April 14, 1961. The
respondent Court correctly observed that there was nothing
surprising in these facts and that the securing of these
residence certificates two days and one day, respectively,
before the execution of the will on April 15, 1961, far from
showing an amazing coincidence, reveals that the spouses
were earlier notified that they would be witnesses to the
execution of Isabel Gabriel's will.
We also agree with the respondent Court's conclusion that the
excursion to the office of Atty. Paraiso was planned by the
deceased, which conclusion was correctly drawn from the
testimony of the Gimpaya spouses that they started from the
Navotas residence of the deceased with a photographer and
Isabel Gabriel herself, then they proceeded by car to Matilde
Orobia's house in Philamlife, Quezon City to fetch her and from
there, all the three witnesses (the Gimpayas and Orobia)
passed by a place where Isabel Gabriel stayed for about ten to
fifteen minutes at the clinic of Dr. Chikiamco before they
proceeded to Atty. Cipriano Paraiso's office.
It is also evident from the records, as testified to by Atty.
Paraiso, that previous to the day that. the will was executed on
April 15, 1961, Isabel Gabriel had requested him to help her in
the execution of her will and that he told her that if she really
wanted to execute her will, she should bring with her at least
the Mayor of Navotas, Rizal and a Councilor to be her
witnesses and that he (Atty. Paraiso) wanted a medical
certificate from a physician notwithstanding the fact that he
believed her to be of sound and disposition mind. From this
evidence, the appellate court rightly concluded, thus: "It is,
therefore, clear that the presence of Isabel Gabriel and her
witnesses Matilde Orobia, Celso Gimpaya and Maria Gimpaya

104
including the photographer in the law office of Atty. Paraiso was
not coincidental as their gathering was pre-arranged by Isabel
Gabriel herself."
As to the appellate court's finding that Atty. Paraiso was not
previously furnished with the names and residence certificates
of the witnesses as to enable him to type such data into the
document Exhibit ' L which the petitioner assails as
contradictory and irreconcilable with the statement of the Court
that Atty. Paraiso was handed a list (containing the names of
the witnesses and their respective residence certificates)
immediately upon their arrival in the law office by Isabel
Gabriel and this was corroborated by Atty. Paraiso himself who
testified that it was only on said occasion that he received such
list from Isabel Gabriel, We cannot agree with petitioner's
contention. We find no contradiction for the, respondent Court
held that on the occasion of the will making on April 15, 1961,
the list was given immediately to Atty. Paraiso and that no such
list was given the lawyer in any previous occasion or date prior
to April 15, 1961.
But whether Atty. Paraiso was previously furnished with the
names and residence certificates of the witnesses on a prior
occasion or on the very occasion and date in April 15, 1961
when the will was executed, is of no moment for such data
appear in the notarial acknowledgment of Notary Public
Cipriano Paraiso, subscribed and sworn to by the witnesses on
April 15, 1961 following the attestation clause duly executed
and signed on the same occasion, April 15, 1961. And since
Exhibit "F" is a notarial will duly acknowledged by the testatrix
and the witnesses before a notary public, the same is a public
document executed and attested through the intervention of
the notary public and as such public document is evidence of
the facts in clear, unequivocal manner therein expressed. It
has in its favor the presumption of regularity. To contradict all
these, there must be evidence that is clear, convincing and
more than merely preponderant. (Yturalde vs. Azurin, 28 SCRA
407). We find no such evidence pointed by petitioner in the
case at bar.
Likewise, the conclusion of the Court of Appeals in holding that
the fact that the three typewritten lines under the typewritten
words "pangalan ' and "tinitirahan" were left blank shows
beyond cavil that the three attesting witnesses were all present
in the same occasion merits Our approval because tills
conclusion is supported and borne out by the evidence found
by the appellate court, thus: "On page 5 of Exhibit "F", beneath
the typewritten words "names", "Res. Tax Cert. date issued"
and place issued the only name of Isabel Gabriel with
Residence Tax certificate No. A-5113274 issued on February
24, 1961 at Navotas Rizal appears to be in typewritten form
while the names, residence tax certificate numbers, dates and
places of issuance of said certificates pertaining to the three
(3) witnesses were personally handwritten by Atty. Paraiso.
Again, this coincides with Atty. Paraiso's even the sale must be
made to close relatives; and the seventh was the appointment
of the appellant Santiago as executrix of the will without bond.
The technical description of the properties in paragraph 5 of
Exhibit F was not given and the numbers of the certificates of
title were only supplied by Atty. Paraiso. "
It is true that in one disposition, the numbers of the Torrens
titles of the properties disposed and the docket number of a
special proceeding are indicated which Atty. Paraiso candidly
admitted were supplied by him, whereupon petitioner contends

that it was incredible that Isabel Gabriel could have dictated


the will Exhibit "F" without any note or document to Atty.
Paraiso, considering that Isabel Gabriel was an old and sickly
woman more than eighty-one years old and had been suffering
from a brain injury caused by two severe blows at her head
and died of terminal cancer a few weeks after the execution of
Exhibit "F". While we can rule that this is a finding of fact which
is within the competency of the respondent appellate court in
determining the testamentary capacity of the testatrix and is,
therefore, beyond Our power to revise and review, We
nevertheless hold that the conclusion reached by the Court of
Appeals that the testatrix dictated her will without any note or
memorandum appears to be fully supported by the following
facts or evidence appearing on record. Thus, Isabel Gabriel,
despite her age, was particularly active in her business affairs
as she actively managed the affairs of the movie business
ISABELITA Theater, paying the aparatistas herself until June 4,
1961, 3 days before her death. She was the widow of the late
Eligio Naval, former Governor of Rizal Province and acted as
coadministratrix in the Intestate Estate of her deceased
husband Eligio Naval. The text of the win was in Tagalog, a
dialect known and understood by her and in the light of all the
circumstances, We agree with the respondent Court that the
testatrix dictated her will without any note or memorandum, a
fact unanimously testified to by the three attesting witnesses
and the notary public himself.
Petitioner's sixth assignment of error is also bereft of merit. The
evidence, both testimonial and documentary is, according to
the respondent court, overwhelming that Matilde Orobia was
physically present when the will was signed on April 15, 1961
by the testatrix and the other two witnesses, Celso Gimpaya
and Maria Gimpaya. Such factual finding of the appellate court
is very clear, thus: "On the contrary, the record is replete with
proof that Matilde Orobia was physically present when the will
was signed by Isabel Gabriel on April '15, 1961 along with her
co-witnesses Celso Gimpaya and Maria Gimpaya. The trial
court's conclusion that Orobia's admission that she gave piano
lessons to the child of the appellant on Wednesdays and
Saturdays and that April 15, 1961 happened to be a Saturday
for which reason Orobia could not have been present to
witness the will on that day is purely conjectural. Witness
Orobia did not admit having given piano lessons to the
appellant's child every Wednesday and Saturday without fail. It
is highly probable that even if April 15, 1961 were a Saturday,
she gave no piano lessons on that day for which reason she
could have witnessed the execution of the will. Orobia spoke of
occasions when she missed giving piano lessons and had to
make up for the same. Anyway, her presence at the law office
of Atty. Paraiso was in the morning of April 15, 1961 and there
was nothing to preclude her from giving piano lessons on the
afternoon of the same day in Navotas, Rizal."
In addition to the testimony of Matilde Orobia, Celso Gimpaya
and Maria Gimpaya that Matilde was present on April 15, 1961
and that she signed the attestation clause to the will and on the
left-hand margin of each of the pages of the will, the
documentary evidence which is the will itself, the attestation
clause and the notarial acknowledgment overwhelmingly and
convincingly prove such fact that Matilde Orobia was present
on that day of April 15, 1961 and that she witnessed the will by
signing her name thereon and acknowledged the same before
the notary public, Atty. Cipriano P. Paraiso. The attestation
clause which Matilde Orobia signed is the best evidence as to
the date of signing because it preserves in permanent form a

105
recital of all the material facts attending the execution of the
will. This is the very purpose of the attestation clause which is
made for the purpose of preserving in permanent form a record
of the facts attending the execution of the will, so that in case
of failure in the memory of the subscribing witnesses, or other
casualty they may still be proved. (Thompson on Wills, 2nd
ed., Sec. 132; Leynez vs. Leynez, 68 Phil. 745).
As to the seventh error assigned by petitioner faulting the
Court of Appeals in holding that the trial court gave undue
importance to the picture-takings as proof that the win was
improperly executed, We agree with the reasoning of the
respondent court that: "Matilde Orobia's Identification of the
photographer as "Cesar Mendoza", contrary to what the other
two witnesses (Celso and Maria Gimpaya) and Atty. Paraiso
said that the photographer was Benjamin Cifra, Jr., is at worst
a minor mistake attributable to lapse of time. The law does not
require a photographer for the execution and attestation of the
will. The fact that Miss Orobia mistakenly Identified the
photographer as Cesar Mendoza scarcely detracts from her
testimony that she was present when the will was signed
because what matters here is not the photographer but the
photograph taken which clearly portrays Matilde Orobia
herself, her co-witnesses Celso Gimpaya. " Further, the
respondent Court correctly held: "The trial court gave undue
importance to the picture takings, jumping therefrom to the
conclusion that the will was improperly executed. The evidence
however, heavily points to only one occasion of the execution
of the will on April 15, 1961 which was witnessed by Matilde
Orobia, Celso Gimpaya and Maria Gimpaya. These witnesses
were quite emphatic and positive when they spoke of this
occasion. Hence, their Identification of some photographs
wherein they all appeared along with Isabel Gabriel and Atty.
Paraiso was superfluous."
Continuing, the respondent Court declared: "It is true that the
second picture-taking was disclosed at the cross examination
of Celso Gimpaya. But this was explained by Atty. Paraiso as a
reenactment of the first incident upon the insistence of Isabel
Gabriel. Such reenactment where Matilde Orobia was
admittedly no longer present was wholly unnecessary if not
pointless. What was important was that the will was duly
executed and witnessed on the first occasion on April 15,
1961 , " and We agree with the Court's rationalization in
conformity with logic, law and jurisprudence which do not
require picture-taking as one of the legal requisites for the
execution or probate of a will.
Petitioner points to alleged grave contradictions, evasions and
misrepresentations of witnesses in their respective testimonies
before the trial court. On the other hand, the respondent Court
of Appeals held that said contradictions, evasions and
misrepresentations had been explained away. Such
discrepancies as in the description of the typewriter used by
Atty. Paraiso which he described as "elite" which to him meant
big letters which are of the type in which the will was
typewritten but which was Identified by witness Jolly Bugarin of
the N.B.I. as pica the mistake in mentioning the name of the
photographer by Matilde Orobia to be Cesar Mendoza when
actually it was Benjamin Cifra, Jr. these are indeed
unimportant details which could have been affected by the
lapse of time and the treachery of human memory such that by
themselves would not alter the probative value of their
testimonies on the true execution of the will, (Pascual vs. dela
Cruz, 28 SCRA 421, 424) for it cannot be expected that the

testimony of every person win be Identical and coinciding with


each other with regard to details of an incident and that
witnesses are not expected to remember all details. Human
experience teach us "that contradictions of witnesses generally
occur in the details of certain incidents, after a long series of
questionings, and far from being an evidence of falsehood
constitute a demonstration of good faith. In as much as not all
those who witness an incident are impressed in like manner, it
is but natural that in relating their impressions, they should not
agree in the minor details; hence the contradictions in their
testimony." (Lopez vs. Liboro, 81 Phil. 429).
It is urged of Us by the petitioner that the findings of the trial
court should not have been disturbed by the respondent
appellate court because the trial court was in a better position
to weigh and evaluate the evidence presented in the course of
the trial. As a general rule, petitioner is correct but it is subject
to well-established exceptions. The right of the Court of
Appeals to review, alter and reverse the findings of the trial
court where the appellate court, in reviewing the evidence has
found that facts and circumstances of weight and influence
have been ignored and overlooked and the significance of
which have been misinterpreted by the trial court, cannot be
disputed. Findings of facts made by trial courts particularly
when they are based on conflicting evidence whose evaluation
hinges on questions of credibility of contending witnesses hes
peculiarly within the province of trial courts and generally, the
appellate court should not interfere with the same. In the
instant case, however, the Court of Appeals found that the trial
court had overlooked and misinterpreted the facts and
circumstances established in the record. Whereas the
appellate court said that "Nothing in the record supports the
trial court's unbelief that Isabel Gabriel dictated her will without
any note or document to Atty. Paraiso;" that the trial court's
conclusion that Matilde Orobia could not have witnessed
anybody signing the alleged will or that she could not have
witnessed Celso Gimpaya and Maria Gimpaya sign the same
or that she witnessed only the deceased signing it, is a
conclusion based not on facts but on inferences; that the trial
court gave undue importance to the picture-takings, jumping
therefrom to the conclusion that the will was improperly
executed and that there is nothing in the entire record to
support the conclusion of the court a quo that the will signing
occasion was a mere coincidence and that Isabel Gabriel
made an appointment only with Matilde Orobia to witness the
signing of her will, then it becomes the duty of the appellate
court to reverse findings of fact of the trial court in the exercise
of its appellate jurisdiction over the lower courts.
Still the petitioner insists that the case at bar is an exception to
the rule that the judgment of the Court of Appeals is conclusive
as to the facts and cannot be reviewed by the Supreme Court.
Again We agree with the petitioner that among the exceptions
are: (1) when the conclusion is a finding grounded entirely on
speculations, surmises or conjectures; (2) when the inference
is manifestly mistaken, absurd or impossible; (3) when there is
a grave abuse of discretion; (4) when the presence of each
other as required by law. " Specifically, We affirm that on April
15, 1961 the testatrix Isabel Gabriel, together with Matilde
Orobia, Celso Gimpaya and his wife Maria Gimpaya, and a
photographer proceeded in a car to the office of Atty. Cipriano
Paraiso at the Bank of P.I. Building, Manila in the morning of
that day; that on the way, Isabel Gabriel obtained a medical
certificate from one Dr. Chikiamko which she gave to Atty.
Paraiso upon arriving at the latter's office and told the lawyer

106
that she wanted her will to be made; that Atty. Paraiso asked
Isabel Gabriel to dictate what she wanted to be written in the
will and the attorney wrote down the dictation of Isabel Gabriel
in Tagalog, a language known to and spoken by her; that Atty.
Paraiso read back to her what he wrote as dictated and she
affirmed their correctness; the lawyer then typed the will and
after finishing the document, he read it to her and she told him
that it was alright; that thereafter, Isabel Gabriel signed her
name at the end of the will in the presence of the three
witnesses Matilde Orobia, Celso Gimpaya and Maria Gimpaya
and also at the left-hand margin of each and every page of the
document in the presence also of the said three witnesses; that
thereafter Matilde Orobia attested the will by signing her name
at the end of the attestation clause and at the left-hand margin
of pages 1, 2, 3 and 5 of the document in the presence of
Isabel Gabriel and the other two witnesses, Celso Gimpaya
and Maria Gimpaya; then, Celso Gimpaya signed also the will
at the bottom of the attestation clause and at the left-hand
margin of the other pages of the document in the presence of
Isabel Gabriel, Matilde Orobia and Maria Gimpaya; that Maria
Gimpaya followed suit, signing her name at the foot of the
attestation clause and at the left-hand margin of every page in
the presence of Isabel Gabriel, Matilde Orobia and Celso
Gimpaya; that thereafter, Atty. Paraiso notarized the will as
Page No. 94, Book No. IV, Series of 1961, in his Notarial
Register. On the occasion of the execution and attestation of
the will, a photographer took pictures, one Exhibit "G",
depicting Matilde Orobia, the testatrix Isabel Gabriel, Celso
Gimpaya, Maria Gimpaya and Atty. Paraiso, taken on said
occasion of the signing of the will, and another, Exhibit "H",
showing Matilde Orobia signing testimony that he had earlier
advised Isabel Gabriel to bring with her at least the Mayor and
a Councilor of Navotas, Rizal to be her witnesses for he did not
know beforehand the Identities of the three attesting witnesses
until the latter showed up at his law office with Isabel Gabriel
on April 15, 1961. Atty. Paraiso's claim which was not
controverted that he wrote down in his own hand the date
appearing on page 5 of Exhibit "F" dissipates any lingering
doubt that he prepared and ratified the will on the date in
question."

fifth was the institution of the petitioner-appellant, Lutgarda


Santiago as the principal heir mentioning in general terms
seven (7) types of properties; the sixth disposed of the
remainder of her estate which she willed in favor of appellant
Lutgarda Santiago but prohibiting the sale of such properties to
anyone except in extreme situations in which judgment is
based on a misapprehension of facts; (5) when the findings of
fact are conflicting, (6) when the Court of Appeals, in making its
findings, went beyond the issues of the case and the same is
contrary to the admissions of both appellant and appellee.
(Roque vs. Buan, et al., G.R. No. L-22459, Oct. 31, 1967;
Ramos vs. Pepsi Cola Bottling Co., G.R. No. L-22533, Feb. 9,
1967; Hilarion Jr. vs. City of Manila, G.R. No. L-19570; Sept.
14, 1967).

It is also a factual finding of the Court of Appeals in holding that


it was credible that Isabel Gabriel could have dictated the will,
Exhibit "F", without any note or document to Atty. Paraiso as
against the contention of petitioner that it was incredible. This
ruling of the respondent court is fully supported by the
evidence on record as stated in the decision under review,
thus: "Nothing in the record supports the trial court's unbelief
that Isabel Gabriel dictated her will without any note or
document to Atty. Paraiso. On the contrary, all the three
attesting witnesses uniformly testified that Isabel Gabriel
dictated her will to Atty. Paraiso and that other than the piece of
paper that she handed to said lawyer she had no note or
document. This fact jibes with the evidence which the trial
court itself believed was unshaken that Isabel Gabriel was
of sound disposing memory when she executed her will.

We rule that the respondent Court's factual findings upon its


summation and evaluation of the evidence on record is
unassailable that: "From the welter of evidence presented, we
are convinced that the will in question was executed on April
15, 1961 in the presence of Matilde Orobia, Celso Gimpaya
and Maria Gimpaya signing and witnessing the same in the the
will on a table with Isabel Gabriel, Celso Gimpaya and Maria
Gimpaya sitting around the table. Atty. Paraiso, after finishing
the notarial act, then delivered the original to Isabel Gabriel
and retained the other copies for his file and notarial register. A
few days following the signing of the will, Isabel Gabriel, Celso
Gimpaya and another photographer arrived at the office of Atty.
Paraiso and told the lawyer that she wanted another picture
taken because the first picture did not turn out good. The
lawyer told her that this cannot be done because the will was
already signed but Isabel Gabriel insisted that a picture be
taken, so a simulated signing was performed during which
incident Matilde Orobia was not present.

Exhibit "F" reveals only seven (7) dispositions which are not
complicated but quite simple. The first was Isabel Gabriel's
wish to be interred according to Catholic rites the second was
a general directive to pay her debts if any; the third provided
for P1,000.00 for her sister Praxides Gabriel Vda. de Santiago
and P2,000.00 for her brother Santiago Gabriel; the fourth was
a listing of her 13 nephews and nieces including oppositorappellee Rizalina Gabriel and the amount for each legatee the

Petitioner's insistence is without merit. We hold that the case at


bar does not fall within any of the exceptions enumerated
above. We likewise hold that the findings of fact of the
respondent appellate court are fully supported by the evidence
on record. The conclusions are fully sustained by substantial
evidence. We find no abuse of discretion and We discern no
misapprehension of facts. The respondent Court's findings of
fact are not conflicting. Hence, the well-established rule that
the decision of the Court of Appeals and its findings of fact are
binding and conclusive and should not be disturbed by this
Tribunal and it must be applied in the case at bar in its full force
and effect, without qualification or reservation. The above
holding simply synthesize the resolutions we have heretofore
made in respect ' to petitioner's previous assignments of error
and to which We have disagreed and, therefore, rejected.
The last assignments of error of petitioner must necessarily be
rejected by Us as We find the respondent Court acted properly
and correctly and has not departed from the accepted and
usual course of judicial proceedings as to call for the exercise
of the power of supervision by the Supreme Court, and as We
find that the Court of Appeals did not err in reversing the
decision of the trial court and admitting to probate Exhibit "F",
the last will and testament of the deceased Isabel Gabriel.

Petitioner's exacerbation centers on the supposed incredibility


of the testimonies of the witnesses for the proponent of the will,
their alleged evasions, inconsistencies and contradictions. But
in the case at bar, the three instrumental witnesses who
constitute the best evidence of the will making have testified in
favor of the probate of the will. So has the lawyer who

107
prepared it, one learned in the law and long in the practice
thereof, who thereafter notarized it. All of them are
disinterested witnesses who stand to receive no benefit from
the testament. The signature
s of the witnesses and the testatrix have been identified on the
will and there is no claim whatsoever and by anyone, much
less the petitioner, that they were not genuine. In the last and
final analysis, the herein conflict is factual and we go back to
the rule that the Supreme Court cannot review and revise the
findings of facts of the respondent Court of Appeals.
WHEREFORE, IN VIEW OF THE FOREGOING, the judgment
appealed from is hereby AFFIRMED, with costs against the
petitioner.
SO ORDERED.
Teehankee, Makasiar, De Castro and Herrera, JJ., concur.

S-ar putea să vă placă și